You are on page 1of 245

Matemàtiques

1r BATXILLERAT

SOLUCIONARI
1a PART: UNITATS 1-7
SOLUCIONARI UNITAT 1. NOMBRES REALS

1. Nombres reals
EXERCICIS PROPOSATS
1. Exercici resolt.

2. Troba la fracció irreductible que correspon als nombres següents racionals.

a) 25,25 b) 25,25 c) 25,25 d) 25,25  25,25  25,25

2525 101
a) 25,25  
100 4

100N  2525,252 525... 2500


b) N  25,25  25,252 525...    99N  2500  N 
N  25,252 525 2... 99

100N  2525,555... 2273


c) N  25,25  25,2555...    90N  2273  N 
10N  252,555... 90

101 2500 2273 150001


d) 25,25  25,25  25,25    
4 99 90 1980

3. Fes les operacions següents i simplifica el resultat.


15 14
a) b)
1 1
1 1
1 1
1 1
2 3
15 15 15 15 3  15 14 14 14 14 4  14
a)     9 b)     8
1 1 2 5 5 1 1 3 7 7
1 1 1 1 1 1
1 3 3 3 1 4 4 4
1 1
2 2 3 3

4. Raona amb exemples si són certes o falses les afirmacions següents.


a) La suma de dos irracionals és sempre irracional.
b) El producte de dos irracionals és sempre irracional.

a) És fals. Per exemple, 2 i  2 són dos nombres irracionals, i la seva suma és 0, nombre racional.

b) Es fals. Per exemple, 2 i  2 són dos nombres irracionals, i el seu producte és 2, nombre racional.

5. Es vol posar una tanca a un camp rectangular. Se sap que un dels seus costats fa tres cinquenes parts de
la mida de l’altre, i la diagonal mesura 30 m. Si un metre de tanca costa 25 € i es desaprofita un 10% del
material usat, calcula el preu que s’haurà de pagar.

3a
Els costats mesuren a i , per tant la diagonal és:
5
2 2
 3a  9a 34a2
D  a2     a 
2
  30  a  25,725 m
 5  25 25

1 Matemàtiques 1r Batxillerat
SOLUCIONARI UNITAT 1. NOMBRES REALS

 3a 
El perímetre mesura 2  a    82,32 m , però com que es negligeix el 10 % del material, necessitem comprar
 5 
82,32 : 0,90  91,47 m, amb un cost de 91,47 · 25  2286,75 €.

6 i 7. Exercicis resolts.

8. Ordena de més petit a més gran en cada cas:


11 3 6 8 7 5 15 11
a) , , , b) , , ,
12 4 7 9 6 4 14 10

3 6 8 11 15 11 7 5
a)    b)   
4 7 9 12 14 10 6 4

9. Ordena de més petit a més gran en cada cas.


11 68 14 27 4 3
a) , , i c) 4 , 3 i 2
4 25 5 10

b) 1,23, 1,23 i 1,23 d) 2,9 , 3 i 3,01

11 275 68 272 14 280 27 270 27 68 11 14


a)  ;  ;  i     
4 100 25 100 5 100 10 100 10 25 4 5

b) 1,23  1,232323…  1,2333…  1,23  1,23  1,23

4
c)
4
4  22  2  1,4142... , 3
3  1,4422...  4 4  2  3 3

29  2
d) 2,9   3  2,9  3  3,01
9

10. Ordena de més petit a més gran en cada cas:

3 2 4 4 2 3 2 3
a) , , , b) 3 , 6 , 5 , 4
2 5 7 9 3 2 3 2

a)    b)   

11. Siguin a i b dos nombres reals negatius. Si a  b, demostra que l’invers de a és més gran o igual que l’in-
vers de b. Què passaria si a fos negatiu i b positiu?

1 1 b 1 b 1 1 1
a  b  a  b   1   1    
a a a b a b b a
Si a  0 i b  0, aleshores:
1 1 b 1 b 1 1 1
a  b  a  b   1   1    
a a a b a b b a
És a dir, si a és negatiu i b positiu i, per tant, a  b, aleshores l’invers de a és més petit que l’invers de b.
Es podria haver deduït de manera més fàcil tenint en compte que l’invers de a és negatiu i el de b positiu i un
nombre negatiu és sempre més petit que un de positiu.

2 Matemàtiques 1r Batxillerat
SOLUCIONARI UNITAT 1. NOMBRES REALS

12. Tres nombres reals positius a, b i c poden representar les mides dels tres costats d’un triangle si i només
si es compleix que la suma dels dos més petits és més gran que el gran. Indica els valors que pot prendre
a perquè els nombres naturals a, 2a  1 i 10 siguin les mides d’un triangle.

Hi ha dues possibilitats:
Si el costat més gran és 2a  1. Aleshores

2a  1  10 a  4,5
    4,5  a  9
a  10  2a  1 a  9

Si el costat més gran és 10. Aleshores

2a  1  10 a  4,5
    3  a  4,5
a  2a  1  10 a  3

Per tant, 3  a  9

13 i 14. Exercicis resolts.

15. Representa en la recta real els nombres següents.


4 12
a) 5 b) c) 2 d) 
7 5

16. Representa en la recta real:

a) 17 c) 11

b) 29 d) 20

 2
2
a) 17 4  1 c) 11   32
2 2

b) 29  5  2 d) 20  2  4
2 2 2 2

17 i 18. Exercicis resolts.

19. Desenvolupa el valor d'aquestes expressions:


a) 5 + |x –1| b) 2x – |x + 4|
Calcula el valor de les expressions anteriors per als casos x = –1, x = 0 i x = 4.

3 Matemàtiques 1r Batxillerat
SOLUCIONARI UNITAT 1. NOMBRES REALS

6 – x si x  1
a) 5  x –1  
4  x si x  1

3 x  4 si x  4
2x  x  4  
b)  x  4 si x  4

x = –1 x = 0 x = 4
5 + |x –1| 7 6 8
2x – |x + 4| –5 –4 0

20. Desenvolupa el valor de les expressions següents.


a) 2x  3  2x  3 b) 2  3x  2  3x

Calcula el valor de les expressions anteriors per als casos x  1, x  0 i x  2.

 3
2x  3  (2x  3) si 2 x  3  0  0 si x 
a) 2x  3  2x  3    2
3
2x  3  (2 x  3) si 2 x  3  0 4 x  6 si x 
 2

 2
2  3 x  (2  3 x ) si 2  3 x  0  0 si x 
b) 2  3 x  2  3 x    3
2
2  3 x  (2  3 x ) si 2  3 x  0 6 x  4 si x 
 3
Per a x  1, el valor de totes dues expressions és 0. Per a x  0, el valor de totes dues expressions és 0. Per a x 
2, el valor de la primera expressió és 2, i el de la segona és 8.

21 i 22. Exercicis resolts.

23. Siguin A  (2, 4), B  (2, 6] i C  [3, ), calcula:


a) A  B  C b) A  B  C c) (A  B)  C d) (A  B)  C

a) A  B  C  C  [3, ) c) (A  B)  C  [3, )


b) A  B  C  A  (2, 4) d) (A  B)  C  B  (2, 6]

24. Expressa com un entorn els intervals (5, 2) i [5, 2].

3 7  3 7
(5, 2) és l’entorn obert de centre  i radi E  , 
2 2  2 2

3 7  3 7
[5, 2] és l’entorn tancat de centre  i radi E  , 
2 2  2 2

4 Matemàtiques 1r Batxillerat
SOLUCIONARI UNITAT 1. NOMBRES REALS

25. Expressa per mitjà d’intervals i gràficament els conjunts de nombres reals següents.
a) x 2  2 b) x 3 1 c) x 1  2

a) (0, 4) b)  ,  4  2,    c)  3,1

26. Exercici interactiu.

27 i 28. Exercicis resolts.

29. Calcula les millors aproximacions per defecte i per excés i l’arrodoniment de 2 a la unitat, la centèsima i
la deumil·lèsima.

Unitat Centèsima Deumil·lèsima


Defecte 1 1,41 1,4142
Excés 2 1,42 1,4143
Arrodoniment 1 1,41 1,4142

12
30. Troba els errors absolut i relatiu que es fan quan s’utilitza 1,7 com a aproximació de .
7

1
12 12 17 1 70 1
Error absolut: Ea   1,7    Error relatiu: Er  
7 7 10 70 12 120
7

31. Troba les aproximacions per defecte i excés amb una, dues i tres xifres decimals de:

a) 5 b) 

Fent servir les aproximacions anteriors, calcula el valor de 5   . En quina d’aquestes l’error absolut és
més gran?

5 1 xifra 2 xifres 3 xifres  1 xifra 2 xifres 3 xifres


Defecte 2,2 2,23 2,236 Defecte 3,1 3,14 3,141
Excés 2,3 2,24 2,237 Excés 3,2 3,15 3,142

5 1 xifra 2 xifres 3 xifres


Defecte 5,3 5,37 5,377
Excés 5,5 5,39 5,379

L’error absolut és òbviament més gran en les aproximacions amb una xifra decimal.

32 i 33. Exercicis resolts.

5 Matemàtiques 1r Batxillerat
SOLUCIONARI UNITAT 1. NOMBRES REALS

34. Calcula les operacions següents i dona el resultat en notació científica.


a) 0,000 48  0,000 059 e) 1,2 ∙ 10  5000 ∙ 2,4 ∙ 10
8 5

2,2  109  7,8  108


b) 35 000 000  720 000 000 f)
7,1 1011

c) 250 000 · 5,5·10


5
g)

0,000 16 25  103  2000 
0,0025

1023  5,5  1012


d) 0,000 000 15 : 0,000 003 h)
3,5  1022  4,3  1021

a) 0,000 48  0,000 059  4,8 · 10  5,9 · 10


–5 –5 –5 –5 –4
 48 · 10  5,9 · 10  53,9 · 10  5,39 · 10
4

b) 35 000 000  720 000 000  3,5 · 10  7,2 · 10  3,5 · 10  72 · 10  68,5 · 10  6,85 · 10
7 8 7 7 7 8

c) 250 000 · 5,5 · 10  2,5 · 10 · 5,5 · 10  13,75 · 10  1,375 · 10


5 5 5 10 11

–7 –6 –1 –2
d) 0,000 000 15 : 0,000 003  1,5 · 10 : 3· 10  0,5 · 10  5 · 10
e) 1,2 ∙ 10  5000 ∙ 2,4 ∙ 10  1,2 ∙ 10  5 · 10 ∙ 2,4 ∙ 10  1,2 ∙ 10  12 ∙ 10  13,2 · 10  1,32 · 10
8 5 8 3 5 8 8 8 9

2,2  109  7,8  108 22  108  7,8  108 14,2  108


f)    2  103
7,1 1011 7,1 1011 7,1 1011

g)

0,000 16 25  103  2000   1,6  10  25  10
4 3
 2  103   1,6  10 4
 27  103
 17,28  102  1,728  103
3
0,0025 2,5  10 2,5  103

1023  5,5  1012 5,5  1011 5,5  1011


h)    0,13995  1010  1,3995  1011
3,5  1022  4,3  1021 35  1021  4,3  1021 39,3  1021

–24
35. Un àtom d’hidrogen (H) té una massa de 1,66 · 10 g aproximadament.
a) Quants àtoms de H es necessiten per obtenir 20 kg d’aquest gas?
26
b) Quina és, aproximadament, la massa de 2,524 · 10 àtoms de H?
c) Si 2 g d’hidrogen molecular ocupen un volum de 22,4 L a 0 °C i a la pressió d’1 atm, quantes molècules d’hidro-
gen hi hauria en un recipient de 5 L en aquestes condicions?

20000
a)  1,205  1028 àtoms de H seran necessaris per obtenir una massa de 20 kg.
1,66  1024

b) 2,524  1026·1,66  1024  419 g  0,419 kg

2 5
c) El recipient de 5 litres conté
22, 4
grams d’hidrogen, és a dir,
10
22,4
: 1,66  1024  2,689·1023 àtoms d’hidro-  
23
gen. Com que cada molècula està composta per dos àtoms, hi haurà 1,345 · 10 molècules en total.

24 22
36. La massa de la Terra és de 5,97 · 10 kg, i la de Plutó, de 1,29 · 10 kg.
a) Quina és la relació entre les dues masses?
b) Si suposéssim que tots dos cossos fossin esferes perfectes amb radis de 6371 i 1160 km, respectivament, cal-
cula la densitat aproximada de cadascun d’aquests.

 463 vegades més gran que la de Plutó.


24 22
a) La massa de la Terra és 5,97·10 : 1,29 · 10

Massa 5,97  1024 kg 5,5  1012  1000


b) Densitat de la Terra    5,5  1012 kg/km3   5,5 g/cm3
Volum 4   63713 km3 1015
3

6 Matemàtiques 1r Batxillerat
SOLUCIONARI UNITAT 1. NOMBRES REALS

Massa 1,29  1022 kg


Densitat de Plutó    1,97  1012 kg/km3  1,97 g/cm3
Volum 4   11603 km3
3
37. Exercici resolt.

38. Simplifica les expressions següents.

 a
3 3
3 1 8  2 32  2 128 a4  a2 
a) 2 8 18 c) e)
2 4 2 3
a5

4 27 2 8
3
a  a3 a 1
b) 144a2  2 a  3a d) f) :3
16 20  5 6
a a3
a

3 1 3·2 3 3 13
a) 2 8 18  2  2 2  2 3 2  2 2
2 4 2 4 4 4

4 27 3 3 3
b) 144a2  2 a  3a  12a  2· 3a  3a  2 3a  3a  3a  3a
16 4 2 2

8  2 32  2 128 2 2  8 2  16 2 26 2
c)    26
2 2 2

8 8
2 8 2 2 2 3 2 2 8
2 8
2 57 2  57
d) 4  4  4   
20  5 2 5 5 3 5 5 8
5 8
5
8
57 5

2 3
 a
3 3 2 3 5 16
a4 a2 a4 a 3 a 2 4   3
e)  5
a 3 2 6  a 3  a16
3
a5 a6
1 1 1 7 1 29
4 1 4 24
3
a a3 a 1 a3 a6 a 1 a3 a7 a3 a a3 a 12
f) :3  : 1
 a3  5
 5
 a 12  a2
6 a 6 12
a5
a a3 a2·a3 a3 a 12 a 12

39. Opera i simplifica les expressions següents.

1 3
a) 128 2  162 2 b) 2 2 2

1 3
a) 128 2  162 2  128  1623  27  23  312  23 2  2  36 2  8 2  1458 2  1466 2

8
b) 2 2 2  23 2  27  27

40. Troba una expressió més senzilla per a les següents.

a a a a
a)  b) 
a a 2 2 a 2 2 a
1 1
1 a 1 a

a a a a
a)     a (1  a )  a (1  a )  a  a  a  a  2a
a a 1 a  a 1 a  a
1 1
1 a 1 a 1 a 1 a

7 Matemàtiques 1r Batxillerat
SOLUCIONARI UNITAT 1. NOMBRES REALS

b)
a

a

a  2 2 a  
a  2 2 a  
2a  2a

2a  2a

2 2 a 2 2 a  2 2 a  2 2 a   2 2 a  2 2 a  2  4a 2  4a

2a  2a  2a  2a 4a 2a
 
2  4a 2  4a 2a  1

41. Racionalitza els denominadors següents.

5 5 6
a) c) e)
2 5 2 5 1 2 3 3 2

5 3 2 3 3 2
b) d) f)
2 4
5 3 2 6 8  27

5 5 5 5 5 5
a)   
2 5 2 5 5 10 2

4 4 4
5 5  53 5  53 53
b)   
2 45 2 4 5  53
4 25 2

c)
5


5 2 5 1  
10 5  5

10 5  5 10 5  5

2 5 1  2 5 1 2 5 1   2 5  2
1 2 4 5 1 19

d)
3

3  3 2 6  
3 3 6 6

3 3 6 6

3 2 6
3 2 6  3 2 6  3 2 6  3  46 21 7

e)
6


6 2 3 3 2  
2 18  3 12

6 2 6 3
 2 3
2 3 3 2 2 3 3 2 2 3 3 2   43  92 6

f)
2 3 3 2

2 3 3 2

2 3 3 2 2 2 3 3  4 6  18  12  9 6 5 6  6

8  27 2 2 3 3 2 2 3 3  2 2 3 3 8  27 19

42. Exercici interactiu.

43 a 46. Exercicis resolts.

47. Aplicant la definició, troba el valor dels logaritmes:


1
a) log2 16 c) log7 e) log3 3 3 g) log3 0,3
49

b) log5 3
25 d) log9 3 3 f) log 1 8 h) log8 0,125
2

a) log2 16  log2 2  4
4

2
3 2
b) log5 3
25  log5 52  log5 5 3 
3

1 1 1
c) log7  x  7x   72  x  2  log7  2
49 49 49
1 1
1 1 1
d) log9 3 3  x  9 x  3 3  3 3  32 x  3 3  2x   x   log9 3 3 
3 6 6
1 3
3 3
e) log3 3 3  x  3 x  3 3  3·3 2  3 2  x   log3 3 3 
2 2

8 Matemàtiques 1r Batxillerat
SOLUCIONARI UNITAT 1. NOMBRES REALS

x 3
 1 3 3
f) log 1 8     8  2 x  23  2 2  x    log 1 8  
2  
2 2 2
2

1
g) log3 0,3  x  3x  0,3   31  x  1  log3 0,3  1
3

1
h) log8 0,125  x  8 x  0,125  23 x   23  3 x  3  x  1  log8 0,125  1
8
48. Prenent log 2 = 0,301 i log 3 = 0,477, troba:

log3 8 b) log 60 c) log 0,012

log8 log23 3log2


a) log3 8     1,893
log3 log3 log3

b) log 60  log (2∙3∙10)  log 2  log 3  log 10  0,301  0,477  1 1,778


1
12  12  2 1 12 1 1
c) log 0,012  log  log    log  (log12  log1000)  (log(22·3)  log103 ) 
1000  1000  2 1000 2 2

1 1
 (2log2  log3  3)  (2·0,301  0,477  3)  0,9605
2 2

49. Aplica logaritmes a l’expressió A  (x ) .


x x

 
log A  log  x x  
  x log x x  x  x log x  x 2 log x
x
 

50. Escriu en forma algebraica les expressions següents.

a) log A  2  2log x  log y c) log C  2log x  log x  log y  3log 3 y

b) log B  3(log x  1)  2(1  log y) d) log D  log x  3log y  log x


3 4

100 x 2 100 x 2
a) log A  log100  log x 2  log y  log A  log A
y y

x3y 2 x3y 2
b) log B  3log x  2log y  5  log  B 
105 105
c) logC  log x  log x  log y  log y  0  log1  C  1

x3y 3 y3 y3
d) log D  log x 3  log y 3  log x 4  log  log  D 
x4 x x

51. Troba el valor dels logaritmes següents fent servir la calculadora.

a) log3 21 b) log0,01 12 c) log 3


19

ln 21 ln12 ln19
a) log3 21   2,771 b) log0,01 12   0,540 c) log 3
19   5,360
ln3 ln0,01 ln 3

52 a 54. Exercicis resolts.

9 Matemàtiques 1r Batxillerat
SOLUCIONARI UNITAT 1. NOMBRES REALS

55. En un cultiu de bacteris, el nombre es duplica cada dos dies. Un dia es compten 3000 bacteris.
a) Calcula el nombre de bacteris que hi haurà 15 dies més tard.
b) Quants dies han de passar perquè hi hagi el triple de bacteris? I si el nombre inicial fos de 6000 bacteris?
c) Se suposa que la població s’estabilitza quan arriba als 20 000 bacteris. Quant de temps ha de passar per arri-
bar-hi?
t
El nombre de bacteris quan han passat t dies és N  3000  2 2 .

a) Per a t  15  N  3000  27,5  543 058 bacteris


t t t
t log3
b) 3N  N  2 2  2 2  3  log2 2  log3  log2  log3  t  2   3,17 dies
2 log2

El resultat anterior és independent del nombre inicial de bacteris.


20
t t t log
20000 20 t 20 3  5,47 dies
c) 20000  3000  2 2  22   log2  log
2  log2  log  t  2
3000 3 2 3 log2

56. Certa substància radioactiva té un període de semidesintegració de 1600 anys. Calcula la quantitat de mas-
sa a què s’haurà reduït 1 quilogram d’aquesta substància després de 10 000 anys.

10000
La massa al cap de 10 000 anys serà: 1 · 0,5 1600  0,01314 kg  13,14 g

57. Una població de conills augmenta anualment un 50 %. Si en el moment inicial hi havia 100 conills, quant de
temps ha de passar perquè el nombre de conills passi a ser de 30 000?

100·1 0,5  30.000  1 0,5  300  log 1,5   log300  t log 1,5   log300  t  14,067
t t t

Al quinzè any.

58. Un cotxe ens ha costat 14 425 € i es deprecia un 15 % anual. Quants anys han de passar perquè el seu va-
lor sigui inferior a 3600 €?
Per calcular quants anys han de passar perquè el valor del cotxe sigui inferior a 3600 €, resolem l’equació següent:
3600 3 600
14 425  0,85t  3 600  0,85t   t log0,85  log  t  8,54 . Hauran de passar almenys 9 anys.
14 425 14 425

59. Exercici interactiu.

61 a 75. Exercicis resolts.

10 Matemàtiques 1r Batxillerat
SOLUCIONARI UNITAT 1. NOMBRES REALS

EXERCICIS
Nombres reals
74. Escriu dos nombres compresos entre:
19 20 22
a) i b) i
23 23 7

19 57 20 60 58 59
a) Resposta oberta.  y  . Entre aquests dos nombres hi ha i .
23 69 23 69 69 69

22
b) Resposta oberta.  3,1428... ,   3,1415... Entre aquests dos hi ha 3,1416 i 3,1417.
7

75. Classifica els nombres següents en racionals i irracionals. En el cas dels racionals, indica’n l’expressió per
mitjà d’una fracció irreductible.
a) 12,121 314 15… c) 12,012 121 2… e) 1,123 123 123…
b) 12,121 212… d) 1,010 010 001… f) 0,001 002 003 004…

a) 12,121 314 15… Irracional

100N  1212,1212... 1200 400


b) 12,121 212…  12,12 Racional   99N  1200  N  
N  12,121212... 99 33

1000N  12012,1212... 11892 1982


c) 12,012 121 2…  12,012 Racional   990N  11892  N  
10N  120,121212... 990 165

d) ,010010001… Irracional

1000N  1123,123123... 1122 374


e) 1,123 123 123…  1,123 Racional   999N  1122  N  
N  1,123123... 999 333

f) 0,001 002 003 004… Irracional

76. Classifica aquests nombres indicant a quins conjunts numèrics pertanyen.

a) 25,012 345 6… c) 4 e) 2 g)  0,0625

3 65
b) 25,425 252 5… d) f) 2,3 h) 
7 13

a) 25,012 345 6… és irracional i real. e) 2 és natural, enter, racional i real.

b) 25,425 252 5… és racional i real. f) 2,3 és irracional i real.

c) 4 és enter, racional i real. g)  0,0625  0,25 és racional i real.

3 65
d) és racional i real. h)  5 és enter, racional i real.
7 13

77. Ordena de més petit a més gran aquests nombres.


126 226
25,0111… 25,01
5 9

126 226 226 126


 25,2 i  25,1111… L’ordre és: 25,01  25,0111…  
5 9 9 5

11 Matemàtiques 1r Batxillerat
SOLUCIONARI UNITAT 1. NOMBRES REALS

78. Calcula el valor de les expressions i expressa el resultat per mitjà de nombres decimals periòdics.
2,23  2,2333...
a) b) 1 1,12  1,12  1,12
2,232323...

223 223  22 1339



2,23  2,2333... 100 90 132 561 3399
a)   300    1,999411764705882352
2,232 323... 223  2 221 66 300 1700
99 99
112 112  11 112  1 21599
b) 1  1,12  1,12  1,12  1      4,3634
100 90 99 4950

79. Representa els nombres reals següents.


12
a) c) 5 e) 7
5

3
b)  d) 6 f) 8
7

80. Indica quins nombres reals representen els punts A i B de la figura.

B  22   5   4  5  3
2
A  12  22  5

Valor absolut i intervals


81. Desenvolupa les expressions següents.
a) 2x  4  x b) x  2x

2x  4  x si x  2 4  x si x  2
a) 2x  4  x   
2x  4  x si x  2 3 x  4 si x  2

 x  2x si x  0 3 x si x  0
b) x  2x   
 x  2 x si x  0 3 x si x  0

Es podia haver fet x  2x  x  2 x  3 x

82. Desenvolupa les expressions següents.


a)x  1 | x  1 b) x+ x  x  2

  x  1   x  1 si x  1 2x si x  1
 
a) x  1  x  1   ( x  1)  x  1 si 1  x  1   2 si 1  x  1
 x  1 x  1
 si x 1 
 2x si x 1

 x  x  x  2 si x  2  x  2 si x  2
b) x  x  x  2   x  x  x  2 si 2  x  0   x  2 si 2  x  0
 x  x  x  2 si x 0 3 x  2 si x 0

12 Matemàtiques 1r Batxillerat
SOLUCIONARI UNITAT 1. NOMBRES REALS

83. Siguin els conjunts A  (2, ), B  (2, 0] i C  [0, 4), calcula A  B  C i A  B  C.

A  B  C  A  (2, ) A  B  C  {0}

84. Expressa per mitjà d’un interval els conjunts de nombres reals x següents i representa’ls a la recta real.

1 1 1
a) x  b) 2x  6  1 c) x 
2 4 3

1 3 1  7 5   1 1 
a)  ,  b) x 3   ,  c)  , 
4 4 2  2 2   3 3

Aproximacions i errors
85. Dona l’expressió aproximada que es demana en cada cas.
23
a) per excés amb tres xifres decimals.
7

b) 5  125 per defecte amb dues xifres decimals.

c) 2  1 arrodonit a tres xifres decimals.

23
a) 3,286
7

b) 5  125 13,41

c) 2  1 5,283

86. Explica com es calcula l’error relatiu que es comet en prendre com a aproximació del nombre auri
1 5
 el nombre racional 1,618.
2

1 5
 1,618
2 0,000 034
Error relatiu: Er    0,000 022
1,618 1,618

87. Escriu les aproximacions fins a les mil·lèsimes, per excés i per defecte, dels nombres 3 i 3. Troba a con-
tinuació aproximacions per defecte i per excés del producte 3∙ 3 .

3 1 xifra 2 xifres 3 xifres 3 1 xifra 2 xifres 3 xifres


Defecte 1,7 1,73 1,732 Defecte 9,4 9,42 9,424
Excés 1,8 1,74 1,733 Excés 9,5 9,43 9,425

3 3 1 xifra 2 xifres 3 xifres


Defecte 16,3 16,32 16,324
Excés 16,4 16,33 16,325

13 Matemàtiques 1r Batxillerat
SOLUCIONARI UNITAT 1. NOMBRES REALS

Aproximant amb tres decimals per defecte tenim:

per a 3  1,732 050 8…: Ea  0,000 050 8…

per a 3  9,424 777 9… : Ea  0,000 777 9…

per a 3 3  16,324 194 2… : Ea  0,000 194 2…


Aproximant amb tres decimals per excés tenim:

per a 3  1,732 050 8…: Ea  0,000 949 1…

per a 3  9,424 777 9… : Ea  0,000 222 0…

per a 3 3  16,324 194 2… : Ea  0,000 825 7…

Notació científica
88. Fes les operacions següents i expressa el resultat en notació científica.
a) 10  4 · 10
8 6
e) 0,000 06 : 45 000 000
–13 –23
b) 0,000 25 · 0,0015 f) 0,0025 · 10 : 10

1,2  108  1,5  107  6  109


c) 235 000 · 0,000 25 g)
0,000 003

7  1020  5  1018
d) 15 000 000 : 45 000 h)
0,000 000 004

–12
a) 10  4 · 10  9,6 · 10 e) 0,000 06 : 45 000 000  1,333… · 10
8 6 7

−7 –13 –23
b) 0,000 25 · 0,0015  3,75 · 10  2,5 · 10
7
f) 0,0025 · 10 : 10

1,2  108  1,5  107  6  109


c) 235 000 · 0,000 25  5,875 · 10 g)  2,035  1015
0,000 003

7  1020  5  1018
d) 150 000 000 : 450 000  3,333…·10  1,2675  109
2
h)
0,000 000 004

Potències i radicals
89. Simplifica les expressions següents.

33  9 22  5 1 3
a) f) 4
390 625 a5b16 k) 16 2  9 2
2( 3)  5
2
 3
4 
1
2    42
3

b) 
2 4 3
2
g) x 3
x x3 l) 81a3  2a 3 24
6
2 3
3 4
   
c)  4  3
2 3 3 3
h) 3 3 3 m) 2 4
2 3
2
2715  75 
40
1 1 2 3
d) i)   2   n) 
4535  15 
60
2 2 3 2

x x
e) 3  2 27  12 j) 3
x

14 Matemàtiques 1r Batxillerat
SOLUCIONARI UNITAT 1. NOMBRES REALS

33  9 22  5 36  3 37
a)  
2( 3)  5 11 11
2
 3 2 1
 2   4  4 
3 1
22 
 2 48 22  22·32
b)   3
62 22·32 24·3
2 3
3 4 22 33
     6
22  33  24  33
c)  4  3
2 3 2
 34 23   34  81
2 3 2 3 32  26
15
2715   75 
40
 33   3  52  345  340  580
40

d)   70 35 60 60  3135  515
45  15 
35  60
 32  5   3  5 
35 60 3 5 3 5

e) 3  2 27  12  3  2  3 3  2 3  5 3

4
f) 4
390 625 a5b16  58 a5b16  52 a b4 4 a  25ab4 4 a

g) x 3
x 4
x 3  12 x 6 x 4 x 9  12 x19  x12 x 7

h) 3 3 3  8 34323  8 37

2
1 1 1 1 1 3 7 3 7 6
i)   2     2   2     
2 2 4 2 2 2 4 2 4 2

x x
4
x3  x 3 3 12
j)   12  x5
3
x 3
x x4
1 3
k) 16  9  16  36  4  27  31
2 2

3
l) 81a3  2a 3 24  3a 3 3  4a 3 3  7a 3 3

3 6 18
m) 3
2 34  23  42  27

2 3 2 3 2 2 3 3 23 5 5 6
n)       
3 2 3 2 6 6 6 6

90. Opera i simplifica.

c) 2  2  3 2    2  3 2  2  3 2 
14 1 2
a)  2   2   2  ...   2 80  4 405  4 5
0 1 2 8
b)
3 2

a)  2   2   2  ...   2  1 2  4  8  16  32  64  128  256  171


0 1 2 8

14 1 1 1 11
b) 80  4 405  4 5   24 5   3 4 5  4 5   4 5
3 2 3 2 6

c) 2  2  3 2    2  3 2  2  3 2   2  4  18  12 2   4  18  30  24 2
2

15 Matemàtiques 1r Batxillerat
SOLUCIONARI UNITAT 1. NOMBRES REALS

91. Racionalitza els denominadors.

a 2
a) d)
a a
6 8 1 2

3y 2 6
b) e)
2 y 5 2 3 2

x2 6 6
c) f)
2 x2 2 3 3 2

3 3
a a 1 a2 a2
a)    
a a8
6
a2
6
a2 a3 a a 3 a a2
3 a2

3y 3y 5 y 3 3y 5 y 3 35 y 3
b)   
25 y 2 25 y 2  5 y 3 2y 2

x2  x  2 x  2  x  2 x  2 x2
c)   
2 x2 2 x2 x2 2  x  2 2

2 2 1  2  2 2
d)    2 2
1 2 1 2 1  2  1 2

2 6 2 6  3  2 2 18  2 12
e)   6 2 4 3
3 2  3  2  3  2  32

6 6 6 6 2 3  3 2  12 18  18 12
f)    3 12  2 18  6 3  6 2
2 3 3 2 2 3  3 2  2 3  3 2  12  18

92. Racionalitza els denominadors.


1
a)
1 2  3

1 2
b)
8 3 2

2 3
c)
2 3 6

1 2  3 1 2  3 1 2  3 1 2  3 1 2  3 2 2 2  6
a)     
1 2 3 1  2 3  1  2 3
2
1 2  2 2  3 2 2 2 2 2 4

1 2 1 2 1 2 1 2  3 2  3  3 2  3 6 6 3 2  3 6 6


b)     
8 3 2 2 2 3 2 3 2 3 3 2  3  3 2  3  18  3 15

2 3  2  3  2  3  6  2  6  12  6  3  18 52 6 2 3 3 2
c)    
2 3 6  2  3  6  2  3  6   2  3 6
2
232 6 6

52 6 2 3 3 2 5  2 6  2 3  3 2  2 6  1
  
2 6 1 2 6  1 2 6  1
10 6  5  24  2 6  4 18  2 3  6 12  3 2 12 6  29  15 2  14 3
 
24  1 23

16 Matemàtiques 1r Batxillerat
SOLUCIONARI UNITAT 1. NOMBRES REALS

Logaritmes
93. Aplicant la definició, calcula el valor dels logaritmes següents.
1 2 2
3
a) log2 d) log 1 27 g) log 2
8 3

b) log 1
1
e) log 8
2 2 h) log 1 3 64
9
3 2

1  1
c) log f) log   i) log0,001 10 000
1000 3
9

1 1
a) log2  x  2x   23  x  3
8 8
x
1  1 1 1
b) log 1  x      9 x  31  32 x  31  2x  1  x 
9
3 9 3 2

1
c) log  x  10x  103  x  3
1000
x 1 3
 1 3
d) log 1 27  x     27 2  3 x  3 2  x  
3 3 2

 8
3x 3
2 2  x 
3x 3
x
e) log 8
 2 2  2 2  22    x 1
2 2

 3
x
 1 x 1 x
f) log  x   32  3 2  32   2  x  4
3
9 9 2

    2 2    23   2 2  2 2  x  9
x 9
2 2 
3 x 3 3
g) log 2
x 2

x 6
 1
h) log 1 3 64  x     2 3  2 x  22  x  2
2 2

4
i) log0,001 10 000  x  0,001x  10 000  104  103 x  104  x  
3

94. Calcula, si és possible, el valor de x en cadascuna de les expressions següents.

a) logx 8  3 c) log3 ( 81)  x e) logx 2 0 g) log3 x  1

b) log– 3 x  9 x  2 f) log1 2  x h) log 1 a  x


2
d) log 1
2 a

3
 1 1
a) logx 8  3  x 3  8  23    x e) logx 2  0 . No existeix x.
2 2

b) log3 x  9 . No està definit. f) log1 2  x . No està definit.

1
c) log3  81  x . No està definit. g) log3 x  1  x  31 
3
2 x
 1   1
  2  2
2
d) log 1 x  2  x    h) log 1 a2  x     a2  a  x  a2  x   2
2
 2 a a

17 Matemàtiques 1r Batxillerat
SOLUCIONARI UNITAT 1. NOMBRES REALS

95. Considerant log 2  0,301 i que log 3  0,477 i log k  0,778, calcula els logaritmes següents.

a) log 50 c) logk 3 e) log2 3 6k

b) log 0,3 d) log 12k f) log 7,2

100
a) log50  log  log100  log2  2  0,301  1,699
2

1
b) log0,3  log  log1  log3  0  0,477  0,477
3
log3 0,477
c) logk 3    0,613
log k 0,778

log12  log k log  22  3   log k 2log2  log3  log k 2  0,301  0,477  0,778
d) log 12k      0,9285
2 2 2 2

log2  log3  log k


log 3 6k 3 0,301  0,477  0,778
e) log2 3
6k     1,723
log2 log2 3  0,301

 log72  log10  log  23  32   1  3log2  2log3  1  3  0,301 2  0,477  1  0,857


72
f) log7,2  log
10

96. Prenent log3 2  0,631 i log3 5  1,465, calcula el valor del logaritme en base 3 de 150.

log3 150  log3  2  3  52   log3 2  log3 3  2log3 5  0,631  1  2  1,465  4,561

97. Calcula el valor de x en cada cas.


–5 –50x
a) 2500  2000 · 1,05 c) 2 · 10  x 2 g) logx 5  1  logx 2
x 6 12
e) 3 · 10
x
b) 20  logx 5  15 d) 0,025  0,5e  ln2 h) ln 3e  2
x 2x
f) ln
2e

2500 5 log1,25
a) 1,05x    log1,05x  log1,25  x log1,05  log1,25  x   4,574
2000 4 log1,05

b) 5  logx 5  x 5  5  x  5 5  1,38

c) x  12 2·106  3,35

0,025
d) e x   0,05  x  ln0,05  2,996
0,5

log  3  105 
e) log  3  105   50 x log2  x   0,3
50log2

f) ln x  ln2  ln e   ln2  ln x  ln e  x  e

2
g) logx 5  logx x  logx 2  logx 5 x  logx 2  5 x  2  x 
5

2  ln3
h) ln3  2x ln e  2  2x  2  ln3  x   0,451
2

18 Matemàtiques 1r Batxillerat
SOLUCIONARI UNITAT 1. NOMBRES REALS

98. Pren logaritmes decimals a les igualtats següents i desenvolupa les expressions.

P  100x 2y 3  log P  log100x 2y 3  2  2log x  3log y

3
2x 2 y 5 x2
a) P  10x 3 yz3 c) R  3 e) y 
3z 3 ax
3
100x 2 (m  2n )n 2
b) Q  d) x  a 4 b3 c 2 f) xy 
xy m  2n

a) P  10x 3yz3  log P  1 3log x  log y  3log z

100 x 2
b) Q   logQ  2  2log x  log( x  y )
xy

2x 2 y 5 log2  2log x  5log y  log3  3log z


c) R  3  log R 
3z 3 3
3
3
d) x  a4 b3 c 2  log x  4log a  3log b  log c
2
3
x2 2 1
e) y   log y  log x  log a  log x   log a  log x
ax 3 3

(m  2n )n 2
f) xy   log x  log y  log(m  2n )  2log n  log(m  2n )
m  2n

99. Expressa el valor de E en cada cas sense que hi surtin logaritmes.

log E  3  4log y  log E  log1000  log y 4  log1000y 4  E  1000y 4

a) log E  2  3log x  log y  5 log z c) log E  log (x  2y)  log (x  2y)


2x  20 3
b) log E  3log 2  4log x  3log y  2log z d) log E  3log (x  10)  log  log
3 2

100y 100y
a) log E  log100  log x 3  log y  log z5  log 3 5
E  3 5
x z x z

8y 3 8y 3
b) log E  log23  log x 4  log y 3  log z2  log 4 2
E  4 2
x z x z

c) log E  log( x  2y )( x  2y )  E   x  2y  x  2y   x 2  4y 2

3
( x  10)3
2  E  9  x  10   9  x  10 2
3
2x  20 3
d) log E  log( x  10)  log
3
 log  log
3 2 2x  20 2  2 x  20  4
3
100. Amb l’ajuda d’una calculadora, obtén aproximacions decimals fins a les mil·lèsimes dels logaritmes següents.

a) log3 20 c) log0,5 60 e) log 2 3 2


5
7
b) log 1 d) log 2
3 f) log 2
3
4
5

19 Matemàtiques 1r Batxillerat
SOLUCIONARI UNITAT 1. NOMBRES REALS

log20 log60 log 3 2


a) log3 20   2,727 c) log0,5 60   5,907 e) log 2 3 2   0,252
log3 log0,5 2
5 log
5
7
log
7 5  0,243 log3 log 3
b) log 1  d) log 3  3,17 f) log 3  1,585
5 1 2
log 2
2
log 2
4 log
4

Síntesi
101. a) Demostra que la suma d’un nombre real positiu més el seu invers és més gran o igual que 2.
b) En quin cas és exactament 2?
c) Sense fer servir la calculadora, demostra que: log2 3  log3 2  2.

2
 1  1 1
a) Sigui a  0,  a    0a 2 0 a  2
 a a a

b) Val 2 quan a  1.
log3 1 1
c) log2 3     log23 i log32 són nombres inversos i, per a l’apartat a, log2 3log3 2  2.
log2 log2 log3 2
log3

102. Calcula el valor de x en cadascuna de les expressions que es donen a continuació.

a) log 3
x  log 3
x2  9

b) log 1 x  log 1 x 4  log 1 x 2  1


9 27 81

9 3
x 3  9   3   x 3  x  3 2  3 2  27  3 3
9 3
a) log 3
x  log 3
x 2  9  log 3

8 log x 
3
log x log x 4 log x 2 log x  4log x  2log x
b) log 1 x  log 1 x 4  log 1 x 2  1       1 
( 2log3)( 3log3)( 4log3) 24 log3 
1 1 1 3
9 27 81 log log log
9 27 81

3
 log x 
  3  log3 x   3  log3 x  3  x  3
3 3
 3 3

 log3 

103. Classifica els nombres reals següents.


 log2 8
3
1 5 
3
1  
10 3 5 2  0,16 
a) 5 log5 10  b)  c) d) 2 2 log2  2 
5 1 2 1  10 
10 8  2  1 
3  2  
 3 
 

10 1 10 1 10 1 10 10 10
a) 5 log5 10           0  Es tracta d’un nombre natu-
10 5log5 10 10 10 10 10 10 10 10
ral.

20 Matemàtiques 1r Batxillerat
SOLUCIONARI UNITAT 1. NOMBRES REALS

1 5 
3
3 5 1  3 5  15  5 5  3  5  5  1 16  8 5 3 5  3  5  5
b)       2  5  2  5   0
8 5 1 8  5  1 5  1 8 4

Per tant, es tracta d’un nombre natural.


 log2 8 3
3 2
1   1  
 3   33· 1  2 
3
2    3  2  27  8  19  Es tracta d’un nombre natural.
3 3
c) 

2 1 1 3
 2  1  27  3 
3  2  
 3 
 

 0,16 
d) 2 2 log2  2   2 2 log2 16  2 2 4  2  Es tracta d’un nombre natural.
 10 

104. Calcula el valor de:


1
2  2
16 5 3  log3
a) log 1 c) 27

2 125  45

b)  log2 log2  2  d) 6
log2  log 4  log8
log2

16 5 16 5 16 5 1
a) log 1  log 1  l og 1  l og 1 2  
2 125  45 2 5 5 3 5 2 8 5 2
2

b)  log2 log2   
2   log2 log2 2 8   log
1
 1
8
  log2 23  3

1
 log3 33
2  2 2  2 2  2  2 2
3  log3 3 3 3
c) 27  

log2  log 4  log8 log2  2log2  3log2 6log2 6


d) 6 6 6  6
log2 log2 log2

105. Per a i b positius i diferents de la unitat, demostra que loga b · logb a  1.

log b log a
loga b  logb a   1
log a log b

QÜESTIONS
106. Indica, raonadament, si les afirmacions següents són certes o falses.
a) La suma de dos nombres irracionals és sempre un nombre irracional.
b) La suma d’un nombre irracional amb un de racional dona com a resultat un nombre irracional.
c) La suma de dos nombres racionals por ser irracional.

a) És fals. Per exemple, 2 i  2 són dos nombres irracionals, i la seva suma és 0, nombre racional.

b) Cert, ja que si la suma d’un irracional I més un de racional R donés com a resultat un altre racional R, tindríem
que I  R  R i això no és possible, la resta de dos racionals és racional.
c) Fals, la suma de dues fraccions dona sempre com a resultat una altra fracció.

21 Matemàtiques 1r Batxillerat
SOLUCIONARI UNITAT 1. NOMBRES REALS

107. Amb l’ajuda d’exemples, estudia si sempre es compleixen les relacions següents.
ab  a  b ab  a  b

No sempre són vertaderes:

 a  b  3  ( 4)  1
a3 b  4    ab  a  b
 a  b  3  4  7

 a  b  3  ( 4)  7
a3 b  4    ab  a  b
 a  b  3  4  1

108. Troba l’error en el raonament següent.


3 4 3 4
1 1  1  1  1  1  1  1
16  8  24  23          log2    log2    3log2    4log2  
23 24 2 2 2 2 2 2

i simplificant: 3  4.

 1
Com que log2    0 quan se simplifica aquesta quantitat negativa en els dos membres de la inequació, se n’ha
2
 1  1
de canviar el sentit, per la qual cosa la conclusió vàlida seria 3log2    4log2    3  4
 
2 2

1
109. Representa a la recta real el conjunt de valors reals x tals que 2 x   1 i determina’l per mitjà d’un in-
3
terval.

1 1 1  1 2
2x   1 x    x   ,
3 6 2  3 3 

PROBLEMES
110 Quan es va fer una enquesta en una localitat sobre el nombre d’ordinadors per habitatge, es va observar
que exactament el nombre d’enquestats que van contestar que hi havia més d’un ordinador a casa seva
era el 40,454 545…% del total.
Quantes persones formaven part de la mostra si se sap que eren menys de 300?

40,4545... 10 000N  4045,454 545... 4005 89


N  0,404 545 45...  0,4045   N  
100 100 N  40,454 545... 9900 220

Per calcular el nombre d’enquestats que van contestar que tenien més d’un ordinador, s’ha de multiplicar per la
89
fracció irreductible .
220
Per tant, el nombre total d’enquestats ha de ser múltiple de 220 i, com que és més petit que 300, és exactament
220.

22 Matemàtiques 1r Batxillerat
SOLUCIONARI UNITAT 1. NOMBRES REALS

111. En Joan ha comprat 2,320 kg de patates a 0,65 €/kg, 4,035 kg de taronges a 2,15 €/kg i 1,475 kg de pomes a
3,25 €/kg. Quan es faci el compte, s’ha d’arrodonir a cèntims d’euro.
a) A quant pujarà aquest compte si primer se sumen els preus i després s’arrodoneix el preu total.
b) I si es fa a la inversa, és a dir, primer s’arrodoneix cadascun dels preus parcials i després se sumen els arrodo-
niments?
c) Quin és el percentatge d’augment en el preu total quan es fa el compte de la segona manera respecte de la pri-
mera?

a) 2,320∙0,65  4,035∙2,15  1,475∙3,25  1,508 8,67525  4,79375  14,977  14,98 €

b) 2,320∙0,65  4,035∙2,15  1,475∙3,25  1,51  8,68  4,79  14,98 €

c) No hi ha variació de preu.

112. La màxima distància de la Terra a la Lluna és de 4,07  108 m i el radi de la Lluna mesura 1737,5 km. Calcula
la distància de la Terra a la Lluna prenent com a unitat el diàmetre de la Lluna.

4,07  108 m 4,07  108 m


  117,12
2  1737,5 km 3 475 000 m

La distància de la Terra a la Lluna és d’aproximadament 117,12 diàmetres lunars.

 I 
113. El nivell d’intensitat del so pot ser expressat en decibels per mitjà de la fórmula   10log   on  és el
 I0 
2
nombre de decibels del so, I és la intensitat del so estudiat mesurat en watts/cm i I0 indica la intensitat del
–12 2
so mínim que l’ésser humà pot sentir (10 watts/m ).
–7 2
a) Calcula els decibels (dB) d’una conversa normal que presenta una intensitat de 10 watts/m .
b) Quants decibels corresponen al so mínim que pot sentir l’ésser humà?
2
c) Quina és la intensitat en watts/cm del llindar de dolor en l’oïda humana sabent que es correspon amb 120 dB?
d) Un so és de 30 dB. Quantes vegades és més intens un so de 20 dB? I si es compara un so de 40 dB amb un
de 30 dB?

 107 
a)   10log  12   10log105  10  5  50 dB
 10 

 1012 
b)   10log  12   10log1  10  0  0 dB
 10 

 I   I  I 2 −4 2
c) 120  10log  12   log  12   12  12  1012  I  100  1 watt/m 10 watt/cm
 10   10  10
  
 I   I   I 12
d)   10log  12   log  12    1010  12  I  1010  1012  I  1010
 10   10  10 10

 30
12
   30  I (30)  10 10  109 I (30) 109
    10 , el so de 30 dB és 10 vegades més intens que el de
20
  20  I (20)  10 10 12  1010 I (20) 1010

20 dB.
Així mateix, el so de 40 dB és 10 més intens que el de 30 dB.

23 Matemàtiques 1r Batxillerat
SOLUCIONARI UNITAT 1. NOMBRES REALS

114. Una empresa fa llaunes de forma cilíndrica amb unes dimensions de 5 cm de radi de la base i 10 cm d’altu-
ra. Després d’un estudi de mercat, decideix canviar la forma de les llaunes: seran ortoedres de base qua-
drada i amb una altura del doble que el costat de la base.
Quines seran les dimensions de la nova forma si la capacitat ha de ser la mateixa? Dona la solució amb
l’aproximació que trobis més adient.

El volum de les llaunes és V    52  10  785,4 cm3 , per tant tenim:

2x 3  785,4  x  7,32 cm

És a dir, les noves llaunes han de mesurar 7,32 cm de costat de la base i 14,64 cm
d’altura.

115. En Xavier pretén posar gespa artificial a un jardí quadrat, sabent que el seu costat té una longitud, en me-
tres, comprès entre 15 i 16.
El cost de cada metre quadrat d’aquesta gespa puja a 30,10 €, i el pressupost que té és de 7000 €.
Calcula els costos màxim i mínim, i decideix si hi haurà pressupost per a l’obra.

15  costat  16  225  àrea  256  6772,5  cost  7705,6

Per tant, el pressupost podria ser insuficient.

2
116. L’àrea d’un quadrat és de10,5 cm . Calcula les àrees dels seus cercles inscrit i circumscrit, arrodonint els
resultats amb dues xifres decimals.

El costat del quadrat mesura x  10,5  3,24 cm.

La diagonal del quadrat mesura 2  10,5  4,58 cm.


2
 3,24 
Àrea del cercle inscrit: S  r 2      8,24 cm
2

 2 
2
 4,58 
Àrea del cercle circumscrit: S  r 2      16,47 cm
2

 2 

117. Experimentalment, s’han obtingut les fórmules següents que expressen el percentatge P d’alçada dels
nens de 6 a 15 anys d’edat, en relació amb l’alçada mitjana d’un adult:
Per a nenes P  31,1 ln(Edat)  16,3
Per a nens P  18,6 ln(Edat)  37
a) Calcula el percentatge d’alçada dels nens de 6, 10 i 15 anys d’edat.
b) Quina edat aproximada es pot esperar que, segons aquest model, tingui una nena amb un 75 % d’alçada mit-
jana de l’edat adulta? I un nen?
c) Si aquest model fos vàlid per als homes de 6 a 18 anys, quina edat tindrien en arribar a l’alçada màxima del
100 %? Analitza el resultat.

a) Per a nenes P(6)72 % P(10)88 % P(15)100 %


Per a nens P(6)70 % P(10)80 % P(15)87 %

24 Matemàtiques 1r Batxillerat
SOLUCIONARI UNITAT 1. NOMBRES REALS

P 16,3 75 16,3
b) Per a nenes. Edat  e 31,1
 Edat  e 31,1
 6,6 anys
P 37 75 16,3
Per a nens. Edat  e 18,6  Edat  e 31,1
 7,7 anys
100 37
c) Edad  e 18,6
 29,6 anys.

Òbviament, el model no serveix per als nois més grans de 15 anys.

1 5
118. Demostra que el nombre auri   compleix les propietats següents.
2
1  1
a) 2    1 b)   1  c) 3 
  1

2
 1 5  1 5  2 5 6  2 5 3  5 1 5
a)        1  1 
 2 
2

  4 4 2 2

2 1   1 1
b) 2  1         1   1 
   

1    1
c)      1      
3 2
1  1

119. L’escala cromàtica del piano està formada per les dotze notes (dotze semitons) que surten a la figura.
El nombre de vibracions per segon de cada nota és igual al producte del nombre de vibracions de la nota
12
anterior pel nombre irracional 2.
a) Troba una expressió que determini el nombre de vibracions per segon segons el lloc que ocupi en l’escala (per
exemple, el Do ocupa el lloc 0, el Si bemoll, el lloc 10 i el Si, l’11) i suposant que el nombre de vibracions per
segon que corresponen a la nota Do és n.
b) Escriu les vibracions per segon corresponents a cada nota, sabent que les del La són 440.

12 440
a) N(k )  n 2 262  N(k )  440 2k 9
k 12 12
b) N(9)  n 29  440  n 
12
29

Do Mi Fa La Si
Do Re Mi Fa Sol La Si
sostingut bemoll sostingut bemoll bemoll
262 277 29 311 33 349 370 392 415 440 466 494
4 0

120. Una empresa cobra pel lloguer d’una furgoneta 80 € diaris. Una altra empresa cobra pel mateix lloguer 60 €
al dia, però a aquesta quantitat cal afegir 200 € independentment del temps que es contracti.
A partir de quants dies és més econòmica la segona empresa? Escriu la solució en forma de desigualtat i
d’interval.
Si es lloga la furgoneta n dies, la primera empresa cobra 80n i la segona, 60n 200. La segona empresa serà més
econòmica quan 60n  200  80n  n  10 dies  10,    .

25 Matemàtiques 1r Batxillerat
SOLUCIONARI UNITAT 1. NOMBRES REALS

121. Quan s’ha mesurat l’alçada d’una persona de 180 cm s’ha obtingut 178 cm. Quan s’ha mesurat l’alçada
d’un edifici de 39 m s’ha obtingut 40 m. Calcula els errors absolut i relatiu de cada mesura i indica raonada-
ment quina de les dues és més precisa.

2
Errors en la mesura de la persona: Ea  180  178  2 cm i Er   0,011
180
1
Errors en la mesura de l’edifici: Ea  39  40  1 m i Er   0,026
39
Com que l’error relatiu és més petit en la mesura de la persona, aquesta mesura és més precisa.

122. La diagonal d’un cub mesura exactament 1,252 cm. Troba la superfície del cub aproximant la seva diagonal
per 1,25 cm. Calcula l’error relatiu comès.

D
2l  D  d  l  2l 2  l 2  3l  l 
2 2
d l2  l2 
3

D2
Superfície del cub: 6l 2  6  2D2
3

La superfície real del cub és 2 · 1,252  3,135 008, l’aproximem per


2

2 · 1,25  3,125, per tant, l’error relatiu és:


2

3,135 008  3,125


Er  0,0032
3,135 008

123. Calcula la mesura de la diagonal d’un paral·lelepípede de costats 10 , 8 i 5 cm, respectivament. Qui-
na mena de nombre és el resultat?
Aproxima el resultat arrodonint a dos decimals i calcula els errors absolut i relatiu comesos.

d  5  10  15 cm  D  15  8  23 cm .
La diagonal és un nombre irracional, que aproximem per 4,80 cm, per
tant:

Ea  23  4,80  4,79558  4,80  0,00417

Ea 0,00417
Er    0,000 87
23 4,79

26 Matemàtiques 1r Batxillerat
SOLUCIONARI UNITAT 1. NOMBRES REALS

124. Un jardí quadrat té 50 m de costat. Dues persones passegen a la mateixa velocitat, una pel perímetre del
quadrat i l’altre recorrent la diagonal. Si surten simultàniament de la mateixa cantonada del parc, tornaran
a trobar-se?

Si es troben, ho faran a la cantonada oposada; en aquest cas, com que van a la mateixa velocitat, han d’haver re-
corregut el mateix espai.
Ara bé, l’espai recorregut per la persona que avança pel perímetre és 100 m i el recorregut per la persona que va
per la diagonal és 50 2 70,71 m, amb la qual cosa no es trobaran.

Ens podem preguntar si acabaran trobant-se si continuen passejant ininterrompudament, l’un seguint el perímetre i
l’altre recorrent la diagonal una i altra vegada.
Per resoldre aquest problema observem que, si es troben, serà en una de les cantonades de la diagonal que re-
corre la segona persona i, com que caminen a la mateixa velocitat, hauran recorregut la mateixa distància..
Ara bé, la persona que va pel perímetre haurà recorregut 100a metres per a algun enter positiu a, mentre que la
persona que avança per la diagonal haurà recorregut 50 2b metres per a algun enter positiu b.

2a
Per tant, obtindríem 100a  50 2b  2  , és a dir, 2 seria racional, cosa que sabem que no és cert.
b
Llavors, deduïm que aquestes dues persones no es trobaran mai, encara que passegin indefinidament.

125. Un tipus determinat de protozous té un diàmetre de2 · 10 m. Calcula quants protozous caldria posar, un a
5

continuació de l’altre, per aconseguir una longitud d’1 cm.

0,01 : (2 · 10 )  500 protozous


5

126. A la taula hi ha les mesures d’una nena i d’una torre.


Alçada
Obtinguda amb instrument
Real
de mesura
92 cm 90 cm
38 m 37 m

Indica quina de les dues mesures ha estat més precisa i justifica la teva resposta.

2 1 1
En el primer cas, l’error relatiu és  . En el segon, l’error relatiu és . Per tant, la mesura de la nena és
92 46 38
més precisa, ja que l’error relatiu és més petit.

127. S’anomena unitat astronòmica (UA) la distància mitjana que separa la Terra del Sol i que equival a
8
1,49598·10 km.
8
a) Sabent que l’1 de gener la distància entre la Terra i el Sol és de 1,471·10 km, expressa-la en unitats astronò-
miques.
b) Sabent que la distància mitjana entre Júpiter i el Sol és de 5,2 UA, expressa-la en quilòmetres.

1,471 108
a)  0,9833 UA b) 5,2·1,49598  108  7,779  108 km
1,49598  108

PER APROFUNDIR
128. Demostra que si a, b i c són nombres positius i diferents, aleshores es compleix la desigualtat següent.

 a  b  c  
1 1 1
  9
a b c

27 Matemàtiques 1r Batxillerat
SOLUCIONARI UNITAT 1. NOMBRES REALS

Utilitzant el resultat de l’exercici 101.a):

 a  b  c  
1 1 1 a a a b b b c c c a b a c b c
             1 1 1       3  2  2  2  9
a b c a b c a b c a b c b a c a c a

129. Calcula de forma exacta el nombre irracional que representa la relació entre la diagonal
d’un pentàgon regular i el seu costat. Comprova que es tracta del nombre auri.
Per fer-ho, segueix els passos següents:
a) Demostra que els triangles DFC i DBC són semblants calculant-ne els angles.
b) Demostra que el triangle BFC és isòsceles.
c) Aplicant el teorema de Tales, troba la relació entre els costats que corresponen a la diagonal i el costat del pen-
tàgon.

 2  5  4   90º  108º .
a) L’angle interior d’un pentàgon regular és
5
El triangle DBC és isòsceles; els angles mesuren 108º, 36º i 36º.
El triangle DFC també és isòsceles; els angles mesuren 36º, 36º i 108º. DFC i DBC són semblants.
b) L’angle BCF mesura 108º  36º  72º, l’angle BFC també mesura 180º  108º  72º. BFC és isòsceles.
c) Sigui un pentàgon regular de costat 1, aplicant el teorema de Tales als triangles semblants:

DB DC x 1 1 1 4 1 5 DB 1  5
    x2  x  1  x2  x  1  0  x   x  
DC DF 1 x 1 2 2 DC 2
(A l’equació de segon grau, l’altra solució és negativa i no té sentit.)

1
130. Racionalitza el denominador de l’expressió:
2 3 2

1 4  23 2  3 4 4  23 2  3 4 4  23 2  3 4 4  23 2  3 4
   
23 2 2  2   4  2
3 3
234  8  4 3 2  23 4  4 3 2  23 4  3 8 82 6

S’ha utilitzat que a  b  (a  b) · (a  ab  b ).


3 3 2 2

28 Matemàtiques 1r Batxillerat
SOLUCIONARI UNITAT 1. NOMBRES REALS

ENTORN MATEMÀTIC

Una piscina amb estètica


En Màrius i la Priscil·la són una parella molt moderna, amb un bon nivell econòmic, una bona vida social i una
família envejable; però últimament s’avorreixen una mica. Decideixen gastar-se l’herència de la tia Edurne en la
construcció d’una piscina a la terrassa del seu àtic del centre de la ciutat. Però, és clar!, no pot ser una piscina
qualsevol.
La Priscil·la recorda haver llegit que les proporcions perquè un rectangle sigui
el més estètic possible són aquelles tals que si al rectangle total li treus un
quadrat de costat la dimensió més petita, el rectangle petit que queda és pro-
porcional al rectangle inicial. Aquesta propietat, si no recorda malament, es
pot trobar en objectes tan quotidians com el DNI o les innombrables targetes
del banc que tenen.
En Màrius no s’ho acaba de creure i mesura el seu carnet d’identitat. Resulta
que fa 8,6 cm de llargada i 5,4 cm d’amplada. Fes els comptes que creguis
convenients per comprovar si efectivament la Priscil·la té o no raó.
La piscina que volen construir ha de tenir aquestes proporcions estètiques i la seva dimensió més gran ha de
ser de 22 5 m. A més, volen posar una franja d’1,5 m de gespa artificial al voltant de tot el perímetre.
a) Calcula la dimensió petita de la piscina.
b) Calcula l’àrea de gespa que volen posar.

c) Calcula el temps que trigaran a omplir la piscina si l’alçària en tots els seus punts és de 5 metres i l’aixeta dona 30
L cada minut.

Si anomenem a i b el costat més gran i més petit, respectivament, d’un rectangle que compleixi les condicions de l’enun-
ciat tindrem:
2
a b a a
  a2  ab  b2  a2  ab  b2  0      1  0
b ab  
b b

1 5
Per tant, el quocient entre el costat gros i el petit ha de ser  1,618 , per això, els rectangles que compleixen
2
aquesta condició s’anomenen rectangles auris.
8,6
El quocient de les dimensions del DNI és  1,593 i, tot i que s’hi acosta, no és exactament un rectangle auri.
5, 4

2  2 5 1 5 4 1  5 
a)  b  4 m.
b 2 1 5

b) La superfície de la gespa serà: S  7  5  2 5   4  2  2 5   35  14 5  8  8 5  27  6 5 m .


2

c) El volum de la piscina és V  5  4   2  2 5   8 5  40 m3   8 5  40 1000 L .

8 5  40  1000
El temps que tardarà a omplir-se serà 1930 minuts 32 hores .
30

29 Matemàtiques 1r Batxillerat
SOLUCIONARI UNITAT 1. NOMBRES REALS

Pintures prehistòriques
La conservació en bones condicions de les pintures prehistòriques de les coves exigeix, entre altres aspectes,
el control de la temperatura a l’interior de la gruta.
Perquè aquesta temperatura es mantingui dins d’uns límits acceptables, un equip d’enginyers ha ideat un sis-
tema de comportes al passadís que dona entrada a la cova. La longitud total d’aquest passadís es dividirà en
compartiments de 30 m de llarg separats per portes de tancament hermètic de manera que la temperatura de ca-
da compartiment serà un 5 % més baixa que la temperatura del compartiment immediatament anterior.
a) Calcula la temperatura del quart compartiment sabent que la del primer és de 25 ºC.
b) Calcula la temperatura al vuitè compartiment si al cinquè hi ha 21 °C. En aquest cas, quina és la temperatura del
primer compartiment?
c) Dissenya un full de càlcul on s’obtinguin les temperatures dels compartiments en relació amb la temperatura inicial.
d) Quants compartiments s’han hagut de construir per tal que la temperatura de l’últim baixi per primer cop a la quarta
part de la temperatura del compartiment inicial? En aquest cas, quina és la longitud total del passadís d’entrada?

a) La temperatura en el compartiment k serà Tk25∙0,95


k–1
.
Per tant: T425∙0,95  20,43 ºC
3

b) T8  T5∙0,95  21∙0,95  18 ºC
8–5 3

T5 21
T5  T1∙0,95
5−1
 T1  4
  25,78 ºC
0,95 0,954
c)

A B
1 COMPARTIMENT TEMPERATURA
2 1 T1
3 A21 B2*0,95
4 Copiar la cel·la Copiar cel·la
immediatament immediatament
superior superior
5 Copiar la cel·la Copiar cel·la
immediatament immediatament
superior superior
… … …

T log0,25
d)  T  0,95k 1  0,95k 1  0,25  k  1  28 compartiments.
4 log0,95

Per tant, el passadís té una longitud de 28∙30  840 m.

30 Matemàtiques 1r Batxillerat
SOLUCIONARI UNITAT 1. NOMBRES REALS

AUTOAVALUACIÓ

Comprova el que has après


1. Calcula el valor i simplifica.
2,4555...  2,555... 1
a) b)
1,222... 1
2
1
3
1
4
5

221 23

2,4555...  2,555... 9  41  4,1 1 1 1 68
a)  90 b)   
1,222... 11 10 1 1 21 157
2 2 2
9 1 5 68
3 3
1 21
4
5

3
2. Representa els nombres  , 68 i 12 en la recta real:
11

12  32   3 
3 2
 68  82  22
11

1 1
3. Desenvolupa l’expressió: 2x  3 x 
2 2

 1 3 1  1
1 1 2 x  2  3 x  2 si x   
2
5 x  1 si x  
2
2x   3 x   
2 2 2 x  1  3 x  3 si x  
1
 x  2 si x  
1
 2 2 2  2

4. Expressa per mitjà de la unió d’intervals el conjunt:

x  / x  2  1  x  / x  5  1

(3,  1)  (4,6)

31 Matemàtiques 1r Batxillerat
SOLUCIONARI UNITAT 1. NOMBRES REALS

24
5. La massa de la Terra és aproximadament 6∙10 kg, la de un àtom d’oxigen és, aproximadament, de
−23
2,65∙10 g, i la d’en Jordi, 75 kg. Calcula les relacions entre les masses de la Terra i d’en Jordi, i entre les
masses d’en Jordi i de l’àtom d’oxigen. Quina és més gran?

mT 6  1024 mJ 75
  8  1022   2,83  1024
mJ 75 mO 2,65  1023

Per tant, és superior la relació entre la massa d’en Jordi i la massa de l’àtom d’oxigen.

6. Troba els errors absolut i relatiu que es fan quan es pren 2,5 com a aproximació de 2,5 .

23 5 1 2,5  2,5 1 23 1
Error absolut: Ea  2,5  2,5    Error relatiu: Er   : 
9 2 18 2,5 18 9 46

7. Donats A  2,3  1012 i B  1,15  1011 . Calcula:

A
b) a) A  B b) A  B c) AB d)
B

a) A  B  2,3  1012  1,15  1011  0,23  1011  1,15  1011  1,38  1011

b) A  B  2,3  1012  1,15  1011  0,23  1011  1,15  1011  0,92  1011  9,2  1012

c) AB  2,3  1012  1,15  1011  2,645  1023

A 2,3  1012
d)   2  101
B 1,15  1011

8. Simplifica les expressions següents.

1024  512  87 10 3 1 1
a) b) c) 1024
2518 3  48 2 8

 
24 7
1024  512  87  2  5   5  2
12 3
224  524  512  221 1 1
a)    23  50  3 
2518  
18
52 536 2 8

10 3 10 3 10 3
b)   2
3  48 3 4 3 5 3

4 2
1 1  1  1  210 210
c) 1024     3  2 
10 8  8 10  8 1  1
2 8 2 2  2 2
4 6
2

9. Racionalitza els denominadors i simplifica tant com puguis les expressions que en resultin.

2 3 1 1 54
a) b) c)
54 4
54 2 3 1

2 3 1 2 3  1 54 2 162  54 2  9 2  3 6 18 2  3 6 6 2  6
a)     
54 54 54 54 54 54 18

4
23  3
4
1 1 23  3 4
24 4
24
b)     
4
54 4
23 3 4
23  2 3
3 4 3 4
2 34 4 23 6

32 Matemàtiques 1r Batxillerat
SOLUCIONARI UNITAT 1. NOMBRES REALS

54 3 6  2 3  1 6 18  3 6 18 2  3 6
c)   
2 3 1 2 3  1 2 3  1 12  1 11

10. Calcula el valor de x a:


1 1 1 1 5
a) log x b) logx  c) log 1 
2
2 3 3 8
4  2x 3

 x   2    2    2   x  1
1 x 1 x 1
a) log 2
2 2
1 3
1 1 1  1 1
b) logx   x3   x    
3 3 3 3 27
5
5
1 5  1 3 1 3·
c) log 1      2 x  2 3  22 x  5  2  x  x  3
8
4  2x 3 8 2 2

50
11. Sabent que log 2  a, calcula, en funció de a el valor de log .
0,08

50 100 8
log  log50  log0,08  log  log  log100  log2  log8  log100  2  log2  log23  2 
0,08 2 100

 4  log2  3log2  4  4log2  4  4a

RELACIONA I CONTESTA

Tria l’única resposta correcta en cada cas

1  loga b
1. Si a i b són nombres reals estrictament positius i diferents de la unitat, l’expressió val:
1  loga b

a
A. logab   B. loga  b  ab  C. logab  ab  D. 1
b

log b log a  log b a


1 log  
1  loga b log a log a log a  log b  b   log  a  , resposta A.
    ab  
1  loga b 1  log b log a  log b log a  log b log ab b
log a log a
1
 ab 
2. Siguin a i b dos nombres reals no nuls tals que a  b 3 . El valor de   és:
 3b  a 

A. 3 B. 3 C. a  b 3 D. 3  3

 ab 
1
 b 3 b



1


3 3

3  3  3 1 3 3 33 3
 3 , resposta B.
 
 3b  a   3b  b 3
  3 1  3  1 3  1 3 1

33 Matemàtiques 1r Batxillerat
SOLUCIONARI UNITAT 1. NOMBRES REALS

3. Si log2 n 0,125  x , aleshores el valor de xn és:

A. 1 B. 2 C. 3 D. x n

1
log2 n 0,125  x  2x  n 0,125  2xn  0,125   23  xn  3 , resposta 3, la C.
8

Assenyala, en cada cas, les respostes correctes

4. Les igualtats següents són certes per a qualssevol valors reals estrictament positius:
 c  
  ab  B.  ab    ac  C. abc   ab 
c c b c c
A. a b D. a b  ab

ab c  abc  ac b , per la qual cosa B i C són certes.

En canvi A i D són falses, per exemple, 2


23   28  256 no coincideix amb  22   43  64 ni amb 22  4 .
3

3 3
5. El nombre  2   2 pertany al conjunt:
2 2

A. B. C. D.

2
 3 3  3  3  3  3 9 1
  2  2    2  2  2   2   2  32  2  3  2  3 1 4
2 2  2  2  2  2 4 2
    

El nombre és 4  2 , natural, i, per tant, totes les respostes són vàlides.

6. Indica quins dels nombres següents són racionals.


A. 0,12122122212222…
B. 0,123412341234…
C. 0,112233445566…
2
D. 2
2

A i C són irracionals, ja que no són periòdics. En canvi B és racional, ja que és periòdic. Finalment,
2 22
2   0 és racional. Per tant, les respostes correctes són la B i la D.
2 2

Tria la relació correcta entre les dues afirmacions donades

7. Siguin P i Q nombres reals. Es consideren les afirmacions:


1. Al menys un dels dos nombres reals P i Q és irracional.
2. PQ és irracional.

A.  2 però 2  1

B. 2  1 però 1  2

C. 1 i 2 són excloents entre si.


D. Cap de les anteriors.

34 Matemàtiques 1r Batxillerat
SOLUCIONARI UNITAT 1. NOMBRES REALS

1 no implica 2, per exemple, si P és irracional i Q  P, tindríem P Q  0 racional.


En canvi 2 sí que implica 1, si P Q és irracional almenys un dels dos nombres reals P i Q és irracional, ja que si
tots dos fossin racionals també ho seria P Q.
Per tant, la relació correcta és la de l’opció B.

Assenyala la dada innecessària per contestar

8. Amb les dades següents:


1. B   0, 6 2. A  B   2, 6 3. A  B  0, 5 

Quin és exactament el subconjunt de nombres reals A?


A. Pot eliminar-se la dada 1.
B. Pot eliminar-se la dada 3.
C. Es pot eliminar qualsevol de les tres dades.
D. No es pot eliminar cap de les dades.

No es pot eliminar cap de les dades, resposta D, les tres són necessàries per deduir que A   2, 5 .

35 Matemàtiques 1r Batxillerat
SOLUCIONARI UNITAT 2. ÀLGEBRA

2. Àlgebra
EXERCICIS PROPOSATS
1 I 2. Exercicis resolts.

3. Fes les operacions següents amb polinomis.


a) (3x  2x  5)(2x  x  3) b) (2x  3)(2x  2)  x (2x  x  1) c) 4(x  x  3)  2(x  3x)(2x  5)
2 2 2 2 3 2

a) (3x  2x  5)(2x  x  3)  6x  3x  9x  4x  2x  6x 10x  5x  15  6x  7x 17x  11x  15


2 2 4 3 2 3 2 2 4 3 2

b) (2x  3)(2x  2)  x (2x  x  1)  4x  4x  6x  6  2x  x  x  6x  7x  5x  6


2 2 3 2 3 2 3 2

c) 4(x  x  3)  2(x  3x)(2x  5)  4x  4x  12  4x  10x  12x  30x  8x  2x  34x  12


3 2 3 3 2 2 3 2

4. Calcula els valors de a i b perquè es verifiquin les igualtats següents.


a) (3x  5x  a)(2x  bx)  6x  19x  19x  6x
2 2 4 3 2

b) 2(x  3)  3(x  a)  2x  bx  3
2 2

3b  10  19
a) (3x  5x  a)(2x  bx) 6x  (3b  10)x  (2a  5b)x  abx  2a  5b  19  a  2, b  3
2 2 4 3 2

ab  6

b) 2(x  3)  3(x  a)  2x  9x  (18  3a) 


2 2
189 b3a  3  a  5, b  9
5. Fes les divisions següents.
a) (3x  2x  x  5) : (3x  2)
3 2 2

b) (3x  2x  x  4) : (x  x 2)
4 2 2

c) (x  3x  x  6) : (2x 2x  3)
3 2 3

3x  2x  x  5 3x  2 c) x  3x  x  6 2x  2x 3
3 2 2 3 2 3
a)
2 3 1
3x  2x x x x
3 3
3 2 2
9
2x  x  5  3x 
2 2
2
4
2x 
2
3
19
x
3
 2x  x  4 x x 2
4 2 2
b) 3x
3x  3x  6x 3x  3x  5
4 3 2 2

3x  8x  x
3 2

3x  3x  6x
3 2

5x  5x  4
2

5x  5x  10
2

10x  14

36 Matemàtiques 1r Batxillerat
SOLUCIONARI UNITAT 2. ÀLGEBRA

6. Fes les divisions següents aplicant la regla de Ruffini.


a) (3x  2x  x  5) : (x  2) d) (4x  3x  5) : ( x  5)
3 2 4 2

b) (3x  2x  x  4) : (x  2) e) (2x  12x  6x  117) : (x  3)


4 2 5 3 2

c) (x  3x  x  6) : (2x  3)
3 2

a)
3 2 1 5
2 6 8 18
3 4 9 23

Quocient: 3x  4x  9 Residu: 23


2

b) 3 0 2 1 4
2 6 12 20 42
3 6 10 21 46

Quocient: 3x  6x  10x  21
3 2
Residu: 46

c) Com que el divisor no és de la forma x  a, abans d’aplicar Ruffini es divideixen el dividend i el divisor per 2.

1 3 1
  3
2 2 2
3 3 27 69
  
2 4 8 16
1 9 23 21
 
2 4 8 16
1 2 9 23  21  21
Quocient: x  x Residu: 2·    
2 4 8  16  8

d) Com que el divisor no és de la forma x  a, abans d’aplicar Ruffini es canvia el signe al dividend i al divisor.

4 0 3 0 5
5 20 100 485 2425
4 20 97 485 2430

Quocient: 4x  20x  97x  485


3 2
Residu: 2430

e) 2 0 12 6 0 117
3 6 18 18 36 108
2 6 6 12 36 9

Quocient: 2x  6x  6x  12x  36 Residu: 9


4 3 2

37 Matemàtiques 1r Batxillerat
SOLUCIONARI UNITAT 2. ÀLGEBRA

7. Fes servir Ruffini per calcular els valors de a i b que fan que les divisions següents siguin exactes.
a) (2x  5x  2x  a) : (x  3) c) (6x  x  ax  16x  4) : (3x  4)
3 2 4 3 2

b) (3x  5x  ax  2) : (2x  3) d) (4x  3x  13x  ax  b) : (x  2)


4 2 4 3 2 2

a)
2 5 2 a
a30  a3
3 6 3 3
2 1 1 a3
b)
3 5 a
0   1
2 2 2
3 9 27 21 63  24a 95  24a 95
   0a
2 4 8 16 32 16 24
3 9 7 21  8a 95  24a
 
2 4 8 16 32
c)
1 a 16 4
2   
3 3 3 3
4 8 28 112  12a 1024  48a 48a  916 229
0a
3 3 9 27 81 27 12
7 28  3a 256  12a 916  48a
2
3 9 27 81

d) La divisió no es pot fer utilitzant Ruffini.


4x  3x  13x  ax  b x 2
4 3 2 2

4x  8x 4x 3x  5
4 2 2

3x  5x  ax
3 2

3x  6x
3

5x  (a 6)x  b Perquè el residu sigui zero a 6 i b 10


2

 10
2
5x
(a6)x  b  10

8 i 9. Exercicis resolts.

10. Troba, sense fer la divisió, el valor de m perquè el polinomi 2x  9x  2x  6x  3m tingui 12 per residu
4 3 2

quan es divideixi per x  2.


32
Pel teorema del residu tenim: 12  2(2)  9(2)  2(2)  6(2)  3m  3m  20  m 
4 3 2
3
11. Calcula el valor de k perquè el polinomi:
a) P(x)  x  x  2x  k sigui divisible per x  2.
3 2

b) P(x)  x  2x  kx  4 sigui divisible per x  2.


3 2

c) P(x)  (k  3)x  5x  k sigui divisible per x 4.


3

d) P(x)  x  (k  3)x  (k  5)x  3 sigui divisible per x  3.


3 2

a) Pel teorema del factor tenim: 2  2  2·2  k  0  k  8  0  k  8


3 2

b) Pel teorema del factor tenim: (2)  2·(2)  k·(2)  4  0  2k  12  0  k  6


3 2

212
c) Pel teorema del factor tenim: (k  3)∙(4)  5∙(4)  k  0  63k  212  0  k 
3
63
d) Pel teorema del factor tenim: 3  (k  3)∙3  (k  5)·3∙ 3  0  6k  36  0  k  6
3 2

38 Matemàtiques 1r Batxillerat
SOLUCIONARI UNITAT 2. ÀLGEBRA

12. En cada cas, factoritza el polinomi donat i troba les seves arrels enteres.
a) 2x  5x  x  6 c) x  16 e) 6x  11x  6x  1 g) x  3x  3x  2
3 2 4 3 2 4 3

b) 9x  12x  4 d) x  x  x  x f) x  4x  3x  11x  6 h) x  9x
2 5 4 2 4 3 2 6 4

a) 2 5 1 6
1 2 7 6
2 7 6 0

 3
7  49  48 7  1  x  
Es calculen les arrels del quocient 2x  7x  6: x 
2
  2
4 4 
 x  2

3
Per tant, 2x  5x  x  6  2(x 1)(x  2)(x  )  (x 1)(x  2)(2x  3). Les arrels enteres són 2 i 1.
3 2
2
2
12  144  144 12  0 2  2
  , així 9x  12x  4  9  x    (3x  2) . No té arrels enteres.
2 2
b) x  
18 18 3  3

c) Utilitzant les igualtats notables: x  16  (x  4)(x  4)  (x  2)(x  2)(x  4). Les arrels enteres són 2 i 2.
4 2 2 2

d) S’extreu el factor comú: x  x  x  x  x(x  x  x  1). Ara per Ruffini:


5 4 2 4 3

1 1 0 1 1 1 0 0 1
1 1 0 0 1 1 1 1 1
1 0 0 1 0 1 1 1 0

x  x  x  x  x (x  1) (x  x  1). Les arrels enteres són 0 i 1.


5 4 2 2 2

e) 6 11 6 1
1 6 5 1
6 5 1 0
 1
x
5  25  24 5  1  2
Es calculen les arrels del quocient 6x  5x  1: x   
2
12 12 x   1
 3

 1  1
Per tant, 6x  11x  6x  1  6 ( x  1)  x    x   . L’arrel entera és 1.
3 2

 2  3 
f) És un exemple de polinomi sense arrels enteres i que no es pot factoritzar de forma senzilla.
g) És un exemple de polinomi sense arrels enteres i que no es pot factoritzar de forma senzilla.
h) S’extreu el factor comú i s’utilitzen les identitats notables: x  9x  x (x  9)  x (x  3)(x  3) . Les arrels en-
6 4 4 2 4

teres són 3, 0 i 3.

13. Exercici interactiu.

14 a 17. Exercicis resolts.


18. Calcula:

b) 
252 
a)   c)      d)           
12 25 25 4 4 4 4

9  250  3  4 0        
1 2 3

39 Matemàtiques 1r Batxillerat
SOLUCIONARI UNITAT 2. ÀLGEBRA

a)   
12 12!
 220
9 9! 12  9  !

b) 
252  252!
  31 626
 250  250!  252  250  !

c)         
25 25 26 26!
 14950
3  4  4 4!  26  4  !

d)             1 4  6  4  15
4 4 4 4
       
0 1 2 3

19. Desenvolupa les potències següents i troba’n el terme sisè.

 
5
 4 
  3a2  2ab 
4 9 8
a) 32 3 b)  2x   c) 2 2 3 d)
 3x 

3  2 3   4  4
 
 4
   4
   4
 
4 2 3 4
a)     34     33  2 3     32  2 3   3 2 3   2 3 
 
0  
1 2 3  4

 81 4  27  2 3  6  9  12  4  3  24 3  144  873  504 3 . El desenvolupament no té terme sisè.

5 2 3 4 5
 4  5 5 4 4 5 3 4  5 2 4  5   4  5  4 
   0   2x    1   2x      2x      3   2x  
5
b)  2x     4  2x       
 3 x      3 x  
2  3 x     3 x     3x   5   3x 

320 3 1280x 2560 2560 1024


 32x 5  x     . El terme sisè del desenvolupament és
3 9 27x 81x 3 243x 5
5
5 4  1024
T6       .
   3x 
5 243 x 5

  9 9
  9
    9
  9
 
9 2 3 4
c) 2 2 3    ( 2)9    ( 2)8 2 3    ( 2)7 2 3    ( 2)6 2 3    ( 2)5 2 3 
 
0  
1  2  3  4

9
  9
   9
   9
   9
 
5 6 7 8 9
   ( 2)4 2 3    ( 2)3 2 3    ( 2)2 2 3    ( 2) 2 3   2 3  16 2 
5  6  7  8  9

288 3  3456 2  16128 3  72 576 2  145 152 3  290 304 2  248 832 3  186 624 2  41472 3 

9
 2  2 3 
4 5
 552 976 2  451872 3 . El terme sisè del desenvolupament és T6      145 152 3 .
5

8 8 8 8 8


 3a2  2ab 
8
d)    (3a2 )8    (3a2 )7(2ab)    (3a2 )6(2ab)2    (3a2 )5(2ab)3    (3a2 )4(2ab)4 
0 1   2  3  4

8 8 8 8


   (3a2 )3 (2ab)5    (3a2 )2(2ab)6    (3a2 )(2ab)7    (2ab)8  6561a16  34 992a15b  81 648a14b2 
5  6 7  8

108 864a13b3  90 720a12b4  48 384a11b5  16 128a10b6  3072a9b7  256a8b8 .

8
El terme sisè del desenvolupament és T6     3a2   2ab   48 384a11b5 .
3 5

 
5

40 Matemàtiques 1r Batxillerat
SOLUCIONARI UNITAT 2. ÀLGEBRA

12
 3 
20. Troba el terme independent del desenvolupament de:  2  5x  .
x 

13  k
 12   3   12  313 k  5k 1  x k 1  12  13 k k 1 3k 27
Tk    2   5x k 1     3 5  x
 k  1  x   k  1 x 26 2k  k  1
El terme independent compleix que 3k  27  0  k  9 .

 12 
Per tant, el terme independent és T9    313 9  591  495  3 4  5 8
8
7
 2y 2 3 
2
21. Calcula el coeficient de y en el desenvolupament:   2 
.
 3 2 y 

8k k 1 8  k k 1 16  2k
7   2y   3 
2
k 1 
 ( 1)k 1 
7  2 ·3 ·y
 ( 1)k 1 
7  9  2k 2k 9 4k 18
Tk  ( 1)  k  1    2  2 3 ·y
   3   2y   k  1 2k 1·38  k ·y 2k 2  k  1

El terme de y compleix que 4k  18  2  k  4 . Per tant, el coeficient de y és ( 1)3   29 8  38 9  
2 2 7 70
3 3
22. Exercici resolt.

23. Calcula el màxim comú divisor i el mínim comú múltiple dels polinomis següents.
a) P(x)  x  3x  4 i Q(x)  x  2x  1
3 2 2

b) P(x)  x  3x  2x  6x  x  3 i Q(x)  x  3x  x  3
5 4 3 2 3 2

c) P(x)  x, Q(x)  x  x i R(x)  x  2x  x


2 3 2

d) P(x)  12x  4x  3x  1 i Q(x)  12x  16x  7x  1


3 2 3 2

e) P(x)  x  6x  8, Q(x)  x  4 i R(x)  2x  4x


2 2 2

a) P( x )  ( x  1)( x  2) , Q( x )  ( x  1)
2 2

m.c.d.P( x ), Q( x )  x  1 m.c.m.P ( x ), Q( x )  ( x  1)2 (x  2)2  x 4  2x 3  3x 2  4x  4


;

b) P( x )  ( x  1) ( x  1) ( x  3) , Q( x )  ( x  1)( x  1)( x  3)
2 2

m.c.d.P( x ), Q( x )  ( x  1)( x  1)( x  3)  Q( x ) m.c.m.P( x ), Q( x )  ( x  1)2 ( x  1)2 ( x  3)  P( x )


;

c) P( x )  x , Q( x )  x( x  1) , R( x )  x( x  1)
2

m.c.d.P( x ), Q( x ), R( x )  x  P( x ) m.c.m.P( x ), Q( x ), R( x )  x( x  1)2  R( x )


;

d) P( x )  (2x  1)(2x  1)(3x  1) , Q( x )  (2x  1) (3x  1)


2

m.c.d.P( x ), Q( x )  (2x  1)(3x  1)  6x 2  5x  1

m.c.m.P( x ), Q( x )  (2x  1)(2x  1)2 (3x  1)  24x 4  20x 3  2x 2  5x  1

e) P( x )  ( x  4)( x  2) , Q( x )  ( x  2)( x  2) , R( x )  2x( x  2)

m.c.d.P( x ), Q( x ), R( x )  x  2 m.c.m.P ( x ), Q( x ), R( x )  ( x  2)( x  2)2x(x  4)  2x 4  8x3  8x 2  32x


;

41 Matemàtiques 1r Batxillerat
SOLUCIONARI UNITAT 2. ÀLGEBRA

24. Simplifica les fraccions algebraiques següents.

2x 4  x 3  11x 2  11x  3 x 3  2x 2  9x  18 x 4  5x 3  9x 2  7x  2
a) b) c)
2x 3  3x 2  8x  3 x 3  7x 2  16x  12 x 4  6x 3  13x 2  12x  4

2x 4  x 3  11x 2  11x  3 ( x  3)( x  1)2(2x  1)


a)   x 1
2x 3  3x 2  8x  3 (x  1)(x  3)(2x  1)

x 3  2x 2  9x  18 ( x  2)( x  3)( x  3) x  3
b)  
x 3  7x 2  16x  12 ( x  3)( x  2)2 x 2

x 4  5x 3  9x 2  7x  2 (x  1)3(x  2) x 1
c)  
x  6x  13x  12x  4 ( x  1)2( x  2)2 x  2
4 3 2

25. Fes les operacions següents amb fraccions algebraiques i simplifica el resultat.

a2 a  b2 a x x a
a)  4 a e) 
ab b x 2  a2 x  a
6 4 16 1
b)   f) 1
2  x 2  x x2  4 1
1
1
1
x
1
2x 2  7x  6 x 2  6x  5 x
c) · g)
x 2  2x  1 2x  3 1

1
x  1 1 x
2x  1
 1 1 x 1
d) ( x  y ) :   
2 2
h) 1 
x y 1 x

x 1 x 1

a  b2 a2b3  a2b2  a2b 4 a b  b  1  b  ab  a  ab2


2 2 2
a2
a)  4 a   
ab b ab 4
ab 4
b2
6 4 16 6( x  2)  4( x  2)  16 6x  12  4x  8  16 10x  12
b)   2   
2 x 2 x x 4 (x  2)(x  2) ( x  2)( x  2) x2  4

2x 2  7x  6 x 2  6x  5 ( x  2)(2x  3) ( x  1)( x  5) ( x  2)( x  5) x 2  3x  10


c) ·  ·  
x 2  2x  1 2x  3 (x  1)2 2x  3 x 1 x 1

 1 1  ( x  y )( x  y ) xy ( x  y )( x  y )
d) ( x 2  y 2 ) :       x 2 y  xy 2
 x y  x  y x  y
xy

a x x a (a  x )(x  a) 1
e)   
x 2  a2 x  a (x  a)(x  a)2 x  a

1 1 1 1 x  1 3x  2
f) 1  1  1  1  1 
1 1 x 2x  1 2x  1 2x  1
1 1 1
1 x 1 x 1 x 1
1
x x
1 1 1
x x x 1 x 2
g)   
1 1 1  x    x  1 2 2x

x  1 1 x  x  1 1  x  1 x 2

42 Matemàtiques 1r Batxillerat
SOLUCIONARI UNITAT 2. ÀLGEBRA

2x  1
x 1  2x  1 x2  1  ( x  1)(2x  1) 3x 2  3x  2
h) 1   1  :   1 
1

x  x  1 ( x  1)( x  1)  x2  1 x2  1
x 1 x 1

26 a 28. Exercicis resolts.

29. Resol les equacions polinòmiques següents.

2x  3 x 3x  1 x 2  1 2x  3 x 2 59
a)    2x  1 d)   
4 2 5 2 4 6 12

b) 2  3x  2  x(x  1)  4 e) 6x 4  13x 3  8x 2  17x  6  0

c) (x  1)3  (x  1)3  7 f) 2x 4  x 3  3x  18  0

2x  3 x 3x  1 9
a)    2x  1  10x  15  10x  12x  4  40x  20  32x  9  x 
4 2 5 32

b) 2  3x  2  x(x  1)  4  6x  4  x 2  x  4  0  x 2  5x  0  x(x  5)  0  x  0, x  5

5 5
c) (x  1)3  ( x  1)3  7  x 3  3x 2  3x  1  (x 3  3x 2  3x  1)  7  6x 2  5  x  ,x
6 6

x 2  1 2x  3 x 2 59
d)     6x 2  6  6x  9  2x 2  59  8x 2  6x  44  0  4x 2  3x  22  0 
2 4 6 12

3  9  352 3  19 11
x   x  , x  2
8 8 4

3 1
e) 6x 4  13x 3  8x 2  17x  6  0  ( x  1)(x  2)(2x  3)(3x  1)  0  x  1, x  2, x   , x 
2 3

3
f) 2x 4  x 3  3x  18  0  (x  2)(x 2  3)(2x  3)  0  x  2, x  
2

30. Escriu una equació de quart grau que tingui aquestes solucions x1 = 0, x2 = 1, x3 = 2 i x4 = 3:

(x  2)(x 2  4x  5)  0  x 3  2x 2  3x  10  0

La solució donada s’obté desenvolupant aquesta


x(x  1)(x  2)(x  3)  x 4  6x 3  11x 2  6x

31. Resol les equacions següents.

a) 2(x  1)4  8x 3  8(x  3)  8  0 c) x 4  17x 2  16  0

9
b) 3x 4  x 3  73x 2  25x  16  0 d)  10  x 2
x2

a) 2(x  1)4  8x 3  8(x  3)  8  2x 4  8x 3  12x 2  8x  2  8x 3  8x  24  8  2x 4  12x 2  14  x 4  6x 2  7  0

z  x2  6  36  28 6  8 z  1  x  1, x  1
  z  6z  7  0  z   
2

z2  x 4  2 2 z  7  No té solució real
b) El polinomi no té arrels enteres, per la qual cosa no és fàcil de factoritzar i, per tant, l’equació no es pot resol-
dre de manera senzilla (de fet, no té arrels reals).

43 Matemàtiques 1r Batxillerat
SOLUCIONARI UNITAT 2. ÀLGEBRA

z  x2  17  289  64 17  15 z  16  x  4, x  4
  z  17z  16  0  z   
2
c)
z2  x 4  2 2 z  1  x  1, x  1

9
d)  10  x 2  9  10x 2  x 4  x 4  10x 2  9  0
x2

z  x2  10  100  36 10  8 z  9  x  3, x  3
4
 z 2  10z  9  0  z   
z x  z  1  x  1, x  1
2
2 2

6 3
32. Resol aquesta equació: x – 19x – 216 = 0. Utilitza la tècnica del canvi d'incògnita.
3
Fem el canvi de variable t = x , i obtenim aquesta equació:

19  192  4   216  t1  8


t 2  19t  216  0  t  
8 t2  27

19  192  4   216  
 x  8  2
3

Desfent el canvi de variable, tenim t 2  19t  216  0  t   1


8  x2  27  3

3

Les solucions són x = –2 i x = 3.

33 a 35. Exercicis resolts.

36. Resol les equacions racionals següents.

1 1 1 7 2x 2x  3 11 2x 3x 6x 2
a)    b)   c)   2
x x2 x3 8 3 x 1 3x  3 x 2 x 2 x 4

1 1 1 7
a)     8x 2  8x  8  7x 3  (x  2)(7x 2  6x  4)  0  x  2 La solució és vertadera.
x x2 x3 8

2x 2x  3 11 
 x  1  2
b)    2x( x  1)  3(2x  3)  11  2x 2  4x  2  0   1 Totes dues són vertade-
3 x 1 3x  3  x2  1  2

res.

2x 3x 6x 2  x  0 Vertadera
c)   2  2x( x  2)  3x( x  2)  6x 2  x 2  2x  0  
x 2 x 2 x 4  x  2 Falsa

37. Troba la solució d’aquestes equacions racionals.


x 1
1
1 x 1 x 1  2
a)  b)
1 x x 1
1 2
x 1 x 1

1 x 1 x 1 x 1
a)    ;x {0, 1}, que anul·len els denominadors de l’equació, són solucions.
1 x x x
1
x 1
x 1
1
x  1  2  2x : x  3  2  2x  2x  2  2x 2  2x  2x 2  4x  6  2x  6  x  3
2
b)
x 1 x 1 x 1 x 2  2x  3
2
x 1

38 i 39. Exercicis resolts.

44 Matemàtiques 1r Batxillerat
SOLUCIONARI UNITAT 2. ÀLGEBRA

40. Resol aquestes equacions amb radicals:

a) x  25  x 2  1 c) x  5x  10  8

x 1 5
b) x  170  x 2  14 d) x
x 2

a) x  25  x 2  1  x  1  25  x 2   x  1  25  x 2  x 2  2x  1  25  x 2 
2

 2x 2  2x  24  0  x  3, x  4

La solució és x = 4, ja que x = –3 no satisfà l'equació original.

b) x  170  x 2  14  14  x  170  x 2  14  x   170  x 2  x 2  28x  196  170  x 2 


2

 2x 2  28x  26  0  x  1, x  13

c) x  5x  10  8  5x  10  8  x  5x  10  8  x   5x  10  x 2  16x  64 
2

 x 2  21x  54  0  x  3, x  18

La solució és x = 3, ja que x = 18 no satisfà l'equació original.


2
x 1 5 x  1 2x  5 2x  2  2x  2  4x 2  8x  4
d) x    x  x  x 
x 2 x 2 2x  5  2x  5  4x 2  20x  25

2 3 2 3
 4x 3  20x 2  25x  4x 2  8x  4  4x 3  24x 2  33x  4  0  x  4, x  ,x
2 2

2 3 2 3
Les solucions són x = 4 i x  , ja que x  no satisfà l'equació original.
2 2

41. Resol les equacions amb radicals següents.

a) x  4  x 1  5 c) x  7  2x  x  1

b) 3(16  x)  2x  5  1 d) 1 4x  2 3  x  3  x

   5  
2 2
a) x  4  x 1  5  x  4  5  x 1  x4 x 1  x  4  25  x  1 10 x  1 

 10 x  1  20  x  1  2  x  1  4  x  5

b) 3(16  x)  2x  5  1  48  3x  1 2x  5  48  3x  1 2x  5  2 2x  5  2 2x  5  5x  52 

264  2 399 264  2 399


 4(2x  5)  25x 2  2704  520x  25x 2  528x  2724  0  x  (Falsa); x 
25 25

c) x  7  2x  x  1  x  7  2x  2 (x  7)2x  x  1  2x 2  14x  4  x  2x 2  14x  16  x 2  8x 

 x 2  6x  16  0  x  8 (Falsa); x  2 (Falsa) L’equació no té solució.

d) 1 4x  2 3  x  3  x  1 4x  3 3  x  1 4x  9(3  x)  1 4x  27  9x  13x  26  x  2

42. Resol les equacions amb radicals següents.

a)
3
x2  1  1  x b)
3
3x 2  9x  5  5  2x

45 Matemàtiques 1r Batxillerat
SOLUCIONARI UNITAT 2. ÀLGEBRA

a) 3
x 2  1  1  x  3 x 2  1  x  1  x 2  1  (x  1)3  (x  1)(x  1)  (x  1)3  0  ( x  1)( x  1 x 2  2x  1)  0 

 (x  1)(3x  x 2 )  0  x(x  1)(3  x)  0  x  0; x  1; x  3

3x 2  9x  5  5  2x  3 3x 2  9x  5  2x  5  3x 2  9x  5   2x  5   8x 3  60x 2  150x  125 


3 3

b)
 8x 3  63x 2  141x  130  0  x  5

 x 2  x  6  0  x  3; x  2 (Falsa)

43. Calcula el valor d’un nombre tal que si se li suma una unitat i després se n’extreu l’arrel quadrada s’obté el
doble que si se li resten 11 unitats i se n’extreu l’arrel quadrada.

Nombre desconegut: x; x  1  2 x  11  x  1  4(x  11)  3x  45  x  15

44 a 46. Exercicis resolts.

47. Resol les equacions logarítmiques següents.

a) log(2x  3)  log(x  1)  2log2  2log3 c) log x 3  log6  2log x

2x  2 log  4  x 
b) log  2log( x  1)  log x d) 2
x log  x  2 

2x  3 2x  3 39
a) log(2x  3)  log(x  1)  2log2  2log3  log  log  22·32    36  2x  3  36x  36  x 
x 1 x 1 34
2x  2
b) log  2log(x  1)  log x  log 2(x  1)  log x  2log( x  1)  log x  log2  log(x  1)  2log(x  1) 
x
 log(x  1)  log2  x  1  2  x  3

c) log x 3  log6  2log x  3log x  log6  2log x  log x  log6  x  6

 8x  8  10x 2  10x  20x  10x 2  2x  8  x  4 (Falsa) L’equació no té solució.

log  4  x 
d)  2  lg(4  x)  2log( x  2)  log(4  x)  log( x  2)2  4  x  x 2  4x  4  x  0; x  5 (Falsa)
log  x  2

48. Resol aquestes equacions logarítmiques.

a) log x 2  2  log x 2

10
 5 log x 2 
2
b) log
x

1
a) log x 2  2  log x 2  log x 2  2  log x 2  log x  2  2log x  log x  2  x  100
2

46 Matemàtiques 1r Batxillerat
SOLUCIONARI UNITAT 2. ÀLGEBRA

10
 5 log x 2   log10  log x  5  2log x   1  log x  5  2log x 
2 2 2
b) log
x
Fem el canvi de variable t = log x

 1
t1  5
1  log x  5  2log x   1  t  5  2t 
2 2
 20t  t  1  0  
2

t   1
 2 4
Només ens interessa la solució positiva. Desfem el canvi de variable:
1
log x   x  5 10
5
49. Troba el nombre que compleix que quan s’afegeix al seu logaritme decimal el valor del logaritme decimal
de 2, el resultat és 1.

Nombre desconegut: x
log x  log2  1  log2x  log10  2x  10  x  5

50. Calcula el valor d’un nombre sabent que el doble del seu logaritme decimal és igual a la suma dels loga-
ritmes decimals de 4 i de 9.

Nombre desconegut: x

x  6
2log x  log 4  log9  log x 2  log36  x 2  36  
 x  6 (Falsa)
51 a 53. Exercicis resolts.

54. Resol les equacions exponencials següents.


x ( x 1)
1 1
 16 2 b) 3·2x  2·3x  2x
 0  3x  5
 16
2 2
a) x
c) 13x d) 22x
2 13

x( x 1) 4 x ( x 1) x  0
1 
a)  16 2  2 x
 2 2   x  2x( x  1)  2x 2
 x  0  x(2x  1)  0   1
2x  x
 2
x
2 2
b) 3·2x  2·3 x      x  1
3 3

1
 2x
  0  13x  2x  131  x 2  2x  1  x 2  2x  1  0  x  1
2 2
c) 13x
13

x  3

3x 5
 16  22x 3x 5
 24  2x 2  3x  5  4  2x 2  3x  9  0  
2 2
d) 22x 3
 x
 2

47 Matemàtiques 1r Batxillerat
SOLUCIONARI UNITAT 2. ÀLGEBRA

55. Resol les equacions següents.

a) 2x  4  8x  0 b) 32x  3x 1  3x 1  1 c) 5x 3  5x 1  3120  0 d) 2  102x  4  3  10x  2  5  0

a) 2x  4  8x  0  2x  4  23x  0  2x  4  23x  x  4  3x  x  2

b) Aplicant el canvi de variable z  3x

 z3
1 1 
32x  3 x 1  3 x 1  1   3 x   ·3 x  3·3 x  1  0  z2  z  3z  3  0  3z 2  10z  3  0  
2
1
3 3 z  3

3 x  3  x  1

 x 1
3   x  1
 3

5x 1 5·3120
c) 5x  3  5x 1  3120  0  5x  53   3120  0  (53  )5 x  3120  5 x  4  25  x  2
5 5 5 1

d) Aplicant el canvi de variable z  10x

2·102x  4  3·10 x  2  5  0  2  104  10 x   3  102  10 x  5  0  20000 z2  300z  5  0 


2

z  0,01  10 x  102  x  2
 4000z 2  60z  1  0  
z  0,025  10  0.025 sense solució real
x

56. Exercici interactiu.

57 a 60. Exercicis resolts.

61. Resol gràficament i per algun mètode algebraic.

2x  2y  6 3x  5y  10 3x  5y  7 2x  3(4  y )  6


 b)  c)  d) 
 x  3y  1 2x  y  2 6x  10y  14 3(2x  9)  5y  1

2x  2y  6  6x  10y  14
a)   y  1, x  2 c)  , Infinites solucions.
2x  6y  2 6x  10y  14

3x  5y  10 2x  3y  18


b)   x  0, y  2 d)   x  6, y  2
10x  5y  10 6x  5y  26

48 Matemàtiques 1r Batxillerat
SOLUCIONARI UNITAT 2. ÀLGEBRA

62. Resol els sistemes d’equacions següents.

2x  y  8

2x  3y  22
2
2x 2  3y 2  32
b) 
3x  4y  48
2 2
x  y  4
c)  2
 x  2y  19
2
d) 
x 2  y 2  13
xy  6

 x  5, y  2
y  8  2x
 
a)   2x  192  12x 2  96x  22  12x 2  94x  170  0 . Solucions:  17 7
  x ,y 
2

2x  3 8  2x  22 
 6 3

3E1 6x2  9y 2  96  y  0, x  4
b)   17y 2  0  y  0 . Solucions: 
2E2 6x  8y  96  y  0, x  4
2 2

y  3, x  1
x  y  4 x  4  y
 x  4  y 
c)  2     2 . Solucions:  1 13
 x  2y  19  4  y   2y  19  3y  8y  16  19 3y  8y  3  0 y  , x
2 2

2 2

 3 3

 2  6 2 x  2, y  3
 x     13

x  2, y  3
x  y  13  
2 2
x  x 4  13x 2  36  0 . Solucions:

d)
xy  6  
 6 x  3, y  2
 y  x  x  3, y  2

63 a 65. Exercicis resolts.

66. Estudia i resol els sistemes d’equacions lineals següents aplicant el mètode de Gauss.

 x  2y  2z  2 2x  4y  z  0 5x  2y  2z  0  2x  y  z  0
   
a) 3x  3y  z  14 c) 3x  3y  2z  1 e) 3x  y  3z  0 g)  x  y  z  1
5x  y  2z  15 3x  3y  2z  5 8x  y  z  1  2x  y  z  2
   

2x  y  z  11 4x  y  5z  5  2x  y  z  5 3x  y  z  2


   
b) 2x  2y  z  8 d) 5x  y  z  13 f)  x  y  2z  3 h) 2x  5y  2z  0
x  y  z  7 4x  2y  3z  14  x  2y  7z  0 x  y  z  1
   

 x  2y  2z  2  x  2y  2z  2
E2  3E1  
a)    9y  7z  20 9E3  11E2    9y  7z  20  z  1, y  3, x  2
E3  5E1
  11y  8z  25   5z  5
 

2x  y  z  11
E2  E1 
b)    3y  3  y  1, z  2, x  4
2E3  E1
  
 y z 3

2x  4y  z  0
2E2  3E1  3 1 25
c)    18y  z  2  z  , y  ,x 
E3  E2 2 36 36
 4z  6

4x  y  5z  5 4x  y  5z  5
4E2  5E1  
d)    9y  21z  27 3E3  E2    9y  21z  27  z  0, y  3, x  2
E3  E1
  3y  2z  9   15z  0
 

5x  2y  2z  0
5E2  3E1 
e)    11y  21z  0 Sistema incompatible. No té solució.
5E3  8E1
  11y  21z  5

49 Matemàtiques 1r Batxillerat
SOLUCIONARI UNITAT 2. ÀLGEBRA

2x  y  z  5 2x  y  z  5
2E2  E1  
f)   y  3z  1 E3  5E2    y  3z  1  z  t, y  3t  1, x  t  2
2E3  E1
 5y  15z  5  00
 

2x  y  z  0
2E2  E1 
g)  y  3z  2  z  1, y  1, x  1
E3  E1
 2z  2

3x  y  z  2 3x  y  z  2
3E2  2E1   1 1
h)   17y  8z  4 17E3  4E2   17y  8z  4  z  , y  0, x 
3E3  E1 2 2
 4y  2z  1  66z  33
 

67. Resol el sistema següent aplicant un canvi de variables que el transformi en lineal.

1 2 3
x    7
y z

2 3 5
    12
x y z
4 4 1
    15
x y z

1 1 1
Fent el canvi de variable: a  , b  , c  . Per tant:
x y z

a  2b  3c  7 a  2b  3c  7 a  2b  3c  7
 E2  2E1  
2a  3b  5c  12  E  4E  b  c  2  E3  4E2  b  c  2  c  5, b  3, a  2
4a  4b  c  15 3 1 4b  11c  43 7c  35
  

1 1 1
Per tant: x  , y , z
2 3 5

68. Calcula les edats de tres germans sabent que:


 Les edats dels tres sumen 44 anys.
 L’edat del germà mitjà és mig any més gran que la mitjana aritmètica de les edats dels altres dos ger-
mans.
 La suma de les edats dels dos germans petits supera en 10 anys l’edat del més gran.

x és l’edat del germà gran en anys, y l’edat del germà mitjà en anys, i z l’edat del germà petit en anys.

x  y  z  44
 x  y  z  44 x  y  z  44
 xz 1  E2  E1 
y    x  2y  z  1
E  E1
 3y  45  x  17, y  15, z  12 .
 2 2 x  y  z  10 3 2x  34
 
y  z  x  10

Per tant, l’edat del germà gran és de 17 anys, la del germà mitjà,15 anys, i la del germà petit, 12 anys.

69. Exercici interactiu.

70. Comprova si els nombres reals indicats pertanyen a la solució de les inequacions corresponents.

a) x  2 de la inequació x 3  x 2  x  6 c) x  0,5 de la inequació 2x  x  2  3x

1
b) x  de la inequació x  ln x  0
e

50 Matemàtiques 1r Batxillerat
SOLUCIONARI UNITAT 2. ÀLGEBRA

a) Sí que forma part de la solució de la inequació, ja que (2)3  (2)2  2  8  4  2  6  6 .

1  1 1 1 e
b) No forma part de la solució de la inequació, ja que  ln     1  0.
e e e e

1 1
c) Sí que forma part de la solució de la inequació, ja que 20,5  0,5  2  30,5   2,5  0
2 3

71. Resol les inequacions següents.


x 3 x 2 x x  38
a)   c) x  2  x  1  3  x  2 
2 8 2 2

x 2  1 3x  2  x  1
2
x 3x  1
b) 2x  3  x d)  
2 6 2 3 6

x 3 x 2 x
a)    4x  12  x  2  4x  x  10  x  10  Solució: x   10,   
2 8 2
x 3x  1
b) 2x  3  x  12x  18  3x  6x  3x  1  0  19  La inequació no té solució.
2 6

x  38
c) x  2  x  1  3  x  2   2x  4x  4  6x  12  x  38  11x  22  x  2  Solució: x   , 2
2

x 2  1 3x  2  x  1
2

d)    3x 2  3  6x  4  x 2  2x  1  2x 2  8x  0  2x( x  4)  0
2 3 6

Solució: x  0, 4
 0 4 +
x  + +
x4   +
Polinomi +  +

72. Troba la solució de les inequacions polinòmiques següents.

a) 2x 3  3x  0 c) x 3  4x  0 e) 3x(2x  1)  x 2  5x  1 g) x 4  1  0

b) 3x 2  x  24  0 d) x 3  x 2  x  1  0 f) x 3  2x 2  x  0 h) (x  1)(x  4)  6

a) 2x 3  3x  0  x(2x 2  3)  0  e) 3x(2x  1)  x 2  5x  1  (x  1)(7x  1)  0


x  3  2x  3  2x  0 
1
3 3  1 
  0  7
2 2
7x  1   
x2    
x1   
x    
Polinomi   
x2    
Polinomi    

 3  3  1
Solució: x   ,     0,  Solució: x   ,   1,   
 2   2   7 

51 Matemàtiques 1r Batxillerat
SOLUCIONARI UNITAT 2. ÀLGEBRA

b) 3x 2  x  24  0  (x  3)(3x  8)  0 f) x 3  2x 2  x  0  x(x  1)2  0

 3
8
  1 0 
(x  1)   
2
3
x3    x   
3x  8    Polinomi   
Polinomi   

8 
Solució: x   ,  3   ,    Solució: x  1  0,   
3 

c) x 3  4x  0  x(x  2)(x  2)  0 g) x 4  1  0  (x  1)(x  1)(x 2  1)  0

 2 0 2   1 1 
x2     x1   
x     x1   
x2     x 1
2
  
Polinomi     Polinomi   

Solució: x   2, 0   2,    Solució: x   ,  1  1,  

d) x 3  x 2  x  1  0  (x  1)(x  1)2  0 h) (x  1)(x  4)  6  (x  1)(x  2)  0

 1 1   2 1 
(x  1)   
2
x2   
x1    x1   
Polinomi    Polinomi   

Solució: x   , 1 Solució: x   2,  1

73. Resol les inequacions racionals següents.


5x  2 4x  5 x4
a) 0 c) 0 e) 0
2x  1 4x 2  x  5 x 9
2

x2  x  2 x3 x
b) 0 d) 0 f) 0
x2  x  2 x 9
2
3  2x  10 

52 Matemàtiques 1r Batxillerat
SOLUCIONARI UNITAT 2. ÀLGEBRA

5x  2 x3 x3
a) 0 d)  0  0
2x  1 x2  9 ( x  3)( x  3)

1 2
    3 0 3 
2 5 x3    
2x  1    x
3
   
5x  2    x3    
Fracció    Fracció    

 1 2 
Solució: x   ,     ,    Solució: x  (3, 0]  (3, )
 2  5 

x2  x  2 (x  1)(x  2) x4 x4
b) 0 0 e)  0  0
x  x 2
2
(x  1)(x  2) x2  9 (x  3)( x  3)

 2 1 1 2   3 3 +
x2      x    
x1      x3   
x1      x
4
+ + +
x2      Fracció +  +
Fracció     
Solució: x  (, 2)  [1, 1)  [2, ) Solució: x  (–3, 3) – {0}
Quan el numerador és 0, la fracció val 0.

4x  5 4x  5 1 x x
c) 0 0 0 f) 0 0
4x 2  x  5  4 x  5  x  1 x 1 3  2x  10  6  x  5

Observa que 4x  5 no es pot anul·lar,


per la qual cosa es pot simplificar i la solució  0 5 
és x  (, 1). x   +
x 5   +
Fracció   +

Solució: x  0, 5 

74. Exercici interactiu.

75 i 76. Exercicis resolts.


77. Resol els sistemes d’inequacions:

2x  1  x  2 3x  4  x  x 2  2x  3  0 3x  2  5x  6


a)  b)  c)  d)  2
3x  1  4x  x  2x  1 2( x  3)  6 x  1

2x  1  x  2  x  1
a)    Solució: x   1, 1
3x  1  4x  x  1

3x  4  x 2x  4 x  2
b)     Sense solució
 x  2x  1 x  1 x  1

 x 2  2x  3  0 ( x  1)  ( x  3)  0
c)    Solució: x  1,   
2( x  3)  6 x  0

53 Matemàtiques 1r Batxillerat
SOLUCIONARI UNITAT 2. ÀLGEBRA

3x  2  5x  6
d)  2 Solució: x   ,  1  1, 4
x  1

78. Resol els sistemes d’inequacions:

0  y  2
x  y  3
  y  2x  4
a)  x  y  2 b) 
x  0 y  x

 y  0

a) b)

4 4
Vèrtexs A(2, 0) B(2,5 ; 0,5) C(3, 0) Vèrtexs: A  , 
3 3

79 a 91. Exercicis resolts.

EXERCICIS
Polinomis
92. Simplifica les expressions polinòmiques següents.

2 3  3 2 6
a) 2(x  2)2  3  3x  2  2  3x  2 3x  2 e)  x    x 2   
3 5  2 5  25

b) (3x  2)2  2(2x  3)2  (2x  5)(x  5) f) x 2


 4 x 2  4  x  2  2x(x 3  10)x

c) (2x 2  2x  13)(3x 2  2x)  3x g) (x 3  2x 2  3x  4)(5x  2)

d) (2x 2  3x  2)(3x 2  x  1)  (6x  10)x 3

a) 2(x  2)2  3 3x  2  2 3x  23x  2  2x 2  8x  8  9x  6  18x 2  8  16x 2  17x  10

b) (3x  2)2  2(2x  3)2  (2x  5)(x  5)  9x 2  12x  4  8x 2  24x  18  2x 2  10x  5x  25  15x 2  3x  3

c) (2x 2  2x  13)(3x 2  2x)  3x  6x 4  4x 3  6x 3  4x 2  39x 2  26x  3x  6x 4  10x 3  35x 2  23x

d) (2x 2  3x  2)(3x 2  x  1)  (6x  10)x 3  6x 4  2x 3  2x 2  9x 3  3x 2  3x  6x 2  2x  2  6x 4  10x 3 

 x 3  7x 2  x  2

2 3  3 2 6 4 9 2 6 6 9 2 4
e)  x    x 2     x3  x x    x3  x  x
3 5  2 5  25 15 10 25 25 10 15

54 Matemàtiques 1r Batxillerat
SOLUCIONARI UNITAT 2. ÀLGEBRA

f) x 2
 4 x 2  4  x  2  2x(x 3  10)x  (x 4  16)  x  2  2x 5  20x 2  x 5  2x 4  16x  32  2x 5  20x 2 

 3x 5  2x 4  20x 2  16x  32

g) (x 3  2x 2  3x  4)(5x  2)  5x 4  2x 3  10x 3  4x 2  15x 2  6x  20x  8  5x 4  8x 3  11x 2  14x  8

93. Demostra la igualtat algebraica:

 x  y  z
2
 x 2  y 2  z2  2xy  2xz  2yz

(x  y  x)2  (x  y  z)(x  y  z)  x 2  xy  xz  yx  y 2  yz  zx  zy  z2  x 2  y 2  z 2  2xy  2xz  2yz

94. Calcula el quocient i el residu de les divisions de polinomis següents.

a)  6x 4
 7x 3  5x 2  6x  6 :  3x 2  2x  1 c)  6x 5
 10x 4  15x 3  7x 2  3 :  4x 2  4x  4

b)  6x 4
 7x 3  x 2  11x  8 :  3x 2  x  2

a) Quocient: 2x 2  x  3

Residu: x  3

5 4
b) Quocient: 2x 2  x
3 9
125x 64
Residu: 
9 9

3 3 13
c) Quocient: x  x2  x 6
2 4
Residu: 37x  21

95. Aplica la regla de Ruffini per resoldre les divisions següents.

 3x 3  2x 2  x  3 :  x  2 
1
a) d) x 5
 3x 4  3x 3  x 2  4x  2 :  2x  4 
 

b)  2x 4
 2x 3  2x 2  2 :  x  3  e)  2x 4
 x 3  2x 2  x  1 :  2x  3 

c) x 5
 2x 3  2x  1 :  2x  4

7 3 1 4 1 3 5 2 9
a) Quocient: 3x 2  x d) Quocient: x  x  x  x  11
2 4 2 2 2 2

21
Residu:  Residu: 46
8
11
b) Quocient: 2x 3  8x 2  26x  78 e) Quocient: x 3  2x 2  4x 
2

31
Residu: 232 Residu: 
2
1 4
c) Quocient: x  x 3  3x 2  6x  11
2

Residu: 45

55 Matemàtiques 1r Batxillerat
SOLUCIONARI UNITAT 2. ÀLGEBRA

96. Determina, si existeixen, les arrels enteres de cadascun dels polinomis següents i factoritza’ls.

a) P(x)  x 3  2x 2  5x  6 e) P(x)  x 4  4x 2

b) P(x)  x 3  x 2  5x  3 f) P(x)  6x 5  11x 4  3x 3  3x 2  x

c) P(x)  2x 4  x 3  5x 2  x  3 g) P(x)  x 6  4x 4  x 2  4

x4 3 2 9
d) P(x)  4x 3  4x 2  11x  6 h) P( x )   x 
4 4 16

a) P(x)  x 3  2x 2  5x  6  (x  1)(x  2)(x  3) . Les arrels enteres són x  1 , x  2 i x  3 .

b) P(x)  x 3  x 2  5x  3  (x  1)2(x  3) . Les arrels enteres són x  1 i x  3 .

 3
c) P(x )  2x 4  x 3  5x 2  x  3  2( x  1)(x  1)2  x   . Les arrels enteres són x  1 i x  1 .
 2

 1  3
d) P(x )  4x 3  4x 2  11x  6  4( x  2)  x    x   . L’única arrel entera és x  2 .
 2  2

e) P(x)  x 4  4x 2  x 2(x  2)(x  2) . Les arrels enteres són x  0 , x  2 i x  2 .

 1  1
f) P( x)  6x 5  11x 4  3x 3  3x 2  x  6x(x  1)2  x    x   . Les arrels enteres són x  0 i x  1 .
 2  3

g) P(x)  x 6  4x 4  x 2  4  (x  1)(x  1)(x  2)(x  2)(x 2  1) . Les arrels enteres són x  1 , x  1 , x  2 i x  2 .


2 2 2
x4 3 2 9  x2  x2 3  3   x2 3 
h) P( x )   x      2· ·        . No té arrels enteres.
4 4 16  2  2 4 4  2 4

97. En cada cas, calcula el m.c.d. i el m.c.m. dels polinomis donats.

a) P(x)  x 2  x  2 i Q(x)  x 2  2x  3 c) P(x)  x  1 , Q(x)  2x  2 i R(x)  3x 2  3

b) P(x)  2x 2  2 i Q(x)  4x  4 d) P(x)  x 2(x  2) , Q(x)  x   x 2  4 i R(x)  x 3  2x 2

a) P(x)  (x  1)(x  2) Q(x)  (x  1)(x  3)

m.c.d.P(x), Q(x)  x  1 m.c.m.P(x), Q(x)  (x  1)(x  2)(x  3)  x 3  4x 2  x  6

b) P(x)  2(x  1)(x  1) Q(x)  4(x  1)

m.c.d.P(x), Q(x)  2(x  1)  2x  2 m.c.m.P(x), Q(x)  4(x  1)(x  1)  4x 2  4

c) P(x)  x  1 Q(x)  2(x  1) R(x)  3(x  1)(x  1)

m.c.d.P(x), Q(x), R(x)  1 m.c.m.P(x), Q(x ), R(x)  6(x  1)(x  1)  6x 2  6

d) P(x)  x 2(x  2) Q(x)  x(x  2)(x  2) R(x)  x 2(x  2)

m.c.d.P(x), Q(x), R(x)  x(x  2)  x 2  2x m.c.m.P(x ), Q(x ), R(x )  x 2 (x  2)(x  2)  x 4  4x 2

98. Calcula el valor de k per tal que x 3  (3k  2)x 2  3 entre x  3:

a) Sigui exacta. b) Tingui residu 15.

56 Matemàtiques 1r Batxillerat
SOLUCIONARI UNITAT 2. ÀLGEBRA

16
a) Pel teorema del residu tenim: 0  (3)  (3k  2)(3)  3  0  27  27k  18  3  k  
3 2
9
7
b) Pel teorema del residu tenim: 15  (3)  (3k  2)(3)  3  15  27  27k  18  3  k  
3 2
3

99. Sense fer les divisions, calcula el seu residu.

a) x 7
 x 3  2x  1 :  x  3  b) x 12
 x 5  x  12 :  x  1

b) R   1   1   1  12  15


12 5
a) R  37  33  6  1  2209

100. Sense fer la divisió, comprova que el binomi x − 3 és un factor del polinomi P  x   2x 3  8x 2  8x  6 .

El valor numèric del polinomi P(x) per a x  3 és 0, per la qual cosa es dedueix que x  3 és un factor del polino-
mi.
101. Determina el valor de k per tal que el polinomi P  x   6x 5 44x 3  88x  k sigui divisible per x  3 .

Com que P(x) és divisible per x  3 , s’ha de complir que el valor numèric de P(x) per a x  3 sigui igual a 0.

P  3  1458  1188  264  k  0  k  6

102. Determina el valor de k per tal que el polinomi P  x   3x 3 kx 2  6k  2 sigui divisible per x  2 .

Com que P(x) és divisible per x  2 , s’ha de complir que el valor numèric de P(x) per a x  2 sigui igual a 0.
P  2  24  4k  6k  2  0  k  13

103. La divisió de x 3 mx  2 entre x  2 dona 6 com a residu. Quant val m? Quant és el quocient?

2  2m  2  6, per la qual cosa m  2, i el quocient és x  2x  2.


3 2

104. Troba un polinomi de segon grau sabent que una de les seves arrels és x  1 i que P(3)  10.

Perquè x  1 sigui una arrel, ha de ser P(x)  (ax  b)(x  1), i com que P(3)  10, aleshores (3a  b)(3  1)  10,
per tant, 3a  b  5. Per exemple, a  1, b  2.
P(x)  (x  2)(x  1)  x  x  2
2

Binomi de Newton
105. Desenvolupa els binomis següents.
4

 
5
x 2  2  2 
2  x 
4
a) c)   2  e) 1 2 2 g)   2
2 x   2 
3 6
 x  3
2  3 3  5 
3 3
b)  2   d)  2x 2   f) h) 2 2 3
 3  x

 4  4  4  4  4
2  x 
4
a)    24    23 x    22 x 2    2x 3    x 4  16  32x  24x 2  8x 3  x 4
0 1  2 3  4

57 Matemàtiques 1r Batxillerat
SOLUCIONARI UNITAT 2. ÀLGEBRA

3 2 3
 x 3 3 x 3  x  3  x  2 1 3
b)  2      23    22     2         8  4x  x 2  x
 3 0 1  3  2  3   3  3  3 27
5 5 4 3 2 2 3 4 5
x 2  5  x   5  x  2  5  x   2   5  x   2   5  x  2   5  2 
c)   2             2       2        2       2      2  
2 x   0   2  1   2  x  2  2   x   3  2   x   4 2  x   5  x 

x5 x4 2 x3 4 x2 8 x 16 32 x 5 5x 2 5 20 40 32
  5  2  10   4  10   6  5   8  10     4  7  10
32 16 x 8 x 4 x 2 x x 32 8 x x x x
d)

6 2 3 4 5 6
 2 3  6  6 2 5 3  6 2 4 3  6 2 3 3 6 2 2 3  6 2  3   6  3 
 2x     0   2x    1   2x     2   2x      3   2x      4   2x      5  2x     6    
2 6

 x     x   x   x   x    x    x 


2916 729
 64x12  576x 9  2160x 6  4320x 3  4860  3  6
x x

1 2 2 
2
e)  1  4 2  (2 2)2  9  4 2

2  3 3 
3
f)  8  3  22  3 3  3  2  (3 3)2  (3 3)3  8  36 3  162  81 3  170  117 3

4
 2 
4
 2 2 2   4 
4
256
g)   2        64
 2      4
 2  2

5   5 2   2    2 3 
3 3 2 2 3
h) 2 2 3 3 5 2 3  35 2 2 3  250 2  300 3  180 2  24 3 

 430 2  324 3

Fraccions algebraiques
106. Simplifica les fraccions algebraiques següents.

x 3  5x 2  8x  4 x4 1 x 3  5x 2  8x  4
a) c) e)
x 3  x 2  8x  12 x 4  2x 3  2x 2  2x  1 x 3  x 2  8x  12

2x 4  5x 3  5x  2 x 3  x 2  8x  12
b) d) f)
2x 4  7x 3  3x 2  8x  4 x2  x  6
x 4  10x 3  21x 2  40x  100
x 4  3x  10

x 3  5x 2  8x  4 (x  1)(x  2)2 x 1
a)  
x 3  x 2  8x  12 ( x  3)( x  2)2 x  3

2x 4  5x 3  5x  2 (x  1)(x  1)(x  2)(2x  1) (x  1)(x  2)(2x  1) 2x 3  3x 2  3x  2


b)     3
2x 4  7x 3  3x 2  8x  4 (x  1)(x  2)2(2x  1) (x  2)2(2x  1) 2x  9x 2  12x  4

x4  1 ( x  1)( x  1)  x 2  1 x  1
c)  
x  2x  2x  2x  1
4 3 2
( x  1)2  x 2  1 x 1

x 3  x 2  8x  12 ( x  3)( x  2)2
d)   x 2
x2  x  6 ( x  3)( x  2)

x 3  5x 2  8x  4 (x  1)(x  2)2 x 1
e)  
x 3  x 2  8x  12 ( x  3)( x  2)2 x  3

x 4  10x 3  21x 2  40x  100 (x  2)(x  2)(x  5)2 (x  2)(x  5)2


f)   3
x  3x  10
4
( x  2)( x  2x  4x  5) ( x  2x 2  4x  5)
3 2

58 Matemàtiques 1r Batxillerat
SOLUCIONARI UNITAT 2. ÀLGEBRA

107. Fes les sumes i les restes de fraccions algebraiques següents i simplifica tant com puguis els resultats.

x2  1 x2 2x 2  x 2x 12x
a)  c)  
x 2  2x  1 x  1 x 3 x  3 9  x2

x 2x  1 50 1 3x  10 2x  7
b)   d)  
x  5 x  5 x 2  25 x  3 x 2  6x  8 x  4

x2  1 x2 x2  1 x2 x 2  1  x 3  x 2 x 3  2x 2  1
a)     
x 2  2x  1 x  1  x  1 2
x 1  x  1
2
 x  1
2

x 2x  1 50 x 2x  1 50 x  x  5    2x  1 x  5   50 x 2  16x  55
b)   2      
x  5 x  5 x  25 x  5 x  5  x  5  x  5   x  5  x  5   x  5  x  5 
  x  11 x  5  11  x
 
 x  5  x  5  x 5

2x 2  x 2x 12x 2x 2  x 2x 12x (2x 2  x)( x  3)  2x( x  3)  12x 2x 3  5x 2  3x


c)        
x 3 x 3 9 x 2
x 3 x  3 (x  3)(x  3) (x  3)(x  3) ( x  3)( x  3)

x( x  3)(2x  1) 2x 2  x
 
( x  3)( x  3) x 3

1 3x  10 2x  7 ( x  4)( x  2)  (3x  10)(x  3)  (2x  7)(x  3)(x  2) 2x 3  21x 2  72x  80


d)  2    
x  3 x  6x  8 x  4 (x  3)(x  4)(x  2) ( x  3)( x  4)( x  2)

(x  4)2(2x  5) 2x 2  13x  20


 
( x  3)(x  4)(x  2) x 2  5x  6

108. Fes els productes i els quocients de les fraccions algebraiques següents i simplifica tant com puguis els
resultats.

x2  1 x2  4 x2  9 x 3  x 4x  4
a)   c) :
x  3 x 1 x  2 2x  4 3x  6

3x 2 y x2  y 2 1 1
b)  d) :
x  y 6xy 2( x  y ) x  y x 2  y 2  2xy

x 2  1 x 2  4 x 2  9 ( x  1)(x  1)(x  2)(x  2)(x  3)(x  3)


a)     (x  1)(x  2)(x  3)  x 3  4x 2  x  6
x  3 x 1 x  2 (x  3)(x  1)(x  2)

3x 2 y x2  y 2 3x 2 y (x  y )(x  y ) x
b)   
x  y 6xy (x  y ) (x  y )6xy 2(x  y ) 2y
2

x 3  x 4x  4 x(x  1)(x  1)3(x  2) 3x(x  1) 3x 2  3x


c) :   
2x  4 3x  6 2( x  2)4( x  1) 8 8

1 1 ( x  y )2
d) : 2   xy
x  y x  y 2  2xy xy

59 Matemàtiques 1r Batxillerat
SOLUCIONARI UNITAT 2. ÀLGEBRA

Equacions polinòmiques
109. Resol aquestes equacions de primer grau.
x  10 2( x  2) 5x  15
a) 2x  2(3x  1)  4(2x  5)  10  8x d)  
2 5 3

3x  1 1 2x x  2 x  3 1
b) 2x  x e)    2x 
3 3 3 12 2 6

7
2x 
3x  1 4  (3x  1)
c)  2x 
4 2

a) 2x  2(3x  1)  4(2x  5)  10  8x  2x  6x  2  8x  20  10  8x  4x  28  x  7

3x  1 1
b) 2x   x   6x  3x  1  3x  1  0x  0  Tots els nombres reals són solució.
3 3

7
2x 
3x  1 4  (3x  1)  3x  1 8x  4x  7  12x  4  3x  3  x  1
c)  2x 
4 2 2 2 2
x  10 2( x  2) 5x  15
d)    15x  150  12x  24  50x  150  23x  276  x  12
2 5 3

2x x  2 x  3 1
e)    2x   8x  x  2  6x  18  24x  2  11x  22  x  2
3 12 2 6
110. Resol les equacions polinòmiques següents.

a) 4x 2  7x  2  0 e) x 4  x 3  5x 2  x  6  0

b) 7x 2  12x  5  0 f) 6x 3  7x 2  14x  15  0

c) x(2x  1)  3x(x  1)  0 g) x 4  125x 2  484  0

x( x  1) ( x  6)2 ( x  2)2 (3x  2)(3x  4)


d)   
15 5 3 15

7  49  32 7  9 1
a) 4x 2  7x  2  0  x    x  2, x  
8 8 4

12  144  140 12  2 5


b) 7x 2  12x  5  0  x    x  , x 1
14 14 7

c) x  2x  1  3x(x  1)  0  2x 2  x  3x 2  3x  0  x 2  4x  0  x(x  4)  0  x  0 x  4

d) x 2  x  3(x 2  36  12x)  5(x 2  4  4x)  9x 2  18x  8  9x 2  17x  128  9x 2  18x  8  x  120

e) x 4  x 3  5x 2  x  6  0  (x  2)(x  3)(x 2  1)  0  x  2, x  3

 3  5 3 5
f) 6x 3  7x 2  14x  15  0  6( x  1)  x    x    0  x  1, x   , x 
 2  3 2 3

g) x 4  125x 2  484  0 
z  x2  125  15625  1936 125  117 z  121  x  11, x  11
  z  125z  484  0  z   
2

z2  x 4  2 2 z  4  x  2, x  2

60 Matemàtiques 1r Batxillerat
SOLUCIONARI UNITAT 2. ÀLGEBRA

Equacions racionals i irracionals


111. Resol les equacions racionals següents.
12 11x  11 1 1 1 x2  1 x 7
a) 2x   7 d)   g)  2 x
2 x 9 x  a x  a x 2  a2 x x 1 6

4 4 x  1 4x  12
b)  3 e) 
x x2 x  1 3x  3

x  9 5  x 12x  12 x  2 x 1 9
c)   2 f)  
x x2 x  2x x  1 x  2 20

12 11x  11 32
a) 2x   7  9(2  x)2x  108  63(2  x)  (2  x)(11x  11)  7x 2  88x  256  0  x  8, x 
2 x 9 7

4 4 4
b)   3  4(x  2)  4x  3x(x  2)  3x 2  2x  8  0  x  2, x  
x x2 3
x  9 5  x 12x  12
c)   2  (x  9)(x  2)  x(5  x)  12x  12  6x  6  0  x  1
x x2 x  2x

1 1 1 1
d)    x  a  x  a  1  2x  1  x 
x  a x  a x 2  a2 2

x  1 4x  12
e)   ( x  1)(3x  3)  (4x  12)( x  1)  x 2  2x  15  0  x  3, x  5
x  1 3x  3
x  2 x 1 9 14
 20  x  2  20  x  1  9( x  1)(x  2)  9x 2  13x  42  0  x  3, x  
2 2
f)  
x  1 x  2 20 9

x2  1 x 7
g)  2  x   6  x 2  1 x 2  1  6x 2  6x 2  x 2  1  7x( x 2  1) 
x x 1 6

1  22
 7x 3  12x 2  7x  6  0  x  2, x 
7

112. Resol aquestes equacions irracionals.

a) 2  3 x  x c) x  1  2x  3  5 e) x  5  x  7x  3

2 x 4 x 5
b) 3x  2x  2  2 2x  2  23 d) 3 3x  1  2 3(2x  1) f) 
4 x 5 x 5

a) 2  3 x  x  3 x  x  2  9x  x 2  4x  4  x 2  5x  4  0  x  4, x  1

59
b) 3x  2x  2  2 2x  2  23  3x  23  2x  2  9x 2  138x  529  2x  2  x  9, x  (Falsa)
9

c) x  1  2x  3  5  x  1  5  2x  3  x  1  25  2x  3  10 2x  3  10 2x  3  x  27 

 200x  300  x 2  729  54x  x 2  146x  429  0  x  143 (Falsa), x  3

d) 3 3x  1  2 3(2x  1)  9(3x  1)  4  3(2x  1)  3x  3  x  1 (Falsa). L’equació no té solució.

e) x  5  x  7x  3  x  5  x  2 x 2  5x  7x  3  2 x 2  5x  5x  8  4x 2  20x  25x 2  64  80x 

16
 21x 2  100x  64  0  x  4, x  (Falsa)
21

2 x 4 x 5 49
f)   2 x 2  5x  21  x  4x 2  20x  441  x 2  42x  x  9, x   (Falsa)
4 x 5 x 5 3

61 Matemàtiques 1r Batxillerat
SOLUCIONARI UNITAT 2. ÀLGEBRA

Equacions logarítmiques i exponencials


113. Resol les equacions logarítmiques següents.
1
a) 2log  2x  2  log  x  1  1 d) 3logx 2  logx 4  5 g) logx 25  logx 2  log 5
x
5

b) log(65  x 3 )  3log(5  x) e) log 7x  51  1  log9  log 2x  67

c) log10 20x  320


 10 x f) log3  x  1  log3  x  2  1 log3  x  6

(2x  2)2 (2x  2)2 4( x  1)2 7


a) 2log  2x  2  log  x  1  1  log  log10   10   10  4( x  1)  10  x 
x 1 x 1 x 1 2

log(65  x 3 )  3log(5  x )  log(65  x 3 )  log(5  x )3  65  x 3  (5  x )3  65  x 3  125  75x  15x 2  x 3 


b)
 15x 2  75x  60  0  x 2  5x  4  0  x  4, x  1

c) log10 20x  320


 10 x  20x  320  10 x  20x  320  100x  x  4
5
 1 1
d) 3logx 2  logx 4  5  logx (23·4)  5  x 5  32  25     x 
 
2 2

7x  51
7x  51 9 9
e) log 7x  51  1  log9  log 2x  67  log   log  
1010 2x  67
2x  67
669
 14x 2  571x  3417  8100  14x 2  571x  4683  0  x  7, x   (Falsa)
14
x 1 3 x 1 3
f) log3  x  1  log3  x  2  1  log3  x  6   log3  log3    x 2  5x  6  3x  6 
x2 x6 x2 x6
x 2  2x  12  0  x  1  13, x  1  13 (Falsa)

1
log
1 log25 5 log x 2log5 log5 2log x 3log5 2log x
g) logx 25  logx 2  log 5
x  2
      
5 log x log x log 5 log x 2log x log5 2log x log5
3 log5
 3  log5   4 log x   log x 
2 2
 log 5 3
x 5 3

114. Resol aquestes equacions exponencials.


1
 25x 5 d) 3x  2  9x 1  90 g) 9x  2  3x 3  810  0
2
a) 4x

b) 4( x 2)  262144 e) 9x  32x 1  3x 1  111 27  3x  2


2 x
h)
x 5
1
c) 9x  5·3x  24  0 f) 22x  4  5  2x 3  1  0 i) 49 3 
7 x 1

1
1
 25x  5  22( x 1)
 25x  5  2x 2  2  5x  5  2x 2  5x  3  0  x  3, x  
2 2
a) 4x
2

b) 4( x  2)  262 144  4( x 2)  49  ( x  2)2  9 


2 2

xx  22  33xx51
z  3  3 x  3  x  1
c) Aplicant el canvi z  3x  9x  5·3 x  24  0  z 2  5z  24  0  
z  8  3  8 sense solució real
x

d) Aplicant el canvi de variable z  3x

z2 z  9  3 x  9  x  2
3x  2  9x 1  90  9z   90  z2  81z  810  0  
z  90  3  90 sense solució real
x
9

62 Matemàtiques 1r Batxillerat
SOLUCIONARI UNITAT 2. ÀLGEBRA

e) Aplicant el canvi de variable z  3x

z  9  3 x  9  x  2
z2 z 
9 3x 2x 1
3 x 1
 111  z    111  4z  z  333  0  
2 2
37 37
3 3  z  3x   sense solució real
 4 4

f) Aplicant el canvi de variable z  2x

z 2 5z z  8  2 x  8  x  3
22x  4  5  2x  3  1  0    1  0  z2  10z  16  0  
z  2  2  2  x  1
x
16 8

z  3  3 x  3  x  1

g) Amb el canvi z  3  9 x x 2
3 x 3
 810  0  3z  z  30  0  
2
10 10
 z  3x   sense solució real
 3 3
3
3
h) x
27  3x  2  3 x  3 x  2   x  2  x 2  2x  3  0  x  1, x  3
x
x 5 2x 10 1 x
1 2x  10 1  x 23
i) 49 3  7 3 7 2    4x  20  3  3x  7x  23  x 
x 1 3 2 7
7

Sistemes d’equacions
115. Resol i classifica els sistemes següents.

 x 1 2x  3(2x  y )  1
2x  3y  13 y  3 
a)  c)  2 e)  x
5x  2y  4  y  2x  10   3y  14
2

2 7 1 3
 3 x  3y  3 2(2x  y )  3(3x  2y )  34  x  y  1  1
 
b)  d)  x y f) 
1 x  3 y  3   2 2  1  3
 5 2 3  x y  1 2
4 5

2x  3y  13 2x  3y  13
a)    y  3, x  2 x  2, y 3
5 x  2y  4   19y  57

2 7
 3 x  3y  3 2x  9y  7 2x  9y  7 2 1
b)    y  ,x
1 x  3 y  3 4x  15y  12   3y  2 3 2
 5 4 5

 x 1
y  3 x 1 13 4
c)  2   3  2x  10  x   , x 
 2 3 3
 y  2x  10

4x  2y  9x  6y  34 5x  8y  34


d)    7x  14  x  2, y  3
3x  2y  12 3x  2y  12

8x  3y  1 8x  3y  1
e)    8(28  6y )  3y  1  y  5, x  2
 x  6y  28  x  28  6y

a  3b  1
1 1  1 1
f) a ,b   3  a  , b   x  2, y  1
x y  1 2a  b  2 2
 2

63 Matemàtiques 1r Batxillerat
SOLUCIONARI UNITAT 2. ÀLGEBRA

116. Resol els sistemes de dues equacions lineals següents.

 8(x  5) 7x  x y  y
   6  2  y  2 2     8
a)  5 3 b)   3 20  2

2  x  6   9  11  3y 3  x  10   2  y  10   0

 8(x  5) 7x
   6  2  y  2 11x  30y  30  0 70 50
a)  5 3  x ,y
 2x  3y  10  0 9 27
2  x  6   9  11  3y

 x y  y
2     8 10x  9y  120  0
b)   3 20  2   x  30, y  20
3  x  10   2  y  10   0 3x  2y  50  0

117. Indica si els sistemes d’equacions lineals següents són compatibles o incompatibles i calcula’n, segons el
cas, totes les solucions.

 x  3y  2z  6  x  2y  3z  3 2x  y  2z  8  x  3y  2z  6
   
a) 2x  3y  2z  8 b) 3x  2y  z  7 c) 2x  4y  3z  2 d) 2x  3y  5z  6
4x  2y  6z  6 5x  2y  5z  1 4x  y  6z  4 5x  3y  8z  6
   

 x  3y  2z  6  x  3y  2z  6  x  3y  2z  6 x  2
   
a) 2x  3y  2z  8  E2  E2  2E1    3y  2z  4     3y  2z  4  y  2
4x  2y  6z  6 E3  E3  4E1   10y  2z  18 E3  E3  E2   7y  14 z  1
   

Sistema compatible determinat.

 x  2y  3z  3  x  2y  3z  3  x  2y  3z  3
  
b) 3x  2y  z  7  E2  E2  3E1    8y  10z  2     8y  10z  2
5x  2y  5z  1 E3  E3  5E1   8y  10z  14 E3  E3  E2  0  12
  

Sistema incompatible. No té solució.

2x  y  2z  8 2x  y  2z  8 2x  y  2z  8 x  2


   
c) 2x  4y  3z  2  E2  E2  E1    5y  5z  10     5y  5z  10  y  0
4x  y  6z  4 E3  E3  2E1   3y  10z  20 E3  5E3  3E2  35z  70 
   z  2

Sistema compatible determinat.

 x  3y  2z  6  x  3y  2z  6  x  3y  2z  6
  
d) 2x  3y  5z  6  E2  E2  2E1    9y  9z  18     9y  9z  18 
5x  3y  8z  6 E3  E3  5E1   18y  18z  36 E3  E3  2E2  00
  
 x  t
 x  3y  2z  6 
   y  t  2 t  Sistema compatible indeterminat
  y  z  2 z  t

118. Estudia la compatibilitat d’aquests sistemes i troba’n, si és el cas, les solucions.

 x  2y  2z  4  x  2y  z  5 2x  y  z  0  x  3y  2z  4
   
a) 2x  5y  2z  10 b) 5x  y  2z  11 c) 3x  2y  2z  15 d) 2x  2y  z  3
4x  9y  6z  18 6 x  y  z  5 x  y  z  7 3x  2y  z  5
   

64 Matemàtiques 1r Batxillerat
SOLUCIONARI UNITAT 2. ÀLGEBRA

 x  2y  2z  4  x  2y  2z  4  x  2y  2z  4
  
a) 2x  5y  2z  10  E2  E2  2E1   y  2z  2   y  2z  2 
4x  9y  6z  18 E3  E3  4E1  y  2z  2 E  E  E  00
  3 3 2

 x  6t
 x  2y  2z  4 
   y  2  2t t  Sistema compatible indeterminat
 y  2z  2 z  t

 x  2y  z  5  x  2y  z  5  x  2y  z  5
  
b) 5x  y  2z  11  E2  E2  5E1    11y  7z  36     11y  7z  36
6x  y  z  5 E3  E3  6E1   11y  7z  35 E3  E3  E2  0  1
  

Sistema incompatible. No té solució.

2x  y  z  0 2x  y  z  0 2x  y  z  0 x  1


   
c) 3x  2y  2z  15  E2  2E2  3E1   y  7z  30   y  7z  30  y  2
x  y  z  7 E3  2E3  E1  y  3z  14 E3  E3  E2  4z  16 z  4
   

Sistema compatible determinat.

 x  3y  2z  4  x  3y  2z  4  x  3y  2z  4 x  2
   
d) 2x  2y  z  3  E2  E2  2E1    4y  5z  5     4y  5z  5  y  0
3x  2y  z  5 E3  E3  3E1   7y  7z  7 E3  4E3  7E2   7z  7 z  1
   

Sistema compatible determinat.

119. Resol els sistemes d’equacions de segon grau següents.

 y
 x  6y  6 3x  2  15 3x 2  5y 2  20
a)  2 c)  e)  2
2x  y  76 2  3 1 4x  y  4
2 2

 x y

 x 2  2( x  y )2  36
3xy  2x 2  26 2x  4y  10 
b)  d)  2 f)  x y
4x  5y  7  x  3xy  8  2  3  5

 y  2, x  6

a) x  6y  6  2  6y  6   y 2  76  73y 2  144y  4  0  
2
2 450
 y  ,x
 73 73

 65 53
4x  7 4x  7 x   ,y
b) y    3x ·  2x 2  26  22x 2  21x  130  0   22 55
5 5 
 x  2, y  3

 y
3x  2  15 6x  y  30
c)  
2  3 1 2y  3x  xy
 x y

 x  4, y  6

y  30  6x  60  12x  3x  30x  6x 2  2x 2  13x  20  0   5
 x  , y  15
 2

 11
5x 5x  x  16, y  
d) y   x 2  3x( )  8  x 2  15x  16  0   2
2 2 
 x  1, y  3

65 Matemàtiques 1r Batxillerat
SOLUCIONARI UNITAT 2. ÀLGEBRA

e) Multiplicant la segona equació per 5 i sumant-li la primera obtenim:

 x  0, y  2
23x 2  0  x  0  
 x  0, y  2

x y 3
f)   5  3x  2y  30  y  15  x
2 3 2

 162 102
x  ,y
2
5 
x 2  2  x  15   36  23x 2  300x  972  0   23 23
2  
 x  6, y  6

Inequacions i sistemes d’inequacions


120. Resol les inequacions de primer grau següents.
x 1 x  2 x  3 8
a) 3x  3(2x  5)  4(x  2)  2  x c)   
3 4 18 9

x x 1 2x  5 2x  3 x 35
b)   1 d)   2( x  1) 
2 6 2 4 2 4

3  3
a) 3x  3(2x  5)  4( x  2)  2  x  3x  6x  15  4x  8  2  x  6x  9  x   Solució: x   , 
2  2

x x 1 2x  5 5 5 
b)   1  3x  x  1  6  6x  15  8x  20  x   Solució: x   ,   
2 6 2 2 2 

x 1 x  2 x  3 8
c)      12x  12  9x  18  2x  6  32  5x  20  x  4  Solució: x   4,   
3 4 18 9
2x  3 x 35
d)   2(x  1)   2x  3  2x  8x  8  35  40  8x  x  5  Solució: x  5,   
4 2 4

121. Calcula les solucions de les inequacions polinòmiques següents.

a) x 2  x  12  0 e) x 3  4x  0

b) 2x 2  3x  0 f) x 3  3x  2  0

c) 4x 2  1  0 g) x 2  1  0

d) 6x 2  x  1  0 h) x 3  7x  6  0

a) x 2  x  12  0  (x  4)(x  3)  0  Les arrels són x  4 i x  3 .

Construïm la taula de signes corresponent i obtenim que la solució és x   ,  4  3,  

3
b) 2x 2  3x  0  x(2x  3)  0  Les arrels són x  0 i x  .
2

 3
Construïm la taula de signes corresponent i obtenim que la solució és x   0, 
 2
1 1
c) 4x 2  1  0  (2x  1)(2x  1)  0  Les arrels són x   i x .
2 2

 1 1
Construïm la taula de signes corresponent i obtenim que la solució és x    , 
 2 2

66 Matemàtiques 1r Batxillerat
SOLUCIONARI UNITAT 2. ÀLGEBRA

1 1
d) 6x 2  x  1  0  (3x  1)(2x  1)  0  Les arrels són x   i x .
2 3

 1 1
Construïm la taula de signes corresponent i obtenim que la solució és x    , 
 2 3

e) x 3  4x  0  x(x  2)(x  2)  0  Les arrels són x  2 , x  0 i x  2 .

Construïm la taula de signes corresponent i obtenim que la solució és x   ,  2  0, 2

f) x 3  3x  2  0  (x  1)2(x  2)  0  Les arrels són x  1 i x  2 .

Construïm la taula de signes corresponent i obtenim que la solució és x   , 2

g) x 2  1  0  (x  1)(x  1)  0  Les arrels són x  1 i x  1 .

Construïm la taula de signes corresponent i obtenim que la solució és x   ,  1  1,  

h) x 3  7x  6  0  (x  1)(x  2)(x  3)  0  Les arrels són x  3 , x  1 i x  2 .

Construïm la taula de signes corresponent i obtenim que la solució és x   ,  3  1, 2

122. Resol les inequacions racionals següents.


5x  2 x 2  5x  4
a) 0 e) 0
2x  1 x 2  5x  6

3x  1  x  4   x  2
b) 0 f) 0
5  10x  x  12

x2  1 x 8
c) 0 g)  4  x
x2 3x

x3  1 x 2  2x  1
d) 0 h) 1
4  x2 4x

5x  2 1 2
a)  0  Les arrels són x   i x  .
2x  1 2 5

 1 2
Construïm la taula de signes corresponent i obtenim que la solució és: x    , 
 2 5

3x  1 1 1
b)  0  Les arrels són x  i x .
5  10x 3 2

 1 1
Construïm la taula de signes corresponent i obtenim que la solució és: x   , 
3 2

x2  1 ( x  1)( x  1)
c) 0  0  Les arrels són x  2 , x  1 i x  1 .
x2 x2

Construïm la taula de signes corresponent i obtenim que la solució és: x   ,  2   1, 1

x3  1 ( x  1)( x 2  x  1) x 1
d) 0 0  0  Les arrels són x  2 , x  1 i x  2 .
4x 2
(2  x)(2  x) ( x  2)( x  2)

Construïm la taula de signes corresponent i obtenim que la solució és: x   2, 1   2,   

(Observem que x 2  x  1 és sempre positiu)

67 Matemàtiques 1r Batxillerat
SOLUCIONARI UNITAT 2. ÀLGEBRA

x 2  5x  4 ( x  1)( x  4)
e) 0  0  Les arrels són x  1 , x  2 , x  3 i x  4 .
x 2  5x  6 ( x  2)( x  3)

Construïm la taula de signes corresponent i obtenim que la solució és: x   , 1   2, 3   4,   

 x  4   x  2
f)  0  Les arrels són x  2 , x  1 i x  4 .
 x  12

Construïm la taula de signes corresponent i obtenim que la solució és: x   2, 1  1, 4

x 8 x 8 4  x2 ( x  2)( x  2)
g)  4  x  4 x  0  0  0  Les arrels són x  2 , x  2 i x  3 .
3x 3x 3x x 3

Construïm la taula de signes corresponent i obtenim que la solució és: x   ,  2  2, 3

x 2  2x  1 x 2  2x  1 x 2  2x  1  4x x 2  2x  1  x  12
h)  1 1  0  0 0 0 Les arrels són
4x 4x 4x 4x 4x
x  0 i x  1.

Construïm la taula de signes corresponent i obtenim que la solució és: x   0, 1  1,   

123. Resol els sistemes d’inequacions amb una incògnita:

3( x  3)  2x  3
 2x  3  x  3
2x  3( x  1)  7 
a)  c) 
3x  2  x  6 x  5
 x  0

( x  3)2  1  0  x
2x   2
b)  2 d)  2
 x  11x  28  0 
2x  3( x  1)  x  2

2x  3( x  1)  7 5x  10 x  2
a)     Solució: x   , 2
3x  2  x  6 2x  4 x  2
b) Observem que la primera inequació sempre és certa, per tant:

( x  3)2  1  0
 2  ( x  7)( x  4)  0  Solució: x   4, 7
 x  11x  28  0

 12
3( x  3)  2x  3 5x  12 x 
 2x  3  x  3 x  6 5
  
 6  Solució: x   , 5
12
c)    x
x  5 x  5 x 5  5 
 x  0  x  0 
 x 0

 4
 x
2x   2  x  3 5 4
d)  2   Solució: x   , 
2x  3( x  1)  x  2  x  5 4 3
 4

124. Resol els sistemes d’inequacions amb dues incògnites:

x  y  6  2x  y  2 x  4  0 3x  4y  12
   
a) 2y  x b)  x  y  2 c)  x  6  0 d) 3x  4y  4
x  0 x  0 y  0 3  y  5 x  4  0
   

68 Matemàtiques 1r Batxillerat
SOLUCIONARI UNITAT 2. ÀLGEBRA

a) c)

Vèrtexs: A(4, 2) B(0, 0) Vèrtexs: A(4, 3) B(6, 3) C(6, 5) D(4, 5)

b) d)

4 
Vèrtexs: A(0, 0) B(1, 0) C(0, 2) Vèrtexs: A  , 2  B(4, 0) C(4, 4)
3 

Síntesi
125. Opera i simplifica.
1 1
1 1
1 a a2 x ab ab a2 x x 1
a)   c) e)   2 g) :
xy xz yz 1
1 2 a  b a  b a  b2 1
1 x 2
x x 1

x2
x x 2
1
 ( x  1)  2  x xy
 a  b  :  a  b 
2
b)  x 
1 1 x2

2 2
d) f) h)
 2 x  2 y2   x2
y 1
xy x 2

1 a a2 z  ay  a 2 x
a)   
xy xz yz xyz

 ( x  1)2  2  x 2x  x 2  x 2  2x  1 2  x 1 2x 1
b)  x       
 2  x  2 2  x 2 2  x 2 2

1 x 1
1
x ( x  1)x 2 x
c)  2x  
1 x  1 x( x  1)( x  1) x  1
1 2
x x2

x2 x 2  xy  x 2
x
xy xy xy ( x  y ) y  x
d)   
y2 xy  y 2  y 2 xy ( x  y ) y x
y
xy xy

ab ab a2 a2  2ab  b2  a2  2ab  b2  a2 4ab  a2 4ab  a2


e)   2    2
ab ab a b 2
(a  b)(a  b) (a  b)(a  b) a  b2

ba
a2  b2  :  a  b   (a  b)(a  b) :
1 1
f)  (a  b)ab  a2b  ab 2
  ab

1 x 1 ( x  1)( x  2)
1
x x 1 x x 1 x x 1
g) :  :  
1 x  2 x  2 x  2 ( x  1)( x  2) x
1
x 1 x 1 x 1

69 Matemàtiques 1r Batxillerat
SOLUCIONARI UNITAT 2. ÀLGEBRA

x 2 x 2 x 2
1
x2  x2 2x( x  2) x 2  2x
h)  
x2 x 2x 2 4( x  2) 2x  4
1
x 2 x 2

a a

126. Racionalitza, opera i simplifica l’expressió: 2 2 a 2 2 a
2a
1  4a 2

a a
a  2 2 a  
a  2 2 a  2a  2a 2a  2a 4a
2 2 a

2 2 a   2 2 a  2 2 a   2 2 a  2 2 a  2  4a

2  4a  2  4a 
2a 2a 2a 2a
1  4a 2 1  4a 2 1  4a 2 1  4a 2
2a
1  2a 2a(1  2a)(1  2a)
   1  2a
2a 2a(1  2a)
(1  2a)(1  2a)

127. Calcula el terme que s’indica en cadascun dels desenvolupaments següents.


a) El cinquè terme de (2  x)
8

6
2 3
b) El tercer terme de   
3 x
c) L’últim terme de (2a b  3a)
2 37

8
a) T5     24  x 4  70  16  x 4  1120x 4
 4
4 2
 6  2   3  16 9 80
b) T3         15   2 
 2  3   x  81 x 3x 2

 37 
c) T38      3a   450 283 905 890 997 363a37
37

 37 

128. Donat el polinomi P(x )  3x 3  x 2  2x  k , calcula el valor de k perquè sigui divisible per x  2.

Pel teorema del residu tenim: P(2)  0  3  23  22  2  2  k  0  24  k  0  k  24

129. Calcula a i b perquè el polinomi 4x 3  4x 2  ax  b sigui divisible per 2x 2  x  1 . Escriu el quocient de la


di-visió

4x 3  4x 2  ax  b (a  5)x  (b  3) a  5  0
 2x  3    a  5 b  3
2x 2  x  1 2x 2  x  1 b  3  0

130. Sigui el polinomi P(x )  2x 4  2x 3  11x 2  ax  b . Calcula el valor de a i b perquè sigui divisible per
x2  x  6 .

Es fa la divisió i s’obté: quocient: 2x 2  23 i residu:  23  a  x   b  138 . Per tant:

23  a  0
  a  23, b  138
b  138  0

70 Matemàtiques 1r Batxillerat
SOLUCIONARI UNITAT 2. ÀLGEBRA

131. Sigui el polinomi P(x )  2x 5  2x 4  3x 3  3x 2  ax  b , troba el valor de a i de b perquè:

 El polinomi sigui divisible per x  1 .


 El valor numèric per a x  1 sigui 12.

Pel teorema del residu tenim P(1)  0, d’altra banda, P( 1)  12, per tant:

P(1)  0 2  2  3  3  a  b  0 a  b  0
    a  1, b  1
P(1)  12 2  2  3  3  a  b  12 a  b  2

132. Calcula l’expressió de P ( x ) sabent que P(2x  1)  8x 2  14x .

P(x) és un polinomi de segon grau. Podem escriure: P(x)  ax 2  bx  c .

Tenim que:

P(2x  1)  a  2x  1  b(2x  1)  c  a(4x 2  4x  1)  2bx  b  c  4ax 2  (4a  2b)x  (a  b  c )


2

Per tant:

4a  8

4a  2b  14  a  2, b  3, c  5  P( x )  2x  3x  5
2

a  b  c  0

2
2x 1
133. Resol l’equació següent: 13 x  0
13

2
 2x
13x  131  x 2  2x  1  x 2  2x  1  0  x  1

134. Troba l’expressió d’un polinomi de tercer grau que compleixi que:
P(0)  0 P(1)  0 P(1)  2 P(2)  6

Sigui P(x)  ax 3  bx 2  cx  d el polinomi buscat. Tenim que:

P(0)  d d  0 d  0
P(1)  a  b  c  d a  b  c  d  0 a  b  c  0
  
  
 P ( 1)  a  b  c  d  a  b  c  d  2 a  b  c  2
P(2)  8a  4b  2c  d 8a  4b  2c  d  6 8a  4b  2c  6

Sumant la segona equació amb la tercera obtenim: 2b  2  b  1 .

Restant a la quarta equació el doble de la tercera obtenim: 6a  2b  10  6a  2  10  a  2 .

Finalment, s’aïlla a la segona equació i s’obté: c  3 .

135. a) Compara les solucions de l’equació de segon grau 3x 2  4x  4  0 amb les de l’equació 4x 2  4x  3  0

b) Demostra que les solucions de l’equació x 2  bx  2  0 són inverses de les de l’equació 2x 2  bx  1  0 .

c) Demostra que les solucions de l’equació ax 2  bx  c  0 són inverses de les de l’equació cx 2  bx  a  0 .

48 2 48 3 1
a) 3x 2  4x  4  0  x   x  2, x   ; 4x 2  4x  3  0  x  x ,x
6 3 8 2 2
Les solucions d’una equació són inverses de les de l’altra.

71 Matemàtiques 1r Batxillerat
SOLUCIONARI UNITAT 2. ÀLGEBRA

b  b2  8 b  b2  8
b) x 2  bx  2  0  x  ; 2x 2  bx  1  0  x 
2 4

b  b2  8 b  b2  8  b    b  8  b2  b2  8 8
2 2

    1
2 4 8 8 8

b  b2  8 b  b2  8  b    b  8  b2  b2  8 8
2 2

De la mateixa manera:     1
2 4 8 8 8
Les solucions d’una equació són inverses de les de l’altra.

b  b2  4ab b  b2  4ac
c) ax 2  bx  2  0  x  ; cx 2  bx  a  0  x 
2a 2c

b  b2  4ac b  b2  4ac  b    b  4ac  4ac


2 2

   1
2a 2c 4ac 4ac
Les solucions són inverses una de l’altra.
I de la mateixa manera es comprova l’altra parella de solucions.

136. a) L’equació polinòmica de tercer grau ax 3  bx 2  cx  d  0 té per arrels x1 , x2 i x3 . Calcula, en funció de a, b,


c i d, els valors de:
i) x1  x2  x3 ii) x1x2  x1x3  x2 x3 iii) x1x2 x3

b) Calcula tres nombres tals que la seva suma sigui 3, la suma dels tres productes de dos d’aquests sigui −6 i el
producte dels tres sigui −8.

a) ax 3  bx 2  cx  d  a(x  x1)(x  x2 )(x  x3 )  a  x 3  (x1  x2  x3 )x 2  (x1x2  x1x3  x2x3 )x  x1x2x3 

b c d
Igualant els coeficients obtenim: x1  x2  x3   , x1x2  x1x3  x2 x3  i x1x2 x3   .
a a a
b) A partir de l’apartat anterior, sigui a  1 , els nombres buscats són les solucions de l’equació de tercer grau
x 3  3x 2  6x  8  0 , per tant, els nombres són x1  1, x2  4 i x3  2 .

137. Calcula el conjunt de nombres reals que compleixen la condició següent: x  2  x  6 . Per fer-ho:

a) Calcula el punt que equidista de 2 i de 6.


b) Raona quins són els punts que es troben més a prop de 2 que de 6.

a) El punt que equidista de 2 i de 6 és 4.

b) x  2 representa la distància del punt x al punt 2 i x  6 representa la distància del punt x al punt 6.

Per tant, es busquen els punts la distància dels quals a 2 sigui més petita que la seva distància a 6. Aquests
són els punts inferiors a 4, és a dir, la solució és x   , 4

QÜESTIONS
138. Per a cada cas, escriu una equació de segon grau que compleixi:
a) Que no tingui cap solució real.
b) Que tingui una sola solució real doble.
c) Que la suma de les arrels sigui 7 i el producte −60.
d) Que el producte de les arrels sigui el doble que la seva suma.
a) x  1  0 b) x  2x  1  0 c) x  7x  60  0 d) x  x  2  0
2 2 2 2

72 Matemàtiques 1r Batxillerat
SOLUCIONARI UNITAT 2. ÀLGEBRA

139. Escriu dos polinomis diferents que tinguin les mateixes arrels:
a) Si són del mateix grau.
b) Si tenen diferent grau.

a) P(x)  x 2  4, Q(x)  2x 2  8 b) P(x)  x, Q(x)  x 3  x

140. Demostra que l’equació x 2  (2  1)x    0 té dues solucions reals diferents per a qualsevol valor del pa-
ràmetre  .

El discriminant de l’equació és   b2  4ac  (2  1)2  4  4 2  1 .

És sempre estrictament positiu, per tant, l’equació té dues solucions reals diferents.

141. Donada l’equació de segon grau x 2  (2  2)x    0 depenent del paràmetre  , es demana:

a) Calcular els valors de  perquè l’equació tingui dues solucions reals diferents.

b) Calcular els valors de  perquè l’equació tingui una única solució real doble.

a) Calcular els valors de  perquè l’equació no tingui cap solució real.

El discriminant de l’equació és   b2  4ac  (2  2)2  4 2  8  4 . Per tant:

1
a) Si    l’equació té dues solucions reals.
2

1
b) Si    l’equació té una única solució real doble.
2
1
c) Si    l’equació no té solució real.
2

142. Un sistema de dues equacions i tres incògnites.


a) Pot ser incompatible? Posa’n un exemple.
b) Pot ser compatible indeterminat? Posa’n un exemple.
c) Pot ser compatible determinat? Posa’n un exemple.

x  y  z  1 x  y  z  1
a)  b)  c) No pot ser compatible determinat.
x  y  z  0  x  y  2z  1

143. a) A l’hora de resoldre una equació racional en la qual la incògnita està en un denominador, pot obtenir-se
alguna solució falsa. Quina és la raó?
b) A l’hora de resoldre una equació irracional pot obtenir-se alguna solució falsa. Quina és la raó?

a) Quan es treuen denominadors pot ser que s’estiguin introduint solucions que anul·len aquest denominador i
siguin falses perquè la divisió no està definida entre zero.
b) Quan s’eleven al quadrat els dos membres d’una equació, es poden introduir solucions falses ja que aquests
membres podrien ser iguals en valor absolut però amb signe diferent.

73 Matemàtiques 1r Batxillerat
SOLUCIONARI UNITAT 2. ÀLGEBRA

144. Amb l’ajuda del desenvolupament del binomi (1  1)n , demostra que es compleix la igualtat:

 n  n  n  n
 0    1    2   ...   n   2
n

       

n n n n n n n n


2n  1  1    1n  10    1n 1  1    1n 2  12  ...    10  1n           ...   
n

0  1  2 n  0  1   2  n

2
145. a) Quines són les condicions que han de satisfer els coeficients b i c del polinomi P(x) = x + bx + c perquè sigui un
quadrat perfecte.
b) Quines són les condicions que han de satisfer els coeficients a, b i c del polinomi P(x)  ax  bx  c perquè
2

sigui un quadrat perfecte?

2
 b
a) P(x) ha de tenir una única solució real. Per tant, b2  4c  0 i P ( x )   x  
 2
2
 b 
2
 b a 
b) P(x) ha de tenir una única solució real. Per tant, b2  4ac  0 i P ( x )  a  x     ax  
 2a   2a 

146. Demostra aquesta igualtat algebraica.

3x  2y 2  3z 
2
 9x 2  12xy 2  18xz  4y 4  12y 2z  9z 2

 3x  2y  3z    3x  2y 2  3z  3x  2y 2  3z   3x  3x  2y 2  3z   2y 2  3x  2y 2  3z   3z 3x  2y 2  3z  
2 2

 9x 2  6xy 2  9xz  6xy 2  4y 4  6y 2z  9xz  9y 2z  9z 2  9x 2  12xy 2  18xz  4y 4  12y 2z  9z 2

147. Demostra que es compleix: log x A  log x 2 A2

log A2 2log A log A


logx 2 A2     logx A
log x 2 2log x log x

PROBLEMES
148. Si es divideix un nombre per 5 i per 13 i se sumen els quocients, el resultat és 72. Troba aquest nombre.

x x 4680
Sigui x el nombre desconegut:   72  13 x  5 x  4680  x   260
5 13 18

149. Si sumem quatre nombres senars consecutius obtenim 72 com a resultat d’aquesta suma. Quins són
aquests nombres?

Siguin x, x  2, x  4, x  6 els nombres buscats. Tenim que:


60
x  x  2  x  4  x  6  72  4x  12  72  x   15 . Per tant, els nombres buscats són: 15, 17, 19 i 21.
4

74 Matemàtiques 1r Batxillerat
SOLUCIONARI UNITAT 2. ÀLGEBRA

150. La suma d’un nombre positiu més el valor de la seva arrel quadrada coincideix amb el triple del nombre. De
quin nombre es tracta?

x  0

Sigui x el nombre desconegut: x  x  3 x  x  2x  x  4 x 2  x(4 x  1)  0   1
 x
 4

151. La suma d’un nombre de dues xifres més el que resulta d’invertir-les és 99. Quant val la suma de les dues
xifres d’aquest nombre?

Sigui  xy 10)  10x  y el nombre desconegut. El nombre invertit serà  yx 10)  10y  x .

Tenim que: 10x  y  10y  x  99  11x  11y  99  x  y  9  Les dues xifres sumen 9.

152. a) Troba un nombre de tres xifres tals que la suma dels quadrats de les dues xifres extremes és 65 i que si
a aquest nombre se li afegeixen 297 unitats, el nombre que en resulta s’escriu al revés del que estem
buscant.
b) Quants nombres d’aquesta mena hi ha?

a) Suposem que el nombre buscat és  xyz 10)  100x  10y  z tenim:

 x  4
 x 2  z 2  65  x 2  z 2  65 z  7  
    x  4 (No vàlida)
100 x  10y  z  297  100z  10y  x  x  z  3 z  4 (No vàlida)

Per tant, un nombre que compleix les condicions és el 407.
b) El resultat no depèn de y, per tant, hi ha 10 solucions possibles: 407, 417, 427, 437, 447, 457, 467, 477, 487 i
497.
153. La suma de tres nombres parells consecutius és 1242. Quins són aquests nombres?
Nombres: x, x 2, x 4
x  x  2  x  4  1242  3x  1236  x  412 . Els nombres són: 412, 414 i 416.

154. D’un nombre de tres xifres sabem que la seva suma és 12, que la xifra de les unitats és igual a la semisuma
de les xifres de les centenes i de les desenes i, per acabar, que el nombre que resulta quan s’inverteixen
les xifres del primer és 198 unitats més petit que aquest. De quin nombre es tracta?

Suposem que el nombre buscat és  xyz10)  100x  10y  z i que, per tant, el nombre que resulta d’invertir-ne les

xifres és  zyx 10)  100z  10y  x . Aleshores, podem escriure:

 x  y  z  12  x  y  z  12  x  6
  
 x  y  2z  0   x  y  2z  0  y  2  El nombre buscat és el 624.
99 x  99z  198  x  z  2 z  4
  

155. Es consideren els nombres A  u  v , B  u  v i C  2uv on u i v són nombres enters positius.


2 2 2 2

a) Comprova que per a qualsevol valor de u i v, els nombres A, B i C formen una terna pitagòrica, és a dir, que A,
B i C poden ser els costats d’un triangle rectangle. Quins són els catets i quin la hipotenusa?
b) Amb l’ajuda de l’apartat anterior, calcula els costats d’un triangle rectangle sabent que les seves longituds són
nombres enters que sumen 90 i que la suma de les longituds dels seus catets val 49.
c) Calcula tres nombres enters que formen una terna pitagòrica, que sumen 132 i tals que la hipotenusa mesura
una unitat més que un dels catets.

75 Matemàtiques 1r Batxillerat
SOLUCIONARI UNITAT 2. ÀLGEBRA

a) La hipotenusa ha de ser A, el nombre més gran, i els catets B i C.


Comprovem que compleix el teorema de Pitàgores:

A2  (u 2  v 2 )2  u 4  v 4  2u 2v 2  u 4  v 4  2u 2v 2  4u 2v 2  (u 2  v 2 )2   2uv   B2  C 2
2

b) La hipotenusa val 90  49  41.


41  2516  5 4  u  5, v  4  A  41, B  9 i C  40
2 2

 A  B  C  132
 
A  B 1 
u  v  u  v  2uv  132
2 2 2 2
2u 2  2uv  132
 u 2  uv  66
 u 2  uv  66
c)   2    
u  v  2uv  1 2uv  u  v  1  (u  v )  1 u  1  v
2 2 2 2
 
 u  6, v  5  A  61, B  11 i C  60

156. Un pare té 48 anys i el seu fill, 15. Quants anys han de passar perquè l’edat del pare sigui just el doble que
la del fill?

Siguin x els anys que han de passar: 48  x  2(15  x )  48  x  30  2x  x  18

D’aquí 18 anys, l’edat del pare serà el doble que la del fill.

157. Les bases d’un trapezi mesuren 10 i 20 cm, respectivament, i l’altura, 8. Calcula l’altura del triangle que re-
sulta quan es prolonguen els dos costats no paral·lels del trapezi.

(20  10)  8
La superfície del trapezi és: ST   120 cm2
2

10 x
La superfície del triangle CED és: SCED   5x
2

20( x  8)
La superfície del triangle ABC és: SABC   10 x  80
2
40
Per tant, tenim que: 10 x  80  5 x  120  5 x  120  80  40  x  8
5
L’altura del triangle ABC és 8  8  16 cm

158. Els catets d’un triangle rectangle mesuren 27 i 36 cm. Prenent com a centre cadascun dels vèrtexs del tri-
angle es fan tres circumferències de forma que són tangents exteriors dos a dos.

Calcula els radis de les tres circumferències.

El valor de la hipotenusa és 362  272  45 cm, per tant:

r1  r3  27 r1  18 cm
 
2 3
r  r  36  r2  27 cm
r  r  45 
1 2 r3  9 cm

76 Matemàtiques 1r Batxillerat
SOLUCIONARI UNITAT 2. ÀLGEBRA

159. Troba una fracció tal que es compleixi que si al numerador i al denominador se’ls suma una unitat, la frac-
1 1
ció equival a , i si se’ls resten 3 unitats, la fracció equival a .
3 5

Anomenem x el numerador i y el denominador.

x 1 1
 y  1  3 3 x  3  y  1 3 x  y  2
    x  7, y  23
x 3  1 5 x  15  y  3 5 x  y  12
 y  3 5

7
La fracció és .
23

160. En un hotel turístic tenen un total de 36 habitacions amb 60 llits. Només hi ha habitacions individuals i do-
bles. Calcula el nombre d’habitacions de cada mena que hi ha.

Sigui x el nombre d’habitacions individuals i y el de dobles.

 x  y  36
  x  12, y  24 . Hi ha 12 habitacions individuals i 24 habitacions dobles.
 x  2y  60

161. Fa cinc anys, l’edat d’una mare era el triple que la del seu fill, i d’aquí deu només serà el doble. Troba les
edats que tenen tots dos ara.

Siguin 3x i x les edats de fa 5 anys.


Les edats actuals són 3x  5 i x  5, i les edats d’aquí 10 anys són 3x  15 i x  15.
Per tant, tenim: 3x  15  2( x  15)  x  15

L’edat actual de la mare és de 50 anys, i la del fill, de 20.

162. Un grup d’amics ha de pagar un total de 360 € per una reserva per un cap de setmana en una casa rural. En
l’últim moment, quatre dels amics confirmen que no hi podran anar; per aquesta raó, cadascun de la resta
del grup haurà de pagar 4,5 € més per la reserva. Quants amics gaudiran el cap de setmana a la casa rural?
Quants diners hi posarà cadascun dels amics?

Sigui x el nombre d’amics inicial i y els diners que aporta, al principi, cadascun:

 xy  360  xy  360  x  20
  
( x  4)( y  4,5)  360 4,5 x  4y  18 y  18

Finalment, van 16 amics a la casa rural i cada un paga 22,5 €.

163. Les dues cordes paral·leles dibuixades a la circumferència de la figura mesuren 12 i 16 cm. La distància
entre elles és de 2 cm.

Troba el radi de la circumferència

77 Matemàtiques 1r Batxillerat
SOLUCIONARI UNITAT 2. ÀLGEBRA

Apliquem el teorema de Pitàgores:

82  x 2  r 2 

 64  x 2  36  4  x 2  4 x  4 x  24  x  6 cm .
2
62   2  x 
2
r 

Per tant, r 2  64  36  100  r  10 cm .

2
164. Si es disminueix en 10 cm el costat d’un quadrat, la seva àrea disminueix en 400 cm . Quina és la mida ori-
ginal del quadrat?

2
Sigui x el costat del quadrat original. La seva àrea és x .

Tenim: ( x  10)2  x 2  400  x 2  20x  100  x 2  400  20x  500  x  25 cm .

165. Dues ciutats A i B, que estan unides per una carretera, es troben a 111 km de distància una de l’altra. Des
de la ciutat A surt cap a B un cotxe a 72 km/h i des de B cap a A, en surt un altre a 76 km/h. Quant trigaran
a trobar-se?

Si els cotxes tarden t hores a trobar-se, en aquestes t hores el primer cotxe haurà recorregut x km i el segon 111 
x. per tant:
x 111  x 7992
  76 x  7992  72x  148 x  7992  x   54 km .
72 76 148

54
El temps que tardaran a trobar-se serà:  0,75 hores  45 minuts.
72
166. Des d’un punt A surt una motocicleta a 72 km/h. Deu minuts després, des del mateix punt i darrere d’a-
questa, surt un cotxe a velocitat constant de 90 km/h. Quant trigarà a atrapar la motocicleta?

 1 2
Si tarda t hores a atrapar-la tenim: 72  t    90t  72t  12  90t  t  hores  40 minuts.
 6  3

167. Es consideren tres barres homogènies de metall compostes de la forma següent:


Primera barra: 30 g d’or, 45 de plata i 75 de coure.
Segona barra: 60 g d’or, 30 de plata i 135 de coure.
Tercera barra: 45 g d’or, 60 de plata i 75 de coure.
Quina quantitat cal prendre de cadascuna de les barres per aconseguir-ne una altra que tingui 64,5 g d’or,
69 de plata i 136,5 de coure?

30 2 45 3 75 5
A la primera barra es compleix que  és or,  és plata i  és coure.
150 10 150 10 150 10
60 4 30 2 135 9
A la segona barra es compleix que  és or,  és plata i  és coure.
225 15 225 15 225 15

45 3 60 4 75 5
A la tercera barra es compleix que  és or,  és plata i  és coure.
180 12 180 12 180 12
Suposem que agafem x g de la primera barra, y de la segona i z de la tercera. Podem escriure el sistema d’equa-
cions lineals:

2 4 3
 10 x  15 y  12 z  64,5
 12 x  16 y  15z  3870  x  90
3 2 4  
 x  y  z  69  18 x  8 y  20 z  4140   y  90
 10 15 12 30 x  36 y  25z  8190 z  90
5 9 5  
 10 x  y  z  136,5
 15 12

Per tant, s’hauran d’agafar 90 g de cada barra.

78 Matemàtiques 1r Batxillerat
SOLUCIONARI UNITAT 2. ÀLGEBRA

168. Troba tres enters consecutius tals que sumant els quadrats dels dos primers s’obté el quadrat del tercer.

Siguin x, x  1 i x 2 els nombres buscats:

x 2  ( x  1)2  ( x  2)2  x 2  x 2  2x  1  x 2  4x  4  x 2  2x  3  0  x  1, x  3

Per tant hi ha dues possibles solucions: 1, 0 i 1 o 3, 4 i 5.

169. Se sap que una certa població d’insectes s’incrementa en un 9 % cada setmana. Calcula el temps que ha
de passar perquè la població es multipliqui per cinc.

Sigui P el nombre inicial d’insectes. Al cap d’una setmana es tindran P ·1,09. Al cap de t setmanes es tindran
t
P ·1,09 insectes. Per tant:
log5
5P  P  1,09t  1,09t  5  t   18,676 setmanes 131 dies.
log1,09

170. L’àrea d’un rectangle és de 35 unitats quadrades. Si s’augmenten dos costats paral·lels en 2 unitats i es
disminueixen els altres dos en 3 unitats, l’àrea disminueix en 17 unitats quadrades. Troba les dimensions
del rectangle inicial.

Siguin x, y les dimensions inicials.

 xy  35  xy  35  11  3x 
   x   35  3x  11x  70  0  x  7 , (solució vàlida)
2

 x  2  y  3   35  17  3 x  2y  11  2 

10
x solució no vàlida. Si x  7 , aleshores y  5 . Per tant, les dimensions són 7 i 5 cm.
3

171. En un concurs de matemàtiques es proposa una prova de 25 preguntes. Cada una de les preguntes té 5
possibles respostes de les quals només una és certa. Per cada resposta encertada s’obtenen 5 punts; si es
respon de forma errònia s’obtenen 0 punts, i si es deixa una pregunta sense resposta, se n’obtenen 2.
a) Escriu l’expressió algebraica que determina la puntuació d’un concursant, anomenant x el nombre de respostes
encertades i y el nombre de respostes incorrectes.
b) Si d’un concursant se sap que ha obtingut 80 punts, com pot deduir-se el nombre de respostes encertades, er-
rònies i no contestades? Dona’n dos possibles exemples.
c) En dues de les preguntes no contestades, aquest mateix concursant dubtava entre dues de les cinc opcions.
Quina puntuació hauria obtingut en el cas d’haver-les contestat de manera correcta?

a) L’expressió algebraica que dona la puntuació és: P( x, y )  5x  2  25  x  y   5x  50  2x  2y  3x  2y  50 .

b) Si el concursant ha obtingut 80 punts, tenim que:


3 x  30 3x
3x  2y  50  80  3 x  2y  30  y  y   15
2 2
Dos possibles exemples poden ser:
x  10  y  0 , és a dir, encerta 10 preguntes i no contesta cap de les altres 15 preguntes.

x  12  y  3 , és a dir, encerta 12 preguntes, en falla 3 i no en contesta 10.

c) Hauria obtingut 6 punts més, és a dir, 86 punts.

79 Matemàtiques 1r Batxillerat
SOLUCIONARI UNITAT 2. ÀLGEBRA

172. S’ha comprat un cert nombre de discos DVD verges per una quantitat total de 17,25 €. Si s’haguessin com-
prat discos d’una qualitat superior, que valen 0,40 € més per unitat, se n’haurien d’haver comprat 8 menys
perquè no canviés el preu total. Quants discos s’han comprat?

17,25
Sigui x el nombre de discos adquirits. El preu de cada un és de euros.
x

 17,25 
Si es compressin discos d’una qualitat superior, el preu de cada disc seria   0,4  .
 x 
Per tant, tenim que:

 17,25  138  x  23
  0,4   x  8   17,25  17,25  0,4 x   3,2  17,25  0,4 x 2  3,2 x  138  0  
 x  x  x  15 (No vàlida)

S’han comprat 23 discos.

173. Per participar en les properes competicions locals d’atletisme s’han de passar dues proves. En la primera
s’elimina el 60 % dels participants, i en la segona, les dues terceres parts dels que queden.
Quants participants es van apuntar en un principi si després de les dues proves queden 10 atletes per
competir a la final?

Sigui x el nombre de participants inicials.


0, 4 x
Després de la primera prova queden 0,4x. Després de la segona prova queden .
3

0,4 x
Per tant,  10  x  75 .
3
S’hi van apuntar 75 participants.

174. Un comerciant compra dos tipus de cafè per torrar, moldre i, posteriorment, barrejar. El de més qualitat té
un preu de 9 €/kg, mentre que per l’altre va pagar 7,50 € per cada quilo. El comerciant vol aconseguir una
barreja que surti al preu de 8,40 €/kg.
Quina haurà de ser la proporció dels dos tipus de cafè?

Sigui x la quantitat de cafè de més qualitat en un kg de mescla i y la quantitat de l’altre cafè en un kg de mescla,
tenim:

x  y  1  x  0,6
 
9 x  7,5y  8,4 y  0,4

Per tant, s’han de mesclar 0,6 kg de cafè de més qualitat per cada 0,4 kg de l’altre, és a dir, s’han de mesclar 3 kg
de cafè de més qualitat per cada 2 kg de l’altre.

175. Un comerciant a l’engròs treballa amb tres classes de televisors. Cada televisor de la primera classe li cos-
ta 180 €, el de la segona classe, 90 €, i el de la tercera, 30 €. Una comanda de 105 unitats té un import total
de 9600 €. Determina el nombre de televisors demanats de cada classe sabent que el nombre de televisors
de la segona classe és el doble que els de la primera i la tercera junts.

Siguin x el nombre de televisors del primer tipus i y el nombre de televisors del tercer tipus. El nombre de televisors
del segon tipus és 2(x  y).
Per tant:

x  2(x  y)  y  105 3 x  3 y  105  x  y  35  x  15


   
180 x  90·2( x  y )  30y  9600 360 x  210y  9600 12x  7y  32 y  20

Es van demanar 15 televisors del primer tipus, 70 del segon i 20 del tercer.

80 Matemàtiques 1r Batxillerat
SOLUCIONARI UNITAT 2. ÀLGEBRA

176. Entre les 6 h i les 9 h de la tarda la velocitat mitjana contracta da de baixada d’arxius disminueix en
80 kB/s, mentre que de 6 h a 9 h del matí, aquesta velocitat mitjana es veu incrementada en 100 kB/s.
L’Andreu ha baixat un arxiu en dues etapes. Al matí ha baixat la primera part de l’arxiu de 12 400 kB i a la
tarda ha baixat els altres 11 264 kB. En total, la baixada de l’arxiu complet ha necessitat exactament un
minut. Quina és la velocitat mitjana de baixada contractada?

Sigui v kB/s la velocitat mitja de baixada contractada. Es baixen 12400 kB amb una velocitat de v  100 i 11264 kB
amb velocitat v  80. El temps total invertit és de 60 segons, per tant:
12400 11264 22464  25536
  60  60v 2  22464v  614400  0  v   11232  12768
v  100 v  80 2
L’única solució que té sentit és v  11232  12768  24000 kB/s.

177. Un ciclista està fent un trajecte a favor del vent. En un primer tram, el vent l’ajuda a raó de 1 km/h, i en un
segon tram l’ajuda a raó de 2 km/h. El ciclista porta una velocitat pròpia constant en tot el recorregut i triga
2 hores i 36 minuts a fer els 40 km. Més tard, en un mapa topogràfic, el ciclista observa que els trams estan
en proporció 3 a 2. Calcula la velocitat pròpia del ciclista.

Sigui x la velocitat del ciclista. Siguin y, 40  y les longituds dels trams . El temps que el ciclista tarda a recórrer el
y 40  y
total del trajecte és:   2,6 .
x 1 x  2

y 3
La relació dels trams és:   2y  120  3y  y  24 .
40  y 2

24 16 32,2  40,6
Per tant,   2,6  2,6 x 2  32,2 x  58,8  0  x  .
x 1 x  2 5,2

32,2  40,6
L’única solució que té sentit és x   14 km/h.
5,2

PER APROFUNDIR
178. Factoritza els polinomis següents.

a) x  y  2xy  z d) x  y
2 2 2 3 3

b) 4x  9y  4z  12yz e) x  y
2 2 2 3 3

c) 4  9x  25y  30xy
2 2

a) x  y  2xy  z  ( x  y )  z  ( x  y  z)( x  y  z)
2 2 2 2 2

b) 4x  9y  4z  12yz  4x  (3y  2z)  (2x  3y  2z)(2x  3y  2z)


2 2 2 2 2

c) 4  9x  25y  30xy  4  (3x  5y )  (2  3x  5y )(2  3x  5y )


2 2 2

d) x  y  ( x  y )( x  xy  y )
3 3 2 2

e) x  y  ( x  y )( x  xy  y )
3 3 2 2

81 Matemàtiques 1r Batxillerat
SOLUCIONARI UNITAT 2. ÀLGEBRA

179. Factoritza el polinomi x 4  2 x 3  4 x 2  3x  2 sabent que és el producte de dos polinomis irreductibles de


segon grau i que el coeficient de primer grau del primer dels factors és 1.

x 4  2x 3  4x 2  3x  2  ( x 2  x  a)( x 2  bx  c )  x 4  (b  1)x 3  (a  b  c )x 2  (c  ab)x  ac

b  1  2
a  b  c  4 a  2
 
  b  1  P ( x )  ( x 2  x  1)( x 2  2x  1)
c  ab  3 c  1

ac  2

180. Calcula el valor de k perquè el sistema d’equacions següent tingui infinites solucions. Per a aquest valor,
escriu les solucions.

 x  2y  2z  4

2x  5y  2z  10
 4x  9y  6z  k

 x  2y  2z  4  x  2y  2z  4
 
Apliquem el mètode de Gauss:  y  2z  2   y  2z  2
 y  2z  k  16 0 z  k  18
 

Per a k 18, s’obté l’equació 0 · z 0, que es compleix per a qualsevol valor de z.

 x  4  2 λ  2(2  2λ )  6λ

Les infinites solucions del sistema venen donades per les fórmules:  y  2  2 λ
z  λ

 x  2y  3z  3

181 Calcula els valors de k perquè el sistema següent sigui incompatible: 3x  2y  z  7 .
5x  2y  5z  k

 x  2y  3z  3  x  2y  3z  3
 
Apliquem el mètode de Gauss: 8y  10z  2  8y  10z  2
8y  10z  k  15 0 z  k  13
 

Quan k 13, l’última equació no té sentit i, per tant, el sistema no té solució.

182. Aplicant el mètode de Gauss, estudia i resol el sistema de quatre equacions lineals amb quatre incògnites
següent.

 x  3 y  2z  2w  12
2 x  2 y  z  w  5


3 x  y  2z  4w  16
3 x  3z  3w  15

 x  3y  2z  2w  12  x  3y  2z  2w  12  x  3y  2z  2w  12
2 x  2 y  z  w  5   8y  3z  3w  19   8y  3z  3w  19
 E2  E2  2E1  
    
3 x  y  2z  4w  16 E3  E3  3E1  8 y  4 z  10w  20 E3  E3  E2 z  7w  1
  
 E  E  3E  E  8E  9E 
3 x  3z  3w  15 4 4 1
  9y  3z  9w  21 4 4 2
  3z  45w  3

 x  3y  2z  2w  12 x  4
  8y  3z  3w  19 y  2
 
  
 z  7w  1 z  1
E4  E4  3E3  
  66w  0 w  0

82 Matemàtiques 1r Batxillerat
SOLUCIONARI UNITAT 2. ÀLGEBRA

ENTORN MATEMÀTIC
Àlgebra perquè que el bebè estigui content
La Maria i l’Antoni estan d’enhorabona, han tingut una nena! Però, és clar, les coses al començament no són
fàcils. La Martina no para de plorar! Es passa el dia i la nit cridant i només quan està exhausta aconsegueix dor-
mir unes dues hores; després, un altre cop els plors. La veritat és que la Maria i l’Antoni comencen a estar des-
esperats. Un bon dia, un amic els comenta que a ell li va passar el mateix amb el seu fill i que ho va solucionar
només posant-li a la seva habitació un mòbil de sostre. Però han de tenir molt de compte que estigui perfec-
tament equilibrat perquè la nena deixi de plorar. Sembla mentida, tan petits i tan exigents!
Decideixen provar la possible solució i el manetes de l’Antoni es disposa a dissenyar un mòbil sonor. La Maria,
que és enginyera, li explica que perquè dues masses M i N penjades en els extrems d’una vareta de massa
negligible estiguin en equilibri, s’ha de verificar la igualtat: xM  yN

Amb aquesta informació, l’Antoni dissenya un mòbil amb cinc figures.

a) Calcula els valors de x, y, z i a perquè el mòbil que ha dissenyat l’Antoni estigui en equilibri.
b) Com que el disseny inicial sembla difícil de fer, l’Antoni decideix simplificar-lo i fer-ne un de tres figures. Per a aquest
mòbil, calcula els possibles valors de x i y que es poden posar tenint en compte que han de ser nombres enters.

a) Imposem les condicions per a l’equilibri i obtenim el sistema:

2  80  az az  160  x  75 g
3 80  z  4 x 4 x  3z  240 
     y  30 g
  
2y  3  20  y  30 z  20 g
2  x  80  z   7  y  20  2x  2z  7y  20 a  8 dm

b) Imposem les condicions per a l’equilibri i obtenim el sistema:

5·10  x  2y
  x  2y  50  x  50  2y
2yx  2yx

Com que x i y han de ser enters positius, hi ha 24 possibles solucions:


x 48 46 44 42 40 38 36 34 32 30 28 26
y 1 2 3 4 5 6 7 8 9 10 11 12

x 24 22 20 18 16 14 12 10 8 6 4 2
y 13 14 15 16 17 18 19 20 21 22 23 24

83 Matemàtiques 1r Batxillerat
SOLUCIONARI UNITAT 2. ÀLGEBRA

Les capses
Un cop resolt el problema, la Maria i l’Antoni volen construir també un joc de capses per guardar les joguines de
la Martina. Per fer-ho, compren cartolines de 84 cm × 56 cm. Per construir cada capsa, retallen quatre quadrats
iguals a les quatre cantonades i dobleguen els costats com es mostra a la figura.

a) Calcula expressions algebraiques que determinin la superfície i el volum de la capsa sense tapa que s’obté en funció
del costat x dels quadrats retallats.
b) Fes un full de càlcul que mostri la superfície i el volum de la capsa per a diferents valors de x.
c) Amb l’ajuda del full de càlcul anterior, estableix la longitud x que fa que el volum de la capsa sigui màxim. Quant val
la superfície en aquest cas?

a) S  84  56  4x 2  4704  4x 2

V  x(84  2x )(56  2x )  4x 3  280x 2  4704x

b)

x S(x) V(x)
5 4604 17020
8 4448 21760
9 4380 22572
10 4304 23040
11 4220 23188
12 4128 23040
15 3804 21060
10,9 4228,76 23186,916
10,95 4224,39 23187,8295
10,97 4222,6364 23187,9867
10,98 4221,7584 23188,0208
10,99 4220,8796 23188,0252

c) El volum màxim s’obté quan es tallen quadrats de 10,99 cm de costat. En aquest cas, el volum serà de
3 2
23 188 cm i la superfície, de 4220,9 cm .

84 Matemàtiques 1r Batxillerat
SOLUCIONARI UNITAT 2. ÀLGEBRA

AUTOAVALUACIÓ

Comprova el que has après


1. Calcula el valor de a perquè la divisió següent sigui exacta.

 2x 4
 
 7 x 3  20 x 2  4ax  a : 2 x 2  3 x  3 

Quan es divideixen els polinomis s’obté com a quocient  x 2  5x  1 i com a residu 4(a  3)x  (a  3) .

Per tant, perquè la divisió sigui exacta, a  3  0  a  3

2. Factoritza el polinomi: 2 x 5  4 x 4  13 x 3  10 x 2  7 x  4

Quan s’aplica la regla de Ruffini, s’obtenen les arrels x  1 (doble), x  4 i un residu de segon grau 2x 2  1 que
no té arrels reals.

Per tant, 2x 5  4x 4  13 x 3  10 x 2  7 x  4  ( x  1) 2( x  4)(2 x 2 1)

 
5
3. Desenvolupa el binomi: 3  2

3  2  5 5
 2    52   3   2   5
 2 5
 2  5
 2
5 2 3 4 5
    35     34  3
    32    3   
0 1   3  4  5

 243  405 2  540  180 2  60  4 2  843  589 2

4. Simplifica les fraccions algebraiques següents.


1
x 1 2 6 1
1
a) 2  b)  
1
1 x 1 x  2 x2  x  2 x  1
x 1

1 x2
x 1
2 1 1 x 1 1 ( x  1)2  2 x 2  2x  1
a)   2     
1
1 x 1 x  2 x 1 2 x 1 2( x  1) 2x  2
x 1 x 1
2 6 1 2 6 1 2x  2  6  x  2 3( x  2) 3
b)        
x  2 x 2  x  2 x  1 x  2 ( x  1)( x  2) x  1 ( x  1)( x  2) ( x  1)( x  2) x  1

5. Resol les equacions:

2 4x  8
a)  x 2 1
3
b) log(2x  1)  log(x  1)  2log(x  1)

49 x  343
c)  7 x 1
8

85 Matemàtiques 1r Batxillerat
SOLUCIONARI UNITAT 2. ÀLGEBRA

2 4x  8
a)  x  2  1  2 4x  8  3  3 x  2  16x  32  9  9x  18  18 x  2  18 x  2  7x  5 
3

x  7

 324x  648  49x 2  25  70x  49x 2  254x  623  0  x   89
 x (Falsa)
 49

b) log(2x  1)  log(x  1)  2log(x  1)  log(2x  1)(x  1)  log(x  1)2  2x 2  3x  1  x 2  2x  1  x 2  5x  0 

 x  0 (Falsa)
 . L’equació no té solució.
 x  5 (Falsa)

c) Fem el canvi z  7x

49x  343 z  49  7x  49  x  2
 7x 1   7x   343  8  7x 1  z2  56z  343  0  z  
2

z  7  7  1  x  1
x
8

6. Resol, si és possible, els sistemes:

5x  3y  4
  x 2  y 2  xy  7
a)  13 b) 
 2x  7y   2x  y  1
 3

1
a) x 1, y
3
b) y  1 2x

 x  1, y  3
x 2  (1  2x)2  x(1  2x)  7  x 2  x  2  0  
 x  2, y  3

7. Troba les solucions de les inequacions:


2x  3 x  8 161 x2  x  6
a)   7x  b) 0
4 8 8 2x 2  x  6

2x  3 x  8 161
a)   7x   4x  6  x  8  56x  161  53x  159  x  3  Solució: x   3,   
4 8 8

x2  x  6 ( x  2)( x  3) x 3 3 
b) 0 0  0  Solució: x   , 3
2x  x  6
2
( x  2)(2x  3) 2x  3 2 

8. Resol els sistemes d’inequacions:

3x  y  12
 x  2y  3
2( x  3)  2x  6 

a)  x  3 b)  x

 3
 2x  6 y  2  2

2x  3y  1

2( x  3)  2x  6 x  3
 4x  12   21
a)  x  3   21  Solució: x    
  2x  6 5x  21  x    5 
 3  5

86 Matemàtiques 1r Batxillerat
SOLUCIONARI UNITAT 2. ÀLGEBRA

b)

Vèrtex: A(1, 1) B(2, 1) C(4, 0) D(3, 3)

9. Per fer un regal de 36 € a un company, cadascun dels amics que hi participa ha de pagar una quantitat. Si
es retiren dos amics, els altres han de pagar 3 € més cada un. Quant ha de pagar cada amic finalment?

En un principi hi ha x amics que paguen y € cada un, tenim:

 xy  36
  x  6, y  6  Cada un dels 4 amics han de pagar 9 €.
( x  2)(y  3)  36

RELACIONA I CONTESTA
Tria l’única resposta correcta en cada cas

1. Quina és la solució real de l’equació següent?

5x  5x  5x  5x  5x  625
A. x  0 C. No té solució real.
B. x  1 D. Té infinites solucions reals.

5x  5x  5x  5x  5x  625  5·5x  25  5x  5  x  1 , la resposta B.

2. La factorització del polinomi P(x )  16x 4  81y 4 és:

A. (2x  3y )4 C. (2x  3y )(2x  3y )(4x 2  9y 2 )

B. (4x  9y )2(4x  9y )2 D. (8x 3  27y 3 )(2x  3y )

16x 4  81y 4  (4x 2  9y 2 )(4x 2  9y 2 )  (2x  3y )(2x  3y )(4x 2  9y 2 ) , la resposta C.

3. L’equació log2 x  1  log2 x  log2 x  1  log2 ( x )

A. Té una solució real. C. Té infinites solucions reals.


B. Té dues solucions reals. D. No té solució real.

 
log2 x  1  log2 x  log2 x  1  log2(x)  log2 x x  1  log2 x x  1  x x  1  x x  1 

 x 3  x 2  x 3  x 2  2x 2  0  x  0 (Falsa), la resposta D.

87 Matemàtiques 1r Batxillerat
SOLUCIONARI UNITAT 2. ÀLGEBRA

Assenyala, en cada cas, les respostes correctes

x 1
4. Es considera la inequació 2
x 2
A. El valor x  3 pertany a la solució.

B. Els valors x  1 i x  1 pertanyen a la solució.


C. El valor x  5 pertany a la solució però el valor x  2 no.

D. Els valors x  2 i x  5 pertanyen a la solució.

x 1 x 1 x 5
2 2  0   0  Solució: x   5,  2 , per tant, les respostes correctes són la A i la C.
x2 x2 x2

2 x  y  z  2
5. Es considera el sistema d’equacions  .
3 x  2 y  z  2

A. Si s’afegeix l’equació 5x  y  0 el sistema no té solució.

B. Si s’afegeix l’equació 5x  y  4 el sistema té infinites solucions.

C. Si s’afegeix l’equació x  y  z  0 el sistema té infinites solucions entre les quals es troba x  1 , y  1, z  1 .

D. Cap de les afirmacions anteriors és certa.

Sumem les equacions del sistema obtenim 5x  y  4 , per tant la A i la B són correctes.

Per comprovar si C és correcta, resolem el sistema:

 10
 x  13
2 x  y  z  2 
  2
3 x  2y  z  2   y   , per la qual cosa la C no és correcta. Per tant, les respostes correctes són la A i la B.
x  y  z  0  13
  8
z   13

Tria la relació correcta entre les dues afirmacions següents

6. Es consideren les afirmacions:


1. x és un nombre que compleix l’expressió: A( x )  B( x )

2. x és un nombre que compleix:  A( x )  B( x )


2 2

A. 1  2 B. 1  2 però 2  1 C. 2  1 però 1  2 D. Cap de les anteriors.

Òbviament 1  2 , però el recíproc no és cert, si  A(x)  B(x) pot ser que A(x)  B(x) o A(x)  B(x) , per tant,
2 2

la relació correcta és la B.

88 Matemàtiques 1r Batxillerat
SOLUCIONARI UNITAT 2. ÀLGEBRA

Assenyala la dada innecessària per respondre

7. Es vol calcular el valor numèric en x  a i y  b de:

1 x 2  2x  1
:
y 1
 xy  x  y  1
x x

Per fer-ho es donen les dades: 1. a  3 2. b  3


A. La dada 1 és innecessària. C. Són necessàries les dues dades.
B. La dada 2 és innecessària. D. Són innecessàries les dues dades.

1 x 2  2x  1 x ( x  1)2 x
:  : 
y 1
 xy  x  y  1 y  1 ( x  1)( y  1) x  1
x x
Per tant, la resposta correcta és la B.

89 Matemàtiques 1r Batxillerat
SOLUCIONARI UNITAT 3. TRIGONOMETRIA

3. Trigonometria
EXERCICIS PROPOSATS
1. Exercici resolt.

2. Expressa en radians les mesures angulars següents.


a) 30º c) 200º
b) 60º d) 330º

30º   200º  10


a) 30º   rad c) 200º   rad
180º 6 180º 9

60º   330º  11


b) 60º   rad d) 330º   rad
180º 3 180º 6

3. Troba la mesura en graus dels angles següents.


7
a) rad c) 4 rad
3

3
b) rad d) 4 rad
2

7 7  180º 4  180º
a) rad   420º c) 4 rad   229º 11
3 3 

3 3  180º 4  180º
b) rad   270º d) 4 rad   720º
2 2 


4. L’angle d’un arc d’una circumferència de radi 9 cm fa rad. Quina és la longitud d’aquest arc?
6

2  9 
  4,17 cm
2 6
3
5. Un arc d’una circumferència fa 15 cm i comprèn un angle de rad. Quant fa el radi de la circumferència?
5

3
r  15  r  7, 96 cm
5

6 i 7. Exercicis resolts.

8. El sinus d'un angle agut d'un triangle rectangle fa 0,35 i la hipotenusa fa 10 cm. Calcula la mida dels catets.
b b
sin Bˆ   0,35   b  3,5 cm
a 10

c  102  3,52  87,75  9,37

90 Matemàtiques 1r Batxillerat
SOLUCIONARI UNITAT 3. TRIGONOMETRIA

9. El cosinus d'un angle agut d'un triangle rectangle fa 0,8 i el catet oposat fa 6 cm. Calcula quant fan els al-
tres costats.

cos Bˆ  0,8  sin2 Bˆ  1  cos2 Bˆ  1  0,82  0,36  sin Bˆ  0,6


b 6
sin Bˆ   0,6   a  10 cm
a a
a2  b2  c 2  102  62  c 2  c  8 cm

Hipotenusa = 10 cm, i catet que falta = 8 cm.

10. Determina les raons trigonomètriques dels angles aguts d'un triangle rectangle els costats del qual fan 8,
15 i 17 cm.
8 15
sin    0,4706 cos   0,8824
17 17
15 8
sin    0,8824 cos    0,4706
17 17

11. Els catets d'un triangle rectangle fan 12 i 35 cm. Calcula el sinus i el cosinus dels angles aguts d'aquest
triangle.

a2  b2  c 2  a2  122  352  1369  a  37 cm

12 35
sin    0,3243 cos   0,9459
37 37
35 12
sin    0,9459 cos    0,3243
37 37

12 i 13. Exercicis resolts.

14. Calcula la cosecant, la secant i la cotangent d'aquests angles.


a) 30º b) 45º c) 60º
30º 45º 60º
cosecant 2 1,414 1,155
secant 1,155 1,414 2
cotangent 1,732 1 0,577

15. L'altura d'un triangle equilàter fa 9 cm. Quant fan els costats?
2
c 3c 2 3c 3c 29
h2  c 2     h 9 c   10,39 cm
2 4 2 2 3

91 Matemàtiques 1r Batxillerat
SOLUCIONARI UNITAT 3. TRIGONOMETRIA

16. La hipotenusa d'un triangle rectangle isòsceles fa 15 cm. Quant fan els catets?
a 15
a2  b2  b2  a2  2b2  b    10,61 cm
2 2

17. Calcula els costats que falten d'aquests triangles rectangles.

b
a) sin30   b  sin30  6  0,5  6  3 cm
6

a2  b2  c 2  62  32  c 2  c  36  9  27  5,20 cm

14 14 14
b) sin 60  a   16,17 cm
a sin 60 sin 60

2 2
 14   14 
a2  b2  c 2     14  c  c   sin 60   14  8,08 cm
2 2 2

 sin 60   

18 i 19. Exercicis resolts.

20. El sinus de l'angle agut  fa 0,6. Determina el cosinus i la tangent d'aquest angle.

sin2  cos2   1  cos   1 sin2   1 0,36  0,8

sin  0,6
tg    0,75
cos  0,8

21. La tangent de l'angle agut  fa 0,4. Determina el sinus i el cosinus d'aquest angle.
cos  = 0,9284 i sin 0,3713

1 1 1
1  tg2   cos     0,9285
cos2  1  tg2 1  0,42

1
sin2  cos2   1  sin   1  cos2   1   0,3714
1  0,42

22. La tangent d'un angle agut  fa 0,9. Calcula quant fa el cosinus de .

1 1 1
1  tg2   cos     0,7433
cos 
2
1  tg 
2
1  0,92

23. Demostra aquestes igualtats:

a) cos2   sin2   1 2sin2 

tg2   2 sin2 
b)  1  2cos2 
tg2 

1 1 1
c)  
cos2  sin2  sin2   cos2 

92 Matemàtiques 1r Batxillerat
SOLUCIONARI UNITAT 3. TRIGONOMETRIA

a)  
cos2   sin2   1  2sin2   1 sin2   sin2   1 2sin2 

tg2   2 sin2  tg2  2 sin2  2 sin2 


 1  2 cos 2
    1  
tg2  tg2  tg2  sin2 
b) cos2 
2 sin2  cos2 
 1  1  2 cos2 
sin2 

1 1 1 sin2   cos2  1
c)    
cos  sin  sin   cos 
2 2 2 2
cos2   sin2  sin2   cos2 

24. Exercici resolt.


25. Troba els angles reduïts i les raons trigonomètriques:

 
a) 3990º b) 9c) 25 200º d) e) 765º f)


3990º  25 200º 765º


angle reduït 30º 0 45º

sin 0,5 0 0 0,7071 0,7071


cos 0,8660 –1 1 0,7071 0,7071
tg 0,5773 0 0 1 1

26. Per als angles següents, indica el signe de totes les seves raons trigonomètriques.
3 4 9
a) c) e) 
4 3 4
11 7
b) d) 
3 6

3 11 4 7 9
  
4 3 3 6 4
Quadrant II IV III II IV
sin  i cosec      
cos i sec      
tg i cotg     

27 i 28. Exercicis resolts.

29. Calcula el valor de les raons trigonomètriques següents reduint-les al primer quadrant.
a) sin 150º d) tg 330º g) sin 240º
b) cos 225º e) cosec 135º h) cotg 300º
c) sin 840º f) tg 1800º i) sec 2295º

1 3 3
a) sin 150º  sen 30º  d) tg 330º  tg 30º   g) sin 240º   sin 60º  
2 3 2

93 Matemàtiques 1r Batxillerat
SOLUCIONARI UNITAT 3. TRIGONOMETRIA

2 1 1 3
b) cos 225º  cos 45º   e) cosec 135º   2 h) cotg 300º    
2 sen 45º tg 60º 3

3 1
c) sin 840º  sin 60º  f) tg 1800º  tg 0º  0 i) sec 2295º     2
2 cos 45º

30. Calcula, en funció de h, sin 303º, sabent que cos 33º  h.

sin 303º  sin 57º  sin (90º  33º)  cos 33º  h

31. Calcula el valor exacte de:


3 11 4 5
a) sin b) sin c) sin d) sin
4 6 3 6

3  2 4  3
a) sin  sin  c) sin   sin  
4 4 2 3 3 2

11  1 5  1
b) sin   sin   d) sin  sin 
6 6 2 6 6 2

32. Exercici interactiu.

33 i 34. Exercicis resolts.

5
35. Transforma 15º i rad en una suma o diferència d’angles i calcula’n les raons trigonomètriques.
12

3 2 1 2 6 2
15º  60  45 : sin15º  sin  60º 45º   sin60ºcos45º  cos 60º sin 45º     
2 2 2 2 4

1 2 3 2 6 2
cos15º  cos  60º 45º   cos60ºcos45º  sin 60º sin 45º     
2 2 2 2 4

tg60º  tg 45º 3 1 2 3 4
tg15º  tg  60º 45º      2 3
1  tg60º tg 45º 1  3 2

5   5       2 3 2 1 6 2
  : sin  sin     sin cos  cos sin     
12 4 6 12 4 6 4 6 4 6 2 2 2 2 4

5       2 3 2 1 6 2
cos  cos     cos cos  sin sin     
12 4 6 4 6 4 6 2 2 2 2 4

  3
tg  tg 1
5   4 6 3  2 3
tg  tg     
12  4 6  1  tg  tg  3
1
4 6 3

94 Matemàtiques 1r Batxillerat
SOLUCIONARI UNITAT 3. TRIGONOMETRIA

36. Calcula tg 75º a partir del sinus i del cosinus.

2 3 2 1 6 2
75º  45º 30º : sin75º  sin  45º 30º   sin45ºcos30º+cos 45º sin30º     
2 2 2 2 4

2 3 2 1 6 2
cos75º  cos  45º 30º   cos45ºcos30º- sin 45º sin30º     
2 2 2 2 4

6 2
sin75º 4
tg75º    2 3
cos 75º 6 2
4

 3 
37. Demostra que sin       cos  .
 2 

 3  3 3
sin      sin cos  cos  sin  sin  0  cos    1   cos 
 2  2 2

38. Exercici resolt.

4
39. Determina el valor del sinus, el cosinus i la tangent dels angles de 120º i rad.
3

3 1 3
sin120º  sin(2·60º )  2sin 60º cos 60º  2· · 
2 2 2
2
 1  3 
2
1
cos120º  cos(2·60º )  cos2 60º sin2 60º       
 2   2  2

2tg60º 2 3
tg120º  tg(2·60º )    3
1  tg 60º 1  3
 
2 2

4  2  2 2 3  1 3
sin  sin  2·   2sin cos  2· ·    
3  3  3 3 2  2 2
2
2  1   3 
2
4  2  2 1
cos  cos  2·   cos2  sin2      
3  3  3 3  2   2  2

2
2tg
4  2  3  2 3
tg  tg  2·    3
 3  1  tg2 2 1   3
 
2
3
3

40 El sinus de l’angle de 20° fa 0,34. Calcula el sinus i el cosinus dels angles de 10° i de 40°.

sin2  cos2   1  cos 20  1  sin2 20  1  0,342  0,9404

20 1  cos 20 1  0,9404


sin  sin 10    0,1726
2 2 2

sin2  cos2   1  cos10  1 sin2 10  1 0,17262  0,9850

sin 2  20  sin 40  2 sin 20 cos 20  2  0,34  0,9404  0,6395

sin2  cos2   1  cos 40  1 sin2 40  1 0,63952  0,7688

95 Matemàtiques 1r Batxillerat
SOLUCIONARI UNITAT 3. TRIGONOMETRIA

3 
41. Si  és un angle del segon quadrant i sin   , calcula les raons de .
5 2

L’angle és del primer quadrant, per tant, les seves raons trigonomètriques són positives. Tenim:
2

4
1
3 9 4  1  cos  5  9 3 10
sin    cos    1  sin2   1   sin   
5 25 5 2 2 2 10 10

4 4
1 1
 1  cos  5  1 10  1  cos  5
cos    tg    9 3
2 2 2 10 10 2 1  cos  1
4
5
42. Calcula sin 32º suposant que sin 8º = 0,14.

sin8º  0,14  cos8º  1 sin2 8º  1 0,142  0,99

sin16º  sin(2·8º )  2sin8º cos8º  2·0,14·0,99  0,2772  cos16º  1 sin216º  0,9608

sin32º  sin(2·16º )  2sin16º cos16º  2  0,2772  0,9608 0,5327

43. Exercici resolt.

44. Resol les equacions següents i dona els resultats en graus i en radians.

a) sin x  1 b) 2cos x  1  0 c) 3tg x  1  0

a) El sinus d’un angle val 1 a 90º, 450º, 810º, etc.



Per tant x  90º  360ºk amb k  o, en radians, x   2k amb k  .
2

 2
1  x  120º 360º k  3  2k
b) 2cos x  1  0  cos x     k  .
2  x  240º 360º k  4  2k
 3

1 3 
c) 3tg x  1  0  tg x    x  30º 180º k   k k  .
3 3 6

45. Resol les equacions trigonomètriques indicant les solucions compreses en l’interval [0, 2].
a) sin x  cos x  0 b) sin2x  sin x  0

 3
 x  135º  rad
 4
a) sin x  cos x  0  sin x   cos x  tg x  1  
 x  315º  7 rad

 4

  x  0º  0 rad
sin x  0   x  180º   rad
 
  x  360º  2 rad
 
b) sin 2 x  sin x  0  2 sin x cos x  sin x  0  sin x  2 cos x  1  0    

1  x  60º  3 rad
cos x   
 2  x  300º  5 rad
 
 3

96 Matemàtiques 1r Batxillerat
SOLUCIONARI UNITAT 3. TRIGONOMETRIA

46. Calcula la longitud dels costats b i c d’un triangle ABC sabent que a  10 cm, Â  45º i B̂  100º.

Apliquem el teorema del sinus:

a c a sin Cˆ 10 sin35º
 c    8,11 cm
sin Aˆ sin Cˆ sin Aˆ sin 45º

47. Donat un triangle ABC amb a  12 cm, b  15 cm i Ĉ  35º, calcula la longitud del costat c.

Aplicant el teorema del cosinus:

c 2  a2  b2  2ab cos Cˆ  122  152  2  12  15cos35º  74,105  c  8,61 cm.

48. Determina els elements desconeguts d’aquests triangles.

a) L'angle  val 180º – 50º –35º = 95º


10 b 10  sin35º
 b  5,76 cm
sin95º sin35º sin95º
10 c 10  sin50º
 c   7,69 cm
sin95º sin50º sin95º
7 9 9  sin 25º
b)   sin Aˆ   0,543  Aˆ  147º 05 '12''
sin 25º sin Aˆ 7

Bˆ  180º 147º 05'12''  7º 54' 47''

9 b 9  sin Bˆ
 b  2,28 cm
sin Aˆ sin Bˆ sin Aˆ

c) a2  b2  c 2  2bc cos Aˆ  122  82  102  2  8  10cos Aˆ  cos Aˆ  0,125  Aˆ 82º 49'09''

b2  a2  c 2  2ac cos Bˆ  82  122  102  2  12  10cos Aˆ  cos Aˆ  0,75  Aˆ 41º 24'34''

c 2  a2  b2  2ab cos Cˆ  102  122  82  2  12  8cos Aˆ  cos Aˆ  0,5625  Aˆ 55º 46'16''

97 Matemàtiques 1r Batxillerat
SOLUCIONARI UNITAT 3. TRIGONOMETRIA

d) a2  b2  c 2  2bc cos Aˆ  a2  92  72  2  9  7cos60º  a2  67  a 8,19 cm

8,19 9 9  sin 60º


  sin Bˆ  0,952  Bˆ  72º 06 ' 54 ''
sin 60º sin Bˆ 8,19

Cˆ  180º Aˆ  Bˆ  47º 53'06''

a b a sin Bˆ 8 sin 40º


 b   5,22
sin Aˆ sin Bˆ sin Aˆ sin80º

a b a sin Bˆ 8 sin 40º


d)  b   5,22
sin Aˆ sin Bˆ sin Aˆ sin80º

49. Resol aquests triangles.

a) Â  80º, B̂  40º, a  8 dm c) a  10 cm, b  15 cm, c  20 cm

b) Â  80º, a  10 m, b  5 m d) Â  75º, b  8 mm, c  12 mm

a) Cˆ  180º Aˆ  Bˆ  60º


Apliquem el teorema del sinus dues vegades:

a b a sin Bˆ 8 sin 40º


 b   5,22 dm
sin Aˆ sin Bˆ sin Aˆ sin80º

a c a sin Cˆ 8 sin 60º


 c    7,04 dm
sin Aˆ sin Cˆ sin Aˆ sin80º
b) Apliquem el teorema del sinus:

a b b sin Aˆ 5 sin80º
  sin Bˆ   0,492  Bˆ  29º 29 55,34
sin Aˆ sen Bˆ a 10

(La possibilitat Bˆ  150º 31 40 no és vàlida)

Cˆ  180º Aˆ  Bˆ  70º 30 4,66


Apliquem el teorema del cosinus:

c 2  a2  b2  2ab cosCˆ  102  52  2  10  5cos70º 30 4,66  91,621  c 9,57 m


c) Apliquem el teorema del cosinus dues vegades:

b2  c 2  a2 152  202  102


cos Aˆ    0,875  Aˆ  28º 57 18
2bc 2·15·20

a2  c 2  b2 102  202  152


cos Bˆ    0,6875  Bˆ  46º 34 3
2ac 2·10·20

Cˆ  180º Aˆ  Bˆ  104º 28 39


d) Apliquem el teorema del cosinus dues vegades:

a2  b2  c 2  2bc cos Aˆ  82  122  2  8  12cos75º  158,307  a  12,58 mm

a2  c 2  b2 12,582  122  82
cos Bˆ    0,789  Bˆ  37º 53 42
2ac 2·12,58·12

Cˆ  180º Aˆ  Bˆ  67º 6 18

98 Matemàtiques 1r Batxillerat
SOLUCIONARI UNITAT 3. TRIGONOMETRIA

50. En un triangle, l’angle més gran fa 10° més que l’angle mitjà i 20° més que l’angle més petit. El costat més
llarg del triangle fa 7 cm. Quant fan els altres costats?

L'angle gran fa  . L'angle mitjà fa Bˆ  Aˆ  10º i l'angle petit fa Cˆ  Aˆ  20º .

180º  Aˆ  Aˆ  10º  Aˆ  20º  3Aˆ  30º  Aˆ  70º, Bˆ  60º, Cˆ  50º


Apliquem el teorema del sinus:
a b c 7 b c 7 b c
         b  6,45 cm, c  5,71 cm
sin Aˆ s i n Bˆ s i n Cˆ sin70º sin60º sin50º 0,9397 0,8660 0,7660

51. Exercici interactiu.

52 a 64. Exercicis resolts.

EXERCICIS
Mesura d’angles
65. Expressa aquestes mesures en radians.

Graus 30º 45º 60º 90º 120º 135º 150º 180º


    2 3 5
Radians 
6 4 3 2 3 4 6

Graus 210º 225º 240º 270º 300º 315º 330º 360º


7 5 4 3 5 7 11
Radians 2
6 4 3 2 3 4 6

66. Passa de graus a radians.


a) 585º b) 450º c) 76º 52 30 d) 382º 30

585 13 76,875 41


a) 585   rad c) 76º 52' 30 ''   rad
180 4 180 96

450 5 382,5 17


b) 450   rad d) 382º 30 '   rad
180 2 180 8

67. Els angles següents estan en radians. Passa’ls a graus.


41 11
a) rad b) 13 rad c) rad d) 5 rad
3 12

41 41 ·180 11 11 ·180


a) rad   2460º c) rad   165º
3 3 12 12

13 ·180 5·180


b) 13 rad   2340º d) 5 rad   286º 28 44
 

99 Matemàtiques 1r Batxillerat
SOLUCIONARI UNITAT 3. TRIGONOMETRIA

Raons trigonomètriques
68. Calcula les raons trigonomètriques dels angles aguts dels triangles rectangles següents.

a) Â  90º, a  29 cm, b  20 cm b) B̂  90º, a  65 cm, c  72 cm

a) c  292  202  441  21 cm

b 20 c 21 b 20
sin Bˆ   cos Bˆ   tg Bˆ  
a 29 a 29 c 21

c 21 b 20 c 21
sin Cˆ   cos Cˆ   tgCˆ  
a 29 a 29 b 20

b) b  652  722  9409  97 cm

a 65 c 72 a 65
sin Aˆ   cos Aˆ   tg Aˆ  
b 97 b 97 c 72

c 72 a 65 c 72
sin Cˆ   cos Cˆ   tgCˆ  
b 97 b 97 a 65

69. Indica els angles següents com la suma d’un nombre enter de voltes completes més l’angle restant.
46 52
a) 2345º b) 1500º c) rad d)  rad
3 7

46 4 4
a) 2345º  6 · 360º  185º  6 voltes  185º c) rad  7·2   7 voltes  rad
3 3 3
52 4 4
b) 1500º  5 · 360º  300º  5 voltes  300º d)  rad  4·2   4 voltes  rad
7 7 7

70. Utilitza la calculadora per trobar el valor de les raons trigonomètriques següents. Aproxima els resultats a
les mil·lèsimes.
a) sin 36º c) cotg 111º e) sec 126º 33
b) tg 331º d) sin 25º 40 f) cotg 121º 22 45

a) sin 36º  0,588 c) cotg 111º  0,384 e) sec 126º 33  1,679
b) tg 331º  0,554 d) sin 25º 40  0,433 f) cotg 121º 22 45  0,610

71. Utilitza la calculadora per trobar el valor de les raons trigonomètriques següents. Aproxima els resultats a
les mil·lèsimes. Tingues en compte que els angles estan donats en radians.
 3 21
a) sin c) cos e) tg
12 7 5
b) cosec 2 d) sec 3 f) cotg 2,75

 3 21
a) sin  0,259 c) cos  0,223 e) tg  0,727
12 7 5
b) cosec 2  1,100 d) sec 3  1,010 f) cotg 2,75  2,422

100 Matemàtiques 1r Batxillerat


SOLUCIONARI UNITAT 3. TRIGONOMETRIA

72. Calcula totes les raons trigonomètriques de l’angle α sabent que:


2
a) És un angle del primer quadrant i cos  
3
b) Pertany al segon quadrant i sen   0,25

c) 180º    270º i tg   2

3
d)    2 i sec   2
2
e) 90º    180º i cotg   3

3 5
f)  i cosec   
2 2

a) Com que és un angle del primer quadrant, totes les raons són positives. Tenim:
2 3
cos    sec  
3 2

4 5 3 3 5
sin2  cos2   1  sin   1 cos2   1   cosec   
9 3 5 5

sin  5 2 2 5
tg   =  cotg   
cos  2 5 5
b) Com que és un angle del segon quadrant, el sinus i la cosecant són positius i la resta de raons són negatives.
Tenim:
1
sin   0,25   cosec   4
4

1 15 4 4 15
sin2  cos2   1  cos    1  sin2    1    sec    
16 4 15 15

sin  1 15
tg      cotg    15
cos  15 15

c) Com que és un angle del tercer quadrant, la tangent i la cotangent són positius, i la resta de raons, negatives.
Tenim:

1 2
tg   2  cotg   
2 2

1 3
1  tg2  sec 2   sec    1  tg2   1 2   3  cos    
3 3

sin  6 3 6
tg    sin   cos  tg     cosec    
cos  3 6 2

d) Com que és un angle del quart quadrant, el cosinus i la secant són positius, i la resta de raons, negatives.
Tenim:

1 2
sec   2  cos   
2 2

1 1 2
sei  cos2   1  sin    1  cos2    1     cosec    2
2 2 2

sin 
tg    1  cotg   1
cos 

101 Matemàtiques 1r Batxillerat


SOLUCIONARI UNITAT 3. TRIGONOMETRIA

e) Com que és un angle del segon quadrant, el sinus i la cosecant són positius i la resta de raons són negatives.
Tenim:
1
cotg   3  tg   
3

1 10 3 3 10
1  tg2  sec 2   sec    1  tg2   1   cos    
9 3 10 10

sin  10 10
tg    sin   cos  tg    cosec    10
cos  10 10

f) Com que és un angle del tercer quadrant, la tangent i la cotangent són positius, i la resta de raons, negatives.
Tenim:
5 2
cosec     sin   
2 5

4 21 5 5 21
sin2  cos2   1  cos    1  sin2    1    sec    
25 5 21 21

sin  2 2 21 21
tg      cotg  
cos  21 21 2

73. Calcula, de forma exacta, el valor de les raons trigonomètriques següents.


7
a) sin 240º d) sin 1215º g) tg j) cotg 225º
3
5 7
b) cos 135º e) cosec 330º h) sec k) sin
3 4
c) cos(600º) f) tg 300º i) sec 120º l) tg( 15 )

3 7 
a) sin 240º  sin 60º   g) tg  tg  3
2 3 3

2 5 
b) cos135º   cos 45º   h) sec  sec  2
2 3 3

1
c) cos  600º   cos 600º   cos 60º   i) sec120º   sec 60º  2
2

2
d) sin1215º  sin135º  sin 45º  j) cotg 225º  cotg45º  1
2

7  2
e) cosec 330º  cosec 30º  2 k) sin   sin  
4 4 2

f) tg300º  tg 60º   3 l) tg  15   tg15   tg   0

102 Matemàtiques 1r Batxillerat


SOLUCIONARI UNITAT 3. TRIGONOMETRIA

74. Calcula, en funció de h, el valor de cadascuna de les raons trigonomètriques següents.


a) sin 123º, sent sin 57º  h. d) cos 250º, sent sin 110º  h. g) tg 290º, sent sin 110º  h.
b) cos 220º, sent tg 40º  h. e) cos 247º, sent sin 113º  h. h) sin 83º, sent cos 7º  h.
c) tg 260º, sent sin 80º  h. f) cosec 701º, sent cotg 199º  h. i) sec 203º, sent cotg 67º  h.

a) sin123º  sen57º  h sen123º  sen57º  h

1 1 1 1
b) 1  tg   sec    cos 220º    
2 2
cos2  1  tg2 220º 1  tg2 40º 1  h2

1 1 1 1 1
c) 1  tg2  sec 2     tg260º  1  1  1 
cos2  1  sin2  1  sin2 260º 1    sin80º  1  h2
2

h2 h
 
1 h 2
1  h2

d) cos250º  cos110º   1  sin2 110º   1  h2

e) cos 247º   1 sin2 247º   1   sin113º    1 h2


2

f) cosec 701º  cosec 341º  cosec19º  cosec199º   1  cotg2199º   1 h2

1 1 h2 h
g) tg290º  tg110º  1  1  
1  sin 110º
2
1 h 2
1  h2 1  h2

h) sin83º  cos7º  h sen83º  cos7º  h

1 1
i) sec 203º   sec 23º     cosec 67º   1  cotg2 67º   1  h2
cos 23º sin67º

75. Determina la raó trigonomètrica que s’indica en cada cas, expressant-la en funció de h.
23 3
a) cosec , sabent que cotg  h 2 . c) tg 348º, sabent que cos 192º  h2 .
5 5
1
b) sec 305º , sabent que cotg55º  .
h

23 3 3
a) cosec  cosec  1  cotg2  1  h4
5 5 5
1 1 1
b) sec 305º     1  tg2 55º  1  h2
cos 305º cos 55º 1
1  tg2 55º

1 1 1 1 1  h4
c) tg348º   1   1   1   1  
2
cos 348º cos2 12º   cos192º 
2
h 4
h2

 3
76. Si  i  i, sent sin   0, 4 i cos   0,5 , calcula:
2 2
a) sin(  ) b) cos(  ) c) tg(  )

cos    1 sin2   0,917 i sin    1  cos2   0,866

103 Matemàtiques 1r Batxillerat


SOLUCIONARI UNITAT 3. TRIGONOMETRIA

a) sen      sen  cos   cos  sen   0,4  0,5  0,917  0,866  0,994

b) sin      sin  cos   cos  sin   0,4  0,5  0,917  0,866  0,994

0,4 0,866
 
tg   tg  0,917 0,5
c) tg(  )    0,738
1  tg  tg  1  0,4  0,866
0,917 0,5

77. Sabent que tg  α = 3, calcula les raons trigonomètriques de l’angle 2α si α és un angle:
a) Del primer quadrant b) Del tercer quadrant

a) Com que tg  >1, 45º <  < 90º, per tant, 2 pertany al segon quadrant. Tenim:

1 1 1 10 3 10
cos      sin   tg  cos  
1  tg 
2
1 9 10 10 10

3 10 10 6 3 3 10 10 6 3
sin 2  2 sin  cos   2      0,6 sen 2  2 sen  cos   2      0,6
10 10 10 5 10 10 10 5

1 9 8 4 1 9 8 4
cos 2  cos2   sin2        0,8 cos 2  cos2   sen2        0,8
10 10 10 5 10 10 10 5
sin 2 3
tg2     0,75
cos 2 4
b) Com que tg  >1, 225º <  < 270º, per tant, 2 pertany al segon quadrant i s’obtenen els mateixos valors de
l’apartat interior per a les raons 2.

78. Calcula, de forma exacta, les raons trigonomètriques dels angles següents.
a) 15º b) 7º 30

3
1
 30º  1  cos 30º 2  2 3 2 3
a) sin15º  sin    
 2  2 2 4 2

3
1
 30º  1  cos 30º 2  2 3 2 3
cos15º  cos    
 2  2 2 4 2

2  3   2 
2
sin15º 2 3
tg15º    3
cos15º 2 3  2  3 2  3 
2 3
1
 15º  1  cos15º 2 2 2 3
b) sin7º 30 '  sin    
 2  2 2 2

2 3
1
 15º  1  cos15º 2 2 2 3
cos 7º 30 '  cos    
 2  2 2 2

2
2  2  3 
2 2 3   2 2 3
tg7º 30 ' 
sin7º 15 '
    
cos 7º 15 ' 2 2 3 2  2  3 2  2  3  2 3
 
  

104 Matemàtiques 1r Batxillerat


SOLUCIONARI UNITAT 3. TRIGONOMETRIA

2 
79. Si cos    i 90º <  < 180º, calcula les raons trigonomètriques de .
3 2

2 2
1 1
 1  cos  3  5 30  1  cos  3  1  6
sin    cos  
2 2 2 6 6 2 2 2 6 6

30
 6  5
tg 
2 6
6

Identitats trigonomètriques
80. Demostra les identitats trigonomètriques següents.
sin   cos  1  cotg
a)  cos  b) tg2   sin2   tg2  sin2  c)  cosec  d) sec 2   1  tg2 
tg   1 sin   cos 

sin   cos  sin   cos  sin   cos   sin   cos   cos 


a)     cos 
tg   1 sin  sin   cos  sin   cos 
1
cos  cos 

b) tg2   sin2  
sin2 
 sen2
  

sin2   sin2  cos2  sin  1  cos 
2 2



sin2 
sin2   tg2  sin2 
cos2  cos2  cos2  cos2 

cos  sin   cos 


1
1  cotg  sin   sin  1
c)    cosec 
sin   cos  sin   cos  sin   cos  sin 

1 1  cos2  sin2 
d) sec 2   1   1    tg2 
cos2  cos2  cos2 

81. Demostra les identitats trigonomètriques següents.


tg 
a)  tg2  tg  e) sin2   sin2   sin(  )sin(  )
cos 2

b) tg   cotg   sec  cosec  f)  cos   cos 2   sin   sin 2  4 sin2
2

   
tg      tg    
sin 2 4  4   tg2
c)  tg  g)
1  cos 2 2

1  cos 2 sin2
2
1  cos  
d)  cos2     h)   sin   tg 
tg2   tg   2sin  1  cos 2

sin2  cos2   sin2 


1
2 tg   2  1  tg2  cos   tg 
2
cos2  tg 
a) tg2  tg    tg   tg    1  tg   tg  
1  tg 
2
 1  tg  
2
1  tg 
2
sin 
2
cos   sin  cos 2
2 2
1
cos2  cos2 

sin  cos  sin2   cos2  1


b) tg   cotg       sec  cosec 
cos  sen  sin  cos  sin  cos 

sin2 2sin  cos  2sin  cos  sin 


c)     tg 
1  cos 2 1  cos2   sin2  2cos2  cos 
sen2 2sen  cos  2sen  cos  sen 
    tg 
1  cos 2 1  cos2   sen2  2cos2  cos 

105 Matemàtiques 1r Batxillerat


SOLUCIONARI UNITAT 3. TRIGONOMETRIA

2
 cos   cos2   sin2  cos2  sin2  cos2  sin2   cos2  1
d) cos2        2    2
 tg   sin2  tg2  tg  tg2  tg2  tg 

e) sin(  )sin(  )   sin  cos   cos  sin  sin  cos   cos  sin   sin2  cos2   cos2  sin2  

sin2  cos2   (1 sin2 )sin2   sin2 (cos2   sin2 )  sin2   sin2   sin2 

f) cos   cos 2   sin   sin 2  cos2   cos2   2cos  cos   sin2   sin2   2sin  sin  
    2  2 
 2  2  cos2  sin2   2  2 1  2 sin   4 sin
 2 2   2  2
g) L’expressió és equivalent a la demostrada a l’apartat d.

1  cos 2 sin2 1  cos2   sin2  2sin  cos  2sin2  2sin  cos 


h)       sin   tg 
2sin  1  cos 2 2sin  1  cos   sin 
2 2
2sin  2cos2 

Equacions trigonomètriques
82. Resol les equacions trigonomètriques següents indicant totes les seves solucions en graus.

1 2 3
a) sin x  d) sin x   g) tg x  
2 2 3

3 1
b) cos x  e) cos x   h) sin x  0 sen x  0
2 2
c) tg x  1 f) 1 cos x  0 i) tg x  0

1  x  30º 360º k
a) sin x   k  f) 1 cos x  0  cos x  1  x  180º 360º k k 
2  x  150º 360º k

3  x  30º 360º k 3  x  150º 360º k


b) cos x   k  g) tg x    k   150º 180º k k 
2  x  330º 360º k 3  x  330º 360º k

 x  45º 360º k
c) tg x  1   k   x  45º 180º k k 
 x  225º 360º k

2  x  225º 360º k
d) sin x    k  h) sin x  0  x  180º k k 
2  x  315º 360º k

1  x  120º 360º k
e) cos x    k  i) tg x  0  x  180º k k 
2  x  240º 360º k

83. Resol les equacions trigonomètriques següents indicant totes les seves solucions en radians.

3 2
a) sin 4 x   b) cos 2 x  c) tg3x  1
2 2

x x 1 3x 3
d) sin 0 e) cos  f) tg 
2 3 2 4 3

106 Matemàtiques 1r Batxillerat


SOLUCIONARI UNITAT 3. TRIGONOMETRIA

 4   
 4x   2k  x  k
3  3  3 2 x x
a) sin 4 x     d) sin  0   k  x  2k
2 4 x  5  x  5   2 2
 2k  k

 3 
 12 2
     x 2
2x   2k  x   k   2k
2   8 x 1   x  2  6k
b) cos 2 x   4  e) cos     3 3 
2 2x  7  2k  x  7  k 3 2  x 4 
 2k  x  4  6k
 4  8  3 3
 3   2k  3 x 5  10 8k


3x 
4
 2k x  4  3
 3x 3  4  6  2k  x  9  3
c) tg3 x  1    f) tg   
3 x  7  2k  x  7  2k 4 3  3 x  11  2k  x  22  8k

 4 
 12 3  4 6  9 3

84. Troba totes les solucions de les equacions trigonomètriques següents.

a) sin x  cos x c) sin x  3 cos x  0

b) sin 2x  sin x  0 d) sin x  cos x  2

a) sin x  cos x  tg x  1  x  45º 180º k

sin x  0  x  180º k

b) sin 2x  sin x  0  2 sin x cos x  sin x  0  sin x(2cos x  1)  0   1  x  60º 360º k
cos x  2   x  300º 360º k
 

c) sin x  3 cos x  0  tg x  3  x  60º 180º k

d) sinx  cos x  2   sinx  cos x   2  1  2 sin x cos x  2  2 sin x cos x  1  sin 2 x  1 


2

 2x  90º  360º k  x  45º 180º k

Quan s’eleva al quadrat, apareixen solucions falses amb k imparell. La solució és


x  45º  360º k amb k  0, 1, 2...

85. Troba, per a l’interval [0, 2π], les solucions de les equacions trigonomètriques següents.

a) sin2 x  tg2 x  0 b) 2sin x  3 tg x  0

sin2 x  0  sin x  0  x  0, x   , x  2
 1  
a) sin x  tg x  0  sin x  1 
2 2 2
0 1
 cos x  1   0  Sense solució real
2
 cos2 x

sin x  0  x  0, x  , x  2
sin x  3  
b) 2 sin x  3 tg x  0  2 sin x  3  0  sin x  2   0 5 7
  3
cos x  cos x  cos x   x ,x
 2 6 6

Resolució de triangles
86. Resol els triangles rectangles següents.

a) Â  90º, a  25 mm, c  14 mm c) Ĉ  90º, Â  20º, a  12 dm

b) B̂  90º, a  28 cm, c  45 cm d) B̂  90º, Â  15º, b  15 m

107 Matemàtiques 1r Batxillerat


SOLUCIONARI UNITAT 3. TRIGONOMETRIA

c 14
a) b  252  142  20,71 mm sen Cˆ    Cˆ  34º 3 ' 21'' Bˆ  90º Cˆ  55º 56' 39''
a 25

12 12
c) Bˆ  90º  Aˆ  70º sin Aˆ   c  35,09 dm tg Aˆ   b  32,97 dm
c b

a c
d) Cˆ  90º  Aˆ  75º sin Aˆ   a  3,88 m cos Aˆ   c  14,49 m
15 15

87. Calcula l’àrea de cadascun d’aquests triangles.

a) Â  90º, a  73 mm, c  55 mm
b) c)

55  48
a) b  732  552  48  Àrea: S   1320 mm2
2

5 2 5 2
b) b  c; c  10 sin 45º  5 2 m  Àrea: S   25 m2
2

16 19,07·16
c) c   19,07 dm  Àrea: S   152,56 dm2
tg 40º 2

88. Resol els triangles següents.

a) b  20 cm, c  28 cm, Ĉ  40º d) a  12 cm, b  15 cm, Ĉ  35º

b) a  41 cm, b  9 cm, c  40 cm e) a  30 cm, B̂  30º, Ĉ  50º

c) a  3 cm, B̂  30º, c  5 cm f) b  25 cm, B̂  55º, Ĉ  65º

b c b sin Cˆ 20·sin 40º


a)   sin Bˆ    0,459  Bˆ  27º19 21
sin Bˆ sin Cˆ c 28

Aˆ  180º Bˆ  Cˆ  112º 40 39

a c c sin Aˆ 28·sin112º 40 39


 a   40,2 cm
sin Aˆ sin Cˆ sin Cˆ sin 40º

b2  c 2  a2 92  402  412
b) cos Aˆ    0  Aˆ  90º
2bc 2·9·40

a2  c 2  b2 412  402  92
cos Bˆ    0,9756  Bˆ  12º 40 ' 58 ''
2ac 2·40·41

Cˆ  180º Aˆ  Bˆ  77º 19 2

108 Matemàtiques 1r Batxillerat


SOLUCIONARI UNITAT 3. TRIGONOMETRIA

c) b2  a2  c 2  2ac cos Bˆ  32  52  2·3·5·cos30º  8,0192  b  2,83 cm

b2  c 2  a2 2,832  52  32
cos Aˆ    0,8484  Aˆ  31º 57 ' 43 ''
2bc 2·2,83·5

Cˆ  180º Aˆ  Bˆ  118º 2 17

d) c  a  b  2ab cos Cˆ  12  15  2·12·15·cos35º  74,1053  c  8,61 cm


2 2 2 2 2

b2  c 2  a2 152  8,612  122


cos Aˆ    0,6006  Aˆ  53º 5 14
2bc 2·15·8,61

Bˆ  180º Aˆ  Cˆ  91º 54 46

e) Aˆ  180º Bˆ  Cˆ  100º

a c a sin Cˆ 30 sin50º
 c    23,34 cm
sin Aˆ sin Cˆ sin Aˆ sin100º

a b a sin Bˆ 30 sin30º
 b   15,23 cm
sin Aˆ sin Bˆ sin Aˆ sin100º

f) Aˆ  180º Bˆ  Cˆ  60º

b c b sin Cˆ 25 sin 65º


 c    27,66 cm
sin Bˆ sin Cˆ sin Bˆ sin55º

b a b sin Aˆ 25 sin 60º


 a   26,43 cm
sin Bˆ sin Aˆ sin Bˆ sin55º

89. Troba l’àrea dels dos triangles que compleixen que   45º, a  6 cm i c  7,5 cm.

 1
A ac sin Bˆ  21,5 cm2
a c ˆ c sin Aˆ Cˆ  62º 6 59, Bˆ  72º 53 1
 
 2
  sin C   0,8839   
sin Aˆ sin Cˆ a  ˆ ˆ
C  117º 53 1, B  17º 6 59 A  1
ac sin Bˆ  6,62 cm2

 2

Síntesi
90. a) Demostra que l’àrea del segment circular de la figura es pot calcular per mitjà
de l’expressió:

r2
A  x  sen x 
2

b) Calcula l’àrea de la zona ombrejada.

109 Matemàtiques 1r Batxillerat


SOLUCIONARI UNITAT 3. TRIGONOMETRIA

r 2 x 1 2
a) Àrea del sector circular: A1  Àrea del triangle: A2  r sin x
360º 2

r 2 x 1 2 r 2  x 
Àrea del segment circular: A  A1  A2   r sin x    sin x 
360º 2 2  180º 

r 2  x  r
2
Considerant els angles donats en radians, l’expressió queda: A    sin x    x  sin x 
2    2
b) Apliquem l’expressió anterior per calcular l’àrea dels dos segments circulars que es formen en totes dues cir-
cumferències.
Àrea del segment circular de la circumferència de radi r1  5 cm:

52  5 2  3 2 r2
cos Aˆ1   0,82  Aˆ1  34,92º  1  2Aˆ1  69,84º  1,22 rad  A1  1 (1  sin 1)  3,51 cm2
2·5·5 2

Àrea del segment circular de la circumferència de radi r2  3 cm:

32  5 2  5 2 r2
cos Aˆ 2   0,3  Aˆ 2  72,54º  2  2Aˆ 2  145,08º  2,53 rad  A 2  2 (2  sin 2 )  8,8 cm2
2·3·5 2

Per tant, l’àrea de la zona ombrejada és A  A1  A2  12,31 cm .


2

91. a) Troba una fórmula que permeti calcular l’àrea d’un rombe coneixent les mesures del seu costat i d’un dels seus
angles.
b) Quina és l’àrea d’un rombe de 15 cm de costat si un d’aquests angles mesura 40°
2
c) Calcula els angles d’un rombe sabent que el seu costat mesura 4 cm i la seva àrea 8 cm .

a) Un rombe de costat x i un dels seus angles  es pot dividir en dos triangles isòsceles iguals d’àrea.

1 2
A x sin  , per tant, l’àrea del rombe és AR  2A  x 2 sin  .
2

b) AR  152 sin 40º  144,63 cm2 .

1
c) 8  42 sin   sin    Els angles del rombe són 30º i 150º.
2

92. Sabent que tg t:
2
 
a) Calcula cos2 i sin2 en funció de t.
2 2
b) Amb l’ajuda de les fórmules de l’angle doble, calcula sinα, cosα i tgα en funció de t.
1 sin   cos  tg 
i) ii) iii)
sin   cos  sin   cos  sin   cos 

  1  1 1   1 t2
a) 1  tg2  sec 2   cos2   i sin2  1  cos2  1  
2 2 cos2  2 1  tg2  1  t 2
2 2 1 t 2
1 t 2
2 2

110 Matemàtiques 1r Batxillerat


SOLUCIONARI UNITAT 3. TRIGONOMETRIA

    t 1 2t
b) sin   sin  2    2 sin cos  2 
 2 2 2 1 t 2 1 t 2 1 t 2

    1 t2 1 t 2
cos   cos  2    cos2  sin2   
 2 2 2 1 t 2 1 t 2 1 t 2

sin  2t
tg   
cos  1  t 2

QÜESTIONS
93. Quins són els valors màxim i mínim de sin α, cos α, tg α, cosec α, sec α i cotg α?

El valor mínim de sin  i cos  és 1 i el valor màxim és 1.


El valor mínim i màxim de la resta de valors no està definit, tg  i cotg  poden tenir qualsevol valor, sec  i cosec
 poden tenir qualsevol valor excepte els que pertanyen a l’interval (1, 1).

94. Indica tots els angles positius i més petits que 360° tals que la seva tangent coincideix amb la seva cotan-
gent.

La tangent coincideix amb la cotangent per a aquells angles en què el valor és 1 i 1. Per tant, els angles positius
inferiors a 360º que compleixen aquesta condició són 45º, 135º, 225º i 315º.

PROBLEMES
95. Un globus està subjecte a una corda de 10 m de llargada. A causa de l’acció al vent, el globus s’ha despla-
çat de la vertical del punt de subjecció i es troba a una altura de 8 m. Calcula la inclinació de la corda res-
pecte de la línia de terra.

8
Sigui  la inclinació buscada, tenim: sin      53º 7 ' 48 '' .
10

96. En una ciutat, al migdia del solstici d’estiu, els raigs solars tenen una inclinació de 73° 3′. Calcula la longi-
tud de l’ombra d’un edifici de 52 m d’altura.

52
Sigui x la longitud de l’ombra, tenim: tg73 º 3 '   x  15,85 m.
x

98. Un senyal de trànsit indica que el pendent d’un tram de carretera és del 8 %, cosa que vol dir que en un
desplaçament horitzontal de 100 m es fa un ascens de 8 m d’altura.
a) Quin angle forma la carretera amb l’horitzontal?
b) Quants metres cal recórrer per pujar 125 m?

8
a) Sigui  l’angle buscat, tenim: tg      4º 34'' 26''
100
125
b) Sigui x el recorregut demanat, tenim: sin    x  1567,5 m
x

111 Matemàtiques 1r Batxillerat


SOLUCIONARI UNITAT 3. TRIGONOMETRIA

99. Des d’un punt, que és a 20 m del peu d’una torre de 10 m d’altura, veiem el punt més alt d’aquesta sota un
cert angle. Quina distància hem de recórrer cap a la torre per veure’l amb un angle que sigui el doble de
l’anterior?

10 2 tg  1 4 10
tg   ; tg2      4 x  50  x  12,5 m
20 1  tg2  1  1 3 20  x
4

100. Dos cotxes, amb velocitats constants respectives de 90 i 80 km/h, viatgen per una carretera que es bifurca
en dues que formen un angle de 82° i són rectes. Si arriben alhora a la bifurcació i cada cotxe pren una de
les branques, quina distància hi haurà entre ells quan portin 15 minuts de viatge.

Siguin e1 i e2 els espais que han recorregut els dos cotxes en 15 min  0,25 h i sigui x la distància buscada, tenim:

e1  90  0,25  22,5 km


  x  22,5  20  2  22,5  20 cos82º  27,95 km
2 2
e2  80  0,25  20 km 

101. Calcula l’altura dels dos edificis de la figura.

Sigui x l’altura del primer edifici i y la del segon. Tenim:


x
tg33º 42'   x  24 tg33º 42'  16 m
24

y x
tg26º 36'   y  x  24 tg26º 36'  12 m  y  12  16  28 m
24

102. Dues ciutats, A i B, estan situades sobre el mateix meridià de l’esfera terrestre, men-
tre que la ciutat C es troba en el mateix paral·lel que A. La latitud de A és de α = 40°
nord.
B
a) Si la ciutat B és 150 km al nord de A, calcula la seva latitud sabent que el radi de la
C A
Terra és d’uns 6370 km.
b) Si la ciutat C està situada sobre el mateix paral·lel, 30° a l’oest de A, quina distància
separa aquestes dues ciutats?

a) Recordem que la longitud d’un arc d’amplitud  graus i d’una circumferència de radi r
r 
és L  :
180º

   40º 6370 
180º   150 
 
180º L
  180º   41º 21
r 6370
r
b) Es calcula en primer lloc el radi del paral·lel corresponent: sin50º   r  4879,7 km
6370

r 
L  2555 km
180º

112 Matemàtiques 1r Batxillerat


SOLUCIONARI UNITAT 3. TRIGONOMETRIA

103. Calcula l’angle de tir del jugador que està situat en el punt B del camp.

 26,34
 tg CBA   0,5268  CBA  27º 46 49
 50
  DBC  DBA  CBA  6º 10 5
tg DBA  33,66  0,6732  DBA  33º 56 54

 50

104. Les bases d’un trapezi isòsceles mesuren 10 i 5 cm, respectivament. L’angle que forma la base més gran
amb cadascun dels costats no paral·lels és de 35°. Calcula l’altura, el perímetre i l’àrea del trapezi.

h
tg35º   h  1,75 cm
2,5

2,5
cos35º   x  3,05 cm  P  21,1 cm
x
(10  5)·1,75
A  13,13 cm2
2

105. Des d’un satèl·lit GPS s’estableix la posició d’un cotxe respecte d’un punt de referència fix a la Terra. Les
distàncies des del punt fix i el cotxe al satèl·lit són 21 364 i 29 079 km, respectivament. Si la línia que uneix
el punt fix amb el satèl·lit forma un angle amb el terra de 71°, i la que uneix el cotxe amb el satèl·lit forma
un angle de 44°, quina distància separa el cotxe del punt fix? A quina altura és el satèl·lit??

Observem el dibuix i tenim:


h  21 364  sin71  20 200 km

x  y  21364·cos71º 29079·cos 44º  27873,12 km

106. Calcula la distància entre els punts A i B.

AD 7,2
  AD  5,77 m
sin50º sin73º

BD 5,25
  BD  4,13 m
sin 48º sin71º

AB2  5,772  4,132  2, 5,77  4,13cos 180º 73º 71º   11,79  AB  3,43 m

113 Matemàtiques 1r Batxillerat


SOLUCIONARI UNITAT 3. TRIGONOMETRIA

107. Calcula l’angle α que formen la diagonal del cub i la diagonal d’una cara d’aquest.

Sigui a l’aresta del cub, D  a2  a2  a2  3a2  a 3 i d  a2  a2  a 2 , per tant, tenim:

d 2 6
cos        35º 15 52
D 3 3

108. Calcula l’amplitud de l’angle  de la figura.

La figura es pot dividir en dos triangles iguals, ja que tenen els tres costats
iguals, per tant:
 a 1 
tg     26º 33 54    53º 7 48
2 2a 2 2

109. Calcula l’altura, el perímetre i l’àrea del trapezi de la figura.

Altura: h  6tg 40º  5,03 cm


Costat restant: x  6cos 40º  4,6 cm
Perímetre: 23,63 cm
10  4
Àrea: ·5,03  35,21 cm2
2

110. Un home que està situat a l’oest d’una emissora de ràdio observa que el seu angle d’elevació és de 45°. Ca-
mina 50 m cap al sud i observa que l’angle d’elevació és ara de 30º. Troba l’altura de l’antena.

La distància inicial a l’antena és igual a l’altura h, ja que l’angle en el primer


punt és de 45º.
h
des del segon punt, la distància a l’antena és  3h .
tg30º

Com que és un triangle del terra rectangle tenim:

 3h 
2
h2  502   3h2  h2  1250  h  35,36 m

114 Matemàtiques 1r Batxillerat


SOLUCIONARI UNITAT 3. TRIGONOMETRIA

111. Dues persones, que estan separades per 2 km de distància, veuen, sobre el seu pla vertical i en el mateix
moment, un núvol sota angles de 73° 18′ i 84° 17′, respectivament.
Calcula l’altura del núvol i la distància que hi ha entre aquest i cadascun dels observadors.

Hi ha dues possibles interpretacions del problema.


Si el núvol està situat entre els dos observadors, tenim:
b 2
  b  5,02 km
sin73º18' sin22º 25'

a 2
  a  5,22 km
sin84º17' sin22º 25'

h  b sin84º 17'  5 km

Si el núvol està situat a un mateix costat dels dos observadors, tenim


b 2
  b  10,05 km
sin73º 18' sin10º 59'

a 2
  a  10,45 km
sin95º 43' sin10º 59'

h  b sin84º 17'  10 km

PER APROFUNDIR
112. Per al triangle de la figura i la circumferència circumscrita a aquest demostra l’afirmació donada en cada
cas.
a b c
a) Es compleix la relació: r    (Tingues en compte la relació
2 sin Aˆ 2 sin Bˆ 2 sin Cˆ
entre els angles B̂ i B̂ )
abc
b) L’àrea del triangle es pot calcular com: A 
4r

a) Bˆ  Bˆ  , ja que tots dos són angles inscrits a la mateixa circumferència i determinen el mateix arc.

b sin Aˆ b b sin Aˆ b a a b c
sin Bˆ  i sin Bˆ     r  r   
a 2r a 2r 2 sin Aˆ 2 sin Aˆ 2 sin Bˆ 2 sin Cˆ

1 1 c abc
b) A  ab sin Cˆ  ab 
2 2 2r 4r

115 Matemàtiques 1r Batxillerat


SOLUCIONARI UNITAT 3. TRIGONOMETRIA

113. Observa la figura següent.

a) Si les diagonals d’un quadrilàter mesuren d i D unitats lineals, respectivament, i formen un angle α, demostra
1
que l’àrea d’aquest quadrilàter pot calcular-se amb la fórmula: A  dD sin 
2
b) Calcula l’àrea d’un quadrilàter les diagonals del qual formen un angle de 80° si mesuren 4 i 5 cm, respectiva-
ment.

a) El quadrilàter es pot dividir en quatre triangles, com que


sin  180º    sin  es pot escriure:

1 1
A sin   xy  x  D  y   y  d  x    D  y   d  x   dD sin 
2 2

1 1
b) A  Dd sin   4  5 sin80º  9,85 cm2
2 2

114. Considera les dues circumferències coplanàries de la figura.

Calcula la inclinació sobre la recta que uneix els centres de:


a) La tangent comuna exterior. b) La tangent comuna interior.

64 64
a) sin      9º 35 39 b) sin      56º 26 34
12 12

116 Matemàtiques 1r Batxillerat


SOLUCIONARI UNITAT 3. TRIGONOMETRIA

ENTORN MATEMÀTIC

Parada inesperada al pont


Un grup d’estudiants és a Londres aprenent
anglès. Com a activitat complementària fan
una visita a Westminster. Tanmateix, quan
van a creuar el pont es troben que està ta-
llat per un accident. Per entretenir-se men-
tre esperen, el seu professor, Mr. Clever, els
proposa el problema següent:
Can you estimate the height of Big Ben To- h
wer and how far we are from it?
x
Els nois estan una mica desconcertats per- A B
què no saben com aconseguir la informació
que els demana Mr. Clever (Internet via
Smartphone is not allowed).
En aquest moment Bonneidée, una estudiant francesa, recorda les seves classes de trigonometria i proposa un
pla per solucionar el problema. Posa l’Edu al punt A, el Fer, al B, i mesura en passes la distància entre tots dos,
obtenint un valor de 125 passes d’uns 80 cm cadascuna. Després, estima els angles α i β formats pel pont i les
línies que uneixen A i B amb el punt més alt del Big Ben, obtenint que α = 18° i β = 26.
Amb aquesta informació, Bonneidée afirma que pot calcular l’altura de la torre i la distància a aquesta. Creus
que és cert o s’està marcant un bluf davant dels seus amics i de Mr. Clever? Si penses el primer, demostra-ho
calculant tu mateix les quantitats demanades (x i h en la figura).

 h
tg26º  x 0,488 x
  h  x tg26º  0,488 x  0,325   x  199,39 m; h  97,3 m
tg18º  h 100 x
 100  x

El London Eye
Com que el pont no s’obre, Mr. Clever proposa, provocant un
altre ensurt al grup, que ja es veu havent de resoldre un nou
problema, una alternativa: A

Would you like to visit the London Eye?


Encara que n’hi ha alguns que els fa por pujar a la gegantina
roda del mil·lenni, a altres els sembla que pot ser excitant i
que les vistes i les fotografies des d’allà dalt valdran la pena.
Al final decideixen caminar pel costat del Tàmesi fins a la fa-
mosa roda i allí el grup es divideix entre els atrevits que hi
pujaran i els més porucs: en Fer, la Clara i, of course, Mr.
Clever que es queden esperant a baix.
Quan han passat uns minuts, la roda s’atura de sobte per una avaria i comencen a sentir-se protestes des de les
cabines. Els nois observen que la cabina on van els seus companys ha quedat en la posició A. Per tenir ocupats
els nois mentre s’arregla el problema, Mr. Clever sense perdre la seva calma britànica comenta:
It seems like the London Eye is not working very well today! Hey guys, can you tell your friends what height
they are at?
En Fer i la Clara tracten de resoldre el problema. Les úniques dades amb què compten són les que s’indiquen a
l’entrada de l’atracció: altura màxima de 135 m i disposa de 30 cabines.
Què haurien de fer aquests nois per estimar l’altura a la qual estan cridant els seus companys?
Observa que l’angle entre dues cabines, mesurat des del centre O de la roda, és de 12º,
per tant: h  67,5sin24º  27,5 m

Per tant, l’altura a la qual es troba la cabina dels seus companys és 67,5  27,5  95 m.

117 Matemàtiques 1r Batxillerat


SOLUCIONARI UNITAT 3. TRIGONOMETRIA

AUTOAVALUACIÓ

Comprova el que has après


1. Converteix en graus o en radians, segons el cas, els angles següents.

a) 65º b) 138º c) 4 rad d) rad
10

65 13 4·180


a) 65   rad c) 4 rad   229º 11
180 36 

138 23   ·180


b) 138   d) rad   18º
180 30 10 10

2. Escriu en funció d’un angle entre 0 i 45° les raons trigonomètriques següents.
a) sin 120º b) cos 480º c) tg (430º)

a) sin120º  sin 180º 40º   sin 40º

b) cos 480º  cos120º  cos 180º  60º    cos60º   sin30º

c) tg  430º    tg430º   tg70º   cotg20º

3. Calcula la raó demanada en cada cas.


1
a) cos , si tg   4 i   III b) tg , si sin    i   IV
5

1 17
a) 1  tg2  sec 2   sec    1  tg2   1 16   17  cos    
17 17

1 6
b) 1  cotg2   cosec 2   cotg    cosec 2   1   25  1   24  2 6  tg    
2 6 12

4. Si sin   0,6 i sin   0,8, sent tots dos angles del primer quadrant, calcula el valor de:
a) sin (  ) b) cos ( − ) c) tg (  )

3 4 3 4 3 4
sin   0,6   cos   i tg   sin   0,8   cos   i tg  
5 5 4 5 5 3

3 3 4 4
a) sin      sin  cos   cos  sin   ·  ·  1      90º
5 5 5 5

4 3 3 4 24
b) cos       cos  cos   sin  sin   ·  ·   0,96
5 5 5 5 25

1
c)     90º  tg      no existeix perquè cos      0 i tg      
0

118 Matemàtiques 1r Batxillerat


SOLUCIONARI UNITAT 3. TRIGONOMETRIA

3 
5. Si  és un angle del segon quadrant i cos    , calcula les raons de i de 2.
4 2


Com que  és un angle del segon quadrant, el sinus és positiu, la tangent negativa i pertany al primer qua-
2
3 7 7
drant. Tenim cos     sin   i tg    i, per tant:
4 4 3

 1  cos  7 7 14  4 2 14


sin       cosec    
2 2 8 2 2 4 2 14 7

 1  cos  1 1 2  4 4 2
cos     sec     2 2
2 2 8 2 2 4  
2 2 2

 1  cos   1 7
tg   7 cotg    
2 1  cos   
2 7 7

D’altra banda:

3 7 8 8 7
sin 2  2 sin  cos    cosec 2  
8 3 7 21

1
cos 2  cos2   sin2   sec 2  8
8

2 tg  1 7
tg2   3 7 cotg2   
1  tg2  3 7 21

6. Resol les equacions trigonomètriques següents.


sin 
a) sin   4 cos  b) tg   
2

a) sin   4cos   tg   4    75,96º 180º k

sin  sin  sin  sin   0    0º 180º k


b) tg      
2 cos  2 cos   2  No té solució real

7. ˆ  70º , Bˆ  30º i b  12 cm
Resol i calcula l’àrea del triangle amb les mesures: A

Cˆ  180º Aˆ  Bˆ  80º

a b b sin Aˆ 12 sin70º c b b sin Cˆ 12 sin80º


 a   22,55 cm  c    23,63 cm
sin Aˆ sin Bˆ sin Bˆ sin30º sin Cˆ sin Bˆ sin Bˆ sin30º

1
Àrea: A  ac sin Bˆ  133,21 cm2
2

8. Els costats d’un rombe mesuren 6 cm i formen entre ells angles de 60° i 120°. Troba les longituds de les dia-
gonals del rombe.

El rombe es divideix en quatre triangles rectangles com el de la figura.

a  6sin30º  3 cm b  6cos30º  3 3 cm

Per tant, la diagonal més gran del rombe mesura 2b  6 3 cm i la diagonal més petita mesura

119 Matemàtiques 1r Batxillerat


SOLUCIONARI UNITAT 3. TRIGONOMETRIA

2a  6 cm .

3
9. Els angles interiors d’un polígon regular tenen per cosinus  .
2
a) Troba el nombre de costats que té aquest polígon.
b) Hi ha més d’una solució a la pregunta a)?

 3
Existeixen dos angles el cosinus del qual és igual a ,   150º i   210º (>180º) aquest darrer no és inte-
2
rior, ja que els interiors són més petits que 180º. Apliquem la fórmula per calcular la mesura dels angles interiors
d’un polígon regular de n costats:
180(n  2) 180(n  2)
  150º   150º n  180º n  360º  n  12
n n
Per tant hi ha una única solució: n  12 costats.

10. Des d’un punt del terra es veu la capçada d’un pi sota un angle de 42°. Si ens allunyem 2,5 m cap a un altre
punt del terra, alineat amb l’anterior i el peu del pi, veiem la capçada amb un angle de 24°. Calcula l’alçada
del pi.

Sigui h l’altura del pi i x la distància del peu del pi al primer punt. Tenim:

h 
tg 42º  
x  0,9 x
  h  x tg 42º  0,9 x  0,445   1,1125  0,445 x  0,9 x  x  2,45 m  h  2,2 m
h  2,5  x
tg24º 

2,5  x 

RELACIONA I CONTESTA
Tria l’única resposta correcta en cada cas

u2  v 2 u2  v 2
1. Un triangle té per costats: a  uv , b  i c amb u i v dos nombres reals positius tals que
2 2
u > v.
A. Es tracta d’un triangle rectangle a A. C. Es tracta d’un triangle rectangle a C.
B. Es tracta d’un triangle rectangle a B. D. No és un triangle rectangle.

Ex compleix que c 2  a2  b2 , per tant, la resposta correcta és la C.

5
2. Donada la funció , el valor mínim s’obté quan:
5  4 cos 5
A.   2k  rad amb k  C.     2k  rad amb k 

 2 
B.    k  rad amb k  D.    2k  rad amb k 
5 5 5

Com que 5  4cos5  0 el valor mínim de la funció s’assoleix quan cos5 és mínim, és a dir, quan cos5  1,
per tant, la resposta correcta és la B.

120 Matemàtiques 1r Batxillerat


SOLUCIONARI UNITAT 3. TRIGONOMETRIA

3. Un dels casos de resolució de triangles següents té dues solucions diferents. Indica quina és.

A. a  4 cm, b  6 cm, c  15 cm C. a  5 cm, b  10 cm, Cˆ  20º

B. a  5 cm, b  10 cm, Aˆ  20º D. a  3 cm, b  4 cm, c  5 cm

La resposta correcta és la B.

Assenyala, en cada cas, les respostes correctes


4. L’equació trigonomètrica sin2 x  cos2 x té:
   3 
A. Una solució s 0,  C. Tres solucions s 0, 
 2  2 

B. Dues solucions s 0,  D. Quatre solucions a 0, 2

 3
sin2 x  cos2 x  tg2 x  1  tg x  1  x   k, x   k , per tant, totes les respostes són correctes.
4 4

5. A la figura següent:

A. Aˆ  Dˆ  180º B. Aˆ  Cˆ  180º C. Bˆ  Dˆ  180º D. Aˆ  Bˆ  Cˆ  Dˆ  360º

La B és correcta, ja que la mesura d’un angle és igual a la meitat de la mesura angular de l’arc que abraça. Si l’an-
x 360º  x
gle inscrit  abraça un arc x, l’angle inscrit Ĉ abraça un arc de 360º  x, i Aˆ  Cˆ    180º . La C és
2 2
correcta pel mateix raonament i la D és correcta perquè es dedueix dels dos casos anteriors.

Tria la relació correcta entre les dues afirmacions que es fan

6. Tres punts del pla, no alineats, determinen un triangle. Es consideren les afirmacions següents:
1. El triangle és isòsceles 2. El triangle té dos angles els sinus dels quals són iguals.
A. 1  2 C. 2  1 però 1  2

B. 1  2 però 2  1 D. 1 i 2 s’exclouen entre elles.

La B és correcta, si un triangle és isòsceles, tindrà dos angles iguals i per tant amb el mateix sinus, però si un trian-
gle té dos angles amb el mateix sinus, pot ser isòsceles (i per tant també equilàter).

121 Matemàtiques 1r Batxillerat


SOLUCIONARI UNITAT 3. TRIGONOMETRIA

Assenyala la dada innecessària per contestar



7. Per calcular la tangent de l’angle es donen les dades següents:
2
1. tg   0 2. cos   0 3. 1  sin2   1,64

A. Pot eliminar-se la dada 1. C. Pot eliminar-se la dada 3.


B. Pot eliminar-se la dada 2. D. Calen les tres dades.


No n’hi ha prou amb la 1 i la 2 per calcular tg , per la qual cosa la dada 3 és necessària.
2
 1  cos 
1 sin2   1,64  sin2   0,64  cos2   1 sin2   0,36  cos   0,6 i tg 
2 1  cos 
També és necessària la 2 per conèixer el signe de cosinus. A més, si cos   0 ,  pertany al segon o al tercer
 
quadrant, per la qual cosa pot pertànyer al primer o segon quadrant i no queda determinat el signe de tg ,
2 2

així doncs, també necessitem la 1 per concloure que  pertany al tercer quadrant i, per tant , pertany al segon
2
i la seva tangent és negativa. La resposta correcta és la D.

122 Matemàtiques 1r Batxillerat


SOLUCIONARI UNITAT 4. VECTORS EN EL PLA

4. Vectors en el pla
EXERCICIS PROPOSATS
1. Exercici resolt.

2. a) Indica tres parelles de vectors equipol·lents.

b) Representa: 2JK  3AB , AB  EF  JK i JK  DC  GI

a) Són equipol·lents les parelles: EF i GI , GH i JK , CD i HI

b) Siguin a, b i c els vectors lliures els representants dels quals són AB, CD i EF respectivament. Tenim

2JK  3AB  2a  3a  5a , AB  EF  JK  a  c  a  c i JK  DC  GI  a  b  c

3. Fes les operacions indicades:

a) 2 v – x – u u
v x
b) –3 w – 2 v + x

c) 2 u + v + x + 2 w
w

a) b) c)

4. Exercici resolt.

123 Matemàtiques 1r Batxillerat


SOLUCIONARI UNITAT 4. VECTORS EN EL PLA

5. a) Troba les coordenades de a, b i c respecte de la base B  u, v .  


1
b) Representa i calcula les coordenades dels vectors d  a  b i e  c a .
2

9 7
a) a   u  3v b   u  3v c  2v
2 2

1 1 9   7  23 7  23 9 
b) d  a  b    u  3v     u  3v    u  v   , 
2 2 2   2  4 2  4 2

 9  9 9 
e  c  a  2v    u  3v   u  v   ,  1
 2  2 2 
2
6. Comprova que els vectors u  (2,  3) i v  (3, 6) formen una base de V .

2 3
  u i v són linealment independents i, per tant, formen una base de V .
2
3 6

7. Determina el mòdul i l’argument d’aquests vectors:


a) u   3, 7  b) v   6, 5 

a) Mòdul: u  32  72  58 7,62

7
Argument:   arctg 66,80 6648 ' 5 ''
3

b) Mòdul: v  62  52  61 7,81

5
Argument:   arctg 39,81  3948 ' 20 ''
6

8 a 11. Exercicis resolts.

12. Troba el punt mitjà del segment comprès entre els punts A(7, 12) i B(33, –10).

 7  33 12  10 
M ,   M (20, 1)
 2 2 

13. Comprova si els punts P, Q i R estan alineats o formen triangle en els casos següents:

 2 1  3
a) P(0, 3), Q(1, 1) i R(2, 1) b) P(3, 0), Q(2, 1) i R(6, 2) c) P(1, 0), Q   ,  i R  0, 
 3 2  2

PQ  (1, 1)  (0, 3)  (1,  2) 


 1 2
a)    P, Q i R estan alineats
PR  (2,  1)  (0, 3)  (2,  4)
 2 4


PQ  (2, 1)  ( 3, 0)  (5, 1)  5 1
b)     P, Q i R no estan alineats; formen un triangle
PR  (6, 2)  ( 3, 0)  (9, 2) 9 2

 2 1  1 1  1 1
PQ    ,   ( 1, 0)   ,  
 3 2   3 2  3 2
c)    3  P, Q i R estan alineats
 3  3  1
PR   0,   ( 1, 0)   1, 
 2  2  2

124 Matemàtiques 1r Batxillerat


SOLUCIONARI UNITAT 4. VECTORS EN EL PLA

14. Calcula els valors de a i b perquè els vectors PQ i RS siguin equipol·lents sabent que:
P(a  4, b), Q(a  1, b), R(4, 1) i S(a, b).

3  a  4
PQ  RS  (a  1  a  4, b  b)  (a  4, b  1)    a  1, b  1
2b  b  1

15. Calcula el valor de k perquè els punts A(4, 1), B(1, 2) i C(k, k  1) estiguin alineats.

AB  ( 1  4, 2  1)  ( 5, 3) 
 5 3 1
    5k  10  3k  12  k 
AC  (k  4, k  1  1)  (k  4, k  2)
 k  4 k  2 4

16. Troba el vèrtex D del paral·lelogram ABCD si A(2, 3), B(1, 1) i C(3, 0).

Perquè ABCD sigui un paral·lelogram n’hi prou que AB i DC siguin equipol·lents, per tant, si D(d1, d2 ) tenim:

1  3  d1
AB  DC  ( 1,  4)  ( 3  d1,  d2 )    d1  2, d2  4  D( 2, 4)
4  d2

17. Exercici resolt.

18. Si u   4, 3  i v   2,  1 , troba: u , v i u  v

u  u12  u22  (4)2  32  25  5 v  v12  v 22  22  (1)2  5

u  v  u1v1  u2v 2  4  2  3(1)  11

19. Calcula, en funció de k, el mòdul de v  (k ,  k ) i w  (k  1, k  1) i el producte escalar.

v  v12  v 22  k 2  (k )2  2k 2  k 2 w  w12  w 22  (k  1)2  (k  1)2  2k 2  2

v  w  v1w1  v 2w2  k (k  1)  k (k  1)  2k

20. Calcula els valors de k perquè l’angle format per v  (k ,  k ) i w  (k  1, k  1) sigui de 45º.

v w 2 2k 2k 1
cos 45º       2k 2  2  2  2k 2  2  4  k  1, k  1
v w 2 k 2  2k 2  2 2k k2 1 k2 1

Si k  1,   45º; si k  1,   135º

21. Exercici interactiu.

22 a 35. Exercicis resolts.

125 Matemàtiques 1r Batxillerat


SOLUCIONARI UNITAT 4. VECTORS EN EL PLA

EXERCICIS
Vectors fixos i vectors lliures en el pla
36. La figura següent representa un octàgon regular.
a) Indica dos vectors equipol·lents tals que els seus orígens i extrems siguin vèrtexs diferents
de l’octàgon.
b) Indica dos vectors equipol·lents tals que els seus orígens i extrems siguin vèrtexs no conse-
cutius de l’octàgon.
c) Indica dos vectors tals que els seus orígens i extrems siguin vèrtexs diferents de l’octàgon i
tinguin el mateix mòdul i la mateixa direcció però diferent sentit.
d) Indica dos vectors tals que els seus orígens i extrems siguin vèrtexs de l’octàgon i tinguin el mateix mòdul i
diferent direcció.
e) Indica dos vectors tals que els seus orígens i extrems siguin vèrtexs de l’octàgon i tinguin diferent mòdul, igual
direcció i diferent sentit.

a) AB i FE b) AC i GE c) AD i EH d) AB i BC e) AB i DG

Dependència lineal
37. Expressa els vectors a i b en funció dels vectors u i v .

Respecte de la base canònica tenim: a  (1,  3) , b  (3, 6) , u  (4,  2) i


v  (6, 2)

1  4a1  6a2 1 1
a  a1  u  a2  v    a1  1, a2    a  u  v
3  2a1  2a2 2 2

3  4b1  6b2 3 3 3 3
b  b1  u  b2  v    b1   , b2   b   u  v
6  2b1  2b2 2 2 2 2

38. Expressa els vectors BC i CB com a combinació lineal de u i v .

AB  BC  AC  BC  AC  AB  3u  3v i CB  BC  3u  3v

39. Decideix si les parelles de vectors següents u i v són linealment


independents o linealment dependents.

2 1  1 1
a) u   4, 2 i v   ,   b) u  16, 32 i v    ,   c) u   2,  16  i v   1,  8
3 3  4 2

4 2
a)   6  u i v són linealment dependents.
2 1

3 3
16 32
b)   64  u i v són linealment dependents.
1 1
 
4 2
2 16
c)   u i v són linealment independents.
1 8

126 Matemàtiques 1r Batxillerat


SOLUCIONARI UNITAT 4. VECTORS EN EL PLA

40. Expressa, en cada cas, el vector a com a combinació lineal dels vectors u i v .

3 7 1 1 1 5
a) a  (12,  2) , u  (4,  3) i v  (1,  2) c) a   ,   , u   ,   i v   ,  
4 6 2 3 4 6

 1  1 
b) a    ,  5  , u   ,  3  i v  (1,  2)
 2  2 

12  4a1  a2
a) a  a1u  a2 v    a1  2, a2  4  a  2u  4v
2  3a1  2a2

 1 1
  a  a
b) a  a1u  a2 v   2 2 1 2  a1  3, a2  2  a  3u  2v

5  3a1  2a2

3 1 1
  a  a
4 2 1 4 2
c) a  a1u  a2 v    a1  1, a2  1  a  u  v
 7   1 a  5 a

 6 3
1
6
2

Operacions amb coordenades


41. Fes les operacions següents amb coordenades de vectors.

 1 1 1 3
a) 2 2  1, 3  3  2, 0  3  4,  3    3 1,  2 c) 2  2  , 1    2  2,  3    , 
 3 4 2 5

1 2 1 3
b) 2  ,     2,  
3 5 3 4

a) 2 2  1, 3  3  2, 0  3  4,  3    3 1,  2  2  40,  21  3,  6   77,  36 

1 2 1 3 2 4 2 1  11 
b) 2  ,     2,     ,     ,     0,  
3 5 3 4 3 5 3 4  20 

 1 1  1 3   10 3   1 3   7 69 
c) 2  2  , 1    2  2,  3    ,    ,    4,  6    ,     , 
 3 4   2 5   3 2   2 5   6 10 

42. En la figura següent:

a) Calcula les coordenades dels vectors lliures de representants: OA , AB ,


BC , CO , OB i AC .

b) Comprova que OA  AB  BC  CO és el vector nul.

c) Calcula les coordenades de OA  AB i OA  2AB  3BC .

A(2, 5), B(7, 5), C(8, 2) i O(0, 0), per tant:

a) OA  (2, 5), AB  (5, 0), BC  (1, 7), CO  (8, 2), OB  (7, 5) i AC  (6, 7)

b) OA  AB  BC  CO  (2, 5)  (5, 0)  (1, 7)  (8, 2)  (0, 0)

c) OA  AB  (2, 5)  (5, 0)  (7, 5) i OA  2AB  3BC  (2, 5)  2(5, 0) – 3(1, 7)  (9, 26)

127 Matemàtiques 1r Batxillerat


SOLUCIONARI UNITAT 4. VECTORS EN EL PLA

43. Calcula les coordenades de l’origen A d’un vector d’extrem B(2, 4) i que és equipol·lent al vector CD , sent
C(5, 1) i D(2, 2).

2  a1  7
CD  (7,  1) , si A(a1, a2 ) : AB  CD  ( 2  a1, 4  a2 )  ( 7,  1)    a1  5, a2  5  A(5, 5)
4  a2  1

44. Donats els punts A(1, 4), B(2, 2) i C(3, 5), calcula:

a) Les coordenades del punt D tal que AB  CD c) Les coordenades del punt D tal que AB  3CD

b) Les coordenades del punt D tal que DB  AC d) Les coordenades del punt D tal que DB  2AC

d1  3  3 d  0
a) AB  CD   3,  2   d1  3, d2  5     1  D(0, 3)
 2
d  5   2 d2  3

2  d1  2 d1  4
b) DB  AC   2  d1,2  d2    2, 1     D(4, 1)
2  d2  1 d2  1

d1  2
3d  9  3   13 
c) AB  3CD   3,  2    3d1  9, 3d2  15    1  13  D  2, 
 2
3d  15  2 d2   3 
 3

2  d1  4 d  2
d) DB  2AC   2  d1,2  d2    4,  2    1  D(2, 4)
2  d2  2 d2  4
45. Donats els punts A(2, 3), B(1, 4) i C(1, 2), calcula les coordenades dels vectors:
1
a) u  2AB  3CA  BC c) w  2u  v
2

b) v  
2
3
1 5
AC  AB  CB
3 6
 
d) x  3 u  v  5v

a) u  2AB  3CA  BC   2,  2   9, 3   2, 2   13, 3 

2 1 5  2  1 1  5 5
b) v   AC  AB  CB   2,     ,    ,     4,  2
3 3 6  3 3 3 3 3

1
c) w  2u  v   26, 6    2,  1   24, 5 
2

 
d) x  3 u  v  5v  3u  2v   39, 9    8,  4    31, 5 

46. Calcula, si és que existeix, el valor de k perquè es compleixin les igualtats següents.

a) (2,  3  k )  3(3,  1)  7(1,  3)

b) (1,  6)  4(k, 2)  3(1, 3  k )

2 k 1 
c)   k,   2  , k   k  2,  3 
3 2 3 

a) (2,  3  k )  3(3,  1)  7(1,  3)  (2,  3  k )  (2,  24)  3  k   24  k  21

4k  3  1  k  1

b) (1,  6)  4(k, 2)  3(1, 3  k )  (1,  6)  (4k  3, 1  3k )   5  No existeix cap valor de k
 1  3k  6  k  
 3
perquè es compleixi la igualtat.

128 Matemàtiques 1r Batxillerat


SOLUCIONARI UNITAT 4. VECTORS EN EL PLA

2 2
  k   2k
 2 k  1   2 k 2  3 k  0
c)   k,   2  , k   k  2,  3     k,     2k, 2k  3k    3  k 0
3 2 3  3 2 3  k
  k k  0

2

47. Calcula els valors de x i de y perquè es compleixin les igualtats següents.


a) 13,  8  5  x,  1  3 1, y 
b)  x, y   2  1,  1 x   4  y, 3
 y x 1
c)  x,    2,    3  x, y 
 4 2 2

13  5 x  3 x  2
a) 13,  8   5  x,  1  3 1, y    
8  5  3y y  1

 x  2  4y  x  4y  2  x  6
b)  x, y   2  1,  1 x   4  y, 3     
 y  2  2 x  12  2 x  y  14 y  2

x  x  3x
 y x 1  x  0
c)  x,    2,    3  x, y    y x 
 4 2 2     3 y y  0
4 4

48. Calcula el mòdul i l’argument dels vectors següents.

1 
a) u   3, 3  c) u   20,  21 e) u   ,  4 
2 

b) u  12,  5  
d) u  3,  3  f) 
u   2,  2 
 u  32  32  18  3 2

a) u   3, 3    perquè pertany al primer quadrant.

arg(u )  arctg1  rad
 4

 u  122  ( 5)2  169  13




b) u  12,  5    perquè pertany al quart quadrant.
 5 
arg(u )  arctg     5,89 rad

  12 

 u  ( 20)2  ( 21)2  841  29




c) u   20,  21   perquè pertany al tercer quadrant.
 21 
arg(u )  arctg    3,95 rad

  20 

 u  32  (  3 )2  12  2 3

 
d) u  3,  3    
arg(u )  arctg  
3  11 perquè pertany al quart quadrant.
 3   rad

   6

 2
  1 65 65
 u     ( 4)  
2
1 
e) u   ,  4    2 4 2 perquè pertany al quart quadrant.
2  
arg(u )  arctg  8   4,84 rad

129 Matemàtiques 1r Batxillerat


SOLUCIONARI UNITAT 4. VECTORS EN EL PLA


   
2 2
 u   2   2  4 2
f)  
u   2,  2  
arg(u )  arctg1  5 rad
perquè pertany al tercer quadrant.


 4

49. Calcula el mòdul i l’argument dels vectors següents.

 a  ( 2)2  22  8  2 2  c  32  32  18  3 2
 
a) a   2, 2    c) c   3, 3   
3 
arg(a )  arctg( 1)  rad arg(c )  arctg1  rad
 4  4

 d  22  ( 3)2  13
 b  32  02  9  3 
 
b) b   3, 0    d) d   2,  3   
 3
arg(b)  arctg0  0 rad
 arg(d )  arctg     5,3 rad

  2

50. Calcula els costats dels triangles ABC en cada cas.


a) A(2, 1), B(1, 5) i C(1, 1)b) A(3, 1), B(2, 3) i C(5, 5)

a) AB   3, 6 , AC   3, 0 y BC  0,  6 

a  BC  02  (6)2  6, b  AC  (3)2  02  3 i c  AB  (3)2  62  45  3 5

b) AB   5, 4 , AC   2, 6 i BC  7, 2

a  BC  72  22  53, b  AC  22  62  40  2 10 i c  AB  (5)2  42  41

51. Classifica els triangles següents, dels quals coneixem els vèrtexs, segons els seus costats.

 3  3 3 3  1
a) A(3, 0), B(0, 1) i C(1, 2) b) A(1, 2), B(2, 4) i C(4, 1) c) A(0, 0), B(3, 1) i C  , 
 2 2 

a) AB   3, 1 , AC   4,  2 i BC  1,  3

a  BC  12  (3)2  10, b  AC  42  ( 2)2  20  2 5 i c  AB  32  12  10

És un triangle isòsceles.

b) AB  1, 2 , AC   3,  1 i BC   2,  3 

a  BC  22  (3)2  13, b  AC  32  (1)2  10  2 5 i c  AB  12  22  5

És un triangle escalè.

130 Matemàtiques 1r Batxillerat


SOLUCIONARI UNITAT 4. VECTORS EN EL PLA

 3  3 3 3  1  3  3 3 3  1
c) AB   3,  1 , AC   ,  i BC   , 
 2 2  2 
  2

2 2 2 2
 3  3   3 3  1  3  3   3 3  1
a  BC      10, b  AC        10 i c  AB  10
 2   2  2 
    2  
És un triangle equilàter.

52. Calcula la mesura dels costats de l’hexàgon de la figura.

A(2, 1), B(2, 2), C(1, 3), D(2, 1), E(1, 2) i F(1, 2), per tant:

AB  0  9  3 BC  9  1  10 CD  1  4  5

DE  1  9  10 EF  4  0  2 FA  1  1  2

Producte escalar
53. Calcula el producte escalar de:

a) u  (3, 4) i v  (2, 5) b) u  (2, 4) i v  (2,  1) c) u  (3,  4) i v  (2, 0) d) u  (1,  3) i v  (1, 3)

a) u  v  6  20  26 b) u  v  6  4  8 c) u  v  6  0  6 d) u  v  1 9  10

54. Troba el mòdul de la projecció ortogonal del vector u  (2,  1) sobre el vector v  (1,  4) .

u ·v 6 6 17
projv u   
v 17 17

55. Calcula els productes escalars que s’indiquen, si els vectors estan determinats per la fi-
gura.

a) AB  BC c) AC  DA

b) CD  BA d) AC  DB

a) AB  BC  (4, 1)  (0,  3)  0  3  3 c) AC  DA  (4,  2)  (3, 4)  12  8  20

b) CD  BA  (1,  2)  (4,  1)  4  2  6 d) AC  DB  (4,  2)  (1, 5)  4  10  6

56. Calcula el valor o els valors de k perquè es compleixin les igualtats següents.

 k
a)  2, k    1, 3  2 b)  3,  k    2,  1  4k c)  5,     k,  k   6k
 2

a)  2, k    1, 3  2  2  3k  2  k  0
b) 3,  k    2,  1  4k  6  k  4k  k  2

 k k2 k  0
c)  5,     k,  k   6k  5k   6k  k 2  2k  0  k (k  2)  0  
 2 2 k  2

131 Matemàtiques 1r Batxillerat


SOLUCIONARI UNITAT 4. VECTORS EN EL PLA

57. Donats els vectors de la figura, resol les operacions que s’indiquen.

a) u  v  u  w

  
b) u  v  w  u  v  w 
  
c) v  2u  3w  3w  2u  v 

u   2, 3 , v   2, 1 i w   1,  2

a) u  v  u  w  1 8  9

   
b) u  v  w  u  v  w   2, 3   3,  1   0, 4    1,  2  9  8  17

   
c) v  2u  3w  3w  2u  v   2, 1   7, 12   7, 12   2, 1  2  2  4

Vectors paral·lels i vectors perpendiculars


58. a) Escriu tots els vectors paral·lels al vector lliure de coordenades u   2,  3  .

b) Escriu tots els vectors perpendiculars al vector lliure de coordenades u   2,  3  .

c) Troba un vector paral·lel a u   2,  3  i unitari.

d) Troba un vector perpendicular a u   2,  3  i unitari.

 2 3   2 13 3 13 
a) (2t,  3t ) amb t  c)  ,  , 
 13 13   13 13 

 3 2   3 13 2 13 
b) (3t, 2t ) amb t  d)  ,    , 
 13 13   13 13 

59. Per a cada cas, calcula tots els vectors unitaris que tenen la mateixa direcció que el vector u . Quins d’a-
quests també tenen el mateix sentit?

1 
a) u  10,  24  b) u   2,7;  3,6  c) u   , 4
2 

 24  10 24   5 12 


  
10
 
  26 26   13 
, , ,
 102  ( 24)2 10  ( 24)2
2 13 
 
a) u  10,  24   
 10 24   10 24   5 12 
  
  26 26   13 
, , ,
 102  ( 24)2 10  ( 24)2
2

13 

El primer té el mateix sentit que u .

 2,7 3,6 
 4,5 ; 4,5    0,6;  0,8 
 
b) u   2,7;  3,6    . El primer té el mateix sentit que u .
 2,7 3,6 
  0,6; 0,8 
 ; 
 4,5 4,5 

132 Matemàtiques 1r Batxillerat


SOLUCIONARI UNITAT 4. VECTORS EN EL PLA

 1 
 4   1 8 
 2 ,  , 
 65 65   65 65 
 
1   2 2 
c) u   , 4    . El primer té el mateix sentit que u .
2   1 
 
2 ,  4    1 ,  8 
   
 65 65   65 65 
 2 2 

60. Calcula les coordenades d’un vector paral·lel a AF i de mòdul 10, sent A(1, 3) i F(4, 7).

AF   3, 4  , els vectors paral·lels a AF són de la forma  3t, 4t  amb t  . Així:  3t, 4t   10 

 9t 2  16t 2  10  5 t  10  t  2 . Per tant, hi ha dues possibles solucions:  6, 8  i  6,  8  .

61. Determina un vector perpendicular a u   5, 12  i que tingui el mateix mòdul que u . Quantes solucions hi
ha?

Hi ha dos vectors perpendiculars a u i amb el mateix mòdul: 12, 5  i  12,  5  .

Angle de dos vectors

62. Calcula l’angle que formen en cada cas els vectors u i v .

a) u   3, 4 i v   5, 12 c) u   1, 2 i v  1,  2 e) u   2,  1 i v  1, 2

b) u   20,  21 i v   1, 1 d) u  1,  1 i v   2, 2 f) u   0, 2 i v   3,  1

u v 63 63
a) cos        14,25º
u v 25 169 65

u v 41 41 41 2
b) cos         178,6º
u v 841 2 29 2 58

u v 5
c) cos     1    180º
u v 5 5

u v 0
d) cos     0    90º
u v 2 8

u v 0
e) cos     0    90º
u v 5 5

u v 2 2 10
f) cos         108,43º
u v 4 10 2 10 10

133 Matemàtiques 1r Batxillerat


SOLUCIONARI UNITAT 4. VECTORS EN EL PLA

63. Troba el vector de m perquè els vectors u   m, 1 i v   2, 3  formin un angle de:

a) 30º b) 135º c) 90º d) 0º

u v 2m  3 2m  3
cos    
u v m  1 13
2
13m2  13

2m  3 3
a)   4m  6  39m  39  16m2  36  48m  39m  39  16m2  87m  3  0 
13m  13
2 2

87  7761 87  7761
m (Falsa), m 
32 32

2m  3 2
b)   4m  6  26m  26  16m2  36  48m  26m  26  16m2  74m  10  0 
13m  13
2 2

37  1209 37  1209
m ,m (Falsa)
16 16

2m  3 3
c)  0  2m  3  0  m 
13m  13
2 2

2m  3
d)  1  2m  3  13m  13  4m2  9  12m  13m  13  4m2  25m  4  0 
13m  13
2

25  689 25  689
m (Falsa), m 
8 8

64. Calcula els angles del triangle de vèrtexs ABC.


a) A(1, 3), B(2, 1) i C(4, 1) b) A(3, 1), B(0, 5) i C(4, 3)

AB  AC 7 7 65
a) AB  1,  2 i AC   3,  2  cos Aˆ     Aˆ  29,74º
AB AC 65 65

BA  BC 2 5
BA   1, 2 i BC   2, 0   cos Bˆ     Bˆ  116,57º
BA BC 2 5 5

CA  CB 6 3 13
CA   3, 2 i CB   2, 0   cos Cˆ     Cˆ  33,69º
CA CB 2 13 13

AB  AC 5 5
b) AB   3, 4  i AC  1, 2   cos Aˆ     Aˆ  63,43º
AB AC 5 5 5

BA  BC 20 2 5
BA   3,  4  i BC   4,  2   cos Bˆ     Bˆ  26,57º
BA BC 10 5 5

CA  CB
CA   1,  2 i CB   4, 2  cos Cˆ   0  Cˆ  90º
CA CB

134 Matemàtiques 1r Batxillerat


SOLUCIONARI UNITAT 4. VECTORS EN EL PLA

65. Classifica els triangles següents segons els angles.


a) A(1, 3), B(3, 0) i C(2, 1) b) A(1, 3), B(2, 1) i C(2, 1) c) A(1, 3), B(2, 1) i C(4, 0)

AB  AC
a) AB   2,  3  i AC   3,  2  cos Aˆ   0  Aˆ  90º . Triangle rectangle a A.
AB  AC

AB  AC 5 5 221
b) AB   3,  2 i AC  1,  4   cos Aˆ     Aˆ  70,35º
AB  AC 221 221

BA  BC 8 4 65
BA   3, 2 i BC   4,  2  cos Bˆ     Bˆ  60,26º
BA  BC 2 65 65

CA  CB 12 6 85
CA   1, 4  i CB   4, 2  cos Cˆ     Cˆ  49,4º
CA  CB 2 85 85

Triangle acutangule.

AB  AC 3 26
c) AB   3,  2 i AC   3,  3   cos Aˆ     Aˆ  101,31º . Triangle obtusangle.
AB  AC 3 26 26

66. Calcula els angles del quadrilàter que té com a vèrtexs A(2, 2), B(2, 4), C(1, 1) i D(1, 1).

Observem que AB  DC   0, 2 , per tant, es tracta d’un paral·lelogram, amb la qual cosa   Cˆ i
Bˆ  Dˆ  180º  Aˆ .

AB  AD 6 2
Així doncs: cos Aˆ     Aˆ  Cˆ  135º i Bˆ  Dˆ  45º .
AB AD 6 2 2

Síntesi
67. Donats els vectors u  6i  8 j i v  4i  3 j , calcula:

v proju v
a) Els mòduls de tots dos vectors. e) Compara els quocients i .
u projv u

b) El producte escalar u  v . f) Troba l’angle que formen u  v i u .

c) L’angle que formen els dos vectors. g) Troba les coordenades de proju v i projv u .

d) Troba el mòdul de la projecció de u sobre v i la de v sobre u .

a) u  u12  u22  62  (8)2  100  10 v  v12  v 22  (4)2  32  25  5

b) u  v  u1  v1  u2  v 2  6  (4)  (8)  3  48

u v 24
c) cos       163,74º
u v 25

u v 48 u v 48
d) projv u    9,6 proju v    4,8
v 5 u 10

135 Matemàtiques 1r Batxillerat


SOLUCIONARI UNITAT 4. VECTORS EN EL PLA

v proju v
e) Són iguals:   0,5
u projv u

52 26 29
f) u  v  2i  5 j  cos       15,07º
10 29 145

g) Com que  és obtús, projv u té sentit oposat a v i projv u té sentit oposat a u . Així:

v  192 144  u  288 384 


projv u  projv u ·  ,  proju v  proju v ·   , 
v  25 25  u  100 100 

68. Troba un vector v de mòdul 5 sabent que u  v  14 , sent u   6, 8  .

Si v   a, b  , tenim:

 44 117
a2  b2  25
2
7  4b  7  4b  b  ,a
 a    b 2
 25  25b 2
 56b  176  0   25 25 
6a  8b  14 3  3  
b  4, a  3

  117 44 
v    25 , 25 
  
v   3,  4 

69. Calcula el vèrtex D del paral·lelogram ABCD, sent A(3, 4), B(2, 4) i C(3, 2). Calcula el punt en què es
tallen les diagonals del paral·lelogram.

Les diagonals d’un paral·lelogram es tallen en el seu punt mitjà. Aquest punt de tall serà, per tant, el punt mitjà seg-
ment AC:

 3  3 4  2 
M ,   M  0, 1
 2 2 

Si D(d1, d2) és el quart vèrtex del paral·lelogram, s’haurà de complir que el punt mitjà del segment BD és M:

 2  d1 4  d2 
 ,    0, 1  d1  2, d2  6  D  2, 6 
 2 2 

70. Donats els vectors de la figura:

a) Calcula les coordenades del vector de projecció de v sobre u .

b) Descompon v com a suma de dos vectors: un d’igual direcció que u i un altre perpendicu-
lar a u .

u   2, 2 i u   2,  1

a) proju v té el mateix sentit que u , per tant:

u u v 2 1  1 1
proju v  proju v ·  u u u , 
2 2
2
u u 8 4

136 Matemàtiques 1r Batxillerat


SOLUCIONARI UNITAT 4. VECTORS EN EL PLA

b) Tots els vectors que tenen la direcció de u són de la forma  t , t  amb t  i tots els vectors que tenen la
direcció perpendicular a u són de la forma  s, s  amb s  , per tant, tenim:

t  s  2 1 3  1 1 3 3
v   t, t    s, s    t  , s   v   ,  ,  
t  s  1 2 2 2 2 2 2

71. Calcula el valor de m perquè els punts del pla A(1, 2), B(2, m  2) i C(3, m) estiguin alineats.

2 m  2
AB   3, m  4  i AC   2,  m  2    2m  8  3m  6  m  14
3 m4

72. Calcula les coordenades de l’extrem B d’un vector que té com a origen A(2, 3) i que és equipol·lent al
vector CD , sent C(2, 3) i D(0, 4).

Si B  b1, b2  tenim: AB  CD  b1  2, b2  3   2,  7   b1  4, b2  4  B 4,  4 

73. El baricentre d’un triangle és el punt en què es tallen les seves tres medianes i està situat a doble distància
del vèrtex que del punt mitjà del costat oposat. Calcula les coordenades del baricentre del triangle
A(3, 3), B(2, 1), C(2, 1).

 2  2 1 1 
El punt mitjà del costat CB és l’origen:  ,   O  0, 0  , per tant, si el baricentre és G  g1, g2  , tenim:
 2 2 

g  3  2g1
AG  2GO   g1  3, g2  3   2   g1,  g2    1  g1  1, g2  1  G  1, 1
g2  3  2g2

2
74. Dels vectors u i v se sap que u  13, v  10 i u ·v  9 .

a) Troba l’angle que formen u i v . b) Troba l’angle que formen u  v i u  v .

u v 9
a) cos       104,45º
u v 10 13

u  v   u  v   u  u  u  v  v  u  v  v  u
2 2
b)  2u  v  v  105

2 2 2 2 2 2
u v  u  2u  v  v  95 u v  u  2u  v  v  131

cos  
u  v   u  v   105
   160,26º
u v u v 12445

   
2
75. Siguin els vectors u i v tals que u  u  v  u  v  v  31 i u  v  37 . Troba el producte escalar u ·v .

   
2 2
u  u  v  u  v  v  31  u  u  u  v  u  v  v  v  31  u  v  31

2 2 2
u v  37  u  2u  v  v  37 . Per tant, 31 2u  v  37  u  v  3

137 Matemàtiques 1r Batxillerat


SOLUCIONARI UNITAT 4. VECTORS EN EL PLA

QÜESTIONS
76. Indica, raonadament, si les afirmacions següents són veritables o falses.
a) El vector nul és linealment dependent amb qualsevol altre vector del pla.
b) Si dos vectors no nuls tenen la mateixa direcció, aleshores el seu producte escalar coincideix amb el producte
dels seus mòduls.

c) u v  u  v

d) AB  BC  CD  DA  0

a) Vertader. El vector nul es pot escriure com el producte de qualsevol vector pel nombre 0.
b) Fals. Només és vertader si tenen el mateix sentit; si tenen sentit diferent, el producte escalar és el producte
dels mòduls multiplicat per 1.

c) Fals. Si u  1, 0 i v   0,1 aleshores u  v  1, 1  2 i u  v  1  1  2

d) Vertader. AB  BC  CD  DA  AC  CD  DA  AD  DA  AA  0

77. Dona un exemple de tres vectors no nuls u , v i w tals que u i v siguin linealment independents, u i w
també siguin linealment independents, però v i w siguin linealment dependents.

u  (1, 0), v  (0, 1) i w  (0, 2)

78. Demostra que si AD  CA  AB  CB aleshores els punts A i D són el mateix.

AD  CA  AB  CB  AD  CA  AB  CB  CB  CB  0  A  D

79. Calcula les coordenades dels vectors a , b , c i d respecte de la base u, v .  


a  2v b  2u c  4u  4v d  u  3v

PROBLEMES
80. El paral·lelogram PQRS compleix que PQ  4 cm, PR  7 cm i PS  5 cm.

a) Calcula PQ  PR , PS  PR , RS  SP i QR  PR .

b) Calcula els angles del paral·lelogram.

Aplicant el teorema del cosinus en els triangles PQR i PRS:


16  49  25 5
cos       44,42º
56 7

25  49  16 29
cos       34,05º
70 35

138 Matemàtiques 1r Batxillerat


SOLUCIONARI UNITAT 4. VECTORS EN EL PLA

5
a) PQ  PR  4  7  cos   4  7   20 RS  SP  7  4  cos      5,6
7

29
PS  PR  5  7  cos   5  7   29 QR  PR  5  7  cos   29
35

b)     78,47º i 180º       101,53º

81. Descompon una força F newtons en dues més que formin amb aquesta angles de 45° i 60°.

 F 2  F 2  F 2  2  F  F  cos 45º  F 2  F 2  225  15  2  F


 2 1 1  2 1 1
 2 2 2
  2 2
 F1  F2  F  2  F2  F  cos 60º  F1  F2  225  15  F2
 
Substituint la segona equació a la primera:
2 2
F2  F2  225  15  F2  225  15 2  F1  15 F2  15 2 F1  450  F2  2 F1  30  F2  30  2 F1

Substituint a la primera equació:

30  
2 2 2 2
2 F1  F1  225  15  2  F1  900  2 F1  60 2 F1  F1  225  15  2  F1 

2 45 2  15 6
 F1  45 2 F1  675  0  F1   13,45 N , F2  30  2 F1  10,98 N
2

82. En Jordi fa una excursió en tres etapes. En la primera es dirigeix cap a l’est i camina 2 km. En la segona
continua caminant 3 km però aquest cop en direcció sud-oest. Finalment, camina 4 km en direcció nord.
Quina distància el separa del punt de partida quan acaba l’excursió? Fes, utilitzant vectors, un esquema
del trajecte seguit.

Considerant el sistema de referència de la figura, el trajecte d’en Jordi és O  A  B  C :

 4  3 2 3 2   43 2 83 2 
O  0, 0  , A  2, 0  , B  2  3 cos 225º,  3 sin 225º    ,  i C  , 
 2 2  2 2
  

2 2
 43 2   83 2 
La distància del punt de sortida és, per tant: OC        29  18 2  1,88 km
 2 2
   

83. La Lola està passejant la seva gossa Lua. En un moment donat, en què són al punt L, la gossa veu un gat
situat a G i va cap a ell amb una velocitat de 8 km/h i amb una direcció de 30° sobre la direcció de passeig,
tal com mostra la figura. Per la seva banda, la Lola va amb una velocitat de 6 km/h en la direcció del seu
passeig.

Calcula el vector velocitat resultant donant-ne el mòdul i direcció.

139 Matemàtiques 1r Batxillerat


SOLUCIONARI UNITAT 4. VECTORS EN EL PLA

Agafant el sistema de referència centrat en L i amb eixos la direcció del passeig i la seva perpendicular, es poden
escriure vectorialment les velocitats de la Lola i la Lua com a   6, 0  i b   8cos30º, 8 sin30º   4 3, 4 ,  
respectivament.

  6  4 3 
2
Per tant, la velocitat resultant és a  b  6  4 3, 4 , MB amb mòdul  16  13,53 km/h i direcció
17,19º respecte de l’horitzontal.

84. Els mòduls de dos vectors valen 15 i 12 unitats de longitud respectivament. El mòdul de la suma d’aquests
vectors és 8 unitats de longitud. Calcula el producte escalar dels vectors i l’angle que formen.

2 2 2
u v  u  2u  v  v  64  225  2u  v  144  u  v  152,5

u v 152,5
cos     0,8472    147,91º
u v 15  12

85. Un pes està suspès en una corda lligada a dos punts A i B d’igual altura i situats en dues parets que disten
10 m, tal com mostra la figura.

La corda té una longitud de 14 m i el pes està situat a 6 m de A i 8 m de B.


La massa del pes és de 10 kg i, per tant, la força que exerceix és F 98 N.
Calcula els valors de les forces F1 i F2 en els quals es descompon la força de F. Quina interpretació pots fer
a aquestes forces? Calcula els angles que formen amb la força F.

El triangle ABP és rectangle a P ja que 10  6  8 , per tant:


2 2 2

8 4 6 3 4 3
tg    , tg    , sin   cos   i cos   sin  
6 3 8 4 5 5

4 3
F1  98  cos   98   78,4 N , F2  98  cos   98   58,8 N ,   53,13º i   36,87º
5 5

86. Un cotxe viatja a 100 km/h durant 45 minuts en direcció nord fins a arribar a una rotonda on fa un gir de
270°. A la nova carretera, circula durant 30 minuts a una velocitat de 90 km/h.
a) Utilitzant vectors, dibuixa un esquema de la situació.
b) Indica quin és el mòdul, direcció i sentit del vector desplaçament total, sent el seu origen el punt en què comen-
ça el recorregut i el seu extrem el punt en què acaba.

a) b) AR  100∙0,75  75 km, RB  90∙0,5  45 km

AB  752  452  87,46 km

45
BAR  arctg  30,96º
75

140 Matemàtiques 1r Batxillerat


SOLUCIONARI UNITAT 4. VECTORS EN EL PLA

87. En Daniel vol travessar un riu d’una banda a l’altra i de manera perpendicular a totes dues. En Daniel acon-
segueix nedar amb una velocitat de 5 km/h però el corrent del riu té una velocitat de 3 km/h.

a) En quina direcció ha de nedar per aconseguir arribar al punt B situat just davant del punt de sortida A?
b) Quina seria la velocitat resultant?
c) Què passaria si la velocitat que aconsegueix en Daniel fos més baixa que la velocitat del corrent?

3
a)   arcsin 36,87º , en Daniel ha de nedar amb 36,87º respecte de la perpendicular al riu.
5

b) La velocitat resultant serà 52  32  4 km/h.


c) No existiria cap direcció amb la qual en Daniel pogués arribar al punt B situat just davant del punt de sortida A,
ja que l’arcsin no estaria definit en aquest cas.

88. Donat el paral·lelogram ABCD, demostra que la suma dels quadrats de les dues
dia-gonals és igual al doble de la suma dels quadrats de dos costats consecutius
del paral·lelogram. Per fer-ho, ajuda’t del càlcul vectorial i demostra que
2 2 2 2
AB  AD  AB  AD  2 AB  2 AD .

Apliquem les fórmules de l’exercici resolt 27:


2 2 2 2 2 2 2 2
AB  AD  AB  AD  AB  AD  2AB  AD  AB  AD  2AB  AD  2 AB  2 AD

PER APROFUNDIR
89. En la figura adjunta, del triangle ABC es consideren els punts D, E i F sobre les
rectes que contenen els seus costats, de manera que AC  3CF , BC  3EC i
3AD  2AB .
Demostra que D, E i F estan alineats. Per fer-ho:

a) Tria una base convenient i escriu els vectors FE i ED com a combinació lineal dels
vectors de la base

b) Amb ajuda de l’apartat anterior, relaciona els vectors FE i ED .


a) S’agafa la base CA, CB , així: 
1 1 1 1  1 1
FE  FC  CE  CF  EC   AC  BC  CA  CB   , 
3 3 3 3 3 3

ED  EC  CA  AD 
1
3
2 1 2
 1 1
  1 1
BC  CA  AB   CB  CA  CB  CA  CA  CB   , 
3 3 3 3 3 3 3

b) Els vectors FE i ED són iguals per la qual cosa D, E i F estan alineats.

141 Matemàtiques 1r Batxillerat


SOLUCIONARI UNITAT 4. VECTORS EN EL PLA

90. En la figura:
 ABCD és un paral·lelogram.
 M és el punt mitjà del segment AD.

 BN  2NM
Demostra que els punts C, N i A estan alineats.

 1
 
A la base DC, DA tenim: A(0, 1), B(1, 1), C(1, 0), D(0, 0) i M  0, 
 2

1 2  1 2
Per tant, si N  n1, n2  : BN  2NM   n1  1, n2  1   2n1, 1 2n2   n1  , n2   N  , 
3 3 3 3

 2 2
D’aquesta manera, CN    ,  i CA   1, 1 són proporcionals, per tant, C, N i A estan alineats.
 3 3

91. En la figura:
 ABC és un triangle qualsevol.
1
 AE  AB
4

 AC  4 AD
Demostra que les rectes BC i ED són paral·leles.

 1  1
 
A la base AD, AB tenim A(0, 0), B(0, 1), C(4, 0), D(1, 0) i E  0,  . Per tant, BC   4,  1 i ED   1,   són
 4   4 
proporcionals; és a dir, les rectes BC i ED són paral·leles.

92. Amb l’ajuda del càlcul vectorial, demostra el teorema del cosinus en un triangle ABC. Per fer-ho, utilitza l’o-
peració del producte escalar.

  
AB  AC  CB  AB  AB  AC  CB  AC  CB  AC  AC  AC  CB  CB  AC  CB  CB 
2 2
  2
 
 AB  AC  2 AC CB cos AC,CB  CB  c 2  b 2  2ba cos 180º Cˆ  a 2  c 2  a2  b2  2ba cos Cˆ

93. Amb l’ajuda del càlcul vectorial, demostra que les tres altures d’un triangle es tallen en un mateix punt.
Considerem un triangle ABC i les seves altures traçades des de A i B, que es tallaran en un punt H. Volem provar
que H també pertany a l’altura traçada des de C.

Per fer-ho, observem que AH i BC són perpendiculars, com ho són BH i AC , i només cal demostrar que també
ho són CH i AB , és a dir, que CH  AB  0 .

  
CH  AB  CB  BH  BC  CA  CB  BC  CB  CA  BH  BC  BH  CA  CB  BC  CB  CA  BH  BC  0 

 
 CB  BC  CB  CA  HB  CB  CB  BC  CA  HB  CB  HA  0

142 Matemàtiques 1r Batxillerat


SOLUCIONARI UNITAT 4. VECTORS EN EL PLA

94. Demostra que la recta que uneix els punts mitjans dels costats no paral·lels d’un trapezi és paral·lela a les
bases.

Considerem un trapezi ABCD amb costats no paral·lels AB i CD, i siguin M i N els respectius punts mitjans d‘a-
quests costats.

Tenim:

MN  MB  BC  CN 
1
2
1 1 1
AB  BC  BC  CD 
2 2 2
1
2
 1 1

AB  BC  CD  BC  ( AD  BC )
2 2

I, com que AD i BC són paral·lels, es conclou que MN també és paral·lel a aquests dos vectors.

143 Matemàtiques 1r Batxillerat


SOLUCIONARI UNITAT 4. VECTORS EN EL PLA

ENTORN MATEMÀTIC

Perdut al desert
Durant les seves vacances i abans de començar la universitat, en Xavier ha decidit fer un viatge per Mongòlia.
Ara mateix està visitant el desert del Gobi, però com que és un despistat incorregible, ha perdut de vista el guia
que acompanya el seu grup i està perdut.
La falta d’aigua i la calor l’estan consumint a poc a poc i, si no troba cap solució, no trigarà gaire a morir-se. De
sobte, i mentre vaga sense cap criteri, troba un pal ancorat a la sorra amb una inscripció en àrab. Sort que en
Xavier va fer cas de la mare que, ja fa temps, li va dir: “Fill: a més de l’anglès seria convenient que apren-
guessis, com a mínim, una altra llengua. I seria millor que fos alguna dels països emergents: el xinès, l’hindi, el
rus… Tampoc estaria malament que aprenguessis l’àrab”. I va aprendre aquesta llengua.
En Xavier aconsegueix traduir el missatge, que sembla cosa d’un
 Partint d’aquest punt, camina 3 km cap
bromista i en vista de l’alternativa de quedar-se a esperar que el
al nord i hi trobaràs sorra. trobin o intentar sortir pels seus propis mitjans, opta per això
 Després, camina 5 km més dirigint-te últim i elabora un esquema:
cap al sud-oest i hi continuaràs trobant
sorra.
 Finalment, agafa una altra direcció fins
que arribis a 4071 m just al sud del
punt inicial del viatge, on continuaràs
trobant sorra.
 Si tornes al punt inicial i recorres la
mateixa distància i amb la mateixa di-
recció que la del tercer tram del camí
indicat, hi trobaràs un OASI!

a) Calcula el mòdul i l’angle amb la direcció oest-est que forma el tercer tram del camí.
b) Tria una referència cartesiana convenient i estableix les coordenades dels punts del camí i del punt en què està si-
tuat l’oasi.

Si es considera el sistema de referència en què A és l’origen de coordenades i la direcció oestest és l’eix d’abscisses,
tenim A(0, 0), B(0,3), C  c1, c2  i D(0; –4,071).

Les condicions del problema impliquen:

 BC  5 c12  (c2  3)2  25


  65 2 5 2  5 2 65 2 
  15 2  c2  , c1    C , 
15 2 
BA  BC  15 cos 45º  3(c2  3)  2 2  2 2 
 2  2

5 2 65 2 
L’oasi es troba en el punt O tal que AO  CD , és a dir, O 
 2
;  4,071    3,5355;  4,6065  .
 2 

Per tant, la longitud del tercer tram és CD 3,53552  ( 4,6065)2 5,8069 km i l’angle que forma amb la direcció
 4,6065 
oestest és arctg   307,51º .
 3,5355 

144 Matemàtiques 1r Batxillerat


SOLUCIONARI UNITAT 4. VECTORS EN EL PLA

Aventura al Tuul
Dies més tard i després de l’aventura al desert, en Xavier, que encara no està prou cansat d’aventures, ha anat
al parc nacional Khustain Nuruu. En un cert moment, es troba al punt A i es vol dirigir al punt B, però travessant
el riu Tuul de manera perpendicular a les seves ribes.

a) Amb l’ajuda de GeoGebra, dibuixa i calcula la distància recorreguda agafant els camins A  C1  D1  B i
A  C2  D2  B .
b) També amb GeoGebra, mitjançant el tempteig i calculant diversos camins, intenta aproximar el camí de mínima lon-
gitud per anar de A a B.

A  C1  D1  B : 5 (7,21  4  10,2)  107,05 m


A  C2  D2  B : 5 (13,6  4  3,61)  106,05 m.
El camí més curt és
A  C  D  B : 5 (11,39  4  5,70)  104,44 m

145 Matemàtiques 1r Batxillerat


SOLUCIONARI UNITAT 4. VECTORS EN EL PLA

AUTOAVALUACIÓ

Comprova el que has après


1. Donats els vectors u i v de la figura:

a) Calcula les coordenades de u  v i 2u  3v

b) Calcula l’angle que formen u i v

a) u   3, 2 i v  1,  3  , per tant, u  v   4,  1 i 2u  3v   6, 4  3,  9   3, 13 

3 3 130
b) cos       105,26º
13 10 130

2. Donats els punts A(2, 4), B(4, 2) i C(3, 1):


a) Calcula el punt D perquè ABCD sigui un paral·lelogram.
b) És l’anterior paral·lelogram un rectangle?

a) Si D  d1, d2  , tenim: AB  DC   6,  2   3  d1,  1 d2   d1  3, d2  1  D  3, 1

b) AB  AD   6,  2  1,  3  6  6  0  AB i AD són perpendiculars i ABCD sí que és un rectangle.

3. a) Calcula l’extrem del vector AB , sabent que A(3, 4) i que és equipol·lent u  (2, 2).

b) Calcula l’origen del vector AB , sabent que B(5, 1) i que és equipol·lent u  (–2, 3).

a) Si B  b1, b2  , tenim: AB  u   b1  3, b2  4   2, 2  b1  1, b2  6  B  1, 6

b) Si A  a1, a2  , tenim: AB  u   5  a1,  1 a2    2, 3  a1  7, b2  4  A  7,  4

4. Calcula els angles del triangle A(3, 2), B(3, 5) i C(2, 0), i comprova que es tracta d’un triangle rectangle.

AB  AC
AB   6, 3  y AC  1,  2  cos Aˆ   0  Aˆ  90º . El triangle és rectangle.
AB AC

BA  BC 45 3 3 10
BA   6,  3  i BC   5,  5   cos Bˆ      Bˆ  18,43º
BA BC 45  50 10 10

CA  CB 5 1 10
CA   1, 2 i CB   5, 5   cos Cˆ      Cˆ  71,57º
CA CB 5  50 10 10

5. a) Calcula tots els vectors que siguin paral·lels a u   9, 12 i que tinguin mòdul 5.

b) Calcula tots els vectors que siguin perpendiculars a u 10,  24  i que tinguin mòdul 13.

a) Tots els vectors paral·lels a u són de la forma  3t, 4t  amb t  , cosa que obliga que tinguin mòdul 5:

5  ( 3t )2  (4t )2  5t  t  1, t  1 . Tenim, per tant, dues solucions, v   3, 4  i w   3,  4 

146 Matemàtiques 1r Batxillerat


SOLUCIONARI UNITAT 4. VECTORS EN EL PLA

b) Els vectors perpendiculars a u són de la forma 12t, 5t  amb t  , cosa que obliga que tinguin mòdul 13:

13  (12t )2  (5t )2  13t  t  1, t  1 . Tenim, per tant, dues solucions, v  12, 5 i w   12,  5 

6. Donats els vectors u   3, 5  i v   10, 3 calcula la projecció de u sobre v .

u v 15 15 109
proyv u   
v 109 109

7. Determina l’angle dels vector u i v sabent que u  4 , v  6 i u  v  8 .

2 2 2
u  v  u v 16  36  64 u v 6 1
u v    6  cos         104,48º
2 2 u v 24 4

8. Escriu el vector a   10, 12  com a combinació lineal dels vectors b   3,  4  i c  1,  1 . Formen b i c
2
una base de V ? Si és així, indica les coordenades de a en aquesta base.

1 1
  b i c formen una base de V .
2
3 4

3a  a  10
a  a1b  a2 c   1 2  a1  2, a2  4  a  2b  4c   2,  4 
4a1  a2  12

9. En l’hexàgon ABCDEF, calcula les coordenades de: AF  EF  ED i 2AB  3EF  4 AF .

AF  EF  ED   2, 0   2, 2   2,  2   2,  4 

2AB  3EF  4AF  2  2,  2  3  2, 2  4  2, 0    10,  10 

Relaciona i contesta
Tria l’única resposta correcta en cada cas

1. Es considera la figura següent.

A. FN  FH  IC C. FN  FH  IC

B. FN  FH  CI D. FN  FH  CI

La resposta correcta és la D.

147 Matemàtiques 1r Batxillerat


SOLUCIONARI UNITAT 4. VECTORS EN EL PLA

2. L’hexàgon de la figura és regular i el seu costat val 1. El producte escalar AC  AE val:

3
A. 1 C.
2

3
B. 2 D.
2

3
L’angle que formen els dos vectors és 60º, i els seus mòduls valen 3 , per tant, AC  AE  3  3  cos 60º  , la
2
resposta C.

3. El producte escalar de dos vectors no nuls és negatiu. L’angle que formen aquests vectors és:
A. Més petit que 90º B. Més gran que 90º C. Més petit que 45º D. 360º

Tenint en compte que el cosinus de l’angle serà negatiu, la resposta correcta és la B.

Assenyala, en cada cas, les respostes correctes

1 3 1
4. Es consideren els vectors u   ,   i v   3, 2 
2 4 4
A. Els vectors tenen el mateix mòdul. C. Els vectors són linealment independents.
B. Els vectors són ortogonals. D. Els vectors són linealment dependents.

1 9 13 9 4 13
La A és correcta, ja que u    i v    .
4 16 4 16 16 4

3 6
La B és correcta, ja que u  v   0.
8 16

La C és correcta, per tant, la D no ho és, ja que les coordenades de u i v no són proporcionals.

5. Es consideren els punts A(2, 1) i B(2, 1).


A. L’origen de coordenades està alineat amb aquests.
B. El punt C(4, 3) determina amb aquests un triangle rectangle.
C. El punt C(4, 3) determina amb aquests un triangle isòsceles.
D. El producte escalar dels vectors de posició de A i B val 5.

La A és correcta, ja que OA  (2, 1) i OB  (2,  1) són proporcionals.

La B és correcta, ja que BA  BC   4, 2   2, 4  8  8  0 .

La C és correcta, ja que BA  16  4  2 5 i BC  4  16  2 5 .

La D no és correcta, ja que OA  OB  4  1  5 .

148 Matemàtiques 1r Batxillerat


SOLUCIONARI UNITAT 4. VECTORS EN EL PLA

Tria la relació correcta entre les dues afirmacions donades


2
6. Siguin v i w vectors de V . Es consideren les afirmacions:

1. El vector u   2,  3  es pot escriure com a combinació lineal de v i w .

2. v i w són linealment independents.

A. 1  2 C. 2  1 però 1  2

B. 1  2 però 2  1 D. 1 i 2 són excloents entre si.

Òbviament 2  1 , però el recíproc no és cert, n’hi ha prou amb agafar com a exemple v  u i w  0 , per tant, la
resposta correcta és la C.

Assenyala la dada innecessària per contestar

7. Es vol calcular l’angle que formen els vectors u i v . Per fer-ho es donen les dades següents:

1. u 8 2. v 6 3. u v  5 4. u  v  37,5

A. Únicament es pot eliminar la dada 1. C. Únicament es pot eliminar la dada 3.


B. Únicament es pot eliminar la dada 2. D. Es pot eliminar qualsevol de les quatre.

2 2 2
Com que u  v  u  2u  v  v , es pot deduir qualsevol d’aquestes dades a partir de les altres tres, per tant, la
resposta correcta és la D.

149 Matemàtiques 1r Batxillerat


SOLUCIONARI UNITAT 5. GEOMETRIA ANALÍTICA

5. Geometria analítica
EXERCICIS PROPOSATS
1 i 2. Exercicis resolts.
3. Comprova si els punts A(2, 3), B(2, 3) i C(2, 5) pertanyen o no a la recta que passa per P(2, 6) i té com a
vector director v  (0, 3). Calcula dos punts més d’aquesta recta.

La recta que passa per P(2, 6) i té com a vector director v  (0, 3), que és paral·lel a l’eix Y, és la recta vertical r :
x 2, per tant, únicament els punts que tenen per abscissa 2 pertanyen a la recta, és a dir, A i C pertanyen a r
però B no.

4. Calcula les equacions de les rectes paral·leles als eixos que passen pel punt A(3, 5).

La recta paral·lela a l’eix X que passa per A és y 5, i la recta paral·lela a l’eix Y que passa per A és x 3.

5. Indica dos punts i el vector director de la recta r: 8x y  7.

Dos punts de la recta són, per exemple, A(1, 1) i B(0, 7), el vector director és AB   1, 8 .

6. En cada cas, calcula l’equació general de la recta que passa pels punts:
a) A(2, 5) i B(1, 3) b) A(2, 4) i B(3, 2)

a) El vector director és AB   1, 2 i la recta passa per A(2, 5). Per tant:

x 2 y 5
  2x  4   y  5  2x  y  1  0
1 2

b) El vector director és AB   5, 2 i la recta passa per A(2, 4). Per tant:

x2 y 4
  2x  4  5y  20  2x  5y  16  0
5 2

7. Troba k perquè r : (k  5)x  (3 k)y  1 k passi per P(2, 3).

Substituint les coordenades de P a l’equació de la recta tenim 2(k  5)  3(3  k )  1 k  0  0 , per tant, el punt P
pertany a r per a qualsevol valor real de k.

8. Troba el valor de k perquè la recta que passa pels punts A(2, 1) i B(3, k) passi pel punt C(1, 4).

C pertany a la recta que passa per A i B si i només si els vectors AB  (1, k  1) i AC  (1,  3) són proporcionals.
Per tant:
1 k 1
  k  1  3  k  2
1 3

9 a 11. Exercicis resolts.

150 Matemàtiques 1r Batxillerat


SOLUCIONARI UNITAT 5. GEOMETRIA ANALÍTICA

12. Indica un vector director i un altre de normal de la recta d’equació 3x  2y  4  0.

Un vector normal és n  (3, 2) i un vector director és u  (2, 3).

 1
13. Troba un vector director i un altre de normal de la recta que passa pel punt A  2,  i per l’origen de coor-
 3
denades.

 1
Vector director: OA   2,  ( 6, 1) Vector normal: n  (1, 6)
 3

14. Una recta té com a vector normal a n  (2, 3) i passa pel punt A(1, 2). Escriu-ne l’equació general.

L’equació general és de la forma 2x  3y k 0. Com que la recta passa per A, ha de ser 2  6  k  0  k  8 .

Per tant, l’equació general de la recta és 2x 3y 8  0.

15. Troba l’equació normal de la recta que passa per P(1, 3) i és perpendicular a la que passa per A(0, 2) i
B(1, 0).

La recta té vector director AB  (1,  2)  (1, 2) i passa pel punt P, per tant, la seva equació normal és:

(x  1)  2(y  3)  0 o x  2y  5  0.

16. Escriu l’equació de la recta perpendicular a 3x−6y = 1 i que passa pel punt A(3, 2).
Totes les perpendiculars a 3x 6y 1 són de la forma 6x 3y k 0. Fent a que A pertanyi a la perpendicular
tenim 18  6  k  0  k  12 i, per tant, l’equació de la recta perpendicular és
6x  3y  12  0  2x  y  4  0.

17. Troba l’equació de la recta perpendicular a 5x  4y  3  0 que talla la recta 6(x  1)  (y  1)  0 a x  1.

Totes les rectes perpendiculars a 5x  4y  3  0 són de la forma 4x  5y  k  0. Fent que talli


6(x  1)  (y  1)  0 en x  1, és a dir, a que passi pel punt (1, 1) tenim 4  5  k  0  k  1. Per tant, l’equació de
la recta perpendicular és 4x  5y  1  0.

18. Troba l’equació de la recta perpendicular al segment d’extrems A(0, 2) i B(1, 4) i que passa pel punt C(3, 0).

La recta té vector normal AB  (1, 6) i passa pel punt C, per tant, la seva equació és:

1( x  3)  6( y  0)  0  x  6y  3  0

19. Sigui el triangle de vèrtexs A(5, 3), B(7, 1) i C(1, 1). Troba l’equació de l’altura corresponent al vèrtex A.

L’altura corresponent al vèrtex A passa per aquest punt i és perpendicular a BC  (6, 0) (1, 0) , per tant, la seva
equació és (x  5)  0(y  3)  0  x  5.

151 Matemàtiques 1r Batxillerat


SOLUCIONARI UNITAT 5. GEOMETRIA ANALÍTICA

20. Exercici resolt.

21. Calcula l’equació de la recta que passa pel punt A(–2, 4) i té pendent m = 3.

y  4  3  x  2  y  4  3x  6  y  3x  10

22. Indica el vector director i el pendent de les rectes:

 x  3 y 1
a) 2x  y  7  0 b)  c) x  4 
y  1   2

2
a) Vector director: u  (1, 2) . Pendent: m  2.
1

1
b) Vector director: u  (3,  1) . Pendent: m   .
3

2
c) Vector director: u  (1, 2) . Pendent: m  2.
1

23. Calcula el pendent i l’ordenada a l’origen de la recta que passa pels punts:
a) A(1, 5) i B(2, 2) b) A(1, 2) i B(2, 1)

Sigui y  mx n l’equació explícita de la recta, on m és el pendent i n l’ordenada en l’origen. Els punts donats han
de complir l’equació, per la qual cosa tenim:

m  n  5 7 8 m  n  2
a)  m ,n b)   m  3, n  5
2m  n  2 3 3 2m  n  1

24. Troba l’equació explícita i la punt-pendent de la recta que passa per A(0, 5) i és perpendicular a
3x  5y  2  0.

5
La recta buscada té vector director u  (3, 5) , per tant, té pendent m 
, a més, passa per A, amb la qual cosa la
3
5 5
seva equació punt-pendent és y  5  x i la seva equació explícita és y  x  5 .
3 3
25. Exercici interactiu.

26 a 28. Exercicis resolts.

29. Indica, en cada cas, si les rectes r i s són paral·leles o secants i, en l’últim cas, obtén el punt d’intersecció:

 x  2  4t  x  2s r : 2x  5y  5  0
a) r :  s: b) 
y  3  2t y  s s : 3 x  5y  5  0

2  4t  2s 2t  s  1
a)   Com que el sistema és incompatible, les dues rectes són paral·leles.
3  2t  s 2t  s  3

2 5 2x  5y  5  0
b)   Són rectes secants. Calculem el punt de tall:   x  0, y  1  P  0, 1
3 5 3 x  5y  5  0

152 Matemàtiques 1r Batxillerat


SOLUCIONARI UNITAT 5. GEOMETRIA ANALÍTICA

30. Calcula l’equació de la recta paral·lela a la recta r: 2x  y  1 0 i que passa pel punt d’intersecció de les
rectes s: x  y  5  0 i t: x  y 1  0.

Es calcula el punt d’intersecció de s i t :

x  y  5  0
  x  3, y  2  P  3, 2 
x  y  1  0
El feix de rectes paral·leles a r té equació 2x  y  K  0 . Com que la recta que es busca passa per P, tenim:

6  2  K  0  K  4 . Per tant, l’equació de la recta és 2x  y  4  0 .

31. Troba l’equació del feix que té per vèrtex P(5, 4).

y  4  m( x  5), m    x  5

32 i 33. Exercicis resolts.

34. Calcula k perquè la distància entre les rectes 5x  12y k  0 i 5x 12y 15  0 sigui 2.

Observem que les rectes donades són paral·leles. Tenim:


k  15 15  k  26 k  41
d (r , s )  2   2  k  15  26   
5  12
2 2  15  k  26 k  11

35. Calcula l’àrea del triangle determinat per O(0, 0) i les interseccions de la recta x  2y  4 amb els eixos.

El triangle té com a vèrtexs O(0, 0),A(0, 2) i B(4, 0).


base·altura
L’àrea del triangle és A  amb: base  d (O, B)  4 u , altura  d (O, A)  2 u
2

base·altura 4·2
Per tant, A    4 u2
2 2

36. Calcula la mida de les altures del triangle de vèrtexs A(4, 1),B(1, 3) i C(0, 4).

AB : 2x  5y  13  0, BC : x  y  4  0 i AC : 3x  4y  16  0 , per tant, les altures mesuren:

20  13 7 7 29 4  1 4 7 7 2
hAB  d (C, AB )    u hBC  d ( A, BC )    u
4  25 29 29 1 1 2 2

3  12  16 7
hAC  d (B, AC )   u
9  16 5

37. Troba la distància del punt C(10, 0) a la recta que passa per A(2, 3) i B(2, 2). Quina és la posició relativa de
A, B i C?
x 2 y 3
La recta AB té vector director AB   4,  1 i passa per A, la seva equació és:   x  4y  10  0 .
4 1

10  10
d (C, AB )   0 u , cosa que significa que C pertany a la recta AB, és a dir, A, B i C estan alineats.
1  16

38. Exercici resolt.

153 Matemàtiques 1r Batxillerat


SOLUCIONARI UNITAT 5. GEOMETRIA ANALÍTICA

39. Calcula l’angle que formen les rectes:


a) r: 3x  4y  0 i s: 2x  2y  3  0 b) r: y x  5 i s: y  2x  2

nr  ns 2 2
a) Vectors normals: nr  (3,  4) i ns  (2, 2) . Per tant: cos   cos(r , s )       81,87º
nr  ns 10 2 10

m1  m2 1 2 1
b) Els pendents són m1  1 i m2  2 , per tant, tg       18,43º .
1  m1m2 1 2 3

40. Calcula l’angle format per r: 4x  2y  7  0 i l’eix Y.

n1  n2 4 2 5
Els vectors normals són n1  (4, 2) i n2  (1, 0) , per tant, cos        26,57º
n1 n2 2 5 5

41. Calcula la recta perpendicular a r : 4x y 3  0 i que passa pel punt P(3, 3).

El pendent de la recta r és m  1 , per tant, el pendent d’una recta perpendicular a aquesta és m  1 .


Així doncs, l’equació de la recta buscada és y  3  x  3  y  x  6

42 i 43. Exercicis resolts.

44. Obtén el simètric de P(2, 3) respecte del punt M(1, 4).

Sigui P(a, b) el punt buscat, M és el punt mitjà del segment PP  . Per tant:

 2  a
 1
 2  a 3  b   2
 ,   1,  4     a  4, b  11  P   4,  11
 2 2   3  b  4

 2

45. Troba la recta simètrica de l’eix d’ordenades respecte de y  x  1.

Calculem el punt Q d’intersecció de totes dues recte ja que és l’únic invariant per la simetria: Q(0, 1).
Per calcular la recta simètrica indicada n’hi ha prou amb determinar el simètric P d’un altre punt qualsevol, P, de
l’eix d’ordenades, ja que la recta buscada quedarà determinada pels punts Q i P.
Agafant P(0, 1), la recta perpendicular a y  x  1 que passa per P és y x  1.

Totes dues rectes es tallen a M(1, 0) i M ha de ser el punt mitjà del segment PP  , per la qual cosa P(2, 1).
Per tant, la recta simètrica que es busca és la que passa per Q i P, l’equació de la qual és y  1.

46. Determina el triangle simètric de k(4, 0), B(1, 6) i C(1, 1) respecte de la simetria central amb centre a l’o-
rigen de coordenades.

El simètric d’un punt P(a, b) respecte de la simetria central amb centre O(0, 0) és P(a, b).
Per tant, els vèrtexs del triangle simètric són A(4, 0), B(1, 6) i C(1, 1).

154 Matemàtiques 1r Batxillerat


SOLUCIONARI UNITAT 5. GEOMETRIA ANALÍTICA

47. Troba el triangle simètric de A(3, 0), B(0, 3) i P(2, 2) respecte de la simetria axial amb eix la recta
y  x.

El simètric d’un punt P(a, b) respecte de la simetria axial amb eix la recta y  x és P(b, a).
Per tant, els vèrtexs del triangle simètric són A(0, 3), B(3, 0) i C(2, 2).

48. Troba l’extrem B del segment AB sent A(2, 1) i sabent que la mediatriu del segment és
r : x  2y  9  0.

Sigui B(a, b), com que la mediatriu és perpendicular a AB   a  2, b  1 , aquest vector ha de ser proporcional a
nr  (1, 2) . A més, el punt mitjà del segment AB ha de pertànyer a r, per tant:

a  2 b 1
 1  2
 2a  b  3
   a  4, b  5  B(4, 5)
 2  a 1  b a  2b  14
2 9  0

 2 2

49. Exercici interactiu.

50. Troba la mediatriu del segment d’extrems A(1, 3) i B(5, 3).

Els punts P(x, y) de la mediatriu compleixen que: d ( A, X )  d (B, X )  ( x  1)2  ( y  3)2  ( x  5)2  ( y  3)2 

 x 2  2x  1 y 2  6y  9  x 2  10x  25  y 2  6y  9  x  y  2  0

51. Troba el punt de la recta r: x  3y  11  0 que equidista dels punts A(2, 3) i B(6, 1).

Els punts que equidisten de A i B pertanyen a la mediatriu del segment AB , d’equació:

( x  2)2  ( y  3)2  ( x  6)2  ( y  1)2  x 2  4 x  4  y 2  6y  9  x 2  12x  36  y 2  2y  1  2x  y  3  0

 x  3y  11  0  2 19 
Per tant, el punt P que es busca és la intersecció de la mediatriu amb r :   P  ,  
2 x  y  3  0  5 5 

52. Donades les rectes r: x  3y  4  0 i s: x y  0, obtén les seves bisectrius, comprova que es tallen en el
punt d’intersecció de r i s i que són perpendiculars.

Els punts P(x, y) de les bisectrius compleixen:

d (P, r )  d (P, s ) 
x  3y  4

xy

x  3y  4

xy
b :
 1

   5  3 y  4  0
5 1 x 

10 2 10 2 b2 :
  5  1 x   5  3  y  4  0

 x  3y  4  0
El punt de tall de les rectes és:   x  1, y  1  P  1 1
x  y  0



P compleix l’equació de cada una de les bisectrius: 
  
5  1 ( 1)  
5  3  1 4   5  1 5  3  4  0


  5  1 ( 1)   5  3 1 4   5  1 5  3  4  0

Els vectors normals a b1 i b2 són, respectivament, n1   5  1, 5  3  i n2   


5  1, 5  3 , que compleixen

n1 ·n2  5  1 5  9  0 , per la qual cosa totes dues bisectrius són perpendiculars.

155 Matemàtiques 1r Batxillerat


SOLUCIONARI UNITAT 5. GEOMETRIA ANALÍTICA

53. Troba els punts de la recta r :y x  6 que equidisten de les rectes s: 3x y 1 i t : 3x  y  5.

Els punts que equidisten de s i t són els que pertanyen a les seves bisectrius, d’equacions:

3x  y  1 3x  y  5 b : y  2
  3x  y  1    3x  y  5   1
10 10 b2 : x  1

Els punts que es busquen són, per tant, els punts de tall de r amb cada una de les bisectrius: P1 (4, 2) i P2 (1, 5).

54. Donat el triangle de vèrtexs A(5, 1), B(3, 7) i C(2, 3):


a) Calcula el circumcentre. b) Calcula l’incentre. c) Calcula el baricentre.

a) El circumcentre, T, és el punt de tall de les mediatrius del triangle. Per tant:

 (5  x )2  (1  y )2  (3  x )2  (7  y )2 4 x  12y  32
 71 125  71 125 
  x ,y T  , 
  2
  2
   2
  2  14 x  4 y  13 38 38  38 38 
 (5 x ) (1 y ) ( 2 x ) (3 y )

b) L’incentre, I, és el punt de tall de les bisectrius interiors del triangle. Per tant:
Recta AB: 3x y 16  0 Recta BC: 4x  5y 23  0 Recta AC: 2x  7y 17  0

 3 x  y  16 4 x  5y  23
 
 10 41 (3 53  2 10 )x  ( 53  7 10 )y  17 10  16 53
   I (2,06; 3,82)
 3 x  y  16  2x  7y  17 (3 41  4 10 )x  ( 41  5 10 )y  16 41  23 10


 10 53

c) El baricentre, G, és el punt de tall de les mediatrius del triangle. Per tant:


1 
Punt mitjà de AB: M1 (4, 4) Punt mitjà de BC: M2  , 5 
2 

CM1 : x  6y  20  0 11  11 
  x  2, y   T  2, 
 AM2 : 8  9 y  49  0 3  3

55 a 62. Exercicis resolts.

EXERCICIS
Equacions de la recta
63. Per a cadascuna de les rectes següents, indica si els punts P(2, 1) i Q(3, 1) pertanyen o no a les rectes i
calcula un punt més de cada una.

 x  2   x 1 y  2
a) r1 :  x, y    7,  2    4,  1 b) r2 :  c) r3 : 2x  5y  1 d) r4 : 
y  1  2 2 5

a) Substituint les coordenades de tots dos punts a l’equació, tenim:

 9
2  7  4   
 2, 1   7,  2    4,  1    4  P  r1
1  2   
  3

3  7  4   1
 3,  1   7,  2    4,  1     Q  r1
 1  2     1

Per calcular un punt més n’hi ha prou amb donar valor a  , per exemple, agafant   0 obtenim el punt R(7,
2).

156 Matemàtiques 1r Batxillerat


SOLUCIONARI UNITAT 5. GEOMETRIA ANALÍTICA

b) Substituint les coordenades de tots dos punts a les equacions, tenim:

2  2     0 3  2     5
   P  r2    Q  r2
1  1  2   0  1  1  2   1

Per calcular un punt més n’hi ha prou amb donar valor a  , per exemple, agafant   1 obtenim el punt R(1,
3).
c) Substituint les coordenades de tots dos punts a l’equació, tenim:

2(2)  5·1  1  P  r3 2·3  5( 1)  1  Q  r3 Un altre punt de la recta és, per exemple, R(7, 3)

d) Substituint les coordenades de tots dos punts a l’equació, tenim:


2  1 1  2 3  1 1  2
  P  r4   Q  r4
2 5 2 5

64. Obtén l’equació vectorial i les equacions paramètriques de cadascuna de les rectes següents.

a) La recta que passa pel punt P(−3, 1) i té la direcció del vector u  (1, 2).
b) La recta que passa pels punts A(2, 3) i B(1, 4).

c) La recta que té com un dels seus vectors de direcció u  (3, 3) i talla la part positiva de l’eix d’abscisses en un
punt que dista 3 unitats de l’origen de coordenades.

 x  3  
a) E. vectorial: r :  x, y    3, 1    1,  2 E. paramètriques: r : 
y  1  2

x  2  
b) Vector director: AB   1, 7  E. vectorial: r :  x, y    2,  3    1, 7  E. paramètriques: r : 
 y  3  7

 x  3  3
c) E. vectorial: r :  x, y    3, 0    3, 3 E. paramètriques: r : 
y  3

65. Per a cadascuna de las rectes següents, determina l’equació contínua i l’equació general.

a) Passa pel punt A(3, 4) i té la direcció del vector u  (1, 2).
b) Passa pels punts P(2, 5) i Q(5, 1).
c) Passa per l’origen de coordenades i pel punt B(3, 4).
x 3 y 4
a) Equació continua:  Equació general: 2x  6  y  4  2x  y  10  0
1 2

x 2 y 5
b) Equació contínua:  Equació general: 6x  12  3y  15  2x  y  9  0
3 6

x y
c) Equació contínua:  Equació general: 4x  3y  0
3 4

66. Troba un vector director i un de normal en cadascuna de les rectes següents.


a) r : 2x  3y  5

3
b) s : x  y 1 0
2
c) Passa pels punts A(2, 5) i B(5, 1).

157 Matemàtiques 1r Batxillerat


SOLUCIONARI UNITAT 5. GEOMETRIA ANALÍTICA

a) Vector normal: n  (2, 3) Vector director: u  (3, 2)

 3 3 
b) Vector normal: n   1,   Vector director: u   , 1
 2 2 

c) Vector director: u  AB   7, 4 Vector normal: n   4, 7 

67. Obtén les equacions dels costats del triangle de vèrtexs P(1, 3), Q(4, 0) i R(2, 1). Per a cada costat, tro-
ba un vector de direcció i un de normal.

Costat PQ. Un vector de direcció és u  PQ   5,  3 . Un vector normal és n   3,  5  .

x 1 y  3
L’equació és   3 x  3  5y  15  3 x  5y  12  0
5 3

Costat QR. Un vector de direcció és u  QR   2,  1 . Un vector normal és n  1, 2  .

x  4 y 0
L’equació és    x  4  2y  x  2 y  4  0
2 1

Costat PR . Un vector de direcció és u  PR   3,  4 . Un vector normal és n   4,  3  .

x 1 y  3
L’equació és   4 x  4  3y  9  4 x  3y  5  0
3 4

68. Troba les equacions punt-pendent de les rectes r, s, t i t  de la figura.

La recta r passa per P1 (1, 1) i té pendent m1  2 , així, la seva equació punt-pendent és y  1  2( x  1) .

La recta s passa per P2 (3, 1) i té pendent m2  2 , així, la seva equació punt-pendent és y  1  2( x  3) .

La recta t passa per P3 (1, 1) i té pendent m3  2 , així, la seva equació punt-pendent és y  1  2( x  1) .

La recta t  passa per P4 (3, 1) i té pendent m4  2 , així, la seva equació punt-pendent és y  1  2( x  3) .

69. Troba les equacions paramètriques de les rectes:


1 3 x 4 y 5
a) r : y  2x  3 b) s : 4x  3y  6  0 c) t : x  y 1  0 d) w : 
2 4 2 3

x  
a) La recta passa per P(0, 3), vector director u  1,  2 . Les equacions són: r : 
y  3  2

 x  3
b) La recta passa per P(0, 2), vector normal n   4, 3  i director u   3, 4  . Les equacions són: s : 
 y  2  4

1 3  x  2  3
c) La recta passa per P(2, 0), vector normal n   ,   2, 3  i director u   3, 2 . Equacions: t : 
2 4  y  2

 x  4  2
d) La recta passa per P(4, 5), vector director u   2, 3  . Les equacions són: w : 
y  5  3

158 Matemàtiques 1r Batxillerat


SOLUCIONARI UNITAT 5. GEOMETRIA ANALÍTICA

70. Troba l’equació normal i l’equació general de la recta que té n   2, 4  com a vector normal i passa pel
punt mitjà del segment AB sent A(0, 2) i B(3, 0).

 3 
El punt mitjà del segment AB és M   ,  1
 2 

 3
L’equació normal és 2  x    4  y  1  0 L’equació general és 2x  4y  7  0
 2

71. Calcula el pendent de les rectes següents:


a) r : y  2x  3

b) r : 2x  3y  5  0

c) Recta que passa pels punts P(1, 2) i Q(1, 3).

 x  3  5
d) r : 
 y  1  2

a) m  2
2 5 2
b) 2x  3y  5  0  y  x m
3 3 3

1
c) El vector director és u  PQ   2, 1 , per tant, el pendent és m 
2

2
d) El vector director és u   5, 2 , per tant, el pendent és m 
5

72. Indica el valor dels pendents i de les ordenades a l’origen de les rectes de la figura i determina, per a cada
una d’elles, la seva equació general.
Recta r :
2 2
m   , n  3  y   x  3  2x  3y  9  0
3 3
Recta s:
1 1
m , n  3  y  x  3  x  3y  9  0
3 3
Recta t:
m   , no té ordenada a l’origen  x  5  0

Recta u: m  0, n  4  y  4  0

73. Troba l’equació explícita de la recta que passa pel punt P(2, 5) i forma amb la part positiva de l’eix d’or-
denades un angle de 60°.

La recta forma un angle de 30º amb la part positiva de l’eix d’abscisses, per la qual cosa té pendent
3
mtg (30º)  .
3

3 3 2 3  15
L’equació explícita és: y  5  ( x  2)  y  x
3 3 3
74. Obtén les equacions explícites de les rectes següents.

159 Matemàtiques 1r Batxillerat


SOLUCIONARI UNITAT 5. GEOMETRIA ANALÍTICA

a) Passa per A(1, 2) i té pendent m 2.


b) Passa pels punts A(1, 3) i B(2, 4).
c) Passa per A(2, 3) i forma amb la part dreta de l’eix d’abscisses un angle de 30°.
d) Passa per A(2, 5) i forma, amb la part esquerra de l’eix d’abscisses, un angle de 120°.

a) y  2  2( x  1)  y  2x  4

x 1 y  3 1 10
b) El vector director és u  AB   3, 1 , l’equació explícita és:   x  1  3y  9  y  x 
3 1 3 3

3 3 3 92 3
c) El pendent és m tg (30º)  , l’equació explícita és: y  3  ( x  2)  y  x
3 3 3 3
d) La recta forma un angle de 60º amb la part dreta de l’eix d’abscisses, per la qual cosa té pendent
 
m tg (60º)  3 , per tant, l’equació explícita és: y  5  3( x  2)  y  3 x  2 3  5 .

Posicions relatives de rectes


 x  2  2t
75. Troba l’equació de la recta que passa pel punt P(2, 6) i és paral·lela a r :  ,t .
 y  1  t

La recta que es busca té vector director u   2,  1 , per tant, l’equació és:

x 2 y 6
   x  2  2y  12  x  2y  14  0
2 1

76. Estudia les posicions relatives dels parells de rectes següents.

 3t
 x
x  1   2
a) r :  s:
y  1   y  1  t

 2

b) r : 3x  2y  7 s : 2x  3y  8

1 1 7
c) r : x  y  7 s :  x y 0
2 2 2

1 1
a) Els vectors directors són ur  1,  1 i us   ,   . Com que són proporcionals i el punt P(1, 1) pertany a to-
2 2
tes dues rectes, les rectes són coincidents.
3 2
b)   Les rectes són secants.
2 3
c) L’equació de s es pot escriure com x  y  7  0, que és la mateixa equació de r, per la qual cosa les rectes són
coincidents.

77. Calcula el punt d’intersecció dels següents parells de rectes secants.

 x  2  3  x  1  4
a) r :  s:
 y  1    y  2  2

23
b) r : 2x  5y   s : 3x  4y  12
2

160 Matemàtiques 1r Batxillerat


SOLUCIONARI UNITAT 5. GEOMETRIA ANALÍTICA

2  3  1  4 3  4  1
a)      1,   1  El punt d’intersecció és P(5, 0)
1    2  2   2  1

 23
2x  5y   4 x  10y  23 3  3
b)  2   x  2, y   El punt d’intersecció és P  2, 
 3 x  4 y  12 2  2
3 x  4y  12

78. Troba l’equació de la recta que passa pel punt d’intersecció de les rectes r: 2x 3y  1  0 i
s: 4x y  3  0 i talla l’eix d’abscisses al punt P(4, 0).

2x  3y  1  0 4 5 4 5
  x  , y   r i s es tallen al punt Q  ,  . El vector director de la recta buscada és
 4 x  y  3  0 7 7 7 7
 24 5  x 4 y 0
u  PQ    ,   24, 5 , per tant, l’equació és:   5x  20  24y  5 x  24y  20  0
 7 7 24 5

79. Troba l’equació dels següents parells de rectes paral·leles a la recta donada.
a) Paral·lela a 2x  5y  5  0 i que passa pel punt A(2, 6).

b) Paral·lela a l’eix d’abscisses i que passa pel punt A(1, 4).


c) Paral·lela a l’eix d’ordenades i que passa pel punt A(1, 4).

 x  1  3t
d) Paral·lela a r :  i que passa pel punt P(2, 4).
y  5  t

a) La recta és de la forma 2x  5y  k  0 . Com que passa per A, ha de ser 4  30  k  0  k  26 . Per tant,
l’equació que es busca és 2x  5y  26  0 .

b) La recta és y  4 .

c) La recta és x  1 .
x2 y 4
d) La recta té vector director u   3, 1 , així, l’equació és:   x  2  3y  12  x  3y  14  0 .
3 1

80. Obtén l’equació de les següents rectes.


a) Perpendicular a x  2y  3  0 i que passa pel punt A(2, 1).
b) Paral·lela a l’eix d’abscisses i que passa pel punt A(4, 8).
c) Paral·lela a l’eix d’ordenades i que passa pel punt A(1, 3).

 x  1  2t
d) Perpendicular a r :  i que passa pel punt P(1, 0).
y  5  t

x  2 y 1
a) La recta té vector director u  1,  2 , per tant, l’equació és:   2x  4  y  1  2x  y  3  0 .
1 2
b) La recta és vertical i passa per A, per tant, l’equació és: x 4.
c) La recta és horitzontal i passa per A, per tant, l’equació és: y  3.

d) La recta té vector normal n   2, 1 , per tant, l’equació és: 2( x  1)  1( y  0)  2x  y  2  0 .

161 Matemàtiques 1r Batxillerat


SOLUCIONARI UNITAT 5. GEOMETRIA ANALÍTICA

81. En cada cas, calcula el valor del paràmetre k perquè les rectes tinguin la posició relativa indicada.
a) r : x ky  1  0; s: kx  4y  3  0, paral·leles.
b) r : kx 2y  4k  0; s: x 3y  4  0, coincidents.
c) r : 2kx  5y  1  0; s: 3x ky  2  0, paral·leles.

1 k 1
a) S’ha de complir que    k 2  4  k  2 .
k 4 3

k 2 4k 2
b) S’ha de complir que   k  .
1 3 4 3

2k 5 1
c) S’ha de complir que    2k 2  15  Impossible, per tant, no poden ser paral·leles.
3 k 2

Feix de rectes
82. Obtén l’equació del feix de rectes secants de vèrtex el punt P(2, 3). Troba la recta d’aquest feix que té pen-
1
dent m   .
2

1 1
L’equació del feix és y  3  m( x  2) . Si m    y  3    x  2   2y  6   x  2  x  2y  4  0 .
2 2

83. Determina l’equació del feix determinat per les rectes secants r: y  2x  3 i s: y  3x  5 i troba el seu vèrtex
i la recta d’aquest feix que passa pel punt P(2, 2).

y  2x  3
L’equació del feix és: 2x  y  3    3x  y  5  0 . Vèrtex:   x  2, y  1  V  2, 1
y  3x  5

9
Si la recta passa per P, tenim que 4  2  3    6  2  5  0     . Per tant, la recta que es busca és:
13

9
2x  y  3   3x  y  5  0  26x  13y  39  27x  9y  45  0  x  4y  6  0
13

84. Determina les equacions dels feixos de rectes d’aquestes figures i troba en cada cas la recta que passa pel
punt P(1, 2).
a) b)

a) És un feix de rectes paral·leles de pendent m  3 , per tant, l’equació és: 3x  y  K  0 .

D’aquestes, la que passa per P compleix 3  2  K  0  K  5 , per tant, l’equació és: 3x  y  5  0 .

b) És un feix de rectes secants de vèrtex V(2, 1), per tant, l’equació és: y  1  m( x  2) .

D’aquestes, la que passa per P compleix 2  1  m(1 2)  m  1 , per tant, l’equació és:

y  1  ( x  2)  x  y  3  0 .

162 Matemàtiques 1r Batxillerat


SOLUCIONARI UNITAT 5. GEOMETRIA ANALÍTICA

Distàncies i angles
85. Calcula la distància entre els punts A i B:

 3 2  2 3
a) A(2, 3) i B(2, 5) c) A  ,  i B  , 
 2 2  2 
  2

 1 1 5 5 1 5 3 
b) A  ,  i B  ,   d) A  ,   i B  ,  3 
2 3 2 3 2 3 5 

a) d ( A, B)   2  22  5  32  16  64  80  4 5 u

2 2
5 1  5 1
b) d ( A, B )           4  4  8  2 2 u
2 2  3 3

2 2
 2 3  2 3 10 10
c) d ( A, B )           u
 2 2 2 2 4 2
   

2 2
 3 1  5 1 16 1609 1609
d) d ( A, B )       3       u
5 2  3 100 9 900 30

x  1
86. Troba la distància del punt P(2, 3) al punt d’intersecció de les rectes r :  i s : 2x  y  3  0 .
 y  1  

Punt d’intersecció A: 2  1   3  0    2  A(1, 1) . Per tant, d ( A, P )  12  ( 4)2  17 u

87. Troba el perímetre del romboide determinat per les rectes següents.
a) x  4y  9  0 b) x  y  4  0 c) x  4y  6  0 d) x  y  6  0

Observem que a i c són paral·leles, igual que b i d, per tant, és un paral·lelogram. Els vèrtexs són:

 x  4y  9  0  x  4y  6  0 18 12  18 12 
  x  5, y  1  A(5, 1)  x ,y   C , 
x  y  4  0  x  y  6  0 5 5  5 5 

 x  4y  9  0  x  4y  6  0 22 2  22 2 
  x  3, y  3  B( 3, 3)  x , y   D , 
x  y  6  0 x  y  4  0 5 5  5 5

 9 9   3 
El perímetre és 2  d ( A, B )  d (B,C )  2  64  4     2  2 17+ 2 18,19 u
 25 25  5 
 

88. Troba la distància del punt P(4, 3) al punt mitjà del segment d’extrems A(1, 3) i B(1, 2).

 1 49 113
El punt mitjà del segment és M  0,   , per tant, d (P, M )  16   u
 2 4 2

163 Matemàtiques 1r Batxillerat


SOLUCIONARI UNITAT 5. GEOMETRIA ANALÍTICA

89. Calcula la distància del punt P a la recta r en els casos següents.


a) P(3, 4) r: 2x  3y  5  0

 x  1  2
b) P(1, 2) r : 
 y  2  2

2   3   3  4  5 1 13
a) d  P, r     u
2 3
2 2 13 13

 x  1  2 x 1 y  2 1 2  1
b)      x  1  y  2  x  y  1  0  d  P, r   0
y  2  2 2 2 12  12

90. Comprova que els parells de rectes següents són paral·leles i calcula, en cada cas, la distància que les se-
para.
2
a) r : 2x y  7 s: 2x y  8 b) r : 2x  3y  2  0 s :  x y 2  0
3

2 1 7 7  8 1 5
a)    Són paral·leles i d (r , s )    u
2 1 8 2  ( 1)
2 2 5 5

2 2 3 2 2  6 8 8 13
b) s:  x  y  2  0  2x  3y  6  0 .    Són paral·leles i d (r , s )    u
3 2 3 6 2  ( 3)
2 2 13 13

91. Calcula les mides dels seus tres costats i classifica els triangles següents.

 3 3
a) A(3, 2), B(5, 4) i C(1, 2) b) A(3, 5), B(1, 1) i C(5, 3) c) A(0, 1), B(0, 2) i C 
 2 2 
,
 

a) d ( A, B)  22  ( 6)2  40  2 10 u d (B, C )  ( 4)2  22  20  2 5 u

d ( A, C )  ( 2)2  ( 4)2  20  2 5 u És un triangle isòsceles.

b) d ( A, B)  ( 4)2  ( 6)2  52  2 13 u d (B, C )  62  ( 2)2  40  2 10 u

d ( A, C )  22  ( 8)2  68  2 17 u És un triangle escalè.

2
 3   1 2
c) d ( A, B)  0  1  1 u
2 2
d (B, C )     1u
 2   2 
 

2
 3   1 2
d ( A, C )    1u
 2   2 
És un triangle equilàter.
 

92. Calcula l’angle que formen les rectes:


a) r : 3x  y  1 i s : x  y  3

b) r : x  2y  2  0 i s : x  y  2  0

164 Matemàtiques 1r Batxillerat


SOLUCIONARI UNITAT 5. GEOMETRIA ANALÍTICA

n1  n2 2 5
a) Els vectors normals són n1  (3, 1) i n2  (1,  1) , per tant, cos        63,43º
n1 n2 10 2 5

n1  n2 1 10
b) Els vectors normals són n1  (1,  2) i n2  ( 1,  1) , per tant, cos        71,57º
n1 n2 5 2 10

93. Troba el perímetre del triangle format pels punts mitjans dels costats del
triangle de la figura.

A(2, 4), B(5, 1) i C(6, 3), de manera que els punts mitjans dels costats són

3 5  1   1
M1  ,  , M2   ,  1 i M3  4,  i, per tant, el perímetre és:
2 2  2   2

49 49 9 121 65  58  137
P  d (M1, M2 )  d (M2, M3 )  d (M3, M1)  4     4  u
4 4 4 4 2

Una altra manera de fer-ho seria:

Perímetre(ABC)   72   3    4    7    11   4     58  65  137 


1 1 2 2 2 2 2 1
P
2 2  2 

94. Calcula l‘àrea i el perímetre del quadrilàter que formen les rectes r : 3x  4y  12 i s : 5x  6y  30 amb els
eixos coordenats.

Els vèrtexs del quadrilàter són A(4, 0), B(6, 0), C(0, 5) i D(0, 3).

El perímetre és P  d( A, B)  d(B,C)  d(C, D)  d (D, A)  4  0  36  25  0  4  16  9  9  61 u .

L’àrea es calcula restant les àrees dels triangles rectangles OBC i OAD: S  15  6  9 u2 .

Punts i rectes simètrics

95. Calcula les coordenades dels extrems del segment simètric del AB respecte de la
simetria central de centre P essent: A(2, 3), B(4, 1) i P(3, 5).

P ha de ser el punt mitjà de A i A, per tant, si A  a1, a2  , tenim:

 2  a1
  3  a1  4
 2
  A '  4, 7 
 3  a2  5  a  7

 2 2

Anàlogament, Pha de ser el punt mitjà de B i B, per tant, si B  b1, b2  , tenim:

 4  b1
  3  b1  2
 2
  B '  2, 9 
 1  b2  5  b  9

 2 2

165 Matemàtiques 1r Batxillerat


SOLUCIONARI UNITAT 5. GEOMETRIA ANALÍTICA

96. Calcula les coordenades dels extrems del segment simètric del AB respecte de
 5
la simetria axial d’eix r essent A(1, 3), B  3,  i r : x y  3.
 2

La recta que passa per A i Aés perpendicular a r, per tant, és de la forma x  y  k  0


i, com que passa per A, tenim 1 3  k  0  k  2 , amb la qual cosa l’equació de la
recta AA és x  y  2  0 .

x  y  2  0 1 5 1 5
El punt d’intersecció d’aquesta recta i r és:   x  , y  M , 
 x  y  3 2 2 2 2

M és el punt mitjà del segment AA , per tant, si A  a1, a2  , tenim:

 1  a1 1
   a1  0
 2

2  A  0, 2 
 3  a2  5  a  2

 2 2
2

Si seguim un procés anàleg per al punt B tenim:


5 1 1
BB : x  y  k  0  3  k  0k    xy   0
2 2 2

 1
x  y   0 7 5 7 5
Punt d’intersecció:  2  x  , y   N , 
 x  y  3 4 4 4 4

 3  b1 7 1
 2  4  a1  2
 1 
Càlcul de B  b1, b2  :  5  B  , 0 
 2  b2 5 2 
   a2  0
 2 4

97. Donades les rectes r : x  4y  2  0 i s : 2x  3y 4:


a) Calcula’n el punt de tall.
b) Demostra que el punt P(1, 2) pertany a s i calcula’n el simètric respecte a la recta r.
c) Calcula l’equació de la simètrica de la recta s respecte a la recta r.

 x  4y  2  0
a) El punt de tall és:   Q  2, 0 
2x  3y  4  0

b) P compleix l’equació de s. En efecte: 2  1 3  2  4  2  6  4  0  P  s .


La recta perpendicular a r que passa per P és: 4x  y  k  0  4  2  k  0  k  6  4x  y  6  0 .

4 x  y  6  0  22 14 
Aquesta recta talla r en el punt:  M  , 
 x  4y  2  0  17 17 

 1  a 22 27
  a
 2  27 6 
M és el punt mitjà de P i el seu simètric P  a, b  , per tant:  17 17  P ' , 
 2  b 14 6  17 17 
 b

 2 17 17

x2 y
c) La recta passa per Q i P, per tant l’equació és:   6x  12  61y  6x  61y  12  0 .
61 6

166 Matemàtiques 1r Batxillerat


SOLUCIONARI UNITAT 5. GEOMETRIA ANALÍTICA

Llocs geomètrics
98. Donats els punts A(1, 4), B(3, 0) i C(3, 2):

a) Determina les mediatrius dels segments AB, AC i BC .

b) Troba les coordenades del punt que equidista de A, B i C.

a) Mediatriu del segment AB :

( x  1)2  ( y  4)2  ( x  3)2  ( y  0)2  x 2  2x  1 y 2  8y  16  x 2  6 x  9  y 2  x  y  1  0

Mediatriu del segment AC :

( x  1)2  ( y  4)2  ( x  3)2  ( y  2)2  x 2  2x  1 y 2  8y  16  x 2  6x  9  y 2  4y  4  x  3y  1  0

Mediatriu del segment BC :

( x  3)2  ( y  0)2  ( x  3)2  ( y  2)2  x 2  6 x  9  y 2  x 2  6 x  9  y 2  4y  4  3 x  y  1  0

b) El punt que equidista de A, B i C el circumcentre, s’obté com a punt d’intersecció de les mediatrius:

x  y  1  0 1 1  1 1
  x  , y  T  , 
 x  3y  1  0 2 2 2 2

 x  2
99. Troba les bisectrius de les rectes r : 3x  2y 2 i s :  i comprova que són perpendiculars.
y  1 

 x  2 x
Equació general de la recta s.    y  1  x  2y  2  0
 y  1   2

Equacions de les bisectrius:

3 x  2y  2

x  2y  2

3 x  2y  2

x  2y  2  1

    
b : 3 5  13 x  2 13  2 5 y  2 5  2 13  0 
13 5 13 5
     
b2 : 3 5  13 x  2 13  2 5 y  2 5  2 13  0 
Els vectors normals i, per tant, les bisectrius, són perpendiculars, ja que:

3  
5  13, 2 13  2 5 3 5  13,  2 13  2 5  45  13  52  20  0

Síntesi
100. Les diagonals d’un rombe són perpendiculars entre elles. Calcula les equacions de les diagonals de la figu-
ra i comprova si es tracta o no d’un rombe.

167 Matemàtiques 1r Batxillerat


SOLUCIONARI UNITAT 5. GEOMETRIA ANALÍTICA

La diagonal AC té vector director AC   6,  3  2, 1 i passa per A, per tant, l’equació és:

x 8 y 3
  x  8  2y  6  x  2y  2  0
2 1

La diagonal BD té vector director BD   2, 5  i passa per B, per tant, l’equació és:

x  6 y 1
  5 x  30  2y  2  5 x  2y  28  0
2 5

No es tracta d’un rombe, ja que les diagonals no són perpendiculars. En efecte, AC ·BD  12  15  3  0

 5
101. Donat el quadrilàter de vèrtexs A(1, 1), B(5, 2), C(3, 3) i D  1,  :
 2
a) Demostra que es tracta d’un trapezi.
b) Calcula el punt on es tallen les diagonals.
c) Comprova que la recta que uneix els punts mitjans dels dos costats no paral·lels és paral·lela a les bases del
trapezi.

 1
a) Els costats AB i DC són paral·lels, ja que els vectors AB   4, 1 i DC   2,  són proporcionals.
 2

 3
Els costats AD i BC no són paral·lels, ja que els vectors AD   0,  i BC   2, 1 no són proporcionals.
 2
Per tant, el quadrilàter és un trapezi de bases AB i DC.
x 1 y 1 x 5 y 2
b) La diagonal AC té equació:   x  y  0 i la BD:   x  8y  21  0 .
2 2 4 1
2

x  y  0 7 7 7 7
Punt de tall:   x  , y  P , 
 x  8y  21  0 3 3 3 3

 7  5
c) R  1,  y S  4,  són els punts mitjans de AD i BC, respectivament. La recta RS és paral·lela als costats AB
 4   2
 1  3
i DC, ja que els vectors AB   4, 1 , DC   2,  i RS   3,  són proporcionals.
 2  4

102. Es considera el quadrilàter de vèrtexs: A(5, 0), B(3, 2), C(5, 8) i D(7, 6)
a) Calcula la mida de les dues diagonals.
b) Comprova que els punts mitjans dels costats formen un paral·lelogram.
c) Calcula el perímetre del paral·lelogram.
d) Comprova que el perímetre trobat coincideix amb la suma de les dues diagonals del quadrilàter inicial.

a) d ( A,C )  102  82  164  2 41 u d (B, D)  ( 10)2  ( 8)2  164  2 41 u

b) Punt mitjà del costat AB: M1  1, 1 Punt mitjà del costat BC: M2  4,  3

Punt mitjà del costat CD: M3  1,  7 Punt mitjà del costat DA: M4  6,  3

Com que M1M4  M2M3   5,  4 , M1M2M3M4 és un paral·lelogram.

c) El perímetre és 2d (M1, M2 )  2d (M2, M3 )  2 5 2  ( 4) 2  2 ( 5) 2  ( 4) 2  4 41 u

d) La suma de les diagonals és d ( A,C )  d (B, D)  4 41 , que coincideix amb el perímetre del paral·lelogram.

168 Matemàtiques 1r Batxillerat


SOLUCIONARI UNITAT 5. GEOMETRIA ANALÍTICA

103. Troba el punt de la recta r : 2x  y  1  0 que equidista dels punts A(2, 2) i B(2, 4).

El punt buscat serà la intersecció de la recta r amb la mediatriu del segment AB.

La mediatriu del segment AB és: ( x  2)2  ( y  2)2  ( x  2)2  ( y  4)2  2x  y  3  0 .

2x  y  1  0
El punt d’intersecció és:   x  1, y  1  P  1, 1
2x  y  3  0

104. Calcula els punts de la recta r: x y 3  0 que estan a distància 1 del punt P(1, 1).

Els punts de la recta són de la forma  t, 3  t  ; perquè estiguin a distància 1 de P s’ha de complir:

t  2
(t  1)2  (2  t )2  1  2t 2  6t  4  0   Així s’obtenen dues solucions, A(2, 1) i B(1, 2).
t  1

105. Donat el triangle de vèrtexs: A(1, 3), B(1, 2) i C(0, 3)


a) Calcula les coordenades del baricentre.
b) Troba les equacions de dues altures i les coordenades de l’ortocentre.
c) Obtén les equacions de dues mediatrius i les coordenades del circumcentre.
d) Calcula el radi de la circumferència circumscrita al triangle.
e) Comprova que el baricentre, l’ortocentre i el circumcentre estan alineats. Escriu la recta que passa per aquests
punts, anomenada recta d’Euler.

 2
a) El baricentre és G  0, 
 3

b) L’altura per A és perpendicular a BC  1,  5  , així, l’equació és: 1( x  1)  5( y  3)  x  5y  14  0 .

L’altura per B és perpendicular a AC   1,  6  , així, l’equació és: 1( x  1)  6( y  2)  x  6y  11  0 .

 29 25 
L’ortocentre és el punt d’intersecció d’aquestes altures: H   , 
 11 11 

c) Mediatriu del costat AB: ( x  1)2  ( y  3)2  ( x  1)2  ( y  2)2  4 x  2y  5  0

Mediatriu del costat AC: ( x  1)2  ( y  3)2  ( x  0)2  ( y  3)2  2x  12y  1  0

 29 3 
El circumcentre és el punt d’intersecció d’aquestes matrius: T  , 
 22 22 

2 2
 29   3  2405
d) El radi de la circumferència circumscrita és d (T , A)    1     3   3,15 u
 22   22  242

 29 53   29 53 
e) GH    ,  i GH   ,  són proporcionals, per tant, el baricentre, l’ortocentre i el circumcentre
 11 33   22 66 
estan alineats. La recta que passa per aquests punts és la recta d’Euler, d’equació:
2
y
x 0 3
  53 x  87y  58  0
29 53

11 33

169 Matemàtiques 1r Batxillerat


SOLUCIONARI UNITAT 5. GEOMETRIA ANALÍTICA

106. A partir de la informació de la figura, calcula:


a) Les equacions de les rectes r, s i t.
b) El punt P d’intersecció entre s i t.
c) El punt P simètric de P respecte de la recta r.
d) El punt C simètric de C respecte de la recta r.
e) El perímetre del quadrilàter PCCP.
f) L’equació de la recta s′ simètrica de s respecte a la recta r.
g) L’angle que formen s i t.
h) La recta paral·lela a s que passa per B.
i) Les rectes que passen pel punt D(1, 3) i formen un angle de 30° amb la recta r.

x 0 y 2 x 2 y 4
a) r :   x  y  2  0 s : y  0  tg60º ( x  2)  y  3 x  2 3 t :   x y 6  0
2 2 2 2

 6  2 3 (6  2 3 )(1  3 )
y  3 x  2 3 x   2 3

b) 
x  y  6  0



1 3 1 3  P 2 3, 6  2 3 .  
 y  6  2 3

c) La perpendicular a r per P és t, que talla r a B. Per tant, B ha de ser el punt mitjà del segment PP ' , per la qual

cosa P ' 4  2 3, 2  2 3 . 
d) La perpendicular a r per C té equació x  y  2  0 i talla r en el punt A, amb la qual cosa A ha de ser el punt
mitjà del segment CC ' i, per tant, C '(2, 4) .

2  2 3    6  2 3  6  2 3   2  2 3 
2 2 2 2
e) d (P,C )  d (C,C ')  d (C ', P ')  d (P ', P )   ( 4)2  42  

 4  4 3    4  4 3 
2 2
  8 3  4 6 8 15,65 u

f) La recta que es busca passa per Pi C, per tant, l’equació és:
x2
62 3

y 4
2  2 3
    
 2  2 3 x  6  2 3 y  20  12 3  0 
g) Els vectors normals de s i t són, respectivament, n1   3,  1  i n2  1, 1 , per tant, l’angle que formen és:

cos  
3 1

 3 1  2
   75º
2 2 4

h) Com que el pendent de s és m  3 , la recta paral·lela a s que passa per B té equació:

y  4  3( x  2)  y  3 x  4  2 3  
i) La recta r té pendent 1, per tant, forma un angle de 45º amb la part positiva de l’eix d’abscisses. Les rectes que
formen un angle de 30º amb la recta r han de formar, per tant, angles de 15º o de 75º amb la part positiva de
l’eix d’abscisses.

3
1
3 tg30º  tg 45º
Per tant, els pendents han de ser m1  tg15º   2  3 i m2  tg75º   2 3 .
3 1  tg30º tg 45º
1
3
D’aquesta manera, les rectes tenen equació:


y 3  2 3   x  1  y  2  3  x  5  3  
y 3  2 3   x  1  y  2  3  x  5  3 

170 Matemàtiques 1r Batxillerat


SOLUCIONARI UNITAT 5. GEOMETRIA ANALÍTICA

QÜESTIONS
107. Existeix algun valor de a per al qual els punts O(0, 0), A(1, 1) i B(1, a) pertanyin a una mateixa recta?

Perquè els punts A, B i C pertanyin a una mateixa recta és necessari i suficient que els vectors OA  1, 1
i OB  1, a  tinguin la mateixa direcció, és a dir, siguin proporcionals. Per tant:

1 1
  a  1  Per a a  1 , els punts O, A i B estan alineats, de fet, A i B són iguals.
1 a

108. a) Indica un vector de direcció i un altre de normal a la recta que té per equació explícita: y  mx  n

b) Indica un vector de direcció i un altre de normal de la recta que té per equació punt-pendent: y  y0  m( x  x0 )

a) Vector de direcció u  1, m  Vector normal n   m,  1

b) Vector de direcció u  1, m  Vector normal n   m,  1

109. Indica, raonadament, si les afirmacions següents són certes o falses.


a) Si la distància d’un punt a una recta és zero, aleshores, obligatòriament, el punt està contingut en la recta.
b) Si l’equació general d’una recta no té terme independent, aleshores l’origen de coordenades pertany a la recta.
c) L’expressió Ax  By  C 0 representa sempre una recta independentment dels valors reals de A, B i C.
a) Certa, si la recta és r : Ax  By  C  0 i el punt és P(a, b) , en ser d (P, r )  0 , es compleix que
Aa  Bb  C  0  Aa  Bb  C  0 , la qual cosa significa que P compleix l’equació de r; és a dir, P pertany a la
recta.
b) Certa, si la recta és r : Ax  By  0 , clarament l’origen de coordenades O(0, 0) compleix l’equació de r, és a dir,
pertany a r.
c) Falsa. Si A  B  0 l’expressió no representa cap recta.

PROBLEMES

110. Al paral·lelogram de vèrtexs ABCD, es coneixen les coordenades dels punts A(0, 3), B(1, 0) i C(6, 1). Calcu-
la la mida de les seves diagonals i l’angle que formen.

Si D(a, b) tenim: AB  DC  1,  3   6  a, 1 b   a  5, b  4  D 5, 4 

Les diagonals mesuren: d ( A,C )  36  4  40  2 10 u i d (B, D)  16  16  32  4 2 u


L’angle que formen és: cos   cos AC, BD   24  8
2 10 4 2

5
5
   63,43º

171 Matemàtiques 1r Batxillerat


SOLUCIONARI UNITAT 5. GEOMETRIA ANALÍTICA

111. Construeix el camí que ha de seguir la bola B(1, 4) per arribar al punt N(8, 1) després de xocar a la banda r :
x y  4  0.

El simètric de B respecte de r és B(0, 3).


El punt de xoc M és el punt d’intersecció de les rectes BN i r:
x y 3
BN:    x  4y  12  x  4y  12  0
8 2

x  4y  12  0 4 8 4 8
  x  , y   P , .
x  y  4  0 3 3 3 3

Camí: BP  PN

112. Calcula les rectes que passen pel punt P(1, 2) i que determinen amb els eixos coordenats un triangle d’àrea
2
4,5 u .

Les rectes que passen per P són de la forma y  2  m( x  1)  y  mx  2  m . Tallen els eixos en els punts
m2
 
 0, 2  m  i  , 0 .
 m 
Tenim:
m2
2  m
m  9  m2  4m  4  9m  m2  5m  4  0  m  4  y  4 x  6

2 2 m  1  y   x  3
Així doncs, hi ha dues solucions.

113. Calcula el valor de k perquè l’àrea del triangle de vèrtexs A(4, 3), B(6, 3) i C(6, k) sigui 20 u .
2

x 4 y 3
La recta que passa per A i per B té d’equació:   3 x  12   y  3  3 x  y  15  0 .
1 3

18  k  15 3k
L’altura per C mesura h  d (C, AB )   u i la base mesura d ( A, B)  4  36  40  2 10 u ,
10 10
1 3k 3  k  20  k  17
per tant, tenim: S    2 10     3  k   20  
2 10 3  k  20  k  23

114. Un raig de llum r, que passa pel punt A(1, 2), incideix sobre l’eix d’abscisses i es reflecteix formant amb
l’eix un angle de 30°
Troba les equacions dels raigs incident i reflectit.

172 Matemàtiques 1r Batxillerat


SOLUCIONARI UNITAT 5. GEOMETRIA ANALÍTICA

El raig incident passa per A i forma amb l’eix d’abscisses un angle de 150º, per tant el pendent és
3 3
m  tg150º   i l’equació és de la forma y   xn.
3 3

3 6 3 3 6 3
Com que ha de passar per A, tenim: 2   n n  y  x
3 3 3 3

 
Per calcular l’equació del raig reflectit observem que el raig incident talla l’eix d’abscisses a B 1  2 3, 0 , per tant,
el raig reflectit passa per B i forma amb l’eix d’abscisses un angle de 30º, amb la qual cosa el seu pendent és
3 3
m  tg30º  i l’equació és de la forma y  x n .
3 3

Com que ha de passar per B, tenim: 0 


3
3
 
1 2 3  n  n  
6 3
3
y 
3
3
x
6 3
3

115. Els vèrtexs oposats d’un quadrat són els punts A(0, 3) i C(4, 0).
a) Quines són les coordenades dels altres dos vèrtexs?
b) Quina és l’àrea del quadrat?

a) La diagonal del quadrat es troba sobre la recta AC: 3x  4y  12  0.

 3
La diagonal mesura d ( A,C )  25  5 u i el seu punt mitjà és M  2,  .
 2
Sigui P(a, b) un dels vèrtexs, MP és perpendicular a AC, per tant, és de la forma (3, 4) .

5 5 5 1
A més, d (M, P )   3 2   4 2  5   
2 2 2 2
Obtenim així els dos vèrtexs que falten:

 1 7 7 7 7
   a  , b   B , 
 3  2 2 2 2 2
MP   a  2, b     3, 4   
 2    1  a  1 , b   1  D  1 ,  1 
  
 2 2 2 2 2

5 5 2
b) Aplicant el teorema de Pitàgores es calcula la longitud del costat del quadrat: 2l 2  52  l   u.
2 2

25 2
Per tant, l’àrea del quadrat és S  l 2  u
2

116. Determina les equacions dels costats d’un triangle que compleixi aquestes condicions:
I. Té un vèrtex a A(2, 7).
II. La recta 3x  y  11  0 és l’altura relativa al vèrtex B.
III. La recta x  2y 7  0 és la mitjana corresponent al vèrtex C.

La recta AC és perpendicular a 3x y  11  0, per tant, és de la forma x  3y k  0.


Com que passa per A, ha de ser k 23 i, per tant, AC: x  3y  23  0.
Per conèixer C n’hi ha prou amb calcular el punt d’intersecció de x 3y  23  0 i l’altura x  2y  7  0. Així s’obté:

 x  3y  23  0
  x  5, y  6  C  5,  6 
 x  2y  7  0

El vèrtex B(a, b) pertany a l’altura 3x y  11  0, per la qual cosa s’ha de complir 3a b  11  0.

173 Matemàtiques 1r Batxillerat


SOLUCIONARI UNITAT 5. GEOMETRIA ANALÍTICA

 a2 b7
D’altra banda, el punt mitjà del costat AB,M  ,  , pertany a la mitjana x  2y  7  0, per la qual cosa es
 2 2 
a2 b7
complirà l’equació  2  7  0  a  2b  2  0 .
2 2
Resolem el sistema format per les dues equacions obtingudes per a i b, i trobem les coordenades del punt B:

3a  b  11  0
  a  4, b  1  B( 4, 1)
a  2b  2  0
Les equacions dels altres dos costats ara es calculen de forma immediata, i s’obté:
AB: 4x  3y  13  0 i BC: 7x  9y  19  0.

PER APROFUNDIR
117. Donades les rectes r : x y  0 i s : x y  7  0 i el segment d’extrems

A(1, 9) i B(5, 8), calcula les coordenades dels extrems d’un segment CD de la
mateixa longitud que AB , paral·lel a aquest i de tal manera que el punt C per-
tanyi a la recta s i el punt D a la r.
Els punts de r són de la forma D(d, d) i els punts de s són de la forma C(c, 7c).

Els vectors AB i CD poden ser iguals o oposats, per tant:

d  c  4 d  c  4
   c  1, d  5  C(1, 6) i D(5, 5)
d  7  c  1 d  c  6

d  c  4 d  c  4
   c  6, d  2  C(6, 1) i D(2, 2)
d  7  c  1 d  c  8

118. Donat el triangle A(2, 1), B(1, 2) i C(1, 3)

a) Calcula el punt P, intersecció de la bisectriu de l’angle C amb el costat oposat AB .

PA CA
b) Demostra que  .
PB CB

x  2 y 1 x  2 y 1
a) AB :   3 x  y  5  0 AC :   2x  3y  7  0
1 3 3 2

x 1 y  3
BC :   5 x  2y  1  0
2 5
Bisectriu interior de l’angle C:
2x  3y  7
13

5 x  2y  1
29
   
 2 29  5 13 x  3 29  2 13 y  7 29  13  0

 45  377 55  3 377 
Resolem el sistema format per les rectes bisectriu i AB i obtenim el punt: P  , 
 16 16
 

PA CA 377
b) En efecte, quan es fa el càlcul s’obté   .
PB CB 29

174 Matemàtiques 1r Batxillerat


SOLUCIONARI UNITAT 5. GEOMETRIA ANALÍTICA

119. Calcula, de forma exacta, les coordenades dels vèrtexs del pentàgon regular de la figura sabent que el seu
costat mesura 2 unitats.

1 5
Comprova que el quocient entre la distància de D a B i la distància de D a C és el nombre auri   .
2
3  180º
L’angle interior d’un pentàgon regular val  108º .
5
C(1, 0), D(1,0)

Pel teorema del cosinus: DB  22  22  2  2  2  cos108º  8  8cos72º

  
Tenint en compte que les longituds de DA i DB són iguals, tenim: A 0, DA2  12  0, 7  8cos72º 
  
B 1 8  8cos72º  cos36º, 8  8cos72º  sen36º E 1  8  8cos72º  cos36º, 8  8cos72º  sen36º 
DB 8  8 cos72º
  2  2cos72º  1,618033989...  
DC 2

175 Matemàtiques 1r Batxillerat


SOLUCIONARI UNITAT 5. GEOMETRIA ANALÍTICA

ENTORN MATEMÀTIC

Donant voltes al riu


D’aquí a uns quants dies, la classe de l’Antoni i la Joana anirà d’excursió al camp. Aprofitant aquesta ocasió i
amb la intenció de fer la classe de matemàtiques més animada, la professora proposa fer un concurs per
equips: qui pot donar la millor aproximació a la longitud que separa els dos punts O i F per la llera d’un riu?
Per aconseguir-ho, proporciona un esbós del riu de la zona
d’acampada, calcat d’un plànol a escala obtingut en una
pàgina de mapes d’Internet. Per facilitar la tasca, hi afegeix
un parell d’eixos coordenats de manera que l’origen de co-
ordenades és el punt O. La quadrícula està formada per qua-
drats de 20 m de costat.
Ben de pressa i amb la intenció de demostrar que són els
més ràpids a l’hora de resoldre problemes i, per tant, me-
reixen guanyar el concurs, l’equip de l’Antoni inventa aquest
mètode:
Triarà cinc punts de la llera que juntament amb O i F forma-
ran una línia poligonal OAABBCCDDEEF. La longitud
d’aquesta poligonal s’aproximarà a la longitud de la llera del
riu.

Després d’exposar la seva idea a la classe, la professora


confirma que el mètode sembla adequat, però indica que
creu que l’aproximació, potser per culpa de la rapidesa amb
què s’ha fet, no és gaire bona i s’hauria de millorar..
Basant-se en la mateixa idea, l’equip de la Joana, després de
pensar-hi una mica més, proposa col·locar dos punts més G
i H de manera que la nova poligonal s’ajusti millor a la línia
de la llera.

a) Calcula l’aproximació que obté l’equip de l’Antoni.


b) Calcula l’aproximació que obté l’equip de la Joana.
c) Afegeix-hi algun punt més i obtén una altra aproximació.

a) O(0, 0), A(1,1), B(3, 2), C(6, 1), D(9, 3), E(11, 6) i F(12, 0)

La poligonal mesura 2  5  18  25  13  37 22,58 u , per tant, l’equip de l’Antoni obté 451,6 m.

b) Si s’hi afegeixen els punts G(8,0) i H(10, 6) la poligonal mesura


2  5  18  5  10  10  1  37 23,54 u , per la qual cosa l’equip de la Joana obté 470,8 m.

c) Si s’hi afegeixen, per exemple, I(4,5; 1) i J(7,5; 1) s’obté com a aproximació 20· 23,98 479,6 m.

176 Matemàtiques 1r Batxillerat


SOLUCIONARI UNITAT 5. GEOMETRIA ANALÍTICA

Piragües a la llacuna?
A la zona on acamparan, el riu desemboca en una llacuna i els
professors de l’escola estan pensant si llogar unes piragües o
no per practicar piragüisme. Com que l’activitat, a més de ser
cara, suposa riscos (no només el d’algun alumne en remull),
només ho faran si la llacuna és prou gran perquè l’experiència
valgui la pena i no ho faran si només és una bassa on poder
fer un parell de palades.
La professora proposa un altre repte als equips: calcular apro-
ximacions de la superfície de la llacuna.
Un altre cop proporciona a tots dos equips un esquema amb
els límits de la superfície que es vol calcular. Els quadrats de
la quadrícula en aquest cas tenen 4 m per costat.
En aquest cas, és més ràpid l’equip de la Joana, que proposa
considerar els punts O, A, B, C, D i E del perímetre del llac i
calcular l’àrea del polígon format per aquests punts mitjançant
triangulació.
a) Calcula l’àrea dels triangles ABC, OAC, OCD i ODE de la figura utilitzant els mètodes de la geometria analítica.
b) Utilitza alguna aplicació de geometria dinàmica per obtenir l’àrea dels triangles anteriors i afegeix-hi alguns punts
més per millorar l’aproximació.

a) O(0, 0), A(1, 4), B(4, 12), C(9, 2), D(12, 0) i E(5; 4,5)



base: d ( A,C )  64  3  68  2 17 u
 1 35 17
ABC :  AC: x  4 y  17  0  S1   2 17   35 u2
 2 17
altura: h  d (B, AC )  4  4  12  17  35  35 17 u
 17 17 17


base: d (O, A)  17 u

 1
OAC : OA : 4 x  y  0  S2   17  2  17 u2
 2
36  2 34
altura: h  d (C,OA)    2 17 u

 17 17

base: d (O, D)  12 u 1 
base: d (O, D)  12 u 1
OCD :   S3   12  2  12 u2 ODE :   S4   12  4,5  27 u2
altura: h  2 u 2 altura: h  4,5 u 2

L’equip de la Joana obté S S1  S2  S3  S4  35  17  12  27  91 u2  91 42  1456 m2

b) La superfície s’aproxima a 103 u  103∙16 1648 m


2 2

177 Matemàtiques 1r Batxillerat


SOLUCIONARI UNITAT 5. GEOMETRIA ANALÍTICA

AUTOAVALUACIÓ

Comprova el que has après


1. Calcula en cada cas l’equació de la recta.
a) Paral·lela a la recta s : 2x  3y  5 i que passa per P(0,3).

b) Perpendicular a la recta s : 4x  3y  5  0 i que passa pel punt P(1, 1).

c) De direcció la del vector AB amb A(2, 4) i B(1,3) i que passa pel punt d’intersecció de les rectes
5x  y  20  0 i y  x  2 .

a) Les rectes paral·leles a s són de la forma 2x  3y  K  0 , la que passa per P és:

9  K  0  K  9  2x  3y  9  0

b) Les rectes perpendiculars a s són de la forma 3x  4y  K  0 , la que passa per P és:

3  4  K  0  K  7  3x  4y  7  0

c) Calculem el punt d’intersecció:

5 x  y  20  0
  x  3, y  5  P  3, 5 
y  x  2

x 3 y 5
Com que AB   3,  1 , la recta buscada té equació:   x  3y  12  0
3 1

2. Calcula el pendent i l’ordenada a l’origen de les rectes r, s i t que apareixen


a la figura. Indica un vector de direcció de cadascuna d’aquestes.

La recta r passa pels punts (3, 0) i (1, 7), per tant, un vector director és
7
ur   2, 7 , la seva equació és r : 7x  2y  21  0 , el seu pendent és mr  
2
21
i l’ordenada a l’origen és nr  .
2
La recta s passa pels punts (1, 3) i (4, 2), per tant, un vector director és
1
us   5, 1 , la seva equació és s : x  5y  14  0 , el seu pendent és ms  i
5
14
l’ordenada a l’origen és ns  .
5

La recta t és la recta horitzontal t : y  2 , amb vector director ut  1, 0  , pendent mt  0 i ordenada a l’origen
nt  2 .

2
3. Calcula l’equació de la recta paral·lela a x + y = 0 i que forma amb els eixos coordenats un triangle de 30 u .
Hi ha una única solució?

La recta ha de tenir com a equació x  y  k , per la qual cosa talla els eixos en els punts A(k, 0) i B(0, k).

k2
L’àrea del triangle OAB serà S   30  k 2  60  k   60  2 15
2


 x  y  60
Per tant, hi ha dues solucions: 
 x  y   60

178 Matemàtiques 1r Batxillerat


SOLUCIONARI UNITAT 5. GEOMETRIA ANALÍTICA

4. Indica el pendent de totes les rectes paral·leles a la recta que passa pels punts P(1, 2) i Q(1, 7).

9
Les rectes tenen vector director u  PQ   2,  9 , per tant, tenen pendent m  .
2
5. Troba per quin valor de b, la recta x by 4b  1 és coincident amb la recta que passa pels punts
P(1, 4) i Q(2, 3).

1  4b  4b  1
La recta donada ha de passar per P i Q, per tant   b  3 , així, la recta és x  3y  11  0.
2  3b  4b  1

2 x  y  5
6. Calcula el punt simètric de P(5,2) respecte del punt d’intersecció de les rectes  .
 x  4y  2

El punt d’intersecció de les rectes és Q(2, 1), per tant, si P(a, b) és el simètric de P, tenim que Q ha de ser el punt
5  a
 2

mitjà de PP, amb la qual cosa:  2  a  1, b  4  P '   1, 4 
 2  b  1

 2
7. Calcula l’àrea del quadrilàter de la figura.

Els vèrtexs del quadrilàter són O(0, 0), A(3, 4), B(8, 5) i C(8, 0).
base  altura d ( A,C )  d ( A,OC ) 8  4
Àrea del triangle OAC: S1     16 u2
2 2 2

base  altura d (B,C )  d ( A, BC ) 5  5 25 2


Àrea del triangle ACB: S2     u
2 2 2 2

57 2
Àrea del quadrilàter: S  S1  S2  u
2

8. Calcula les coordenades dels vèrtexs i el perímetre del triangle determinat per les rectes:
r : x  3y  1  0
s : 3x  2y  4  0
t : 2x y  2  0

Els vèrtexs del triangle són:

 x  3y  1  0  x  3y  1  0
  x  2, y  1  A  2, 1   x  1, y  0  B  1, 0 
3 x  2y  4  0 2x  y  2  0

3 x  2y  4  0
  x  0, y  2  C  0,  2
2x  y  2  0

Per tant, el perímetre és P  d ( A, B)  d (B,C )  d (C, A)  9  1  1  4  4  9  10  5  13 u

179 Matemàtiques 1r Batxillerat


SOLUCIONARI UNITAT 5. GEOMETRIA ANALÍTICA

RELACIONA I CONTESTA
Tria l’única resposta correcta en cada cas

 1
x  2    x  1 
1. Les rectes r :  i s: 2 :
 y  3  2    
 y 1

A. Són paral·leles. C. Són secants i es tallen al punt R(1, 1).


B. Són secants i es tallen al punt Q(2, 3). D. Són la mateixa recta.

 1
x  2   x 2 y 3 x  1  x 1
r :    2x  y  1  0 s :  2   y  1  2x  y  1  0
y  3  2 1 2  1
y  1   2
La resposta correcta és la D.

2. La distància del punt (0, –1) a la recta x – y = 1 és:

A. 2 u B. 2 2 u C. 2 u D. Cap de les anteriors.


Es pot observar que el punt donat forma part de la recta. Per tant, la distància del punt a la recta és nul·la. La res-
posta correcta és la D.

x y
3. L’àrea del triangle determinat per l’origen de coordenades i les interseccions de la recta   1 amb els
a b
eixos coordenats és:
ab a b
A. A  ab B. A  C. A  2ab D. A  ·
2 2 2

ab
Els punts d’intersecció són (a, 0) i (0, b), per tant, l’àrea del triangle és A  , la resposta B.
2

Assenyala, en cada cas, les respostes correctes

4. Es consideren els punts del pla A(a, 1), B(a  1, 4) i C(1, a).
A. Per a a  2 els punts estan alineats.

1
B. Per a a  , ABC és un triangle rectangle a A.
2
C. Per a a  2 , ABC és un triangle isòsceles.
D. Per a a  2 , B és el punt mitjà del segment AC.

 a  1 1 a 
S’observa que AB   1, 3 , AC   1 a, a  1 , BC   a, a  4  i el punt mitjà de AC és  , .
 2 2 

Si a  2 tenim que AB   1, 3 i AC  1,  3  són proporcionals, per la qual cosa la resposta A és correcta, pe-
 3 1 
rò el punt mitjà de AC,  ,  no coincideix amb B, per la qual cosa la resposta D és incorrecta.
 2 2 

1  3 1
Si a  tenim que AB   1, 3  i AC    ,   són perpendiculars, per la qual cosa la resposta B és cor-
2  2 2 
recta.

180 Matemàtiques 1r Batxillerat


SOLUCIONARI UNITAT 5. GEOMETRIA ANALÍTICA

Si a  2 tenim que AB   1, 3 i AC   3,  1 tenen la mateixa longitud, però és diferent que la longitud de
BC   2,  2 , per la qual cosa la resposta C també és correcta.

5. Per a r, s i t, siguin mr , ms i mt els pendents i nr , ns i nt les ordenades a l’o-


rigen.
A. mr  2, ms  2 y mt  0 C. mt  1 y nt  2

1 1
B. mr  , ms  1 y mt  0 D. mr  y nr  2
2 2

1
mr  , ms  1, mt  0 y nr  ns  nt  2 , per tant, les respostes correctes són B i D.
2

Tria la relació correcta entre les dues afirmacions donades

6. Es considera el feix de rectes 2 x  y  ( x  y  3)  0 de vèrtex el punt (1, 2) i es consideren les afirma-


cions:
1. L’equació de la recta r s’obté en substituir algun valor real de λ a l’expressió del feix.
2. La recta r passa pel punt (1, 2).
A. 1  2 C. 2  1 però 1  2

B. 1  2 però 2  1 D. Cap de les anteriors.

L’equació del feix representa totes les rectes que passen per (1, 2) excepte la recta x  y  3  0 , per tant, la rela-
ció correcta és la B.

Assenyala la dada necessària per respondre

7. Per calcular l’equació d’una única recta s paral·lela a la recta r i que disti d’aquesta k u es donen les dades
següents:
1. L’equació general de r és 3x  4y  3  0 .

2. k  3 u
3. L’ordenada a l’origen de r és més gran que la de s.

A. La dada 1 és innecessària. C. La dada 2 és innecessària.


B. La dada 2 és innecessària. D. Cap dada no és innecessària.

Òbviament necessitem les dades 1 i 2, però amb aquestes dades només podem trobar dues rectes paral·leles a r
que distin d’aquesta k u, una a cada costat de r. Perquè hi hagi una única solució també necessitem la dada 3, per
tant, la resposta correcta és la D.

181 Matemàtiques 1r Batxillerat


SOLUCIONARI UNITAT 6. CÒNIQUES

6. Còniques
EXERCICIS PROPOSATS
1. Exercici resolt.

2. Comprova que l’equació 3 x 2  3y 2  6 x  12y  14  0 representa una circumferència i determina’n el cen-


tre i el radi.

14 29
 0   x  1   y  2 
2 2
Tenim 3x 2  3y 2  6x  12y  14  0  x 2  y 2  2x  4y  , equació d’una circum-
3 3
29
ferència de centre C(1, 2) i radi r  .
3

3. Estudia si el punt P és interior, exterior o pertanyent a la circumferència x 2  y 2  10 x  0 .

a) P(2, 4) b) P(2, 2) c) P(2, 5)

La circumferència té centre C(5, 0) i radi r  25  5 .

a) d (P, C )  9  16  25  5  r  P pertany a la circumferència.

b) d (P, C )  9  4  13  r  P és interior a la circumferència.

c) d (P, C )  9  25  34  r  P és exterior a la circumferència.

4. Indica la posició relativa de la recta 3 x  4y  25 respecte de la circumferència x 2  y 2  6 x  8y  25  0 .

La circumferència té centre C(3, 4) i radi r  9  16  25  50  5 2 .

9  16  25 32
La distància del centre C a la recta és   r , per tant, la recta i la circumferència són secants.
9  16 5

5. Estudia la posició relativa de cada parell de circumferències.

a) x 2  y 2  25 y 2x 2  2y 2  3y  3  0 b) x 2  y 2  2x -16  0 y x 2  y 2  5x  3y  9  0

 3
a) La primera circumferència té centre C1(0, 0) i radi r1  25  5 , la segona té centre C2  0,   i radi
 4
33 33 9 3
r2   . Com que d (C1, C2 )   , tenim d (C1, C2 )  r1  r2 i, per tant, la segona circumferència
16 4 16 4
és interior a la primera.

 5 3 35
b) La primera circumferència té centre C1(1, 0) i radi r1  17 , la segona té centre C2   ,   i radi r2 
 2 2 2
9 9 3 2
Com que d (C1, C2 )    , tenim r2  r1  d (C1, C2 )  r2  r1 i, per tant, les circumferències són
4 4 2
secants.

182 Matemàtiques 1r Batxillerat


SOLUCIONARI UNITAT 6. CÒNIQUES

6. Exercici resolt.

2 2
7. Calcula la potència dels punts següents respecte de la circumferència x + y – 4x – 6y – 3 = 0. Indica en
cada cas la posició relativa d’aquests punts respecte de la circumferència.
a) P(3, –4) c) R(6, 3)

1 4
b) Q(2, 1) d) S  , 
2 5

a) PotC (P )  32   4  4  3  6   4  3  34 . P és exterior a la circumferència.


2

b) PotC (Q)  22  12  4  2  6  1 3  12 . Q és interior a la circumferència.

c) PotC (R )  62  32  4  6  6  3  3  0 . R pertany a la circumferència.

2 2
 1  4  1 4
d) PotC (S )        4     6     3  8,91 . S és interior a la circumferència.
2 5 2 5

8. En cada cas, determina la posició relativa entre el punt i la circumferència x 2  y 2  2 x  2y  1  0 .

 3  2 2 2 2  8 1  1
a) P  1,  b) Q  ,  c) R  ,  d) P  2, 
 2  2 2  5 5  4

2
3 3 3
a) PotCf (P )  12     2  1  2   1   . P és interior a la circumferència.
2 2 4
2 2
 2 2   2 2  2 2 2 2
b) PotCf (Q )     2  2  1  0 . Q pertany a la circumferència.
 2   2  2 2
   
2 2
 8  1 8 1
c) PotCf (R )        2   2   1  0 . R pertany a la circumferència.
5 5 5 5
2
 1 1 9
d) PotCf (S )  22     2  2  2   1  . S és exterior a la circumferència.
4 4 16

9. Exercici resolt.

10. Calcula l’eix radical de les circumferències i comprova que és perpendicular a la recta que passa pels cen-
tres.

a) C1 : x 2  y 2  9 i C2 : x 2  y 2  4x  2y  1  0

b) C1 : 12x 2  12y 2  27 i C2 : x 2  y 2  6x  6y  14  0

c) C1 : x 2  y 2  4x  2y  20 i C2 : ( x  4)2  ( y  3)2  4

a) Eix radical: 4x  2y  10  0  2x  y  5  0

Els centres són, respectivament, C1  0, 0  i C2  2,  1 , per tant, la recta que els uneix és x  2y  0 , perpendi-
cular a l’eix radical.

183 Matemàtiques 1r Batxillerat


SOLUCIONARI UNITAT 6. CÒNIQUES


12x  12y  27  0
2 2
b) Eix radical:   72x  72y  195  0  24 x  24y  65  0
12x  12y  72x  72y  168  0
 2 2

Els centres són, respectivament, C1  0, 0  i C2  3, 3  , per tant, la recta que els uneix és x  y  0 , perpendi-
cular a l’eix radical.

 x 2  y 2  4 x  2y  20  0

c) Eix radical:   4 x  4y  41  0
 x  y  8 x  6y  21  0
 2 2

Els centres són, respectivament, C1  2, 1 i C2  4, 3  , per tant, la recta que els uneix és x  y  1  0 , perpendi-
cular a l’eix radical.

11. Siguin ( x  2)2  (y  4)2  20 i ( x  11)2  (y  1)2  50 . Troba els punts de tall de les dues circumferències i
comprova que pertanyen al seu eix radical.


( x  2)  ( y  4)  20
2 2

 x  y  4 x  8y  0
2 2
    18 x  6y  72  0  3 x  y  12  0  y  3 x  12
( x  11)2  ( y  1)2  50
  x 2  y 2  22x  2y  72  0

L’eix radical és y  3x  12 , a més, substituint aquesta equació a, per exemple, la primera equació, obtenim els
punts de tall que, òbviament, compliran l’equació de l’eix radical i, per tant, pertanyeran a aquest.

 x  6, y  6  P(6, 6)
x 2  (3 x  12)2  4 x  8(3 x  12)  0  10 x 2  100 x  240  0  x 2  10 x  24  0  
 x  4, y  0  Q(4, 0)

12. Donades les circumferències següents:

C1 : x 2  (y  1)2  4 C2 : x 2  y 2  8x  2y  16  0 C3 : x 2  y 2  2x  2y  1  0

a) Troba els eixos radicals de totes les possibles parelles entre les circumferències donades.
b) Raona si existeix centre radical i, si és així, troba’l.

19
a) Eix radical de C1 i C2 : 8 x  19  0  x  Eix radical de C1 i C3 : 2x  4  0  x  2
8

3
Eix radical de C2 i C3 : 10 x  15  0  x 
2
b) Els eixos radicals són rectes paral·leles i, per tant, no es tallen. No hi ha centre radical. També podríem haver
comprovat que els centres C1  0, 1 , C2  4, 1 o C3  1, 1 estan alineats, ja que pertanyen a la recta y  1, per
la qual cosa no existeix centre radical.

13. Exercici interactiu.

14 i 15. Exercicis resolts.

16. Dibuixa i indica els elements de les el·lipses següents.

( x  2)2 ( y  3)2 ( y  1)2


a)  1 b) x 2  1
9 4 4

c 5
a) a  3 , b  2 , c  a2  b2  5 , e  
a 3

Centre: C(2, 3)   


Focus: F 2  5,  3 , F ' 2  5,  3 
Vèrtexs: A(5, 3), A(1, 3), B(2, 1), B(2, 5)

184 Matemàtiques 1r Batxillerat


SOLUCIONARI UNITAT 6. CÒNIQUES

c 3
b) a  2 , b  1 , c  a2  b2  3 , e  
a 2

Centre: C(0, 1)   
Focus: F 0, 1  3 , F ' 0, 1  3 
Vèrtexs: A(0, 3), A(0, 1), B(1, 1), B(1, 1)

17. Troba l’equació reduïda d’una el·lipse centrada en l’origen, si un focus és F(12, 0) i el seu semieix major
val 13.

x2 y2
a  13 , c  d (O, F )  12 i b  a  c  25  5 . Per tant, l’equació és
2 2
  1.
169 25

18. Troba l’equació de l’el·lipse de centre (3, 4), e  0,5 i F(8, 4).

c ( x  3)2 ( y  4)2
c  d (C, F )  5 , e   a  10 i b  a2  c 2  75  5 3 . Per tant, l’equació és  1.
a 100 75

19 i 20. Exercicis resolts.

21. Dibuixa i indica els elements de les hipèrboles següents.

( x  2)2 ( y  2)2 x2 y 2
a)  1 b)   1
4 2 9 16

c 6
a) a  2 , b  2 , c  a2  b2  6 , e  
a 2

Centre: C(2, 2)   
Focus: F 2  6, 2 , F  2  6, 2 
Vèrtexs: A(0, 2), A( 4, 2)

2 2
Asímptotes: y  2  ( x  2) , y  2   ( x  2)
2 2

c 5
b) a  4 , b  3 , c  a2  b2  25  5 , e  
a 4

Centre: C(0, 0) Focus: F  0, 5 , F '  0,  5  Vèrtexs: A(0, 4), A(0, 4)

4 4
Asímptotes: y  x, y  x
3 3

22. Troba l’equació canònica de les hipèrboles següents:


a) Hipèrbola amb excentricitat 1,5 i semieix a  4 . b) Hipèrbola amb focus a F(2, 0) i que passa per P(2, 3)

c x2 y 2
a) a  4 , e   c  6 , b  c 2  a2  20  2 5 , per tant, l’equació és   1.
a 16 20

185 Matemàtiques 1r Batxillerat


SOLUCIONARI UNITAT 6. CÒNIQUES

x2 y2
b) c  2 , per tant, l’equació és de la forma   1 . Si fem que la hipèrbola passi per P tenim:
a 2
4  a2

4 9 a2  16, b2  12



  1  16  4a2  9a2  4a2  a 4  a 4  17a2  16  0   2
a 2
4a 2
a  1, b  3
 2

x2 y 2
Òbviament, la primera possibilitat no és vàlida, per tant, l’equació és   1.
1 3
23. Exercici resolt.

24. Calcula les coordenades del focus i del vèrtex, les equacions de l’eix i de la directriu, i dibuixa les paràbo-
les següents.

a) x  10  y 2 b) y 2  4y  2  3x c) x 2  6y  13  5 d) x 2  4x  6y  28

a) x  10  y 2  y 2  x  10 , paràbola amb obertura a la dreta.

1
Vèrtex: V(10, 0) Eix: y  0 p
2

39  41 
Directriu: x  Focus: F  , 0 
4  4 

b) y 2  4y  2  3x  ( y  2)2  4  2  3x  ( y  2)2  3( x  2) ,

paràbola amb obertura cap a l’esquerra.


3
Vèrtex: V(2, 2) Eix: y  2 p
2

11 5 
Directriu: x  Focus: F  ,  2 
4  4 

c) x 2  6y  13  5  x 2  6  y  3 , obertura cap avall.

Vèrtex: V(0, 3) Eix: x  0 p3

3  9
Directriu: y   Focus: F  0,  
2  2

d) x 2  4x  6y  28  ( x  2)2  4  6y  28  ( x  2)2  6( y  4) ,

obertura cap amunt.


Vèrtex: V(2, 4) Eix: x  2 p3

5  11 
Directriu: y  Focus: F  2, 
2  2

25. Troba l’equació d’una paràbola, sabent que té el seu vèrtex en el punt (2, 4) i que la seva directriu és la rec-
ta y  2 .

p x2 x 9
 d (V , d )  2  p  4 , per tant, l’equació és ( x  2)2  8(y  4)  y    .
2 8 2 2
26. Exercici interactiu.

27 a 34. Exercicis resolts.

186 Matemàtiques 1r Batxillerat


SOLUCIONARI UNITAT 6. CÒNIQUES

EXERCICIS
Circumferència
35. Calcula l’equació de les circumferències següents.
a) b)

c) De centre, C(2, 3), i passa pel punt P(5, 1).


d) De centre, el punt C(5, 2), i tangent a l’eix d’abscisses.
e) Passa pels punts A(3, 2) i B(1, 2), i té el seu centre en la recta r : 3x  y  6 .

f) Passa pel punt A(3, 4), el seu radi val r = 5 i el seu centre es troba en l’eix d’abscisses.
g) El centre és C(3, 6) i és tangent a la bisectriu del primer quadrant.
h) Passa per A(7, 3), B(5, 1) i C(2, 8).

a) Centre C(2, 1) i passa per P(5, 5). El radi és d (C, P )  25  5 , i l’equació,  x  2   y  1  25 


2 2

 x 2  y 2  4x  2y  20  0 .
b) A(1, 2) i B(3, 4) són diametralment oposats, per tant, el centre serà el punt mitjà, C(2, 3), i el radi, la meitat
2 . L’equació és  x  2   y  3  2  x 2  y 2  4x  6y  11  0 .
2 2
de d ( A, B) , és a dir,

c) r  d C, P   25  5   x  2  ( y  3)2  25  x 2  y 2  4x  6y  12  0 .
2

d) El radi és la distància del punt C a la recta y  0 , és a dir, r  2 . L’equació és ( x  5)2  ( y  2)2  4 

 x 2  y 2  10x  4y  25  0 .
e) El centre és la intersecció de la mediatriu del segment AB i la recta r.

Mediatriu de AB: d ( X , A)  d ( X , B)  ( x  3)2  ( y  2)2  ( x  1)2  ( y  2)2  x  2y  2  0

 x  2y  2
Centre:   C  2, 0  Radi: r  d ( A, C )  5
3 x  y  6

Equació: ( x  2)2  y 2  5  x 2  y 2  4x  1  0

f) El centre serà de la forma C  a, 0  , així, d ( A, C )  5  (3  a)2  42  25  (3  a)2  9 

3  a  3  a  0  Equació: x 2  y 2  25


3  a  3  a  6  Equació: ( x  6)  y  25  x  y  12x  11  0
 2 2 2 2

3 2
g) El radi és la distància del punt C a la recta y  x , és a dir, r  .
2
9
L’equació és ( x  3)2  ( y  6)2   2x 2  2y 2  12x  24y  81  0 .
2
49  9  7 A  3B  C  0  A  4
 
h) x 2  y 2  Ax  By  C  0  25  1  5 A  B  C  0  B  6  x 2  y 2  4 x  6y  12  0
4  64  2A  8B  C  0 C  12
 

187 Matemàtiques 1r Batxillerat


SOLUCIONARI UNITAT 6. CÒNIQUES

36. Calcula l’equació de la circumferència que té per centre el punt C(1, 4) i és tangent a la recta
3 x  4y  4  0 .

3  16  4 15
r    3  ( x  1)2  ( y  4)2  9  x 2  y 2  2x  8y  8  0 .
9  16 5

37. Determina el centre i el radi de la circumferència que passa pels punts A(0, 0), B(0, 2) i C(2, 4).

A partir de l’equació de la circumferència x 2  y 2  Dx  Ey  F  0 tenim:

F  0 F  0
 
4  2E  F  0  4  2E  0  E  2  Eq. Circumferència: x 2  y 2  6 x  2y  0
4  16  2D  4E  F  0 20  2D  4  2   0  D  6
 

 D E D2 E 2
Així: C  c1, c2     ,    C  3, 1 i r    F  9  1  10
 2 2 4 4

38. Donada la circumferència 4 x 2  4y 2  24 x  4y  33  0 , calcula l’equació d’una altra circumferència con-


cèntrica amb aquesta i de radi la meitat.

 1 1 33
La circumferència donada té centre C  3,   i radi r  9    1 , per tant, l’equació que es busca és:
 2 4 4
2
 x  3 2   y 
1 1
   x  y  6x  y  9  0
2 2

 2 4

39. Troba l’equació de la circumferència que passa pel punt P(2, 9) i és tangent als dos eixos de coordenades.

El centre és de la forma C(r , r ) on r és el radi, per tant, l’equació és  x  r    y  r   r 2 


2 2

 x 2  y 2  2rx  2ry  r 2  0 . Si fem que passi pel punt P tenim:

r  17  Ecuación: x 2  y 2  34 x  34y  289  0



4  81  4r  18r  r 2  0  r 2  22r  85  0  
r  5  Ecuación: x  y  10 x  10y  25  0
 2 2

40. Confirma si les equacions representen una circumferència. En cas afirmatiu, dibuixa-les i indica el centre i
el valor del radi.

a) x 2  y 2  4x  6y  9  0 c) 2x 2  2y 2  6x  0 e) 2x 2  2y 2  2x  6y  3  0

b) x 2  y 2  2x  2y  3  0 d) x 2  y 2  2x  2y  0

a) C(2, 3) i r  4  9  9  2

b) C(1, 1) i r  1 1 3  1 no és real.


No representa una circumferència.

3  9 3
c) C  , 0  i r  
2  4 2

d) C(1, 1) i r  1 1  2

1 3 1 9 3
e) C  ,   i r    2
2 2 4 4 2

188 Matemàtiques 1r Batxillerat


SOLUCIONARI UNITAT 6. CÒNIQUES

41. Troba la posició relativa de cada punt i de la circumferència x 2  y 2  4 x  2y  20  0 .

a) A(5, 5) b) B(3, 3) c) C(4, 3)

a) PotCf ( A)  25  25  20  10  20  0 . El punt A pertany a la circumferència.

b) PotCf (B)  9  9  12  6  20  20 . El punt B és interior a la circumferència.

c) PotCf (C )  16  9  16  6  20  15 . El punt B és exterior a la circumferència.

42. Calcula la màxima i la mínima distància del punt P(5, 2) a la circumferència x 2  y 2  6 x  8y  0 .

PotCf (P )  25  4  30  16  75 , així, el punt és exterior a la circumferència.

Per tant, la recta que uneix P amb el centre C tallarà la circumferència primer a una distància x i després a una
distància x + 2r , que seran les distàncies mínima i màxima que es busquen.

Com que C(3, 4) i r  9  16  25  5 , tenim:

x  5
x  x  2r   75  x 2  10 x  75  0    La distància mínima és 5, i la màxima, 15.
 x  15 no vàlida

43. Per a cada cas, estudia la posició relativa de la recta amb la circumferència que s’indica.

a) 2x  y  1  0 amb x 2  y 2  4x  6y  9  0 . c) x  7y  30 amb x 2  y 2  10x  0 .

b) x  2  0 amb x 2  y 2  2x  2y  1  0 . d) y  3 amb 2x 2  2y 2  14x  2y  21  0 .

a) La circumferència té centre C(2, 3) i radi r  4  9  9  2 .

4  3 1 8 5
La distància del centre C a la recta és   r , per tant, la recta és exterior a la circumferència.
4 1 5

b) La circumferència té centre C(1, 1) i radi r  1 1 1  1 .

1 2
La distància del centre C a la recta és  1  r , per tant, la recta és tangent a la circumferència.
1

c) La circumferència té centre C(5, 0) i radi r  25  5 .

5  30 5 2
La distància del centre C a la recta és   r , per tant, la recta és secant a la circumferència.
1  49 2

7 1 49 1 21
d) La circumferència té centre C  ,   i radi r     2.
2 2 4 4 2

1
 3
2 5
La distància del centre C a la recta és   r , per tant, la recta és exterior a la circumferència.
1 2

189 Matemàtiques 1r Batxillerat


SOLUCIONARI UNITAT 6. CÒNIQUES

44. Estudia la posició relativa de la circumferència 2 x 2  2y 2  6 x  6y  7  0 amb cada una de les circumfe-
rències següents.
1
a) x 2  y 2  d) x 2  y 2  2x  3y  3  0
4

b) 2x 2  2y 2  5 e) x 2  y 2  3y  2  0

c) 8x 2  8y 2  16x  24y  25  0 f) x 2  y 2  3 x  3y  4  0

3 3 9 9 7
La circumferència 2x 2  2y 2  6x  6y  7  0 té centre C1  ,  i radi r1     1.
2 2 4 4 2

1 1
a) La circumferència x 2  y 2  té centre C2   0, 0 i radi r2  .
4 2

3 2 3
Com que d (C1, C2 )   r1  r2  , les circumferències són exteriors.
2 2

5 10
b) La circumferència 2x 2  2y 2  5 té centre C2   0, 0 i radi r2   .
2 2

10  2 3 2 2  10
Com que r2  r1   d (C1, C2 )   r1  r2  , les circumferències són secants.
2 2 2

 3 51 102
c) La circumferència 8x 2  8y 2  16x  24y  25  0 té centre C2   1,  i radi r2   .
 2 8 4

1 102  4
Com que d (C1, C2 )   r2  r1  , la primera circumferència és interior a la segona.
2 4

 3 1
d) La circumferència x 2  y 2  2x  3y  3  0 té centre C2   1,  i radi r2  .
 2 2

1
Com que d (C1, C2 )   r1  r2 , les circumferències són tangents interiors.
2

 3 1
e) La circumferència x 2  y 2  3y  2  0 té centre C2   0,  i radi r2  .
 2 2

3 3
Com que d (C1, C2 )   r1  r2  , les circumferències són tangents exteriors.
2 2

3 3 1 2
f) La circumferència x 2  y 2  3x  3y  4  0 té centre C1  ,  i radi r2   , per tant, les circum-
2 2 2 2
ferències són concèntriques i x 2  y 2  3x  3y  4  0 és a l’interior de 2x 2  2y 2  6x  6y  7  0 .

45. Calcula la potència de cada punt respecte de la circumferència indicada i assenyala’n la posició relativa.

a) P(1, 3) i 8x 2  8y 2  79x  32y  95  0

b) P(1, 1) i 2x  2y  x  0
2 2

c) P(5, 3) i x  y  7x  8y  0
2 2

a) PotCf (P )  8  72  79  96  95  0 . El punt pertany a la circumferència.

b) PotCf (P )  2  2  1  3 . El punt és exterior a la circumferència.

c) PotCf (P )  25  9  35  24  25 . El punt és interior a la circumferència.

190 Matemàtiques 1r Batxillerat


SOLUCIONARI UNITAT 6. CÒNIQUES

46. Calcula l’eix radical de les parelles de circumferències següents i representa gràficament la situació en ca-
da cas.

a) x 2  y 2  4x  2y  4  0 i x 2  y 2  6x  0

b) x 2  y 2  8y  6  0 i 2x 2  2y 2  4y  0

c) 4x 2  4y 2  4x  8y  1  0 i 4x 2  4y 2  4x  24y  1  0

 x 2  y 2  4 x  2y  4  0

a) Eix radical:   2x  2y  4  0  x  y  2  0 .
x  y  6x  0
 2 2


 x  y  8y  6  0
2 2
b) Eix radical:  2  6y  6  0  y  1 .
 x  y  2y  0
 2

4 x 2  4 y 2  4 x  8 y  1  0

c) Eix radical:   16y  0  y  0 .
4 x  4y  4 x  24y  1  0
 2 2

47. Troba el centre radical de les circumferències:

C1 : x 2  y 2  16 C2 : x 2  y 2  2 x  4y  4  0 C3 : x 2  y 2  6 x  6y  14  0

Eix radical de C1 i C2 : x  2y  6  0 Eix radical de C1 i C3 : x  y  5  0

 x  2y  6
Centre radical:   x  16, y  11  P  16,  11
 x  y  5

48. Calcula les tangents a les circumferències següents en el punt donat.

a) x 2  y 2  26 en P(1, 5) b) 3x 2  3y 2  4x  17y  23  0 en P(1, 2)

La tangent en un punt P és perpendicular al segment CP, on C és el centre de la circumferència.

a) C(0, 0), per tant CP   1, 5  i l’equació de la tangent és: 1( x  1)  5( y  5)  0  x  5y  26  0 .

 2 17  1 5 1 5
b) C  ,   , per tant CP   ,  i l’equació de la tangent és: ( x  1)  ( y  2)  0  2x  5y  8  0 .
3 6  3 6 3 6

191 Matemàtiques 1r Batxillerat


SOLUCIONARI UNITAT 6. CÒNIQUES

49. Donada la circumferència ( x  3)2  (y  1)2  25 , calcula les equacions de les seves tangents paral·leles a
la recta 3 x  4y  16  0 .

Les equacions que es busquen seran de la forma 3x  4y  K  0 , perquè siguin tangents a la circumferència, la
distància del centre a la recta ha de coincidir amb el radi, així:
9  4  K K  5  25  K  30  3 x  4y  30  0
 5  K  5  25  
9  16 K  5  25  K  20  3 x  4y  20  0

50. Calcula les equacions de les tangents a la circumferència x 2  y 2  4 x  4y  17  0 que siguin perpendi-
culars a la recta d’equació 3 x  4y  14 .

Les equacions que es busquen seran de la forma 4x  3y  K  0 , perquè siguin tangents a la circumferència, la
distància del centre a la recta ha de coincidir amb el radi; així, com que C(2, 2) i r  4  4  17  5 , tenim:

86K 2  K  25  K  23  4 x  3y  23  0
 5  2  K  25  
25 2  K  25  K  27  4 x  3y  27  0

El·lipse
51. Per a cada una de les el·lipses, indica les mesures dels seus semieixos i de la seva semidistància focal, es-
criu les coordenades dels vèrtexs i dels focus, i calcula el valor de l’excentricitat. Escriu-ne l’equació.
a) b)

c 7
a) a  4 , b  3 , c  a2  b2  7 , e  
a 4
Vèrtexs: A(4, 0), A(4, 0), B(0, 3), B(0, 3)

Focus: F   
7, 0 , F '  7, 0 
x2 y 2
Equació:  1
16 9

c 5
b) a  3 , b  2 , c  a2  b2  5 , e  
a 3

Vèrtexs: A(0, 3), A(0, 3), B(2, 0), B(2, 0)

  
Focus: F 0, 5 , F ' 0,  5 
x2 y 2
Equació:  1
4 9

192 Matemàtiques 1r Batxillerat


SOLUCIONARI UNITAT 6. CÒNIQUES

52. Donada l’el·lipse x 2  4y 2  4 x  12  0 , dibuixa-la i calcula’n el centre, els semieixos, la semidistància fo-
cal, els focus, els vèrtexs i l’excentricitat. Quina és l’equació de l’el·lipse que té els mateixos elements però
el centre és a l’origen de coordenades?

( x  2)2 y 2
x 2  4y 2  4x  12  0  ( x  2)2  4y 2  16   1
16 4

c 3
a  4 , b  2 , c  a  b  12  2 3 , e  
2 2
a 2

Centre: (2, 0)   
Focus: F 2  2 3, 0 , F  2  2 3, 0  Vèrtexs: A(2, 0), A(6, 0), B(2, 2), B(2, 2)

x2 y 2
Equació reduïda:  1
16 4

53. Calcula l’equació de les el·lipses següents. (Tret que s’indiqui, el centre és l’origen de coordenades.)
a) a  5 , c  3
b) Els radis vectors d’un punt mesuren 7 i 3, i c  4
c) Focus, F(3, 0), i vèrtex, A(4, 0)
d) Vèrtexs, A(6, 0) i B(0, 3).
e) Focus, F ' (2, 0), i excentricitat, e  0,4
f) Passa pels punts P(1, 2) i Q(2, 0).
3
g) Passa per P(5, 0) i la seva excentricitat és e 
5
h) Focus, F ' (0, 2), i semieix major, a  3
i) Centre, el punt C(2, 1), a  13 i b  5

x2 y 2
a) b  a2  c 2  16  4   1
25 16

x2 y 2
b) 2a  7  3  a  5, b  a2  c 2  9  3   1
25 9

x2 y 2
c) a  4, c  3, b  a2  c 2  7   1
16 7

x2 y 2
d) a  6, b  3   1
36 9
c x2 y 2
e) c  2, a   5, b  a2  c 2  21   1
e 25 21
1 4
   1 a  2
x2 y 2  a2 b2  x2 y 2
f)   1    4 3   1
b 
2 2
a b  4 1 4 16
  3
 a2 3

x2 y 2 25 x2 y 2
g) 2
 2  1  2  1  a  5, c  a  e  3, b  a2  c 2  16  4   1
a b a 25 16

x2 y 2
h) c  2, b  a2  c 2  5   1
5 9

( x  2)2 ( y  1)2
i)  1
169 25

193 Matemàtiques 1r Batxillerat


SOLUCIONARI UNITAT 6. CÒNIQUES

54. Per a cadascuna de les el·lipses següents, indica’n les mesures dels semieixos i de la semidistància focal,
escriu les coordenades dels vèrtexs i dels focus, i calcula el valor de l’excentricitat. Dibuixa-les.

x2 y2 x 2 ( y  5)2
a)  1 c)  1 e) 2( x  1)2  y 2  2
169 144 6 8

( x  3)2 ( y  2)2
b) 16x 2  25y 2  400 d)  1 f) 9x 2  y 2  18x  6y  9  0
10 6

c 5
a) a  13 , b  12 , c  a2  b2  25  5 , e  
a 13
Vèrtexs: A(13, 0), A(13, 0), B(0, 12), B(0, 12)
Focus: F(5, 0), F ' (5, 0)

x2 y 2
b) 16 x 2  25y 2  400   1
25 16
c 3
a  5 , b  4 , c  a b  9  3, e  
2 2
a 5
Vèrtexs: A(5, 0), A(5, 0), B(0, 4), B(0, 4)
Focus: F(3, 0), F ' (3, 0)
c 1
c) a  8  2 2 , b  6 , c  a2  b2  2 , e  
a 2

  
Vèrtexs: A 0, 5  2 2 , A ' 0, 5  2 2 , B    
6, 5 , B '  6, 5 
  
Focus: F 0, 5  2 , F ' 0, 5  2 
c 2 10
d) a  10 , b  6 , c  a2  b2  4  2 , e   
a 10 5

      
Vèrtexs: A 3  10,  2 , A ' 3  10,  2 , B 3,  2  6 , B ' 3,  2  6 
Focus: F(5, 2), F ' (1, 2)

y2
e) 2( x  1)2  y 2  2  ( x  1)2  1
2

c 1 2
a  2 , b  1 , c  a2  b2  1  1 , e   
a 2 2

   
Vèrtexs: A 1, 2 , A ' 1,  2 , B  2, 0  , B '  0, 0 

Focus: F 1, 1 , F ' 1,  1

( y  3)2
f) 9x 2  y 2  18x  6y  9  0  9( x  1)2  ( y  3)2  9  ( x  1)2  1
9

c 2 2
a  3 , b  1, c  a  b  8  2 2 , e  
2 2
a 3

Vèrtexs: A 1, 0 , A 1,  6 , B 0,  3 , B  2,  3

  
Focus: F 1,  3  2 2 , F  1,  3  2 2 

194 Matemàtiques 1r Batxillerat


SOLUCIONARI UNITAT 6. CÒNIQUES

Hipèrbola
55. Troba l’equació de les hipèrboles següents. (Tret que s’indiqui, el centre és l’origen de coordenades.)
a) a  3 , c  5
b) Semidistància focal 5 i els radis vectors d’un punt mesuren 10 i 2.
c) Focus, F(4, 0), i vèrtex, A(2, 0)
d) Vèrtexs, A(6, 0) i B(0, 3)
e) Focus, F ' (6, 0), i excentricitat, e  1,25

f) Passa pels punts P(3, 0) i Q(5, 3).


g) Passa per P(2, 0) i la seva excentricitat és e  1,5 .

h) Passa per P(15, 4), i la seva distància focal val 2 90 .

i) Centre, C(2, 3), a  8 i c  10

x2 y 2
a) b  c 2  a2  16  4   1
9 16

x2 y 2
b) 2a  10  2  a  4, b  c 2  a2  9  3   1
16 9

x2 y 2
c) a  2, c  4, b  c 2  a2  12  2 3   1
4 12

x2 y 2
d) a  6, b  3   1
36 9

c 24 576 18 x2 y2
e) c  6; a   ; b  c 2  a2  36     1
e 5 25 5 576 324
25 25

9
x2 y 2  a2  1

a  3
 x2 y 2
f)   1     9   1
a2 b2  25  9  1 b  4 9 81

 a2 b2 16

x2 y 2 4 x2 y 2
g) 2
 2  1  2  1  a  2, c  a  e  3, b  c 2  a2  5   1
a b a 4 5

 225 16
  2 1 x2 y 2
h)  a2 b  a  9, b  3   1
a2  b2  c 2  90 81 9

( x  2)2 ( y  3)2
i) b  102  82  6   1
64 36

 
56. Troba l’equació de la hipèrbola que té per focus els punts F(3, 0) i F(3, 0) i que passa pel punt P 8, 5 3 .

 64 75
  1 64 75 
b  5  a  4
2 2
x2 y 2
c  3   a2 b2    1  b 4
 130b 2
 675  0     1
a2  b2  9 9b 2
b 2
b  135  No vàlida
 2 4 5

195 Matemàtiques 1r Batxillerat


SOLUCIONARI UNITAT 6. CÒNIQUES

57. Calcula l’equació d’una hipèrbola si un focus és el punt F(0, 10) i una asímptota la recta y  x .

x2 y 2
Tenim a  b i c 2  a2  b2  100 , per tant, a2  b2  50 i l’equació de la hipèrbola és   1.
50 50

58. Per a cadascuna de les hipèrboles següents, indica’n les mesures dels semieixos i de la semidistància fo-
cal, escriu les coordenades dels vèrtexs i dels focus, i calcula les equacions de les asímptotes i el valor de
l’excentricitat. Dibuixa-les.

x2 y 2 ( x  1)2 ( y  2)2
a)  1 c)  1 e) 4y 2  x 2  4
144 25 8 6

y2 x2
b) 36x 2  64y 2  2304 d)  1 f) 2( y  1)2  x 2  2
225 64

c 13
a) a  12 , b  5 , c  a2  b2  13 , e  
a 12
Vèrtexs: A(12, 0), A(12, 0) Focus: F(13, 0), F ' (13, 0)
5 5
Asímptotes: y  x, y  x
12 12

x2 y 2
b) 36 x 2  64y 2  2304   1
64 36
c 10
a  8 , b  6 , c  a  b  10 , e  
2 2
a 8
Vèrtexs: A(8, 0), A(8, 0) Focus: F(10, 0), F ' (10, 0)
3 3
Asímptotes: y  x, y  x
4 4

c 14 7
c) a  8 , b  6 , c  a2  b2  14 , e   
a 8 2

  
Vèrtexs: A 1  8, 2 , A ' 1  8, 2    
Focus: F 1 14, 2 , F ' 1 14, 2 
6 3 3
Asímptotes: y  xy  x, y  x
8 2 2

c 17
d) a  15 , b  8 , c  a2  b2  17 , e  
a 15
Vèrtexs: A(0, 15), A(0, 15) Focus: F(0, 17), F ' (0, 17)
8 8
Asímptotes: x  y, x y
15 15

x2
e) 4y 2  x 2  4  y 2  1
4
c
a  1, b  2 , c  a  b  5 , e   5
2 2
a

Vèrtexs: A(0, 1), A(0, 1)   


Focus: F 0, 5 , F ' 0,  5 
Asímptotes: x  2y , x  2y

196 Matemàtiques 1r Batxillerat


SOLUCIONARI UNITAT 6. CÒNIQUES

x2
f) 2( y  1)2  x 2  2  ( y  1)2  1
2
c
a  1, b  2 , c  a  b  3 , e   3
2 2
a

Vèrtexs: A(0, 0), A(0, 2)   


Focus: F 0,  1 5 , F ' 0,  1 5 
Asímptotes: x  2( y  1) , x   2( y  1)

59. Donada la hipèrbola x 2  y 2  2y  2  0 , dibuixa-la i calcula’n el centre, els semieixos, la semidistància


focal, els focus, els vèrtexs i l’excentricitat. Quina és l’equació de la hipèrbola que té els mateixos elements
però el centre és a l’origen de coordenades?

x 2  y 2  2y  2  0  x 2  ( y  1)2  1

c
a  1, b  1, c  a  b  2 , e   2
2 2
a
Centre: C(0, 1)
Vèrtexs: A(1, 1), A(1, 1)

Focus: F   
2,  1 , F   2,  1 
Equació reduïda: x 2  y 2  1

60. Per a cadascuna de les hipèrboles següents, indica’n les mesures dels semieixos i de la semidistància fo-
cal, escriu les coordenades dels vèrtexs i dels focus, i el valor de l’excentricitat. Troba’n l’equació i les de
les seves asímptotes.
a) b)

c
a) a  1 , c  3 , b  c 2  a2  8  2 2 , e  3
a

y2
Equació: x 2  1
8
Vèrtexs: A(1, 0), A(1, 0)

Focus: F  3, 0 , F '  3, 0 

Asímptotes: y  2 2 x , y  2 2 x

197 Matemàtiques 1r Batxillerat


SOLUCIONARI UNITAT 6. CÒNIQUES

3 13 c 13
b) a  , b  1 , c  a2  b2  , e 
2 2 a 3
2
 7
x  
Equació: 
2
 ( y  3)2  1
9
4
Vèrtexs: A(5, 3), A(2, 3)

7 13  7 13 
Focus: F   , 3, F '   , 3
2 3   
  2 3 

2 7 2 7
Asímptotes: y  3  x   , y 3   x  
3 2 3 2

Paràbola
61. Calcula l’equació de les paràboles següents. (A c, d, e i f, el vèrtex és l’origen de les coordenades.)
a) Focus, F(2, 0), i directriu, x  6 f) Paràmetre, p  8 i oberta cap avall

b) Focus, F(0, 4), i directriu, y  1 g) Vèrtex, V(2, 1), i directriu, y  2

c) Paràmetre, p  2 i oberta cap a la dreta h) Vèrtex, V(2, 2), i focus, F(2, 6)

d) Paràmetre, p  4 i oberta cap a l’esquerra i) Vèrtex, V(0, 1), i directriu, x  7

e) Paràmetre, p  6 i oberta cap amunt

a) Oberta cap a l’esquerra, V(4, 0), p  4  y 2  8( x  4)

 5  5
b) Oberta cap amunt, V  0,  , p  3  x 2  6  y  
 2   2

c) y 2  4x

d) y 2  8x

e) x 2  12y

f) x 2  16y

g) Oberta cap amunt, p  6  ( x  2)2  12( y  1)

h) Oberta cap avall, p  8  ( x  2)2  16( y  2)

i) Oberta cap a l’esquerra, p  14  ( y  1)2  28x

62. Per a les paràboles següents, troba el vèrtex, el focus i l’equació de la directriu.

a) y 2  10x b) x 2  2y c) ( y  1)2  8( x  1) d) x 2  3y

5  5
a) Vèrtexs: V(0, 0) Focus: F  , 0  Directriu: x  
2  2

 1 1
b) Vèrtexs: V(0, 0) Focus: F  0,  Directriu: y  
 2 2

c) Vèrtexs: V(1, 1) Focus: F  3, 1 Directriu: x  1

 3 3
d) Vèrtexs: V(0, 0) Focus: F  0,   Directriu: y 
 4 4

198 Matemàtiques 1r Batxillerat


SOLUCIONARI UNITAT 6. CÒNIQUES

63. Per a cadascuna de les paràboles següents, calcula’n el vèrtex, el focus i la directriu, el valor del paràme-
tre p i l’equació reduïda.
a) b) c) d)

a) Vèrtexs: V(0, 0) Focus: F(1, 0) Directriu: x  1 Paràmetre: p  2 Equació: y 2  4x

b) Vèrtexs: V(0, 0) Focus: F(1, 0) Directriu: x  1 Paràmetre: p  2 Equació: y 2  4x

c) Vèrtexs: V(0, 0) Focus: F(0, 1) Directriu: y  1 Paràmetre: p  2 Equació: x 2  4y

d) Vèrtexs: V(0, 0) Focus: F(0, 1) Directriu: y  1 Paràmetre: p  2 Equació: x 2  4y

64. Troba el vèrtex, el focus i l’equació de la directriu en cadascuna de les paràboles següents.

a) y 2  4y  2x  2  0 b) x 2  2x  y  1  0 c) y 2  4y  6x  5  0

a) y 2  4y  2x  2  0  ( y  2)2  2( x  1)

 1  3
Vèrtex: V(1, 2) Focus: F   , 2  Directriu: x  
 2  2

b) x 2  2x  y  1  0  ( x  1)2  y

 1 1
Vèrtex: V(1, 0) Focus: F  1,  Directriu: y  
 4 4

 3
c) y 2  4y  6 x  5  0  ( y  2)2  6  x  
 2

 3 
Vèrtex: V   , 2  Focus: F(0, 2) Directriu: x  3
 2 

Síntesi
65. Calcula el valor de m perquè l’eix radical de les circumferències donades sigui l’eix d’abscisses.

C1 : x 2  y 2  mx  6y  0 C2 : 2 x 2  2y 2  8 x  y  0


 x  y  mx  6y  0
2 2

2x  2y  2mx  12y  0
2 2
 2    (2m  8)x  13y  0
2x  2y  8 x  y  0
 2x  2y  8 x  y  0

2 2 2

Perquè l’eix radical sigui l’eix d’abscisses s’ha de complir que 2m  8  0  m  4 .

66. Troba els llocs geomètrics següents.


a) Punts dels pla que equidisten de les rectes paral·leles r : x  y  5 i s : x  y  9 .

b) Punts del pla la distància dels quals a l’origen de coordenades sigui el doble que la distància al punt (2, 0).
c) Punts del pla tals que la suma dels quadrats de les seves distàncies a P(4, 0) i Q(4, 0) és 40.
d) Punts del pla tals que la seva distància al punt A(4, 0) és el doble que la seva distància a la recta x  1 .

199 Matemàtiques 1r Batxillerat


SOLUCIONARI UNITAT 6. CÒNIQUES

Sigui X(x, y) un punt genèric del lloc geomètric sol·licitat:


x  y 5 x  y 9  x  y  5  x  y  9  5  9 No vàlida
a) d ( X , r )  d ( X , s )   
2 2 x  y  5  x  y  9  x  y  7
S’obté la recta paral·lela a r i s que “es troba entre totes dues”.

b) d ( X , O)  2d  X , (2,0)  x 2  y 2  2 ( x  2)2  y 2  x 2  y 2  4( x  2)2  4y 2  3x 2  16x  3y 2  16  0

8  4
S’obté la circumferència de centre C  , 0  i radi r  .
3  3

c) d( X, P )2  d( X, Q)2  40  ( x  4)2  y 2  ( x  4)2  y 2  40  x 2  y 2  4


S’obté la circumferència de centre C(0, 0) i radi r  2 .
x 1
d) d ( X , A)  2d ( X , x  1)  ( x  4)2  y 2  2   ( x  4)2  y 2  4( x  1)2  3 x 2  y 2  12  0 .
1

S’obté la hipèrbola de centre C(0, 0) i semieixos a  2 i b  12  2 3 .

67. Troba, en funció del paràmetre positiu a, la posició relativa de la circumferència d’equació ( x  2)2  y 2  a
i la recta d’equació y  x .

200 2
La circumferència té centre C(2, 0) i radi r  a . La distància del centre a la recta és   2 , per
1 1 2
tant, si a  2 , la recta és tangent, si a  2 , és secant, i si a  2 , és exterior a la circumferència.

68. Donats els punts A(2, 3) i B(6, 1), troba l’equació i descriu el lloc geomètric dels punts P(x, y) del pla tals
que el triangle APB sigui rectangle P.

Els vectors AP   x  2, y  3 i BP   x  6, y  1 han de ser perpendiculars, per tant:

( x  2)( x  6)  ( y  3)( y  1)  0  x 2  y 2  8x  4y  15  0  ( x  4)2  ( y  2)2  5

es tracta d’una circumferència de centre C(4, 2) i radi r  5 .

69. Identifica cadascuna de les còniques següents i estableix-ne els elements més importants.

a) x 2  4x  4y  0 d) 3x 2  4y 2  18x  16y  31  0

b) x 2  y 2  6x  10y  33  0 e) 25x 2  144y 2  288y  3744  0

c) 9x 2  4y 2  24y  72  0

a) x 2  4x  4y  0  ( x  2)2  4( y  1) . Paràbola oberta cap amunt amb vèrtex V(2, 1).

b) x 2  y 2  6x  10y  33  0  ( x  3)2  ( y  5)2  1 . Circumferència de centre C(3, 5) i radi r  1 .

x 2 ( y  3)2
c) 9 x 2  4y 2  24y  72  0  9 x 2  4( y  3)2  36    1 . Hipèrbola de centre C(0, 3) i semieixos
4 9
a2 i b3.

 x  3 2   y  2 2
d) 3 x 2  4y 2  18 x  16y  31  0  3  x  3   4  y  2   12 
2 2
 1 . El·lipse de centre C(3, 2)
4 3
i semieixos a  2 i b  3 .
x 2 ( y  1)2
e) 25 x 2  144y 2  288y  3744  0  25x 2  144( y  1)2  3600    1 . Hipèrbola de centre C(0, 1) i
144 25
semieixos a  12 i b  5 .

200 Matemàtiques 1r Batxillerat


SOLUCIONARI UNITAT 6. CÒNIQUES

70. Calcula els punts d’intersecció de les parelles de còniques següents.

x2 y 2 x2 y 2
a)   1 amb  1 c) 9x 2  4y 2  36 amb x 2  y 2  43
16 12 10 15

x2 y 2 x2 y 2
b)   1 amb x 2  y 2  6x  1  0 d)   1 amb y 2  36x  144  0
16 12 16 64

 x2 y 2
  1

a)  162 122  P1  2, 3  , P2  2, 3  , P3  2,  3  , P2  2,  3 
x  y 1

 10 15

 x2 y 2
  1
b)  16 12  P1  2, 3  , P2  2,  3 
x 2  y 2  6x  1  0


9 x  4y  36
       
2 2
c)  2  P1 4, 3 3 , P2 4,  3 3 , P3 4, 3 3 , P4 4,  3 3
 x  y  43
 2

 x2 y 2
  1
d)  16 64  P1  4, 0  , P2  5, 6  , P3  5,  6 
 y 2  36 x  144  0

x2 y2
71. Per a quins valors del paràmetre k l’equació   1 representa una el·lipse? Comprova que to-
25  k 16  k
tes aquestes el·lipses tenen els mateixos focus.

25  k  0
L’equació representa una el·lipse si   k  16 .
16  k  0

En aquest cas tindríem a2  25  k i b2  16  k , amb la qual cosa c  a2  b2  9  3 i els focus són F(3, 0) i
F ' (3 , 0).

201 Matemàtiques 1r Batxillerat


SOLUCIONARI UNITAT 6. CÒNIQUES

72. Siguin A(1, 1) i B(1, 1) dos punts del pla.


a) Determina les equacions de totes les circumferències que passen pels punts A i B raonant on estan situats els
seus centres.
b) D’entre les circumferències de l’apartat anterior troba:
I) El centre i el radi d’aquella que és tangent a la recta y  x .

II) Els centres d’aquelles que tenen per radi 5.

a) Els centres de les circumferències han de pertànyer a la mediatriu del segment AB ja que els punts d’aquesta
recta equidisten de A i B. Com que la mediatriu del segment AB és l’eix d’ordenades, els centres de les
circumferències són de la forma C(0, c) i, per tant, el radi corresponent és r  1  (1 c )2 .

D’aquesta manera, les equacions de les circumferències que passen per A i B són de la forma
x 2  ( y  c )2  1 (1 c )2  x 2  y 2  2yc  2c  2  0 .
b) I) El punt A pertany a la circumferència i a la tangent, per tant, ha de ser el punt de tangència. Així:

c c2
d (C, y  x )  r   1  (1  c )2   1  (1  c )2  c 2  4c  4  0  c  2
2 2

Per tant, el centre i el radi de la circumferència que es busquen són, respectivament, C(0, 2) i r  2 .

c  1  24  1  2 6  C 0, 1  2 6
 1
II) r  1  (1  c )2  5  (1  c )2  24  
1  
c2  1  24  1  2 6  C2 0, 1  2 6
  

73. Es considera una vareta AB de longitud 1. L’extrem A d’aquesta vareta recorre completament la circum-
ferència d’equació: x 2  y 2  4 x  2y  1  0 ; mantenint-se la vareta tangent a la circumferència en tot
moment.
a) Determina el lloc geomètric descrit per l’extrem B de la vareta.
b) Obtén l’equació d’aquest lloc geomètric.

a) La circumferència té centre C(2, 1) i radi r  2 . Observem que CAB és un triangle rectangle a A, per la qual
cosa la distància entre C i B es manté constant i igual a 22  12  5 , d’aquesta manera, el punt B descriu
una circumferència de centre C i radi 5.

b) ( x  2)2  ( y  1)2  5 .

QÜESTIONS
74. Indica si les afirmacions següents són certes o falses.
a) L’excentricitat d’una circumferència és 0.
b) Una circumferència és una el·lipse en la qual els dos semieixos mesuren igual.

x2 y 2
c) Si a  b , l’equació   1 representa una el·lipse l’eix major de la qual està contingut en l’eix Y.
a2 b2

x2 y 2
d) L’equació   1 representa una hipèrbola l’eix real de la qual està contingut en l’eix Y.
a2 b2

202 Matemàtiques 1r Batxillerat


SOLUCIONARI UNITAT 6. CÒNIQUES

c 0
a) Cert, e   0. c) Cert.
a r

x2 y 2 y 2 x2
b) Cert, a  b  r . d) Cert,   1    1.
a2 b2 b2 a2

75. Indica els punts del pla des dels quals es pot traçar almenys una tangent a la circumferència x 2  y 2  4 .

Es pot traçar almenys una tangent des de qualsevol punt que no sigui interior a la circumferència.

76. Indica per a quins valors de a l’equació y 2  ay  x  0 representa una paràbola.

 a
2
 a2 
y 2  ay  x  0   y      x   . Per a qualsevol valor real de a l’equació representa una paràbola oberta
 2  4 
 a2 a a2  1
cap a l’esquerra de vèrtex V  ,   i directriu x  .
 4 2 4

PROBLEMES
77. La màxima distància que separa la Terra de la Lluna en la seva òrbita és de 63 vegades el radi terrestre.
L’excentricitat de l’el·lipse de l’òrbita és molt baixa, e = 0,0678. La Terra es troba en un dels focus de l’el-
lipse.
Calcula la distància mínima, en quilòmetres, que pot separar la Terra de la Lluna. Considera que el radi ter-
restre mesura aproximadament 6357 km.

La distància màxima i mínima de entre la Lluna i la Terra és, respectivament, a  c i a  c . Per tant:

a  c  63  6357  400 491


 a  375 062
c   a  c  349 633 km .
  0,0678 c  25 429
a

78. La Terra gira al voltant del Sol descrivint una el·lipse en un dels focus de la qual hi ha el Sol. El punt en el
qual la distància entre la Terra i el Sol és màxima s’anomena afeli, i el punt on és mínima, periheli.

Amb les dades de la figura, calcula l’excentricitat de l’òrbita de la Terra i interpreta-la.

a  c  152  106
 a  149,5  106
 c
    e   0,0167 .
a  c  147  10

6
c  2,5  10

6 a

L’òrbita és una el·lipse molt poc aplatada, és gairebé una circumferència.

203 Matemàtiques 1r Batxillerat


SOLUCIONARI UNITAT 6. CÒNIQUES

79. Troba la longitud d’una circumferència que passa per l’origen de coordenades i pel punt (2, 4) i té el seu
centre en la recta determinada pels punts (7, 2) i (3, 2).

El centre C de la circumferència ha de ser la intersecció de la recta determinada per (7, 2) i (3, 2) i la mediatriu del
segment d’extrems O(0, 0) i el punt (2, 4).

x  y  5 x  5
   C(5, 0), r  (2  5)2  (4  0)2  25  5
 x  2 y  5 y  0
Per tant, la longitud de la circumferència és 2r  10 u.

80. Un segment AB de longitud 5 unitats llisca de manera que l’extrem A sempre està sobre l’eix d’ordenades, i
l’extrem B, sobre el d’abscisses.

a) Determina el lloc geomètric que descriu el centre del segment durant el lliscament.
b) Calcula el lloc geomètric que descriu el punt del segment que dista 2 unitats de A i 3 de B.

Siguin A(0, a) i B(b, 0), de forma que a2  b2  25 , i sigui X(x, y) un punt genèric del lloc geomètric demanat.

 a
x
 2  a  2x  4 x 2  4 y 2  25 . Circumferència de centre l’origen i radi 5 .
a)  
y  b b  2 y 2
 2

 2b  5x
x b
 
2 2 2 2
5  2  25y  25 x  25  x  y  1 . El·lipse de centre l’origen,
b)   a  3 , b  2 i eix major si-
 y  3a a  5 y 9 4 4 9
 5  3
tuat a l’eix d’ordenades.

81. Troba les equacions de les circumferències inscrita i circumscrita al triangle de vèrtexs A(0, 1), B(3, 3) i
C(4, 4).

Circumferència circumscrita:
Observem que es tracta d’un triangle isòsceles i rectangle a A.

7 1
Per tant, el circumcentre estarà situat en el punt mitjà de la hipotenusa BC, T  ,  .
2 2
2 2
 7  1 25
Així, l’equació de la circumferència circumscrita és  x     y    .
 2  2 2

204 Matemàtiques 1r Batxillerat


SOLUCIONARI UNITAT 6. CÒNIQUES

Circumferència inscrita:
x y 1
Bisectriu de l’angle A: AT :   x  7y  7
7 1
3 x  4y  4 7x  y  24
Bisectriu de l’angle C:   (3 2  7)x  (4 2  1)y  4 2  24  0
5 5 2

 x  7y  7
  7 2 2
Incentre:   I 7  , 
 2 
(3 2  7)  (4 2  1)y  4 2  24  0
  2

5 2
Radi: 5 
2
2 2 2
 7 2  2  5 2
Equació:  x  7     y     5  
 2 2 2 
    

82. Quant es xuta una pilota amb el peu, la trajectòria que descriu aquesta és una paràbola que depèn de l’an-
gle amb el qual es xuta la pilota i de la velocitat inicial que se li dona.
Un jugador A ha xutat una pilota cap al seu company B i ha aconseguit les distàncies següents.
 Altura màxima aconseguida per la pilota: 2,75 m.
 Distància fins al punt on la pilota bota: 12,5 m.
Amb aquestes dades:
a) Escriu l’equació de la trajectòria prenent una referència adequada.
b) Indica les coordenades del focus i del vèrtex, i l’equació de la directriu.
c) Si un jugador C es troba a 5 m del jugador A, a quina altura passa la pilota per la seva vertical?

a) Agafant com a origen el punt A l’equació és de la forma ( x  6,25)2  2p  y  2,75  .

Com que ha de passar per l’origen, tenim: 6,252  2p  2,75  p  7,1  ( x  6,25)2  14,2  y  2,75

b) Vèrtexs: V(6,25; 2,75) Focus: F(6,25; 0,8) Directriu: y  6,3

c) x  5  (5  6,25)2  14,2  y  2,75  y  2,64 m

PER APROFUNDIR
83. Troba les longituds de les cordes comunes a la paràbola y 2  2 x  4 i l’el·lipse
2x 2  y 2  8 .

 y 2  2x  4
  x  1, y   6

 2  2x 2  2x  4  8  x 2  x  2  0  
    x  2, y  0

2
2 x y 8

   
Els punts comuns de les còniques són: A 1, 6 , B 1,  6 i C  2, 0  .

Les longituds de les cordes seran, per tant: d ( A, B)  2 6 u , d ( A, C )  15 u i d (B, C )  15 u .

205 Matemàtiques 1r Batxillerat


SOLUCIONARI UNITAT 6. CÒNIQUES

84. Quan es gira una hipèrbola equilàtera, x 2  y 2  a2 , 45º segons es mostra en les figures següents, les
asímptotes de la hipèrbola coincideixen amb els eixos de coordenades. Demostra, fent servir les noves co-
ordenades dels focus i la definició d’hipèrbola com a lloc geomètric, que respecte d’aquests nous eixos
a2
l’equació de la hipèrbola s’escriu en la forma xy  .
2

Com que c  a2  a2  2a , les coordenades dels nous focus seran F (a, a) i F '(a,  a) .

Per la definició d’hipèrbola:

( x  a)2  ( y  a)2  ( x  a)2  ( y  a)2  2a  ( x  a)2  ( y  a)2  2a  ( x  a)2  ( y  a)2 

x 2  y 2  2xa  2ya  2a2  4a2  x 2  y 2  2xa  2ya  2a2  4a x 2  y 2  2xa  2ya  2a 2 

x 2  y 2  2xa  2ya  2a2  x  y  a  x 2  y 2  2xa  2ya  2a2  x 2  y 2  a2  2xy  2ax  2ay 

a2
2xy  a2  xy  .
2

x2 y 2
85. Donada l’el·lipse d’equació   1 , determina els vèrtexs del rectangle PQRS inscrit a l’el·lipse sabent
36 16
que el costat PQ és paral·lel a l’eix major i talla l’eix menor en dues parts iguals. Calcula l’àrea del rectan-
gle.

Sigui Q(x, y). si el rectangle talla l’eix menor en dues parts iguals vol dir que la seva ordenada és igual a 2. Llavors
y = 2.
Atès que el punt Q és de l’el·lipse, es compleix:

x2 y 2 x 2 22 x2 1
  1   1  1  x 2  27  x  5,19
36 16 36 16 36 4

Els quatre vèrtexs són:  5,19, 2 ,  5,19, 2 ,  5,19,  2 i  5,19,  2


2
La base del rectangle és SR = 2 · 5,19 = 10,38, i l’altura és QR= 4. La seva área val A = 10,38 · 4 = 41,52 u .

206 Matemàtiques 1r Batxillerat


SOLUCIONARI UNITAT 6. CÒNIQUES

86. Els tres tipus de còniques es poden definir tots alhora de la manera següent: «Una cònica és el lloc geo-
mètric dels punts del pla tals que el quocient de distàncies a un punt fix anomenat focus i a una recta fixa
anomenada directriu és constant».
 Si la constant és més petita que la unitat, la cònica és una el·lipse.
 Si la constant és igual a la unitat, la cònica és una paràbola.
 Si la constant és més gran que la unitat, la cònica és una hipèrbola.
Comprova les afirmacions anteriors trobant:
25
a) El lloc geomètric dels punts del pla tals que el quocient de distàncies al punt F(3, 0) i a la recta x  és 0,6.
3
16
b) El lloc geomètric del punts del pla tals que el quocient de distàncies al punt F(5, 0) i a la recta x  és 1,25.
5

( x  3)2  y 2 3
   
2
  3 x  25   25 x 2  6 x  9  y 2  9 x 2  150 x  625 
2
a)   5 ( x  3)2  y 2
25 5
x
3

x2 y 2
 16 x 2  25y 2  400    1 . Equació d’una el·lipse.
25 16

( x  5)2  y 2
   
2
5
  5 x  16   16 x 2  10 x  25  y 2  25 x 2  160 x  256 
2
b)   4 ( x  5)2  y 2
16 4
x
5

x2 y 2
 9 x 2  16y 2  144    1 . Equació d’una hipèrbola.
16 9

207 Matemàtiques 1r Batxillerat


SOLUCIONARI UNITAT 6. CÒNIQUES

ENTORN MATEMÀTIC

El millor lloc per a un tafaner


L’Elena i l’Eloïsa són al metro xerrant animada-
ment mentre esperen que arribi el tren. El sostre
de l’estació té forma el·líptica, tal com mostra el
dibuix, i les amigues estan, curiosament, situades
justament en un dels focus (F) de la secció el-
líptica.
En Lluís, un amic seu una mica tafaner, vol sentir
la conversa i com que aquest curs està estudiant
les seccions còniques a la classe de matemàti-
ques, sap que per escoltar bé què diuen ha de po-
sar-se en l’altre focus (F′ ) de l’el·lipse.

a) Suposant les distàncies i el sistema de referència que s’indiquen al dibuix, escriu l’equació de la secció el·líptica.
Quines són les coordenades dels punts on es troben l’Eloïsa, l’Elena i en Lluís?

 9
b) Troba l’equació de la recta tangent a l’el·lipse en el punt  4,  , sabent que coincideix amb una de les bisectrius
 5
dels radis vectors de l’el·lipse que passen per aquest punt.
c) Comprova que els angles que formen els radis vectors del punt anterior amb la tangent trobada són iguals.

x2 y 2
a) Equació:  1 Focus: F  4, 0  i F   4, 0 
25 9
x4 y
b) Els radis vectors són PF : x  4 i PF  :   9 x  40y  36  0 .
8 9/5

9 x  40y  36
Les bisectrius dels radis vectors són    x  4  . Com que el pendent ha de ser negatiu, la bisectriu
41
que ens interessa és 9x  40y  36  41( x  4)  4x  5y  25  0 .

4 4
c) Angle format per PF i la tangent: cos 1    1  51,34º
1 16  25 41

9  4  40  5 164 4
Angle format per PF i la tangent: cos 2     2  51,34º
81  1600 16  25 41 41 41

208 Matemàtiques 1r Batxillerat


SOLUCIONARI UNITAT 6. CÒNIQUES

Els fars del cotxe


Les seccions dels fars dels cotxes tenen forma parabòlica, ja que aquestes còniques tenen la propietat que tots
els raigs reflectits de raigs que surten del seu focus surten sempre en direcció paral·lela a l’eix de la corba.
A la gràfica inferior es pot veure la secció del far d’un cotxe amb
forma parabòlica y 2  2 x . La bombeta està situada en el focus
F i emet raigs en totes direccions.
Comprova que la direcció de qualsevol raig que rebota és paral-
lela a l’eix de la paràbola. Per fer-ho:
a) Calcula les coordenades del focus.
b) Considera, per exemple, el punt S(2, 2) de la paràbola i troba la
tangent a la paràbola en aquest punt. Fes servir el fet que la tan-
gent i la paràbola tenen un punt en comú.
c) Comprova que són iguals els angles que forma la tangent amb la
recta FS i la y  2 .

d) Amb l’ajuda d’un programa de geometria analítica comprova que


aquesta propietat es compleix per a tots els punts de la paràbola.
Et sembla convenient que les seccions dels focus dels cotxes
tinguin forma parabòlica?

1 
a) F  , 0 
 2 
b) La tangent a S(2, 2) serà de la forma y  2  m( x  2) amb m  0 . El sistema format per les equacions de la tangent
i de la paràbola ha de tenir solució única, així:

y  2  m( x  2)
 2  m2 ( x 2  4 x  4)  4m( x  2)  4  2 x  m2 x 2  (4m2  4m  2)x  (4m2  8m  4)  0 
 y  2 x

1
   b2  4ac  0  (4m2  4m  2)2  4m2 (4m2  8m  4)  0  16m2  16m  4  0  m 
2
1
Per tant, l’equació de la recta tangent és y  2  ( x  2)  x  2y  2  0 .
2
x 2 y 2
c) FS:   4 x  3y  2  0
3 2
2

46 2
Angle format per la tangent i FS: cos 1    1  26,57º
1  4 16  9 5

2 2
Angle format per la tangent i la recta y  2 : cos 2    2  26,57º
1 4 1 5

209 Matemàtiques 1r Batxillerat


SOLUCIONARI UNITAT 6. CÒNIQUES

AUTOAVALUACIÓ

Comprova el que has après


1. Calcula el centre i el radi d’aquesta circumferència.

x 2  y 2  4 x  10y  4  0

La circumferència té centre C(2, 5) i radi r  4  25  4  25  5 .

2. Calcula l’equació de la circumferència que té el seu centre en la intersecció de les rectes 2 x  y  5 i


2 x  y  3 i passa pel punt P(5,5).

2x  y  5
Centre:   x  2, y  1  C(2, 1) Radi: r  d (C, P )  32  42  25  5
2x  y  3

Equació: ( x  2)2  ( y  1)2  25  x 2  y 2  4x  2y  20  0

3. Troba l’equació de la circumferència que passa pels punts A(0, 0), B(1, 3) i C(8, 4). Calcula’n el centre i
el radi

L’equació serà de la forma x 2  y 2  Dx  Ey  F  0 , per tant:

F  0 F  0
 
1  9  D  3E  F  0  D  3E  10  D  10, E  0, F  0
64  16  8D  4E  F  0 8D  4E  80
 

L’equació de la circumferència és x 2  y 2  10x  0 , el seu centre és (5, 0) i el radi r  5 .

 
4. Calcula l’equació reduïda de l’el·lipse que passa pels punts 3,  2 3   i  4,

4 5
3
 . Troba els semieixos,
 
la semidistància focal i els vèrtexs. Quina és la seva excentricitat?

x2 y 2
L’equació és de la forma   1 , per tant:
a2 b2

 9 12
 a2  b2  1

 80  a2  36, b2  16  a  6, b  4
 16
 2  92  1
a b

x2 y 2
Equació:  1 Semieixos: a  6 i b  4 Semidistància focal: c  a2  b2  20  2 5
36 16

c 5
Vèrtexs: A(6, 0), A(6, 0), B(0, 4) i B(0, 4) Excentricitat: e  
a 3

210 Matemàtiques 1r Batxillerat


SOLUCIONARI UNITAT 6. CÒNIQUES

5. Troba l’equació i l’excentricitat de la hipèrbola de vèrtexs (3, 0) i (−3, 0) i asímptotes 3y  4 x  0 i


3y  4 x  0 .

b 4 x2 y 2
Tenim a  3 ,   b  4 i c  a2  b2  25  5 , per tant, l’equació és   1 i l’excentricitat és
a 3 9 16
c 5
e  .
a 3

6. Donada la paràbola y 2  2y  4 x  9  0 :

a) Calcula’n el focus i la directriu, i dibuixa-la.


b) Calcula l’abscissa del punt l’ordenada del qual és y = 5 i comprova que la distància d’aquest punt al focus coin-
cideix amb la distància a la directriu.

a) y 2  2y  4x  9  0  ( y  1)2  4( x  2)

Paràbola oberta cap a la dreta amb vèrtex V(2, 1) i paràmetre p  2 , per tant, el
seu focus és F(3, 1) i la directriu és d : x  1 .
b) y  5  25  10  4x  9  0  x  6  P(6, 5)

6 1
d (P, F )  9  16  25  5 d (P, d )  5
1

7. Els punts del pla (x, y) tals que la distància a un punt fix P(6, 0) és constant i igual a 5 configuren un lloc
geomètric.
a) Determina l'equació d'aquest lloc geomètric.
b) Descriu la figura obtinguda.

a) ( x  6)2  y 2  5  ( x  6)2  y 2  25  x 2  12x  36  y 2  25  x 2  y 2  12x  11  0

b) És una circumferència.

RELACIONA I CONTESTA
Tria l’única resposta correcta en cada cas

1. Escull l’equació que no representa una circumferència:

A. ( x  3)2  ( y  1)2  1 C. 3x 2  3y 2  2x  3y  3  0

x2 y2 1
B. 2x 2  2y 2  2x  3y  0 D.  
1 1 2
 
2 2

L’equació A representa una circumferència de centre (3, 1) i radi r  1 .

1 3 1 9 13
L’equació B representa una circumferència de centre  ,  i radi r    .
2 4 4 16 4

 1 1
L’equació C no representa una circumferència, ja que el seu centre seria  ,  i el seu radi
3 2
1 1 23
r   1  , que no és un nombre real.
9 4 6

211 Matemàtiques 1r Batxillerat


SOLUCIONARI UNITAT 6. CÒNIQUES

x2 y2 1 1 1
L’equació D,     x 2  y 2  , representa una circumferència de centre (0, 0) i radi r  .
1 1 2 4 2
 
2 2
Per tant, la resposta correcta és la C.

2. L’equació de l’el·lipse de focus els punts (3, 2) i (−3, 2) i que passa pel punt P(5, 2) és:

A. 16x 2  25y 2  500 C. 16x 2  25( y  2)2  500

x 2 ( y  2)2
B. 16x 2  25( y  2)2  1 D.  1
25 16

Els apartats B i C queden descartats, ja que P no compleix les equacions corresponents. D’altra banda, l’el·lipse ha
de tenir centre C(0, 2), la qual cosa descarta l’apartat A, per tant, la solució correcta ha de ser la D.

x 2 ( y  2)2
En efecte, el centre de l’el·lipse   1 és C, el punt P pertany a l’el·lipse i tenim a  5 i b  4 , amb la
25 16
qual cosa c  a2  b2  9  3 i els focus són (3, 2) i (3, 2).

1
3. La paràbola y 2  x 0:
1  a2
A. Té el focus a la part positiva de l’eix X. C. Té el focus a la part negativa de l’eix X.
B. Té el focus a la part positiva de l’eix Y. D. Té el focus a la part negativa de l’eix Y.

És una paràbola oberta cap a l’esquerra, amb vèrtex a l’origen de coordenades i eix l’eix X, per tant, la resposta
 1 
correcta és la C; de fet, el focus és F   , 0 .
 
 4 1  a2  

Assenyala, en cada cas, les respostes correctes

4. La paràbola y 2  y  2 x compleix que:

 1 1  1
A. El seu vèrtex és el punt  ,  . C. El seu focus és el punt  0,   .
8 2  2
B. La seva directriu és paral·lela a l’eix Y. D. La distància entre el seu focus i la seva directriu és 1.

2
 1  1
y 2  y  2x   y    2  x  
 2  8

 1 1 3 1
La paràbola està oberta cap a la dreta, el seu vèrtex és el punt   ,   , té paràmetre p  1 , focus  ,   i
 8 2 8 2
5
directriu x   , per tant, les respostes correctes són la B i la D.
8

212 Matemàtiques 1r Batxillerat


SOLUCIONARI UNITAT 6. CÒNIQUES

5. L’el·lipse 2 x 2  y 2  4 x  4y  0 verifica que:

A. El seu centre és el punt C(1, 2). C. La seva excentricitat val 3.


B. El seu eix major és paral·lel a l’eix X. D. Passa pel punt (2, 4).

( x  1)2 ( y  2)2
2x 2  y 2  4 x  4y  0   1
3 6

c 2
El centre és C(1, 2), a  6 , b  3 , c  a2  b2  3 i e   , per tant, les respostes correctes són la A i
a 2
la D.

Tria la relació correcta entre les dues afirmacions donades

6. Es considera el lloc geomètric dels punts del pla tals que la suma de les seves distàncies als punts F(10, 0)
i F ' (10, 0) és 2a, sent a un nombre real positiu. Es consideren les afirmacions:
1. El lloc geomètric és una el·lipse 2. a  10
A. 1  2 C. 1 i 2 són excloents entre elles.
B. 1  2 però 2  1 D. Cap de les anteriors.

Òbviament, la relació correcta és la A.

Assenyala la dada innecessària per contestar

x2 y 2
7. Es considera l’equació   1 , on a i b son nombres reals no nuls, i es vol deduir que representa una
a 2 b2
el·lipse l’eix major de la qual està contingut en l’eix Y. Es tenen, a més, les afirmacions següents:
1. b  a

2. La suma de distàncies d’un punt qualsevol de l’el·lipse als focus és 2b .

A. Cada afirmació és suficient per ella mateixa. C. 2 és suficient per ella mateixa, però 1 no.
B. 1 és suficient per ella mateixa, però 2 no. D. Son necessàries les dues afirmacions.

L’equació representa una el·lipse d’eixos continguts en els eixos de coordenades. Perquè l’eix major estigui
contingut a l’eix Y n’hi ha prou que es doni 2, però no n’hi ha prou amb 1, per exemple, si b  1 i a  2 es
compleix 1, però l’eix major de l’el·lipsi està contingut a l’eix X.
Per tant, la resposta correcta és la C.

213 Matemàtiques 1r Batxillerat


SOLUCIONARI UNITAT 7. NOMBRES COMPLEXOS

7. Nombres complexos
EXERCICIS PROPOSATS
1. Exercici resolt.

2. Representa gràficament els nombres z1  2  2i , z2  2i i z3  6 . Troba’n el mòdul i l’argument. Fes el


mateix amb els seus respectius conjugats i oposats.

2
z1  ( 2)2  ( 2)2  8  2 2 tg 1   1  Arg z1  1  225º
2

z2  02  22  2 Arg z2  90º

z3  ( 6)2  02  6 Arg z3  180º

z1  z1  z1  2 2 z2  z2  z2  2 z3  z3  z3  6

Arg z1  360º  Arg z1  135º Arg z2  360º  Arg z2  270º Arg z3  360º  Arg z3  180º

Arg(z1)  Arg z1  180º  45º Arg(z2 )  Arg z2  180º  270º Arg(z3 )  Arg z3  180º  0º

3. Donats z1  1  3i i z2  3  i , representa’ls gràficament i troba’n el mòdul i l’argument. Fes el mateix


amb els seus respectius conjugats i oposats.

 12   
2 3
z1  3 2 tg 1    3  Arg z1  1  120º
1

 3 1 3
2
z2   12  2 tg 2    Arg z2  2  30º
3 3

z1  z1  z1  2 z2  z2  z2  2

Arg z1  360º  Arg z1  240º Arg z2  360º  Arg z2  330º

Arg(z1)  Arg z1  180º  300º Arg(z2 )  Arg z2  180º  210º

4 a 6. Exercicis resolts.

214 Matemàtiques 1r Batxillerat


SOLUCIONARI UNITAT 7. NOMBRES COMPLEXOS

7. Si z1  1  3i i z2  3  i , troba el mòdul i l’argument dels nombres:

z1
a) z1z2 b) z12 c) d) z21
z2

a) z1z2  1 3i   
3  i   3  i  3i  3i 2  2 3  2i

 2 3  2 3
2
z1z2   22  4 tg     Arg  z1z2     150º
2 3 3

 
2
b) z1  1  3i  1 2 3i  3i 2  2  2 3i
2

 22   2  2 3
 
2
z12  3 4 tg    3  Arg z12  240º
2

c)
z1 1  3i
 
1  3i   3  i    3  i  3i  3 i 2 4i
 i
z2 3 i 3 i   3  i  3 1 4

z1 z 
 02  12  1 Arg  1   90º
z2  z2 

z2 3 i 3 1
d) z21  2
   i
z2 4 4 4

1
 3   1 2 1
2

z2 1
   
 4   4 

2
tg   4 
1

3
3

 Arg z21    330º 
  3 3
4

8. Determina quin nombre real a situa l’afix del complex  2  i   a  i  en la bisectriu del primer i tercer qua-
drant. Quins en són el mòdul i l’argument?

 2  i a  i   2a  2i  ai  i 2   2a  1  a  2 i . Perquè l’afix estigui en la bisectriu del primer i tercer quadrant, la


part real i imaginària han de coincidir, per tant, 2a  1  a  2  a  3 .

5
El nombre complex resulta que és 5  5i , de mòdul (5)2  (5)2  5 2 i argument tg    1    225º .
5

9. Justifica que (1  i )8 és un nombre real positiu.

(1 i )2  2i  (1 i )8  (2i )4  16i 4  16

Una altra possible resposta seria:

 
Arg(1 i )  45º  Arg (1 i )8  8  45º  360º  0º  (1 i )8 és un nombre real positiu.

10. Opera i escriu en forma binòmica aquests complexos:


1 i 2  3i 2 3 1 1 i
a) (1 i )(3  2i ) b) 2i (3  4i ) c) d) e)  f) 
i 4  2i 1 i 1 i 4  3i i

a) (1 i )(3  2i )  3  2i  3i  2i 2  5  i

b) 2i (3  4i )  6i  8i 2  8  6i

215 Matemàtiques 1r Batxillerat


SOLUCIONARI UNITAT 7. NOMBRES COMPLEXOS

1  i (1  i )i i  i 2
c)    1 i
i i2 1

2  3i (2  3i )(4  2i ) 8  4i  12i  6i 2 2  16i 1 4


d)      i
4  2i (4  2i )(4  2i ) 16  4 20 10 5

2 3 2(1  i )  3(1  i ) 2  2i  3  3i 1  5i 1 5
e)       i
1 i 1 i (1  i )(1  i ) 1 1 2 2 2

1 1 i 1 i (1  i )( 3  4i ) 3  4i  3i  4i 2 1  7i 1 7
f)        i
4  3i i 3  4i ( 3  4i )( 3  4i ) 9  16 25 25 25

a  2i
11. Calcula a perquè sigui imaginari pur.
5  12i

a  2i (a  2i )(5  12i ) 5a  12ai  10i  24i 2 24  5a 10  12a 24  5a 24


    i 0a
5  12i (5  12i )(5  12i ) 25  144 169 169 169 5

12. Escriu en forma binòmica 1  i  i 2  i 3  ...  i 100 .

i 101  1 i 425 1  1 i  1
1  i  i 2  i 3  ...  i 100    1
i 1 i 1 i 1

13 i 14. Exercicis resolts.

15. Expressa en forma polar aquests nombres complexos:

a) 1  3i b) 2 + 2i

a) Sigui el nombre complex a + bi. El seu mòdul en forma polar s’escriu z  a2  b2 , i l’argument,
b
Argz  arctg .
a

2 b 3
Si a = 1 i b  3 , aleshores z  12  3  4  2 i Argz  arctg  arctg  60º . El nombre complex és
a 1
z  260º .

b 2
b) a = b = 2. Així doncs, z  22  22  8 i Argz  arctg  arctg  45º . El nombre és z  8 45º .
a 2

16. Expressa en les tres formes habituals els complexos:

a) (1  i )2 b)
1
i
c) i 7  i 17 d)
1
2

1  i  1 i 4 

a) (1 i )2  1 i 2  2i  2i  290º  2 cos90º i sin90º 

1 i
b)   i  1270º  1 cos 270º i sin270º 
i i2

c) i 7  i 17  i 3  i 1  i  i  0 , no té forma polar ni trigonomètrica.

216 Matemàtiques 1r Batxillerat


SOLUCIONARI UNITAT 7. NOMBRES COMPLEXOS

d) 1  i 4  1 
1
i4
1 1
 
 1   2  1  i  1  i 4  1  i  2 45º  2 cos 45º i sin 45º 
1 2

17. Resol les operacions indicades pels complexos:


z1  260º z2  1  i
, i z3  2  cos210º  i sin210º  .

z1 z2
a) z1z2z3 b) c) z3 4 d) z22
z3

z1  260º , z2  1  i  2 135º i z3  2  cos210º i sin210º   2210º

 2 
a) z1z2z3  260º  2 135º  2210º  4 2   
 4 2   4 2  cos 45º i sin 45º   4 2 
 2

2
2
i   4  4i

405º 45º
 

z1z2 260º  2 135º


b)   2 285º  2 75º  2  cos 75º  sin75º   2 cos(45º 30º )  i sin(45º 30º ) 
z3 2210º

 2  cos 45º cos30º  sin 45º sin30º   i  sin 45º cos30º  cos 45º sin30º  

 2 3 2 1  2 3 2 1  3 1 3 1
 2     i       i

 2 2  
2 2  2 2 2 2   2 2

 1 3 
c) z3 4   2210º   16840º  16120º  16  cos120º i sin120º   16   
4
i   8  8 3 i
 2 2 
 

  1 1  1  1 1
2
d) z22  2 135º  2270º  z22   0º       i
z 2
2270º  2 270º  2 90º 2

18. Escriu en forma polar z  2  cos30º  i sin30º  .

z  2 cos30º i sin30º   2   cos30º i sin30º   2 cos210º i sin210º   2210º

També podríem haver observat que z és l’oposat de 2  cos30º i sin30º   230º i, per tant, z  2180º 30º  2210º .

19. Expressa en forma polar tots els complexos z que compleixin:


z
a) (2z  3)(iz  5)  0 c) 3z  1  i e)  1 i
z  2i

b) 2  3iz  4iz  9 d) (1 2i )z  3  5i f) 2z  i z  1

 3 3
 2z  3  0  z     
 2  2 180º
a) (2z  3)(iz  5)  0  
iz  5  0  z   5   5i  5i  5

 i i2
90º

7
b) 2  3iz  4iz  9  iz  7  z    7i  790º
i
1 1 1 1  2
c) 3z  1  i  z   i z  i 
3 3 3 3  3 
 315º

3  5i (3  5i )(1  2i ) 7  11i 7 11  34 
d) (1  2i )z  3  5i  z      i 
1  2i (1  2i )(1  2i ) 5 5 5  5 
 237,53º

e)
z
z  2i
 1  i  z  (1  i )z  2i (1 i )  iz  2  2i  z 
2  2i
i
 2  2i  2 2   45º

217 Matemàtiques 1r Batxillerat


SOLUCIONARI UNITAT 7. NOMBRES COMPLEXOS

2a  b  1 2 1
f) 2z  i z  1  2(a  bi )  i (a  bi )  1  (2a  b )  (2b  a )i  1   a  ,b  
a  2b  0 3 3

2 1  5
z  i 
3 3  3 
 333,43º

x  yi
20. Troba tots els nombres reals x i y tals que  x  yi . Expressa en forma polar i trigonomètrica x  yi i
x  yi
x  yi .

x  0, y  0
x  1, y  0

x  yi x  x2  y 2  1 3
 x  yi  x  yi  ( x  yi )2  x 2  y 2i 2  2xyi  ( x 2  y 2 )  2xyi    x  ,y 
x  yi y  2xy  2 2
 1 3
x  ,y 
 2 2

S’obtenen, per tant, quatre possibilitats per a z  x  yi i z  x  yi :

z  z  0 , que no té forma polar ni trigonomètrica.

z  z  1  10º  1 cos0º i sin0º 

1 3 1 3
z  i  1120º  1 cos120º i sin120º  i z     1240º  1 cos 240º i sin 240º 
2 2 2 2

1 3 1 3
z  i  1240º  1 cos 240º i sin 240º  i z     1120º  1 cos120º i sin120º 
2 2 2 2

21 i 22. Exercicis resolts.

1 3
23. Calcula les arrels cinquenes de  i expressant el resultat en forma polar.
2 2

1 3 60º 360º k
 i  160º  5 160º  s , amb s  5 1  1 i    12º 72º k  k  0, 1, 2, 3, 4  , així, les arrels
2 2 5
1 3
cinquenes de  i són z1  112º , z2  184º , z3  1156º , z4  1228º i z5  1300º .
2 2

24. Calcula i representa gràficament les arrels quadrades de –4.

180º 360º k
4  4180º  4180º  s , amb s  4  2 i    90º 180º k  k  0, 1 , per tant,
2
les arrels quadrades de –4 són z1  290º  2i i z2  2270º  2i .

25. Troba els complexos z que compleixen que: z  4 16

180º 360º k
16  16180º  4 16180º  s , amb s  4 16  2 i    45º 90º k  k  0, 1, 2, 3  , per tant, obtenim
4
quatre solucions: z1  245º  2  2i , z2  2135º   2  2i , z3  2225º   2  2i i z4  2315º  2  2i .

218 Matemàtiques 1r Batxillerat


SOLUCIONARI UNITAT 7. NOMBRES COMPLEXOS

26. Si una arrel sisena de z és 1 + i, calcula i representa gràficament les altres cinc arrels sisenes de z.

 
6
Tenim z  (1  i )   8270º , per tant, les arrels sisenes de z són de la forma s
6
2 45º
270º 360º k
amb s  6 8  2 i    45º 60º k  k  0, 1, 2, 3, 4, 5  , obtenim així les
6
arrels sisenes de z: z1  2 45º  1 i , z2  2 105º , z3  2 165º , z4  2 225º , z5  2 285º i
z6  2 345º .

Podem resoldre el problema de manera més simple recordant que si 1 i  2 45º és una arrel sisena de z, les
altres cinc arrels s’obtenen multiplicant 2 45º per 160º , 1120º , 1180º , 1240º i 1300º , i s’obté el mateix resultat que amb
el mètode anterior.

27. Un vèrtex d’un octàgon regular inscrit en una circumferència centrada en l’origen és el punt A(12, 5). Cal-
cula els dos vèrtexs adjacents.

360º
L’angle central d’un octògon regular és  45º , així, per trobar els vèrtexs adjacents demanats, n’hi ha prou
8
amb girar el vèrtex A respecte de l’origen de coordenades 45º i –45º.

2 2 2 2
És a dir, només cal multiplicar 12  5i per 145º   i i 145º   i , sent els vèrtexs adjacents els
2 2 2 2
afixos dels nombres que s’obtenen. D’aquesta manera:

 2 2  7 2 17 2  7 2 17 2 
12  5i    i 

 i  Vèrtex: 
 2
, 
 2 2  2 2  2 

 2 2  17 2 7 2  17 2 7 2
12  5i    i 

 i  Vèrtex: 

, 
 2 2  2 2  2 2 

28. Demostra que si k  cn  r amb r  0, 1, 2, …, n – 1 aleshores les raons trigonomètriques de l’angle


  360º k   360º r
coincideixen amb les del .
n n

Observem que tots dos angles es diferencien en un múltiple de 360º, per la qual cosa les seves raons trigonomè-
triques coincidiran.
  360º k   360º r 360º (k  r ) 360º cn
En efecte:     360º c .
n n n n

29. Exercici interactiu.

30 i 37. Exercicis resolts.

219 Matemàtiques 1r Batxillerat


SOLUCIONARI UNITAT 7. NOMBRES COMPLEXOS

EXERCICIS
Els nombres complexos
38. Siguin els nombres complexos assenyalats en la figura.

a) Troba’n la forma binòmica. c) Determina’n l’argument.


b) Calcula’n el mòdul. d) Hi ha cap parell que siguin conjugats entre si?

Identifiquem punts i vectors amb el nombre complex corresponent.

a) P  a  bi , Q  c  di i S  e  fi . Observem que R s’obté traslladant P segons el vector OQ , per tant,


R  (a  bi )  (c  di )  (a  c )  (b  d )i . Anàlogament, T s’obté traslladant S segons el vector OQ , per tant,
T  (e  fi )  (c  di )  (e  c )  (f  d )i .

b) P  a2  b2 , Q  c 2  d 2 , R  (a  c )2  (b  d )2 , S  e2  f 2 i T  (e  c )2  (f  d )2

b
c) Arg P  arctg   , s’ha d’agafar la solució del primer quadrant.
a

d 
ArgQ  arctg   , s’ha d’agafar la solució del primer quadrant.
c

 bd 
Arg R  arctg   , s’ha d’agafar la solució del primer quadrant.
 ac 

f   f d 
Arg S  arctg    90º  Arg R ArgT  arctg    90º  Arg R  ArgS
e ec 
d) Si hi hagués algun parell que fossin conjugats entre si serien simètrics respecte de l’eix X, cosa que no passa.

39. Representa els nombres complexos següents i, sense cap càlcul, obtén-ne els arguments.
a) 1  i c) 3i e) –3 g) 2  2i i) –5i

b) 5 d) 1 i f) 4  4i h) 1  3i j) 3 i

a) Arg 1 i   45º f) Arg  4  4i   315º

b) Arg  5  0º g) Arg  2  2i   135º

c) Arg  3i   90º 
h) Arg 1  3i  60º
d) Arg  1 i   225º i) Arg  5i   270º

e) Arg  3  180º j) Arg  


3  i  330º

220 Matemàtiques 1r Batxillerat


SOLUCIONARI UNITAT 7. NOMBRES COMPLEXOS

40. Siguin els nombres complexos: z1  2  i , z2  1  4i , z3  3  2i , z4  5i , z5  4 , z6  2  3i .

a) Representa’ls juntament amb els seus oposats i els seus conjugats.


b) Troba’n el mòdul i l’argument.
c) Determina el mòdul i l’argument dels seus oposats.
d) Determina el mòdul i l’argument dels seus conjugats.

a)

b) z1  5 , z2  17 , z3  13 , z4  5 , z5  4 i z6  13

 1 2
Arg z1  arctg     153,43º , Arg z2  arctg  4  75,96º , Arg z3  arctg    213,69º , Arg z4  270º ,
 2 3

 3
Arg z5  0º i Arg z6  arctg     303,69º
 2
c) El mòdul de cada oposat coincideix amb el mòdul del nombre complex del qual és oposat i els arguments es
diferencien en 180º:

z1  z1  5 , z2  z2  17 , z3  z3  13 , z4  z4  5 , z5  z5  4 i z6  z6  13

Arg  z1   Arg  z1   180º  333,43º , Arg  z2   Arg  z2   180º  225,96º , Arg  z3   Arg  z3   180º  33,69º

Arg  z4   Arg  z4   180º  90º , Arg  z5   Arg  z5   180º  180º i Arg  z6   Arg  z6   180º  123,69º

d) El mòdul de cada conjugat coincideix amb el mòdul del nombre complex del qual és conjugat i els arguments
sumen 360º:

z1  z1  5 , z2  z2  17 , z3  z3  13 , z4  z4  5 , z5  z5  4 i z6  z6  13

     
Arg z1  360º  Arg  z1   206,57º , Arg z2  360º  Arg  z2   314,04º , Arg z3  360  Arg  z3   146,31º ,

     
Arg z4  360º  Arg  z4   90º , Arg z5  360º  Arg  z5   360º  0º i Arg z6  360º  Arg  z6   56,31º

221 Matemàtiques 1r Batxillerat


SOLUCIONARI UNITAT 7. NOMBRES COMPLEXOS

41. Escriu en la forma a  bi , amb a i b reals, els nombres complexos següents:

3  6i 4
a) z  12  3i  4(5  8i ) d) z   2  i  1 2i 
2
g) z  
3i 3  4i

 2  2 2 1  2i  3 
2


b) z  3  i 5 3  i 5   e) z   
 2
2
i  
2   2
i 
2 
h) z   
5  3i  1  i 

5  15i  4  6i   1  3i 
c) z   4  3i  z
2
f) z i)  
1  2i  2  3i   3  2i 

a) z  12  3i  4(5  8i )  12  3i  20  32i  32  35i

  
b) z  3  i 5 3  i 5  9  5i 2  9  5  14

c) z   4  3i   16  9i 2  24i  16  9  24i  7  24i


2

 
d) z   2  i  1 2i   4  i 2  4i 1 2i   3  4i 1 2i   3  6i  4i  8i 2  11 2i
2

2
 2 2  2 2  2 2 1 1 2 
e) z    i   i     i      i  i   i
 2 2  2 2  2 2 
  2 2 

5  15i  5  15i 1  2i  5  10i  15i  30i 2 35  5i


f) z     7i
1  2i 1  2i 1  2i  1 4 5

3  6i 4  3  6i  3  i   4 3  4i   3  21i  12  16i  3  21 i  12  16 i  39  73 i
g) z   
3i 3  4i  3  i  3  i   3  4i 3  4i  10 25 10 10 25 25 50 50

1 2i  5  3i    3 1  i    1  13i   3  3i 2   1  13 i  9  9 i 2  9 i 
2 2
1  2i  3 
h) z        
5  3i  1  i   5  3i  5  3i   1  i 1  i   34  2  34 34 4 4 2

1 70
  i
34 17

 4  6i   1  3i
z
  2(2  3i )   1  3i  1 3i  3  2i   2 9  7i  18  14 i
i)    2
 2  3i   3  2i   2  3i   3  2i  3  2i 3  2i  13 13 13

42. Si z1  3  2i , z2  1  3i i z3  1  i , calcula:

z   1 
a) Re  1  b) Im  z1  2z2  z3 
 z2   2 

z1

3  2i

 3  2i  1  3i   3  11i  3  11 i  Re  z1   3
a)  
z2 1  3i  1  3i  1  3i  10 10 10  z2  10

1 1 1 11 9  1  9
b) z1  2z2  z3  3  2i  2  6i   i   i  Im  z1  2z2  z3   
2 2 2 2 2  2  2

222 Matemàtiques 1r Batxillerat


SOLUCIONARI UNITAT 7. NOMBRES COMPLEXOS

43. Per a cada complex z  x  iy , definim el complex Z  z 2  z . Prova que Re Z  x 2  y 2  x i que


Im Z  y (2x  1) .

Z  ( x  iy )2  ( x  iy )  x 2  i 2y 2  2xyi  x  iy  ( x 2  y 2  x )  y (2x  1)i  Re Z  x 2  y 2  x, Im Z  y (2x  1)

44. Resol les equacions d’incògnita z següents.

a) (1 i )z  3  i c)  2z  1 i  z  3  0
z 1
b) 2z  1 i  iz  2 d)  2i
z 1

3  i (3  i )(1  i ) 2  4i
a) (1  i )z  3  i  z     1 2i
1 i (1  i )(1  i ) 2

1 i (1  i )(2  i ) 1 3i 1 3
b) 2z  1  i  iz  2  (2  i )z  1  i  z      i
2  i (2  i )(2  i ) 5 5 5

 1 1
2z  1  i  0  z    i
c)  2z  1  i  z  3   0   2 2
z  3  0  z  3

z 1 1  2i ( 1  2i )(1  2i ) 3  4i 3 4
d)  2i  z  1  2iz  2i  (1  2i )z  1  2i  z      i
z 1 1  2i (1  2i )(1  2i ) 5 5 5

45. Considera el polinomi: P (z )  z 3  (2  3i )z 2  (13  i )z  6  10i

Són els nombres complexos i, 3 i 1  i arrels de P (z ) ?

P(i )  i 3  (2  3i )i 2  (13  i )i  6  10i  i  2  3i  13i  1 6  10i  3  i  0  i no és arrel de P.

P(3)  33  (2  3i )  32  (13  i )  3  6  10i  27  18  27i  39  3i  6  10i  42  14i  0  3 no és arrel de P.

P(1 i )  (1 i )3  (2  3i )(1 i )2  (13  i )(1 i )  6  10i  2  2i  4i  6  14  12i  6  10i  0  1 i és arrel de P.

46. Resol els sistemes d’equacions d’incògnites z1 i z2 :

3z  z  2  5i 2iz  z  2i
a)  1 2 c)  1 2
z1  z2  2  i 3z1  iz2  1

3z  z  5  2i 3z  2iz2  3  11i


b)  1 2 d)  1
z1  z2  1  2i z1  (i  1)z2  3  5i

a) Sumant les equacions tenim 4z1  4i  z1  i i, substituint, i  z2  2  i  z2  2  2i .

b) Restant les equacions tenim 4z1  4  4i  z1  1 i i, substituint, 1 i  z2  1 2i  z2  2  i .

c) Multiplicant la primera equació per i i sumant-li la segona equació tenim z1  1 i, substituint,


2i  z2  2i  z2  4i .

d) Restant a la primera equació el triple de la segona tenim:

12  4i 12  4i  3  5i  56  48i 28 24


( 3  5i )z2  12  4i  z2      i i, substituint,
3  5i  3  5i  3  5i  34 17 17

 28 24  52 4 1 81
z1   i  1    i   3  5i  z1   i  3  5i  z1   i.
 17 17  17 17 17 17

223 Matemàtiques 1r Batxillerat


SOLUCIONARI UNITAT 7. NOMBRES COMPLEXOS

47. En cada un dels casos següents, representa el conjunt dels afixos dels nombres complexos z que complei-
xen les condicions següents:

a) z 3 c) Im z  1 e) Re z  Im z

b) Re z  2 d) Im z  2 f) Re z  1

a) c) e)

b) d) f)

48. Sigui z  x  iy i w  iz  z  z  2i .

a) Comprova que w  w  2i ( x  2y  2) .

b) Demostra que l’afirmació “l’afix de w se situa sobre l’eix d’abscisses” és equivalent a “l’afix de z se situa en la
recta 2y  x  2 ”.

   
a) w  i z  z  z  2i  w  w  i z  z  2 z  z  4i  2i Re z  4i Im z  4i  2i Re z  2Im z  2  2i  x  2y  2 

b) Si l’afix de w se situa sobre l’eix d’abscisses tenim w  w i, per tant, x  2y  2  0 , és a dir, l’afix de z se situa
a la recta 2y  x  2 . Recíprocament, si l’afix de z es troba a la recta 2y  x  2 tenim w  w  0 , és a dir,
w  w i, per tant, l’afix de w se situa sobre l’eix d’abscisses.

iz
49. Determina tots els complexos z que facin que sigui un nombre imaginari pur.
z2

iz b  ai ( b  ai ) (a  2)  bi  2b  (b2  a2  2a)i


Si z  a  bi tenim    , que serà imaginari pur si i
z  2 (a  2)  bi (a  2)  bi (a  2)  bi  (a  2)2  b2
només si b  0 , per tant, els complexos z buscats són els nombres reals.

224 Matemàtiques 1r Batxillerat


SOLUCIONARI UNITAT 7. NOMBRES COMPLEXOS

Formes polar i trigonomètrica. Operacions


50. Escriu en forma polar els complexos següents.

1 3
a) z  2  2 3i f) z  i
4 4

4
b) z   2  2i g) z 
1 i

c) z  4  4i h) z  2  cos150º i sin150º 

d) i i) z  2   cos 45º i sin 45º 

e) z  2i j) z  cos60º i sin30º

a) z  4  12  4, Arg z  arctg 3  60º  z  460º

b) z  2  2  2, Arg z  arctg  1  135º  z  2135º

c) z  16  16  4 2, Arg z  arctg  1  315º  z  4 2  


315º

 1
d) z  0  12  1, Arg z  arctg    90º  z  190º . La unitat imaginària i s’escriu (0 + 1i) i està situada a l’eix
0
vertical.

e) z  0  4  2, Arg z  270º  z  2270º

f) z 
1

3
16 16 2
1
   1
 , Arg z  arctg  3  120º  z   
 2 120º

g) z 
4
1 i

40º
2 315º
 4 
 
 2 0º 315º
 2 2  315º

 2 2  45º

h) z és l’oposat de 2150º , per tant, z  2150º 180º  2330º .

i) z és l’oposat de 2 45º , per tant, z  2 45º 180º  2 225º .

1 1  2
z  cos 60º i sen30º   i 
 2 
j)
2 2  45º

Observem que el primer cas no és possible, ja que aleshores la part real de z seria cos(90º )  sin  , que no
coincideix amb sin(  )  sin excepte que   0º .

Així, ha de ser z  1270º  , i, en efecte, 1270º   cos(270º )  i sin(270º  )   cos(90º  )  i sin(90º  ) 

sin  i cos   z .

225 Matemàtiques 1r Batxillerat


SOLUCIONARI UNITAT 7. NOMBRES COMPLEXOS

51. Escriu en forma binòmica, polar i trigonomètrica els nombres complexos que tenen com a afixos els que
s’assenyalen a la figura.

A  4  5i  41128,66º  41  cos128,66º i sin128,66º 

B  2i  290º  2  cos90º i sin90º 

C  3  2i  13 33,69º  13 cos33,69º i sin33,69º 

D  1 3i  10 288,43º  10 cos288,43º i sin288,43º 


E  4  2i  2 5 333,43º
 2 5  cos333,43º i sin333,43º 

F  3  3180º  3  cos180º i sin180º 

3  7i
52. Escriu en forma trigonomètrica el complex: z  .
2  5i

3  7i  3  7i  2  5i  29  29i
z    1  i  2  cos 225º i sin 225º 
2  5i  2  5i  2  5i  29

3 i
53. Donats els complexos: z1  2(1  i ) i z2   .
2 2
a) Escriu z1 i z2 en forma trigonomètrica.

z1
b) Si w  , escriu w en forma binòmica i trigonomètrica i dedueix el valor exacte de cos 75º i sin 75º.
z2

a) z1  2(1 i )  2  2i  2 cos 45º i sin 45º  z2  1 cos330º i sin330º 

z1 2  cos 45º i sin 45º 


b) w    2  cos( 285º )  i sin( 285º )   2 cos75º i sin75º 
z2 1 cos330º i sin330º 

 3 1 
2  2i
 2  2i  2
 i
2 
6 2

6 2
i
6 2 6 2
z
w 1     2 2   i
z2 3 1  3 1  3 1  1 2 2
 i   i   i
2 2  2 2   2 2 

6 2 6 2 6 2 6 2
Per tant es dedueix que 2cos 75º   cos 75º  i 2 sin75º   sin75º 
2 4 2 4

226 Matemàtiques 1r Batxillerat


SOLUCIONARI UNITAT 7. NOMBRES COMPLEXOS

54. Si z  3  i , calcula z 2 , z 2 i z .
2

2
z  3  i  2330º  z2  4660º  4300º , z2  4 i z  22  4

55. Si z  6  2  i  
6  2 , escriu en forma binòmica els complexos z 2 , z 6 , z 9 i z 12 .

S’escriu z en forma polar:

   
2 2
z  6 2 6 2  6  2  2 12  6  2  2 12  16  4 , d’altra banda, a l’exercici 62 s’ha provat

6 2 6 2 6 2
que sin75º  i cos 75º  , d’on tg75º  . Així, si Argz   , tenim
4 4 6 2
6 2 1
tg       cotg75º    345º , per tant, z  4345º .
6 2 tg75º

D’aquesta manera:

 3 1 
z2   4345º   16690º  16330º  16  cos330º i sin330º   16 
2
 i   8 3  8i
 2 2 

z6   4345º   40962070º  4096270º  4096i


6

 
z9   4345º   218
9
  3105º
 
 218
225º
 218  cos 225º i sin 225º   218  
 2
2

2
2
i   217 2  217 2i

 

   
2 2
  4096i   212 i
2
z12  z6  224 i 2  224

30
 1 i 3 
56. Calcula el nombre complex següent:   .
 1 i
 
30
 1 i 3 
1 i 3  2 
   
2 30
1  i 3  260º i 1 i  2 45º , per tant,  60º     2 15º i    2 15º  215 
1 i 2 45º  2 15º  1 i
450º

 
 215
90º
 215 i

 
n
57. Decideix els valors de l’enter n per al qual el complex 3 i sigui:

a) Un nombre real b) Un nombre real positiu c) Un nombre imaginari pur

   
n
  230º   2n
n
3  i  230º  3 i
30º n

a) 30º n  180º k  n  6k per a algun enter k.

b) 30º n  360º k  n  12k per a algun enter k.

c) 30º n  90º 180º k  n  3  6k per a algun enter k.

227 Matemàtiques 1r Batxillerat


SOLUCIONARI UNITAT 7. NOMBRES COMPLEXOS

58. Resol l’equació següent: 2z  (1  i )z  1  5i .

Sigui z  a  bi :

a  b  1
2(a  bi )  (1  i )(a  bi )  1  5i  2a  2bi  a  bi  ai  b  1  5i  (a  b)  (3b  a)i  1  5i   
a  3b  5

a  b  1
  a  1, b  2  z  1  2i
a  3b  5

Una altra manera de resoldre l’equació és agafar conjugats, amb la qual cosa s’obté un sistema lineal amb in-
cògnites z i z .

Radicació de nombres complexos


59. Resol les equacions següents i escriu-ne les solucions en forma binòmica.

a) z3  i  0 c) z 4  1  0 e) z6  1  0

b) z6  64  0 d) z4  81  0 f) z4  81  0

90º 360º k
a) z3  i  0  z  3 i  3 190º  s , amb s  3 1  1 i    30º 120º k  k  0, 1, 2 , així:
3

3 1 3 1
z1  130º   i , z2  1150º    i i z3  1270º  i .
2 2 2 2

0º 360º k
b) z6  64  0  z  6 64  6 640º  s , amb s  6 64  2 i    60º k  k  0, 1, 2, 3, 4, 5  , així:
6
z1  20º  2 , z2  260º  1 3i , z3  2120º  1 3i , z4  2180º  2 , z5  2240º  1 3i i
z6  2300º  1  3i .

180º 360º k
c) z4  1  0  z  4 1  4 1180º  s , amb s  4 1  1 i    45º 90º k  k  0, 1, 2, 3  , i s’obté:
4
2 2 2 2 2 2 2 2
z1  145º   i , z2  1135º    i , z3  1225º    i i z4  1315º   i.
2 2 2 2 2 2 2 2

180º 360º k
d) z4  81  0  z  4 81  4 81180º  s , amb s  4 81  3 i    45º 90º k  k  0, 1, 2, 3  , així:
4
3 2 3 2 3 2 3 2 3 2 3 2 3 2 3 2
z1  345º   i , z2  3135º    i , z3  3225º    i i z4  3315º   i.
2 2 2 2 2 2 2 2

0º 360º k
e) z6  1  0  z  6 1  6 10º  s , amb s  6 1  1 i    60º k  k  0, 1, 2, 3, 4, 5  , s’obté: z1  10º  1 ,
6
1 3 1 3 1 3 1 3
z2  160º   i , z3  1120º    i , z4  1180º  1 , z5  1240º    i i z6  1300º   i.
2 2 2 2 2 2 2 2

0º 360º k
f) z4  81  0  z  4 81  4 810º  s , amb s  4 81  3 i   90º k  k  0, 1, 2, 3  , per tant:
4
z1  30º  3 , z2  390º  3i , z3  3180º  3 i z4  3270º  3i .

228 Matemàtiques 1r Batxillerat


SOLUCIONARI UNITAT 7. NOMBRES COMPLEXOS

60. A partir de les dades de la figura, calcula de forma exacta i raonada la forma binòmica dels afixos corres-
ponents als vèrtexs del quadrat que es representa.

S’identifiquen punts i vectors amb el complex corresponent.


Calculant en primer lloc el centre de la circumferència s’observa que els punts P  1 3i i Q  2  4i són els
1 1
extrems d’un diàmetre, per tant, el centre serà el punt mitjà del segment PQ, R   i .
2 2
Fent una translació que porti el punt R a l’origen de coordenades O, és a dir, una translació de vector
1 1
RO    i els punts A, B, C i D es transformen en els punts A’, B’, C’ i D’, vèrtexs d’un quadrat centrat en O,
2 2
per tant, D '  iA ' , C '  iD '  A ' i B '  iC '  D ' . A més, D '  2 5  3i  A ' , per tant:

2 5  3i 2 5  3 2 5  3 2 5 3 2 5 3
iA '  2 5  3i  A '  ( 1  i )A '  2 5  3i  A '    i, D'    i,
1  i 2 2 2 2
2 5 3 2 5 3 2 5 3 2 5 3
C'    i i B'   i.
2 2 2 2

Finalment, desfent la translació, obtenim A   5 2     


5 2 i , D  2 5    
5 1 i ,

  
C   1 5  1 5 i i B    5 1   5 2 i . 

61. Si P  
3, 1 és un vèrtex d’un hexàgon regular inscrit en la circumferència centrada en l’origen i de radi 2,
calcula els restants de les dues maneres següents.

a) Multiplicant z  3  i per determinats nombres complexos.

 
6
b) Calculant les arrels sisenes de 3 i .

a) Els restants vèrtexs s’obtenen girant P 60º reiteradament respecte de l’origen de coordenades, és a dir, multipli-
cant z  3  i  230º per 160º , 1120º , 1180º , 1240º i 1300º , i s’obtenen els punts:

 
230º  160º  290º  2i  0, 2 , 230º  1120º  2150º   3  i   3, 1 , 230º  1180º  2210º   3  i   3,  1 ,  
230º  1240º  2 270º  2i   0,  2 i 230º  1300º  2330º  3  i   3,  1 
 
6
  230º   64180º , és a dir, són els
6
b) Els vèrtexs de l’hexàgon són els afixos de les arrels sisenes de 3 i
180º 360º k
afixos dels complexos s amb s  6 64  2 i    30º 60º k  k  0, 1, 2, 3, 4, 5  :
6

230º  3  i  P  
3, 1 , 290º  2i   0, 2 , 2150º   3  i   3, 1 ,   
2210º   3  i   3,  1 , 
2270º  2i   0,  2 i 2330º  3  i   3,  1

229 Matemàtiques 1r Batxillerat


SOLUCIONARI UNITAT 7. NOMBRES COMPLEXOS

62. Troba els nombres reals a i b perquè es compleixi la relació  a  bi   i de dues maneres:
2

a) Calculant les arrels quadrades de i.

b) Desenvolupant  a  bi  .
2

 2 2 2 2
k  0  145º   i a ,b
 2 2 2 2
a) a  bi  i  190º  190º 360º k  145º 180º k 
 2 2 2 2
k  1  1225º   2  2 i  a   2 , b   2
2

a2  b2  0 a  b
b)  a  bi 2  a2  b2i 2  2abi  a2  b2   2abi   
2ab  1 2ab  1

1 2 2 2
Si a  b obtenim 2a2  1  a2  a , és a dir, obtenim dues solucions, a  b  i ab .
2 2 2 2
Si a  b obtenim 2a2  1 , que no té solució real.

63. Sigui z un nombre complex.

a) Calcula els nombres reals a, b i c perquè es compleixi la igualtat: z3  8  (z  2)(az2  bz  c ) .

b) Troba de dues maneres diferents les arrels cúbiques de –8.

a) (z  2)(az2  bz  c )  az3  (2a  b)z2  (c  2b)z  2c  a  1, 2a  b  0, c  2b  0, 2c  8  a  1, b  2, c  4

180º 360º k
b) 3
8  3 8180º  s , amb s  3 8  2 i    60º 120º k  k  0, 1, 2 , per tant, les arrels cúbiques
3
de –8 són 260º  1 3i , 2180º  2 i 2300º  1  3i .

Una altra manera de calcular les arrels cúbiques de –8 és observar que són les solucions de z3  8  0 , és a
2  12 2  2 3i
dir, segons l’apartat anterior, z  2  0  z  2 i z2  2z  4  0  z    1  3i .
2 2

64. a) Justifica que (1 i )  8i .


6

b) Considera l’equació z2  8i :


i) Dedueix de l’apartat a) una solució de l’equació.

ii) Dedueix de l’apartat a) una solució per a z3  8i .

1 i 6   
6
a) 2 45º  8270º  8i

 
   2 2  2 2
3
b) i) Segons l’apartat a), una solució és z  (1  i )3  2 45º  2 2   i   2  2i .
 2 2 
135º
 

ii) Segons l’apartat a), una solució és z  (1 i )2  2i .

230 Matemàtiques 1r Batxillerat


SOLUCIONARI UNITAT 7. NOMBRES COMPLEXOS

Equacions i nombres complexos


65. Resol aquestes equacions de segon grau:
2
a) x + 3x + 6 = 0
2
b) x – x + 3 = 0
2
c) x + 2x + 10 = 0
2
d) –x + 4x + 5 = 0
2
e) x + 2x + 8 = 0
2
f) 3x + 5x + 3 = 0

3  9  24 3  15 3  15  1 3  15  i
a) x    
2 2 2 2

1  1  12 1  11 1  11  1 1  11  i
b) x    
2 2 2 2

2  4  40 2  36 2  36  1 2  6  i
c) x      1  3i
2 2 2 2

4  16  20 4  36 4  6  x  5
d) x    
2 2 2  x  1

2  4  32 2  28 2  28  1 2  2 7  i
e) x      1 7  i
2 2 2 2

5  25  36 5  11 5  11  1 5  11  i
f) x    
6 6 6 6

66. A partir de les solucions de les equacions de segon grau, determina l'equació d'aquestes:
a) 2 + 3i, 2 – 3i
b) 1 + 4i, 1 – 4i
c) 3 – 2i, 3 + 2i
d) 5 + i, 5 – i

a)  x   2  3i    x   2  3i    x 2   2  3i  x   2  3i  x   2  3i  2  3i  

 x 2   2  3i  2  3i  x  22   3i   x 2  4x  13
2

b)  x  1 4i    x  1 4i    x 2  1 4i  x  1 4i  x  1 4i 1 4i  

 x 2   1 4i  1 4i  x  12   4i   x 2  2x  17
2

c)  x   3  2i    x   3  2i    x 2  3  2i  x  3  2i  x  3  2i 3  2i  

 x 2   3  2i  3  2i  x  32   2i   x 2  6x  13
2

d)  x   5  i    x   5  i    x 2   5  i  x   5  i  x   5  i  5  i  

 x 2   5  i  5  i  x  52   i   x 2  10x  26
2

231 Matemàtiques 1r Batxillerat


SOLUCIONARI UNITAT 7. NOMBRES COMPLEXOS

67. Resol aquestes equacions de tercer grau. Una solució és real i les altres són complexes.
3
a) x + 5x – 6 = 0
3
b) x – x + 6 = 0
3 2
c) x + 3x + 4x + 4 = 0
3 2
d) x + 2x + 6x + 5 = 0

x  1  0  x  1

 
a) x 3  5x  6  ( x  1) x 2  x  6  0   1  23 1  23  i
x  x  6  0  x  
2
 2 2

 x  2  0  x  2

b) x  x  6  ( x  2)  x  2x  3   0  
3 2
2  8 2  8  i
 x  2x  3  0  x    1 2  i
2
 2 2

 x  2  0  x  2

 
c) x 3  3x 2  4x  4  ( x  2) x 2  x  2  0   1  7 1  7  i
x  x  2  0  x  
2
 2 2

 x  1  0  x  1

 
d) x 3  2x 2  6x  5  ( x  1) x 2  x  5  0   1  19 1  19  i
x  x  5  0  x  
2
 2 2

QÜESTIONS

68. Escriu en funció de z el conjugat dels complexos w següents.


1  iz
a) w  2  3z b) w  (1 iz)(1 2z) c) w  d) w  z3  2iz2  1 3i
3z

1 i z 3 2
a) w  2  3z b) w  (1 i z)(1 2z) c) w  d) w  z  2i z  1 3i
3z

1 1
69. Justifica que si z  0 i z   1 , aleshores Re z  .
z 2

1 1 3 1 3 1
z  1  z2  1  z  z2  z  1  0  z   i, z   i , en qualsevol cas, Re z  .
z 2 2 2 2 2

70. Si z és real, és cert que z  z ?

No, només és cert si z > 0.

71. Per a qualsevol complex z, es compleix que Re(z 4 )  4Re z ?

No, per exemple, si z  i tenim Re(z 4 )  1 i 4Re z  0 .

232 Matemàtiques 1r Batxillerat


SOLUCIONARI UNITAT 7. NOMBRES COMPLEXOS

72. Si z  z és un nombre real, què pots dir de z?

Recordem que z  z  2i Im z , per tant, si és un nombre real ha de ser Im z  0 , és a dir, z és real.

73. Observa la figura i raona si els afixos corresponents als vèrtexs del pentàgon poden correspondre a les ar-
rels cinquenes d’un nombre real.

0º 360º k
Les arrels cinquenes d’un nombre real positiu tenen com a argument  72º k (k  0, 1, 2, 3, 4) i les ar-
5
180º 360º k
rels cinquenes d’un nombre real negatiu tenen com a argument  36º 72º k (k  0, 1, 2, 3, 4) . En cap
5
cas es corresponen amb els arguments dels vèrtexs del pentàgon de la figura, per tant, aquests no poden ser les
arrels cinquenes d’un nombre real.

74. És cert que si z  1 , aleshores z  1 o z  1 ?


4

Si z 4  1 , z és una de les arrels quartes de 1, però no necessàriament z  1 o z  1, també podria ser z  i o
z  i .

 1  i n  1  i n
75. És cert que si n és un enter positiu, aleshores el complex z  es un nombre real?
2

No, de fet, z és sempre imaginari pur, ja que 1  i n i 1  i n són conjugats, per tant,
2i Im 1  i  
n
1  i  n
 1  i 
n
   i Im  1  i n  .
z 

  
2 2

76. Demostra aquestes igualtats:

a) z1  z2  z1  z2 b) z1  z2  z1  z2

a) z1  z2  z1  z2

z1  a  bi  z1  a  bi 

Si   z1  z2  (a  c )  (b  d )i
z2  c  di  z2  c  di 

Per altra banda:

z1  z2  a  bi  c  di  (a  c)  (b  d )i  z1  z2  (a  c)  (b  d )i

b) z1  z2  z1  z2


z1  a  bi  z1  a  bi 
  z1  z2  (a  c )  (b  d )i
z2  c  di  z2  c  di 

Per altra banda:

z1  z2  a  bi  (c  di )  (a  c)  (b  d )i  z1  z2  (a  c)  (b  d )i

233 Matemàtiques 1r Batxillerat


SOLUCIONARI UNITAT 7. NOMBRES COMPLEXOS

PROBLEMES
77. Siguin a, b, c, d nombres reals. Si z1  a  bi i z2  c  di :

a) Calcula el mòdul z1z2 de dues maneres diferents i demostra que  ac  bd    ad  bc   a2  b2 c 2  d 2 .


2 2
  
b) Demostra que 34 · 122 es pot escriure com la suma de dos quadrats de nombres enters.

a) z1z2  z1 z2  a2  b2 c 2  d 2

z1z2   ac  bd    ad  bc  i  z1z2  ac  bd 2  ad  bc 2

Per tant,  ac  bd 2  ad  bc 2  a 2  b 2 c 2  d 2  ac  bd   ad  bc


2
2  a 2  b 2 c 2 d 2 

  
b) 34  122  52  32 112  12   5  11 3  1  5  1  3  11  522  382
2 2

78. a) Demostra que el polinomi P(z)  2z 4  3z2  3 3z  9 admet una arrel de la forma (1  i ) sent  un nombre
real. Determina el valor de  .


b) Demostra que: P(z)  2  z  (1  i ) z  (1  i ) z2  3z  3 . 
c) Escriu en forma polar totes les arrels de P ( z ) .

  
a) P (1 i )  2 (1 i )  3 (1 i )  3 3 (1 i )  9  8 4  3 3  9  62  3 3 i
4 2

3
La part imaginària s’anul·la si   0 o    , en el primer cas la part real no s’anul·la, però sí en el segon,
2
4
 3  3 9 9 3 3
ja que 8    3 3  9    9  0   (1  i ) és una arrel de P ( z ) i   
 2   2 
.
    2 2 2 2

b) Com que els coeficients de P ( z ) són nombres reals, també (1  i ) és arrel de P ( z ) , amb la qual cosa tindrem
 
P(z)  2  z  (1 i ) z  (1 i ) z2  bz  c . Per calcular b i c, en lloc de multiplicar i identificar coeficients, és

millor dividir P ( z ) entre 2  z  (1 i ) z  (1 i )  2z2  2 3z  3 per obtenir que z2  bz  c  z2  3z  3 ,
la qual cosa demostra b).

3 3 3  6 3 3 3  6
c) Les arrels de P ( z ) són z   (1  i )    i  , z (1  i )    i 
 2   2 
i
2 2 2  225º 2 2 2  135º
 3 3
  i  3 60º
3  9 3  3i  2 2
z2  3z  3  0  z    .
2 2  3 3
  i  3 300º
 2 2

234 Matemàtiques 1r Batxillerat


SOLUCIONARI UNITAT 7. NOMBRES COMPLEXOS

79. En cada un dels casos següents, troba el conjunt d’afixos dels z que compleixen les condicions donades:
a) |z| = 4

b) z2  3

c) zi 1

h) z  1 i  2

a) Punts la distància dels quals al punt (0, 0) és 4, és a dir, la circumferència de centre (0, 0) i radi 4.
b) Punts la distància dels quals al punt (2, 0) és 3, és a dir, la circumferència de centre (2, 0) i radi 3.
c) Circumferència de centre (0, –1) i radi 1. Efectivament:
si z = x + yi  z + i = x + yi + i = x + (y +1)i 

z  i  1  x  (y  1)i  1  x 2  (y  1)2  1  x 2  (y  1)2  1

d) Circumferència de centre (1, –1) i radi 2. Efectivament:

z  1 i  2  x  1 (y  1)i  2  (x  1)2  (y  1)2  2  (x  1)2  (y  1)2  4

80. Expressa en forma polar els complexos en què els seus afixos se situen en les zones acolorides (excloses
les fronteres).
a) c)

b) d)

2 2
 1  1 4
a) Els punts interiors de la circumferència són els (x, y) tals que  x     y    .
 3  3 9

Ho podem expressar com a suma de dos complexos:

 2 2
z = OC = OB + BC =  + r , amb 0  r  i 0º    360º
 3  3
 45º

235 Matemàtiques 1r Batxillerat


SOLUCIONARI UNITAT 7. NOMBRES COMPLEXOS

b) Els punts demanats són els que compleixen 2x – y > –1.


Ho podem expressar com a suma de dos complexos:
z  190º  r , amb r > 0 i (arctg 2 – 180º) <  < arctg 2, és a dir, –118º 33’ 54,18’’<  < 63º 26’ 5,82’’

c) r amb 0  r  1 i 0º    90º

d) r60º amb r  0

PER APROFUNDIR
81. Si l’afix de z  x  iy està alineat amb els afixos de i i de iz, quina relació existeix entre x i y?

z  x  iy  iz  y  ix i ens demanen la relació entre x i y sabent que els punts A(x, y), B (0, 1) i C (y, x) estan
alineats.
x 1 y
D’aquesta manera,   x 2  y 2  x  y  0 , és a dir, (x, y) són les coordenades d’un punt de la cir-
y  x x  y
 1 1 2
cumferència de centre  ,  i radi .
2 2 2

 
m
 1  i  .
n
82. Troba els enters positius m i n més petits que compleixen que 1  i 3

   2  i 1  i  2 315º  1  i    2 
m n
1  i 3  260º  1  i 3 m n
60º m  315º n
n
Perquè tots dos nombres siguin iguals s’ha de complir que 2m  2  n  2m i que 315º n  60º m sigui múltiple
de 360º, és a dir, 630º m  60º m  570º m ha de ser múltiple de 360º, l’enter més petit per al qual això es compleix
és m  12 i, per tant, n  24 .

83. Si z1 i z2 són dos nombres complexos de mòdul 1, demostra que


 z1  z2 2 és un nombre real no nega-
z1z2
tiu.

Com que
 z1  z2 2 
z12  z22  2z1z2 z z z z
 1  2  2 , només s’ha de demostrar que 1  2 és un nombre real
z1z2 z1z2 z2 z1 z2 z1
més gran o igual que 2.
z1 z
Per fer-ho, observem que i 2 són dos nombres complexos inversos i de mòdul 1, per tant han de ser
z2 z1
z1 z z z
conjugats, per exemple  w i 2  w , així 1  2  2Re w és un nombre real.
z2 z1 z2 z1

z1 z2
D’altra banda, com que w té mòdul 1, tenim Re w  1 , per la qual cosa   2Re w  2 .
z2 z1

236 Matemàtiques 1r Batxillerat


SOLUCIONARI UNITAT 7. NOMBRES COMPLEXOS

84. Calcula la part imaginària del complex:

z   cos12º  i sin12º  cos 48º  i sin48º 


6

Volem calcular la part imaginària de z  112º  148º  , per fer-ho, observem que
6

36º
48º 12º  36º , per tant, l’argument de 112º  148º és 12º   30º (vegeu la figura).
2
6
 
Per tant, 112º  148º  r30º i z   r30º   r 6
180º
, és a dir, Im z  0 .

85. Sigui z0  cos72º  i sin72º .

a) Comprova que z0 és arrel del polinomi P(z)  z5  1 .

b) Prova la relació:
2
 1  1
 z0     z0    1  0
 z0   z0 

1 5 1
c) Comprova que z0   2cos 72º i demostra que cos 72º  .
z0 4

d) Construeix un pentàgon regular utilitzant solament el compàs i un regle no graduat, basant-te en els resultats
dels apartats anteriors.

a) z0  cos72º i sin72º  172º  P(z0 )  z05  1  1360º  1  1 1  0

b) Com que P(z)  z5  1  (z  1)(z4  z3  z2  z  1) i z0  1 , tenim z04  z03  z02  z0  1  0 .

Com que z0  0 podem dividir per z0 2 per obtenir:


2
1 1 1 1  1  1
z02  z0  1    0  z02  2  z0   1  0   z0     z0    1  0
z0 z02 z0 z0  z0   z0 

1 1
c) Com que z0  1 tenim  z0 i, per tant, z0    cos72º i sen72º   cos72º i sen72º  2cos72º .
z0 z0

D’aquesta manera, segons l’apartat anterior, 2cos72º és la solució positiva de l’equació z2  z  1  0 , és a


1  5 5 1
dir, 2cos 72º  , d’on cos 72º  .
2 4

5 1
d) Una vegada tenim que cos 72º  i, atès que l’angle central en un pentàgon regular és   72º , agafant
4
com a vèrtex del pentàgon regular el punt A(1, 0), la construcció d’un vèrtex adjacent és immediata: construïm
amb el compàs 5  22  12 i posteriorment 5  1 sobre l’eix X.
Dividint aquest últim segment en quatre parts iguals, aixequem una perpendicular per la primera divisió i el punt
on talla la circumferència centrada a l’origen i radi 1 és el vèrtex buscat.

86. a) Resol en l’equació z2  2 2z  4  0 .

b) Si representem per z1 la solució en la qual la part imaginària és positiva i per z2 l’altra solució, escriu en forma
2
z 
polar z1 , z2 i  1  .
 z2 

237 Matemàtiques 1r Batxillerat


SOLUCIONARI UNITAT 7. NOMBRES COMPLEXOS

2 2  8 2 2  2 2i 
z  2  2i
a) z2  2 2z  4  0  z   
2 2 z  2  2i

2 2
z  2 
b) z1  2  2i  245º , z2  2  2i  2315º i  1    45º   1270º   190º   1180º
2 2

 z2   2315º 

87. Resol aquestes equacions:

a) x 4  x 3  9x 2  9x  0 b) x 4  26x 2  25  0

x  0
a) x  x  9x  9x  x(x  1)(x  9)  0 
4 3 2 2
x  1  0  x  1
 x 2  9  0  x 2  9  x   9   9  1  3i


 x  1  0  x  1  x   1  i
2 2
b) x 4  26x 2  25  ( x 2  1)(x 2  25)  0 
 x  25  0  x  25  x   25   25  1  5i
 2 2

238 Matemàtiques 1r Batxillerat


SOLUCIONARI UNITAT 7. NOMBRES COMPLEXOS

ENTORN MATEMÀTIC
Baralles matemàtiques
Avui a classe, l’Ivan i la Sara han tingut una forta discussió perquè tots dos deien que havien resolt abans un
problema difícil de geometria. El professor veu que la seva alta competitivitat pot eclipsar el seu talent per a les
matemàtiques i, malgrat que a ells no els fa cap gràcia, els proposa un exercici:
Estem treballant amb nombres que són una eina fonamental en un munt de camps científics i tècnics. En els
seus inicis, van donar lloc a una amarga baralla entre dos grans matemàtics del Renaixement italià: Cardano i
Tartaglia que, malgrat el seu geni, no van saber reconèixer en l’altre un matemàtic equiparable a ell. Vull que
reflexioneu sobre això i que treballeu sobre el problema següent:
Un bon exemple de la utilitat d’aquests nombres, que anomenem imaginaris, és la resolució de l’equació cúbica
x 3  px  q que Tartaglia va enunciar així:

“Per resoldre l’equació x 3  px  q , troba dos nombres en què la seva resta sigui q i el seu producte sigui
3
 p
  . La solució serà la diferència de les arrels cúbiques dels dos.”
3
En efecte, donats dos nombres qualssevol u i v, tenim que:

u 3  v 3   u  v   3 u  v  v  3 u  v  v 2
3 2

Extraient factor comú i operant, resulta:

u 3  v 3  u  v   3 u  v  v u  v   v   u  v   3 u  v  uv
3 3

i anomenant x  u  v , s’arriba a x 3  3uvx  u3  v 3 , que s’assembla molt a la nostra equació. N’hi ha prou de
3uv  p
triar u i v amb la condició:  3 . D’aquesta manera, x  u  v serà la solució buscada.
u  v  q
3

La vostra tasca és revisar el mètode de Tartaglia, aplicar-lo a la resolució de l’equació x 3  15 x  4 , identificar


aquests nombres imaginaris i explicar als vostres companys com els heu fet anar.
Ets capaç de resoldre el problema que han plantejat a l’Ivan i la Sara?

3uv  15 uv  5 5


Per resoldre el sistema  3  3 , s’aïlla a la primera equació v   i se substitueix a la segona per
u  v 3
 4 u  v 3
 4 u
3
 5 125
obtenir u 3      4  u 3  3  4 .
 u u

125
Anomenem z  u 3 i tenim z   4  z2  4z  125  0 i és en resoldre aquesta equació quan apareixen els nom-
z
bres complexos, ja que les solucions són z  2  11i i z  2  11i .

Agafant, per exemple, z  2  11i (si s’agafés z  2  11i el resultat final seria el mateix), tenim u3  2  11i , amb la qual
5 5
cosa es poden calcular les tres arrels cúbiques de 2  11i , u1 , u2 i u3 , calcular els corresponents v1   , v 2  
u1 u2
5
i v3   i trobar així les tres solucions x1  u1  v1 , x2  u2  v 2 i x3  u3  v 3 .
u3
3
Per simplificar els càlculs només s’ha d’observar que 2  11i  125  5 2 , per la qual cosa les arrels cúbiques de
1
5 5u
2  11i tenen mòdul 5 2  5 . Així, si u és una d’aquestes arrels cúbiques, tindrem v     2  u i
u u
u  v  u  u  2Re(u ) . Això prova, en particular, que les tres solucions de x 3  15x  4 seran reals.

239 Matemàtiques 1r Batxillerat


SOLUCIONARI UNITAT 7. NOMBRES COMPLEXOS

 1 3 
Només s’ha de trobar una arrel cúbica u1 de 2  11i , ja que aleshores es tindrà u2  u1  1120º     i  u1 i
 2 2 

 1 3 
u3  u2  1240º     i  u1 .
 2 2 

Es calcula, per tant, una arrel cúbica u1 de 2  11i , per fer-ho, sigui   Arg(2  11i ) l’angle del primer quadrant tal que
11   
tg   ( 79,695º ), aleshores u1  5   5  cos  i sin   2  i .
2 3  3 3

Aquests càlculs, que són els que s’han de fer amb més cura si es vol obtenir la resposta correcta, es poden evitar si es
té en compte que z1  2  i és una arrel cúbica de 2  11i , ja que (2  i )3  23  3  22  i  3  22  i 2  i 3  2  11i .

Les solucions de x 3  15x  4 són, per tant: x1  2Re u1  2Re(2  i )  4


,
 1 3    1 3  
x2  2Re u2  2Re    i   2  i   2  3 i x3  2Re u3  2Re    i   2  i   2  3 .
   
 2 2    2 2  

El conjunt més bonic de les matemàtiques


L’Anna està preocupada, ha d’estudiar per l’examen de matemàtiques i ha de fer una composició per l’assig-
natura de dibuix, però no sap si tindrà temps. Anant cap a l’institut li ho comenta al seu company Xavi, un
freaky dels ordinadors. Sorprenentment, en Xavi es posa a riure i li diu: “No t’amoïnis, pots fer les dues coses
alhora”. L’Anna es pensa que s’està rient d’ella i és a punt de deixar-lo amb la paraula a la boca, però en Xavi
comenta: “Ho dic de debò, es poden aconseguir gràfics espectaculars utilitzant els nombres complexos que
has d’estudiar per a mates”. I li comença a explicar com fer-ho:

 z1  z

A partir d’un nombre complex z has de fer dues successions:  , i la que formen els seus mòduls
 zn  1  zn  z
2

z1 , z2 , z3 ,…

Per a un nombre inicial z, pot passar que la successió de mòduls estigui fitada o no ho estigui. En el primer cas,
z pertany a l’anomenat conjunt de Mandelbrot i el seu afix corresponent es pinta de negre. En el segon cas, z no
pertany a aquest conjunt i el punt no es pinta.
Per exemple, per a z = 1 la successió dels mòduls és 1, 2, 5, 26, 677…, que no és fitada i, per tant, el punt (1, 0)
és blanc. No obstant això, si s’agafa z = −1, la successió de mòduls és 1, 0, 1, 0, 1…, que sí que és fitada i l’afix
de z, (−1, 0), és negre. Repetint aquest procés per a un nombre suficient de punts inicials arribaries a la figura
de la dreta.
Si vols dibuixos amb tonalitats diferents, com el de l’esquerra, pots donar una coloració diferent segons la ve-
locitat a la qual creixen els mòduls dels termes de la successió. En la figura, els valors de z pels quals la suc-
cessió creix més ràpid són d’un vermell més intens que aquells que porten a un creixement més lent.
L’Anna decideix fer un gràfic amb el conjunt de Mandelbrot. Ajuda-la trobant els tres primers termes de la suc-
cessió per a z  1  2i , z  3i i z  1 2i i decideix quin d’aquests tindrà un vermell més fosc en la represen-
tació.

Si z  1 2i obtenim z1  1  2i  5 , z2  2  6i  40 i z3  31  22i  1445

Si z  3i obtenim z1  3i  9 , z2  9  3i  90 i z3  72  51i  7785

Si z  1 2i obtenim z1  1  2i  5 , z2  4  2i  20 i z3  11 18i  445

Cap de les tres successions està fitada; la que creix més ràpid i es representarà amb un vermell més intens és la
corresponent a z  3i , la que creix més a poc a poc i es representarà amb un vermell menys intens és la corresponent
a z  1 2i .

240 Matemàtiques 1r Batxillerat


SOLUCIONARI UNITAT 7. NOMBRES COMPLEXOS

AUTOAVALUACIÓ

Comprova el que has après


4i
1. Escriu en forma binòmica el complex z  .
1  2i

4i (4  i )(1  2i ) 2  9i 2 9
z     i
1  2i (1  2i )(1  2i ) 5 5 5

1  i
2. Calcula el mòdul i l’argument de z  .
3 3  3i

2 135º  2   2 2
1 i  2 135º i 3 3  3i  6150º , per tant, z   
 6   , és a dir, el mòdul de z és i
6150º  6
 15º  6 345º
l’argument és 345º.

1 z
3. Si z  x  iy , calcula en termes de x i y el conjugat de: w  .
1 i

1  x  iy (1  x  iy )(1  i ) (1  x  y )  (1  x  y ) 1  x  y 1  x  y 1 x  y 1 x  y
w     i w   i
1 i (1  i )(1  i ) 2 2 2 2 2

4. Determina el conjunt M dels afixos dels z  x  iy tals que w  iz 2  (1  i )z  1 és un nombre real.

tenim w  i ( x  iy )2  (1 i )( x  iy )  1  (2xy  x  y  1)  ( x 2  y 2  x  y )i , per tant, w serà un nombre real si


x 2  y 2  x  y  0  ( x  y )( x  y  1)  0 , és a dir, el conjunt M està format pels punts de les rectes x  y  0 i
x  y 1 0 .

6 i 2 z
5. Si z1  i z2  1  i , escriu en forma trigonomètrica z1 , z2 i 1 .
4 z2

 2
z1  
 2 

2
2
 cos 330º i sin330º  , z2   2  2  cos315º i sin315º i
z1 1
  cos15º i sin15º  .
z2 2
 330º
315º

6. Escriu, en forma polar, les arrels cúbiques de i.

i  190º , per tant, les arrels cúbiques de i són z1  130º , z2  1150º i z3  1270º .

7. Descriu el conjunt dels nombres complexos z tals que z  i  10 .

És una circumferència de centre (0, 1) i radi 10 .

241 Matemàtiques 1r Batxillerat


SOLUCIONARI UNITAT 7. NOMBRES COMPLEXOS

2
8. Resol l'equació de segon grau x + 4x + 6 = 0. Observa que les solucions són nombres complexos.

4  8 4  8  1 4  2 2  i
x 2  4x  6  0  x     2  2  i
2 2 2

 7
9. Si un vèrtex d’un triangle equilàter inscrit en una circumferència centrada en l’origen és el punt A  6,  ,
 4
calcula els altres dos.

Els altres dos vèrtexs s’obtenen girant A respecte de l’origen 120º i 240º, és a dir, són els afixos dels complexos
7 1 3 1 3
que s’obtenen quan es multiplica 6  i per 160º    i i per 1120º    i . Així:
4 2 2 2 2

 7  1 3   7 3   7  7 3 7
 6  i     i    3     3 3   i  B  3  ,3 3 
 4  2 2   8   8  8 8 

 7  1 3   7 3   7  7 3 7
 6  i     i    3     3 3   i  C  3  , 3 3  
 4  2 2   8   8   8 8 

10. Si q(z )  2z 2  12z  26 , aleshores pots afirmar que per a tot z  es compleix que q (z ) és sempre positiu
com passa amb p( x )  2 x  12 x  26 si x és qualsevol nombre real?
2

No, ja que q( z ) pot agafar valors complexos, per exemple, q(i )  2i 2  12i  26  24  12i , i per a aquests nombres
no té sentit parlar de si són positius o negatius.

RELACIONA I CONTESTA
Tria l’única resposta correcta

1. El producte dels nombres complexos que tenen com a afixos els punts A i C del dibuix és el complex d’a-
fix:

A. E B. B C. F D. D

Com que A i C tenen mòdul 1, el seu producte també tindrà mòdul 1 i l’argument del producte serà la suma dels ar-
guments de A i C, és a dir, la resposta correcta és la D.

2. Si S  z  tals que (3  4i )z   , els afixos de z constitueixen:


A. Un triangle equilàter C. Una recta
B. Una circumferència D. Una paràbola

Si z  x  iy tenim (3  4i )z  (3  4i )( x  iy )  (3x  4y )  (3y  4x )i , que serà un nombre real si 3y  4x  0 , és a


dir, la resposta correcta és la C.

242 Matemàtiques 1r Batxillerat


SOLUCIONARI UNITAT 7. NOMBRES COMPLEXOS

3. El valor de (1  i )20  (1  i )20 és:

A. 0 B. –1 C. –i D. i

     2       
20 20
(1 i )20  (1 i )20  2 45º 2 315º 10
 210  210  210  0 , és a dir, la resposta cor-
900º 6300º 180º 180º
recta és la A.

Considera el nombre complex z  1  i 3   1  i 3  . Podem assegurar que:


2 2
4.

A. z és un nombre real.
B. z és un nombre imaginari pur.

z2
C. z 
2

z2
D. z  
2

Es calcula z  1  i 3   1  i 3   1  2i 3  i 2  3   1  2i 3  i 2  3    1 2i 3  3  1 2i 3  3  4i 3
2 2 2 2

 
Segons això, z és un imaginari pur.

 
2
z2 4i 3 48
D’altra banda,    24  z
2 2 2
La resposta és la B.

1 2
5. De les solucions de l’equació x  x  3 podem afirmar que:
2
A. Les dues són nombres imaginaris purs.
B. Les dues tenen el mateix mòdul.
C. Si z0 és una solució, –z0 també ho és.

D. Si z0 és una solució, també ho és.

1
1 1 4  3
1 1 2
Si x 2  x  3  x 2  x  3  0  x   1 5 i
2 2 1
2
2

 
2
Les dues solucions són dos complexos conjugats i, per tant, tenen el mateix mòdul: z0  12   5  6

Les respostes correctes són la B i la D.

Assenyala les respostes correctes


6. Si z0 és una solució de l’equació z 2  (1  i )  0 , aleshores:

Re z0   Im z0   1
2
z0  2
2 2
A. C.

243 Matemàtiques 1r Batxillerat


SOLUCIONARI UNITAT 7. NOMBRES COMPLEXOS

2
B. z0  2 D. z0 també és solució d’aquesta equació.

Com que z0 és una arrel quadrada de 1  i  2 45º , tenim que z0  4 2 45º o z0  4 2 45º 180º .
2 2

2  45º   45º 
En tots dos casos, z0  2 , és a dir, A és falsa i B vertadera. A més, com que cos     cos   180º  i
 2   2 
 45º   45º 
 180º  , en els dos casos el valor de Re z0   Im z0  és el mateix, en concret
2 2
sin    sin 
 2   2 
2 2
 45º   4 45º   2 45º 45º 
Re z0   Im z0    4 2 cos
2 2
   2 sin   2  cos  sin2   2 cos 45º  1 , per tant, C tam-bé
 2   2   2 2 
és certa.

Finalment, la D és falsa, ja que les dues solucions de l’equació, 4


2 45º i 4
2 45º 180º no són conjugades.
2 2

Tria la relació correcta entre les dues afirmacions


2
7. Considera l’equació de segon grau ax + bx + c = 0, on a, b i c són nombres reals, i les dues afirmacions
següents:
1. Una solució de l'equació és 4.
2. Una solució de l'equació és –2 + i.
Aleshores:

A. 1 ⇒ 2 però 2 ⇏ 1 C. 1 ⇔ 2

B. 2 ⇒ 1 però 1 ⇏ 2 D. 1 i 2 s'exclouen entre elles

La resposta correcta és la D.

Assenyala la dada innecessària per contestar

8. Per determinar un nombre complex z ens donen les dades següents. Determina quina dada és inneces-
sària.
1. El quadrat de la part imaginària és un nombre negatiu.
2. La suma amb el seu conjugat és un nombre positiu.
3. La seva quarta potència és –119 + 120i.
4. El quadrat de la part imaginària és –4.
A. Es pot eliminar la dada 1.
B. Es pot eliminar la dada 2.
C. Es pot eliminar la dada 3.
D. Es pot eliminar la dada 4.

La dada innecessària és la 1.

244 Matemàtiques 1r Batxillerat

You might also like